Anda di halaman 1dari 441

Dermoscopy

An Illustrated Self-Assessment Guide


NOTICE

Medicine is an ever changing science. As new research and experience broaden


our knowledge, changes in treatment and drug therapy are required. The authors
and the publisher of this work have checked with sources believed to be reliable
in their efforts to provide information that is complete and generally in accord
with the standards accepted at the time of publication. However, in view of the
possibility of human error, or changes in medical sciences, neither the authors
nor the publisher nor any other party who has been involved in the preparation
or publication of this work warrents that the information contained herein is in
every respect accurate or complete, and they disclaim all responsibility for any
errors or omissions or for the results obtained from use of the information con-
tained in this work. Readers are encouraged to confirm the information con-
tained herein with other sources. For example and in particular, readers are
advised to check the product information sheet included in the package of each
drug they plan to administer to be certain that the information contained in this
work is accurate and that changes have not been made in the recommended dose
or in the contraindications for administration. This recommendation is of par-
ticular importance in connection with new or infrequently used drugs.
Dermoscopy
An Illustrated Self-Assessment Guide
Robert H. Johr, MD
Clinical Professor of Dermatology and
Associate Clinical Professor of Pediatrics
Pigmented Lesion Clinic
University of Miami School of Medicine
Miami, Florida

Prof. Wilhelm Stolz, MD


Director Clinic of Dermatology, Allergology and Environmental Medicine
Hospital München Schwabing, and
Professor of Dermatology, Faculty of Medicine
Ludwig-Maximilians-Universität Munich
Germany

New York Chicago San Francisco Lisbon London Madrid Mexico City
Milan New Delhi San Juan Seoul Singapore Sydney Toronto
Copyright © 2010 by The McGraw-Hill Companies, Inc. All rights reserved. Except as permitted under the United States Copyright Act of 1976, no part of
this publication may be reproduced or distributed in any form or by any means, or stored in a database or retrieval system, without the prior written permission
of the publisher.

ISBN: 978-0-07-161356-9

MHID: 0-07-161356-0

The material in this eBook also appears in the print version of this title: ISBN: 978-0-07-161355-2,

MHID: 0-07-161355-2.

All trademarks are trademarks of their respective owners. Rather than put a trademark symbol after every occurrence of a trademarked name, we use names in
an editorial fashion only, and to the benefit of the trademark owner, with no intention of infringement of the trademark. Where such designations appear in this
book, they have been printed with initial caps.

McGraw-Hill eBooks are available at special quantity discounts to use as premiums and sales promotions, or for use in corporate training programs. To contact
a representative please e-mail us at bulksales@mcgraw-hill.com.

TERMS OF USE

This is a copyrighted work and The McGraw-Hill Companies, Inc. (“McGrawHill”) and its licensors reserve all rights in and to the work. Use of this work is
subject to these terms. Except as permitted under the Copyright Act of 1976 and the right to store and retrieve one copy of the work, you may not decompile,
disassemble, reverse engineer, reproduce, modify, create derivative works based upon, transmit, distribute, disseminate, sell, publish or sublicense the work or
any part of it without McGraw-Hill’s prior consent. You may use the work for your own noncommercial and personal use; any other use of the work is strictly
prohibited. Your right to use the work may be terminated if you fail to comply with these terms.

THE WORK IS PROVIDED “AS IS.” McGRAW-HILL AND ITS LICENSORS MAKE NO GUARANTEES OR WARRANTIES AS TO THE ACCURACY,
ADEQUACY OR COMPLETENESS OF OR RESULTS TO BE OBTAINED FROM USING THE WORK, INCLUDING ANY INFORMATION THAT
CAN BE ACCESSED THROUGH THE WORK VIA HYPERLINK OR OTHERWISE, AND EXPRESSLY DISCLAIM ANY WARRANTY, EXPRESS OR
IMPLIED, INCLUDING BUT NOT LIMITED TO IMPLIED WARRANTIES OF MERCHANTABILITY OR FITNESS FOR A PARTICULAR PURPOSE.
McGraw-Hill and its licensors do not warrant or guarantee that the functions contained in the work will meet your requirements or that its operation will be
uninterrupted or error free. Neither McGraw-Hill nor its licensors shall be liable to you or anyone else for any inaccuracy, error or omission, regardless of cause,
in the work or for any damages resulting therefrom. McGraw-Hill has no responsibility for the content of any information accessed through the work. Under
no circumstances shall McGraw-Hill and/or its licensors be liable for any indirect, incidental, special, punitive, consequential or similar damages that result
from the use of or inability to use the work, even if any of them has been advised of the possibility of such damages. This limitation of liability shall apply to
any claim or cause whatsoever whether such claim or cause arises in contract, tort or otherwise.
Contents

Foreword vii
Preface ix
Acknowledgments xi

Chapter 1 Dermoscopy from A to Z 01


Chapter 2 Scalp, Face, Nose, and Ears 27
Chapter 3 Trunk and Extremities 95
Chapter 4 Palms, Soles, Nails 325
Chapter 5 Genitalia 389
Index 419
This page intentionally left blank
Foreword

The new textbook, “Dermoscopy: An Illustrated Self- lesions dermoscopically, helping to separate those lesions
Assessment Guide” by Dr. Robert H. Johr and Prof. Wilhelm which deserved follow-up from those which deserved biopsy.
Stolz, is a welcome addition to the evolving world of dermo- To this day, I continue to use dermoscopy not only to evalu-
scopic knowledge. I have reviewed all of the images and the ate pigmented lesions in children, but I also use it as an
text and feel that the contributions are indeed unique. First of advanced “pocket magnifier”. Specifically, hair shaft disor-
all, the exquisite nature of the photographic images deserves a ders, scabies, questionable molluscum lesions, etc., are read-
comment. Also, the clarity of labeling for each image separates ily identified when separated out by means of dermoscopy.
this text from all others I have seen. Specifically, all of the key When I forget my dermascope or when I find it uncharged,
features are isolated by means of circles, rectangles, asterisks, I actually feel less qualified to evaluate my patients.
etc., and this clear demarcation has succeeded where many I firmly believe that dermoscopy should be taught as an
similar textbooks fail. Secondly, the questions, statements and essential skill in every residency program and that all practition-
the answer format draws the reader into the dialogue. You ers should strive to learn this technique. I certainly learned it
really cannot escape committing yourself to a diagnosis, or when I was in my mid 50s, hence, a mature dog can learn new
differential diagnosis, as you answer the questions and state- tricks. I am still surprised by the fact that virtually all children
ments posed. I think these two unique aspects of the textbook referred to me for evaluation of pigmented lesions have not
serve the learning process in a very positive way. been viewed with dermoscopy by their referring dermatologists.
I should also add that I am an unabashed enthusiast of This needs to change. This technique adds a dimension which
dermoscopy. Although I presently confine my clinical prac- clearly makes us better diagnosticians and better dermatolo-
tice to pediatric dermatology, until 7 years ago, I saw adults gists. The book by Drs, Johr and Stolz will help us in this effort.
on a daily basis and evaluated all of their pigmented lesions
with dermoscopy. I have been committed to learning the Ronald C. Hansen, MD
technique for the last 14 years. Most of that learning curve Chief, Pediatric Dermatology
has involved discussions and teaching by Dr. Robert Johr. Phoenix Children’s Hospital, Phoenix, Arizona
Early on, we saw patients with complex pigmentary disorders Professor, Dermatology and Pediatrics
such as xeroderma pigmentosum and evaluated numerous University of Arizona College of Medicine, Tucson, Arizona
This page intentionally left blank
Preface

Dermoscopy is itself a language full of terms that have specific the important features of each case. Our goal is to fully
meanings and even connotations, depending on any given demonstrate the global features and local criteria of each
lesion being examined. As in any language, its vocabulary is a lesion. This is another very important unique teaching point
work in progress based in consensus among its “expert” of our book.
speakers who are striving to create a system to communicate Each case has a discussion of all of its salient features.
dermoscopic findings based on observational data. In order Not in long drawn out paragraphs, but in outline form. We
for there to be mutual comprehension among users of this realize that your time is valuable and want to make the learn-
powerful technique, the language of dermoscopy must be ing and recall process as easy as possible.
spoken properly. This is not an easy task since there is a signifi- Series of cases are organized into groups. For example,
cant learning curve to master the technique as well as its ter- there are lesions in which the major feature might be pigment
minology. It takes study, practice, and dedication. network, dots and globules, regression, pink, blue or black
Dermoscopy is the standard of care around the world and color, or vascular structures. There are similar-looking clini-
is becoming very popular in the United States. Dermatologists cal and/or dermoscopic images grouped together in specific
and other groups of physicians are realizing what a valuable body locations, such as brownish spots on an ear lobe or in
tool it is. The goal of this book is to teach what we believe are the genital area. This simulates real-life encounters. One case
the important general principals and specific points of der- often flows into the next and knowledge gained from the pre-
moscopy and to allow for users to “self-assess” their knowledge vious case is needed to solve the next case. Melanocytic, non-
and skills using the techniques taught here. melanocytic, benign, malignant, or inflammatory pathology
In an era of information overload, we designed the book from head to toe with 95 melanomas and their most impor-
to be short, sweet, and to the point. We want it to be an easy, tant simulators.
enjoyable, and practical read. Important principles are often Each case ends with a series of dermoscopic and/or
repeated which is a good way for them to be remembered. clinical pearls based on years of experience treating patients
We “keep it real” with 191 cases that any busy clinician with skin cancer. The patients’ well-being trumps political
may have the opportunity to see in general dermatology clinic correctness. The book is sprinkled with general principles
on a daily basis. Great clinical and dermoscopic images with and specific points that are controversial but strongly embed-
short histories are followed by five “true or false” statements. ded in our core beliefs.
As in real life, then comes the decision making in check box This book was a coproduction between United States
form: what is the potential risk and what is the diagnosis? and Germany. The text was developed in Florida and most of
Finally, the disposition of the case; whether to effect no inter- the cases were seen in Munich.
vention, follow-up, or to make a histopathologic diagnosis? Being a competent diagnostician must include the
The concept of dermoscopic differential diagnosis is found tissue-sparing and potentially life-saving technique called
throughout the book. In most cases, we do not get into the dermoscopy.
controversial issue of the best technique to make a Each of us has a profound responsibility for the well-
histopathologic diagnosis. We leave that up to you. being of every patient that walks through the door. Always
Turn the page, and the answers to the statements are regard each patient as someone’s precious loved one as if they
given in a format that separates our book from the others. were your own!
The dermoscopic images are presented again with an exten-
sive description of the criteria in the lesion. It is essential to Robert H. Johr Prof. Wilhelm Stolz
evaluate as much as possible before making a diagnosis. Boca Raton, Florida Munich, Germany
There are many circles, boxes, arrows, and stars to point out
This page intentionally left blank
Acknowledgments

I would like to thank James A. Ida, MD academic chief resi- First and foremost, I would like to express my deepest gratitude
dent in the Department of Dermatology at Tufts Medical to my wife, Karola, who has lovingly shared my many dermoscopic
Center in Boston as well as a friend, for his excellent efforts as and academic pursuits over the last two decades. For her tireless
resident editor. James’s passion for language and grammatical enthusiasm and skill with the beautiful color photography, a cor-
style as well as keen interest in dermoscopy and in helping nerstone of our text, I thank Mrs. Ulrike Brückl. Special thanks
produce a superb text, made the tedious job of writing a book also go to our nurses, Mss. Carolin Mertens, Antje Angilotti, and
much more enjoyable. Thanks! Christa Meinhold, whose consistent passion and professionalism
I want to thank my wife of 38 years, Irma, for letting me be contributed greatly to our text. To the many physicians who sup-
free in mind and spirit to spend the many hours it took to write ported me in my dermoscopy clinic and in the case preparations,
this book. Her encouragement and willingness to listen to me most especially Drs. Brigitte Coras, Stefanie Guther, Anette
when I thought or wrote something exciting was very helpful. Michael, Katrin Ramrath, Alexandra Tillmann, and Ulrike
Weigert, I wish to extend my sincerest thanks. Finally, special
Dr. Robert H. Johr thanks go to Mrs. Agnes Kaldewei, my long-time clinical and sci-
entific assistant. Without her continuous and ever pleasant sup-
port, this text would not have been possible.

Dr. Wilhelm Stolz


This page intentionally left blank
Dermoscopy
An Illustrated Self-Assessment Guide
This page intentionally left blank
Chapter 1

Dermoscopy from A to Z
This page intentionally left blank
Chapter 1 Dermoscopy from A to Z 3

SYNONYMS ■ Helps to avoid unnecessary surgery


■ Helps to plan surgery
■ Dermatoscopy
■ Helps one to work better with their pathologist (asym-
■ Skin surface microscopy
metrical high risk criteria, collision tumors, dermo-
■ Epiluminescence microscopy (ELM)
scopic–pathologic correlation)
■ Digital dermoscopy/digital ELM
■ Patient reassurance
■ Auflichtmikroskopie (German)
■ Allows for follow up of patients with multiple nevi digi-
■ Dermoscopia/Dermatoscopia (Spanish/Italian)
tally to find changes over time
■ Grammatically, dermatoscopy is correct English; however,
dermoscopy is the term used by the International
Dermoscopy Society (IDS), experienced dermoscopists, Dermoscopic Digital Monitoring
and is most commonly used in the literature. This might ■ There are pigmented skin lesions that are not high risk
change in the future and dermatoscopy could become the enough to warrant immediate histopathologic diagnosis,
term of choice again. yet not so banal that there is no concern at all
■ There are melanomas that do not appear to be high risk
DEFINITION clinically or with dermoscopy
■ They are only diagnosed after monitoring for dermoscopic
■ Dermoscopy is an in vivo, noninvasive technique in which
changes over time when comparing baseline with subse-
oil or fluid (eg, mineral oil, gels, alcohol, and water) is
quent digital images
placed on the lesion
■ Short-term monitoring is performed every three or four
■ Fluid eliminates reflection of light from the surface of
months
the skin allowing visualization of color and structure in
■ Any change over time could be a melanoma and is an
the epidermis, dermo-epidermal junction, and papillary
indication to make a histopathologic diagnosis
dermis
■ Long-term monitoring is done at six month to yearly
■ The color and structure visualized cannot be seen with
intervals
the naked eye or with typical magnification that clini-
■ Important changes include asymmetrical enlarge-
cians use
ment, asymmetrical changes within the lesion without
■ Polarizing light and digital instrumentation do not
enlargement, the appearance of high risk criteria (eg,
require fluid
irregular pigment network, dots, globules, streaks,
■ When using polarized light dermoscopy
blotches, regression, polymorphous vessels), disap-
■ Light from a polarized light source penetrates the stra-
pearance of well-developed local criteria, new colors
tum corneum with less scatter
■ Single or multiple suspicious pigmented skin lesions can
■ A second polarizer screens out scattered surface light
be chosen for short- or long-term digital monitoring
resulting in the physician seeing primarily light from
the deeper structures
■ This removes the need for contact with the skin and the

need for immersion fluids, resulting in faster examina-


THE TWO-STEP ALGORITHM
tion times ■ The analysis of a suspicious skin lesion is a two-step
process
■ Step one: determines if it is melanocytic or
BENEFITS OF DERMOSCOPY nonmelanocytic
■ Helps to differentiate melanocytic from nonmelanocytic ■ Step two: if it has the criteria for a melanocytic lesion,

skin lesions the second step is to determine if it is low, intermediate,


■ Helps to differentiate benign from malignant skin lesions or high risk using the melanocytic algorithm of your
■ With dermoscopy the diagnostician’s sensitivity to diag- choice
nose melanoma is 90% compared to 74% when the ■ Pattern analysis was the first melanocytic algorithm
technique is not used developed for this purpose and is most often used by
■ Dermoscopy improves the diagnosis of melanoma by experienced dermoscopists. Variations of pattern analysis
16% (the simplified algorithms) have also been developed,
■ The number of benign lesions excised for each including:
melanoma found (malignant/benign ratio) is signifi- ■ The ABCD rule of dermatoscopy (the second algorithm

cantly decreased with dermoscopy developed) (Table 1-1)


■ Increases the diagnosis of early melanoma ■ The seven-point checklist (Table 1-2)

■ Increases the diagnosis of melanoma incognito (false ■ Menzies method (eleven-point checklist) (Table 1-3)

negative melanoma) ■ The latest three-point checklist (Table 1-4)


4 DERMOSCOPY: AN ILLUSTRATED SELF-ASSESSMENT GUIDE

Table 1-1 ABCD RULE OF DERMATOSCOPY. Table 1-3 MENZIES SCORING METHOD.
IDENTIFY CRITERIA AND GIVE THEM THE ELEVEN-POINT CHECKLIST
POINTS TO DETERMINE THE TOTAL
DERMATOSCOPY SCORE (TDS) DERMOSCOPIC CRITERION

DERMOSCOPIC CRITERION DEFINITION SCORE WEIGHT Negative Features


FACTOR Symmetry of pattern
Presence of single color
Asymmetry in 0, 1, or 2 perpendicular axes; assess contour,
colors and structures 0-2 Positive Features
Border abrupt ending of pigment pattern at periphery in
Blue-white veil
0-8 segments 0-8
Multiple brown dots
Color presence of up to 6 colors (white, red, light-brown,
Pseudopods (streaks)
dark-brown blue-gray, black) 1-6
Radial streaming (streaks)
Dermoscopic structures presence of network, structureless
Scar-like depigmentation
(homogeneous) areas, branched streaks, dots, and
Peripheral black dots/globules
globules 1-5 Multiple (5 or 6) colors
Formula for calculating total dermatoscopy score (TDS): (A score x 1.3) + Multiple blue/gray dots
(B score x 0.1) + (C score x 0.5) + (D score x 0.5) = TDS. Interpretation of total Broadened network
score: <4.75. Benign melanocytic lesion 4.75-5.45; suspect lesion (close follow-up
or excision recommended); >5.45, lesion highly suspect for melanoma. For melanoma to be diagnosed both negative features must be absent and
one or more of the 9 positive features must be present.

Criteria Defined
Step One: Identification of Criteria Melanocytic Lesion
Look for the criteria associated with a melanocytic lesion PIGMENT NETWORK/NETWORK
(Table 1-5). If one does not find them, the search is on
■ On the trunk and extremities
for the criteria associated with seborrheic keratosis, basal
■ Shades of black or brown
cell carcinoma, dermatofibromas, vascular lesions, and
■ Honeycomb-like, reticular, web-like line segments
others.
(elongated and hyperpigmented rete ridges) with
■ Not all of the possible criteria are needed to make a
hypopigmented holes (dermal papilla)
diagnosis
■ There are nonmelanoctyic lesions that can have a
■ When there is an absence of criteria for a
pigment network (ie, solar lentigo, seborrheic keratosis,
melanocytic lesion, seborrheic keratosis, basal cell
dermatofibroma)
carcinoma, dermatofibroma, or vascular lesion, you
will have to deal with a melanocytic lesion by PSEUDONETWORK/PSEUDOPIGMENT NETWORK
default ■ Because the skin of the head and neck is thin and does
■ The “default category” is the last criterion used to not have well-developed rete ridges, one sees
diagnose a melanocytic lesion (Figure 1-1) ■ Appendageal openings/adnexal structures (sebaceous

glands, hair follicles)


■ Uniform, round white or yellowish structures

Table 1-2 SEVEN-POINT CHECKLIST Table 1-4 THREE-POINT CHECKLIST TO DIAGNOSE


DERMOSCOPIC CRITERION SCORES HIGH RISK LESIONS (MELANOMA, BASAL
CELL CARCINOMA)
1. Atypical pigment network (major criteria) 2
2. Blue-whitish veil 2 Asymmetry of color and/or structure
3. Atypical vascular pattern 2 Irregular pigment network
4. Irregular streaks (minor criteria) 1 Blue and / or white color
5. Irregular dots/globules 1 2 out 3, 3 out 3 → excise
6. Irregular blotches 1 The three-point check list is based on simplified pattern
7. Regression structure 1 analysis and is intended to be used by nonexpert dermo-
By simple addition of the individual scores a minimum total score of 3 is scopists as a screening technique. Its aim is to diagnose
required for the diagnosis of melanoma, whereas a total score of less than 3 is melanocytic and nonmelanoctyic potentially malignant
indicative of nonmelanoma. pathology.
Chapter 1 Dermoscopy from A to Z 5

■ When they penetrate areas of diffuse pigmentation,


Table 1-5 CRITERIA FOR VARIOUS LESIONS reticular-like structures are formed that are referred to as
the pseudonetwork
CRITERIA FOR A MELANOCYTIC LESION
■ Monomorphous appendageal openings can often be seen
Pigment network (trunk and extremities) on the skin of the face without any pigmentation
Aggregated globules
■ They should not be confused with the milia-like cysts
Homogeneous blue color of a blue nevus
Parallel patterns on acral sites
seen in seborrheic keratosis
By default ■ It is not always possible to make the differentiation
■ Consequences could be misdiagnosing lentigo maligna for
CRITERIA FOR A SEBORRHEIC KERATOSIS a seborrheic keratosis
Milia-like cysts ■ This criterion can also be seen with nonmelanocytic
Pseudofollicular openings lesions (ie, solar lentigo, seborrheic keratosis)
Fissures and ridges ■ It is not 100% diagnostic of a melanocytic lesion
Fat fingers
Fingerprint pattern DOTS AND GLOBULES
Hairpin-shaped vessels ■ Roundish structures distinguished only by their relative sizes
Moth-eaten borders ■ Dots (0.1mm) are smaller than globules (greater than
Sharp demarcation 0.1mm)
CRITERIA FOR A BASAL CELL CARCINOMA ■ Black, brown, gray, or red
■ When black, they can represent melanin or atypical
Absence of pigment network
Arborizing blood vessels melanocytes in the epidermis
Pigmentation ■ Regular brown dots and globules represent nests of

Ulceration melanocytes at the dermo-epidermal junction


Spoke-wheel structures ■ Irregular brown dots and globules represent nests of

atypical melanocytes at the dermo-epidermal junction


CRITERIA FOR A DERMATOFIBROMA
■ Fine grayish dots (“peppering”) represent free melanin
Central white patch
and/or melanophages in the papillary dermis, which
Peripheral pigment network
can be seen in regression areas along with other criteria
CRITERIA FOR A VASCULAR LESION or alone in benign pathology such as late stage lichen
Vascular spaces (lacunae ) planus-like keratosis or posttraumatic
■ Reddish globules can be seen in melanoma (neovascu-

larization)
■ It is written and taught that globules identify a
melanocytic lesion with no mention of the smaller dots.
The reality is that both dots and globules define a
melanocytic lesion (Figure 1-2)

Figure 1-1 Amelanotic Melanoma. This is a melanocytic lesion by


default because there is an absence of criteria for a melanocytic lesion, seb-
orrheic keratosis, basal cell carcinoma, dermatofibroma, or hemangioma.
The blue-white color (arrow) is a clue that this might be a melanocytic
lesion. There are pinpoint/dotted (yellow boxes) and irregular linear (black
boxes) vessels plus a general milky-red background color. Note: This inter-
digital melanoma was mistakenly treated as a tinea for two years. Figure 1-2 Acquired Nevus. This is a melanoctic lesion because it has
(Reproduced, with permission, from Journal of Drugs in Dermatology. New pigment network (black boxes) and globules (circles). There is a small
Methods and Technologies. May 2008-Vol 7-Issue 5. Fig 1b.) hemangioma adjacent to the nevus (arrow).
6 DERMOSCOPY: AN ILLUSTRATED SELF-ASSESSMENT GUIDE

Figure 1-3 Blue Nevus. The classic homogenous blue color of a blue nevus. Figure 1-4 Acral Nevus. This is a melanocytic lesion on acral skin with the
benign parallel furrow pattern. Pigmentation is in the thin furrows (arrows)
with globules (boxes) in the thicker ridges (stars).

HOMOGENEOUS BLUE PIGMENTATION


■ Structureless blue color usually in the absence of local crite- ■ Thick fibrillar variant with thick oblique lines
ria such as pigment network, dots, or globules (Figure 1-3) ■ Thin filamentous variant with fine thin lines
■ Many variations of homogeneous blue color with or with- ■ Pressure can change the lattice-like pattern into a fibrillar
out the white color of scarring usually represent a blue pattern
nevus ■ Irregular thick and pigmented line segments could be
■ The history is important because there is a differential seen in an acral melanoma
diagnosis which could include GLOBULAR PATTERN (BENIGN)
■ A lesion as banal as a radiation tattoo
■ Brown globules without a parallel component
■ One more ominous such as nodular or cutaneous
RETICULAR PATTERN (BENIGN)
metastatic melanoma
■ A lesion with only pigment network
PARALLEL PATTERNS/ACRAL PATTERNS/VOLAR SURFACES ON GLABROUS
NON HAIR BEARING SKIN HOMOGENEOUS PATTERN (BENIGN)
■ Furrows (also referred to as fissures) and ridges on the ■ Brown homogeneous color
skin of the palms and soles ■ Absence of local criteria (ie, pigment network, globules)
■ Dermatoglyphics are the skin markings that can form

whorls, loops, and arches


■ Can create parallel patterns
PARALLEL FURROW PATTERN (BENIGN PATTERN)
■ Single thin or thick brown parallel lines in the furrows of
the skin (crista superficialis limitans)
■ Variations include two brown lines on both sides of

the hypopigmented furrows with or without dots and


globules
■ Single line of dots and globules along the furrows

(single-dotted variant)
Double line of dots and globules parallel to the hypopig-
mented furrows (double-dotted variant) (Figure 1-4)
LATTICE-LIKE PATTERN (BENIGN PATTERN)
■ Brown parallel lines in the furrows
■ Brown lines running perpendicular to the furrows forming
a ladder-like picture (Figure 1-5)
Figure 1-5 Acral Nevus. Brown lines in the furrows (black arrows) and
FIBRILLAR PATTERN (BENIGN PATTERN) perpendicular to the furrows (yellow arrows) characterize the lattice-like
■ Uniform brown lines that run in an oblique (///////) pattern. Pressure on the foot can change this into the fibrillar pattern with
direction fine oblique (/////) lines.
Chapter 1 Dermoscopy from A to Z 7

Figure 1-6 Acral Melanoma. The parallel ridge pattern diagnoses this Figure 1-8 Acquired Nevus. There is an increased incidence of acral
acral melanoma with pigmentation in the thicker ridges (black arrows). The melanoma in darker skinned races. This nevus on the palm of an African-
thin white lines are the furrows (yellow arrows). American was without change and demonstrates the benign parallel ridge
pattern. Pigmentation is seen in the ridges of the nevus (yellow arrows) and
in the ridges of the entire palm (white arrows).

PARALLEL RIDGE PATTERN (IN SITU AND EARLY INVASIVE ACRAL MELANOMA)
■ Pigmentation is in the thicker ridges of the skin (crista
profunda intermedia) (Figure 1-6) ■ Parallel ridge pattern can be found on normal skin in
■ Sometimes there are monomorphous round white darker skinned races (Figure 1-8)
structures in the ridges that represent the acrosyringia DIFFUSE VARIEGATE PATTERN (MELANOMA)
of the sweat ducts that are said to look like a “string ■ Pigmented blotches
of pearls” ■ Black, brown, or gray
■ They are an important clue to help identify the ridges
MULTICOMPONENT PATTERN (MELANOMA)
■ They are never found in the furrows, always in the

ridges ■ Filled with regular and irregular criteria


■ Parallel ridge pattern is not 100% diagnostic for ■ Multiple colors plus areas with several acral patterns
melanoma and can also be created by blood (black heel, (parallel furrow, lattice-like, fibrillar)
talon noir) (Figure 1-7) NONSPECIFIC PATTERN (MELANOMA)
■ If one cannot determine any of the above benign or
malignant patterns; this represents a red flag for concern

PEARLS
■ There can be exceptions to every dermoscopic rule
■ The history and clinical appearance of the lesion are
important and should not be ignored
■ If a pigmented lesion on the palms or soles is rapidly changing
yet has a typical benign pattern, it still could be melanoma
■ A supposedly benign acral pattern with irregularity to the
components could be high risk
■ The presence of blood at acral sites (palms, soles, nails)
may or may not be associated with melanoma
■ Look carefully for other important criteria
■ If in doubt, cut it out!
Seborrheic Keratosis
MILIA-LIKE CYSTS
Figure 1-7 Acral Hemorrhage. The parallel ridge pattern created by ■ Variously sized white or yellow structures
blood (white arrows). ■ Small or large, single or multiple
8 DERMOSCOPY: AN ILLUSTRATED SELF-ASSESSMENT GUIDE

■ They can appear opaque or bright like “stars in the sky”


(epidermal horn cysts)
■ Melanocytic lesions can have a few milia-like cysts
PSEUDOFOLLICULAR OPENINGS/COMEDO-LIKE OPENINGS
■ Sharply demarcated roundish structures
■ Pigmented or nonpigmented
■ Shape and size can vary, not only within a single lesion, but
from lesion to lesion in an individual patient (Figure 1-9)
■ Large keratin-filled irregularly-shaped openings are called
crypts
■ When pigmented, they can be brownish-yellow or even
dark brown and black (keratin-filled invaginations of the
epidermis)
■ Pigmented pseudofollicular openings can be hard to dif-
ferentiate from the pigmented dots and globules of a
melanocytic lesion Figure 1-10 Seborrheic Keratosis. A striking brain-like pattern created
■ A few pseudofollicular openings can be found in by pigmented fissures (yellow arrows) and light ridges (black arrows). Many
melanocytic lesions of the ridges look like “fat fingers.”
FISSURES AND RIDGES
■ Fissures (sulci) and ridges (gyri) seen in papillomatous
seborrheic keratosis can create several patterns ■ All of these patterns are commonly seen in this ubiquitous
■ Cerebriform or brain-like in which they resemble a benign skin lesion (Figure 1-10)
sagittal section through the cerebral cortex FINGERPRINT PATTERN
■ Mountain-like with variously sized or uniformly ■ Brown fine/thin parallel line segments that resemble
roundish structures representing mountains fingerprints
(ridges) and fine pigmented lines representing ■ Differ from the pigment network where the line segments
valleys (fissures) are honeycomb-like or reticular
■ Possible to confuse the mountain and valley pattern ■ Fingerprint pattern can be seen in flat seborrheic keratosis
with the cobblestone pattern of a melanocytic lesion or in solar lentigines
■ Pigmented fissures should not be confused with the ■ Some authors believe that solar lentigines are flat seborrheic
pigment network of a melanocytic lesion keratosis
■ Hypo- and hyperpigmented ridges can be digit-like
HAIRPIN VESSELS
(straight, kinked, circular, or branched) and are referred
■ Elongated vessels (capillary loops) resembling hairpins
to as “fat fingers”
(Figure 1-11)

Figure 1-9 Seborrheic Keratosis. Sharp borders (red arrows), milia-like


cysts (black arrows), and pseudofollicular openings (boxes) characterize this Figure 1-11 Seborrheic Keratosis. An especially well-formed hairpin
seborrheic keratosis. vessel (box) in a seborrheic keratosis.
Chapter 1 Dermoscopy from A to Z 9

■ May or may not be surrounded by hypopigmented halos


■ Light halo indicates a keratinizing tumor and may be
found in keratoacanthomas
■ Irregular and thick hairpin vessels can be seen in melanoma
MOTH-EATEN BORDERS
■ Flat or slightly raised brown seborrheic keratoses
■ Well-demarcated, concave borders that are felt to resemble
a “moth-eaten” garment
SHARP DEMARCATION
■ The majority of seborrheic keratoses have sharp,
well-demarcated borders
■ Not always indicative of melanoma in a pigmented lesion
(Figure 1-9)
Basal Cell Carcinoma
ABSENCE OF THE CRITERIA SEEN IN A MELANOCYTIC LESION
Figure 1-12 Basal Cell Carcinoma. This pigmented basal cell carcinoma
■ Specifically, absence of a pigment network
has classic arborizing vessels (black arrows), linear vessels (yellow box), gray
■ Pigmented dots and/or globules found in some basal cell blotches (black boxes), blue globules (yellow arrows), and fine gray dots (circles).
carcinomas The three different presentations of pigmentation point out to how variable
■ Raises the issue of dermoscopic differential diagnosis of this criterion can be. (Reproduced, with permission, from Journal of Drugs in
individual criterion Dermatology. New Methods and Technologies. Sep 2007-Vol 6-Issue 9. Fig 2b.)

ARBORIZING VESSELS
■ One of the most sensitive and specific vascular structures ■ Blue
seen with dermoscopy ■ Red
■ Red tree-like branching telangiectatic blood vessels
■ White
■ In focus vessels because they are on the surface of the ULCERATION
lesion ■ Single or multiple areas, where there is loss of epidermis
■ Out-of-focus arborizing vessels should raise a red flag with oozing blood or congealed blood and crusts (Figure 1-13)
for concern that the lesion might be a melanoma ■ There should be no recent history of trauma
■ Can be thick or thin SPOKE-WHEEL STRUCTURES
■ Most often there are different caliber vessels in a single
■ To date, spoke-wheel structures are the only criterion with
lesion dermoscopy, which are 100% diagnostic
■ Can also be found in ■ Can be found in up to 10% of basal cell carcinomas
■ Benign nevi
■ May or may not be associated with the other criteria
■ Recurrent nevi
used to make the diagnosis
■ Sebaceous gland hyperplasia
■ Scars

■ On sun-damaged skin

■ Melanoma

PIGMENTATION
■ Basal cell carcinoma may or may not contain pigment
(pigmented nests or island of basal cell carcinoma in the
dermis) that can range from
■ Fine dots to large leaf-like structures (bulbous exten-

sions forming a leaf-like pattern)


■ Not necessary to try to determine if “leaf-like” structures

(maple leaf-like areas) are present since in reality this


is a difficult task
■ Blue-gray ovoid nets

■ Multiple blue-gray globules


■ Colors that can be seen (Figure 1-12)
■ Black

■ Brown
Figure 1-13 Basal Cell Carcinoma. Arborizing vessels (black arrows) and
■ Gray ulceration (yellow arrows) characterize this nonpigmented basal cell carcinoma.
10 DERMOSCOPY: AN ILLUSTRATED SELF-ASSESSMENT GUIDE

■ Well-defined pigmented radial projections meeting at a


darker central globule/central axle/hub
■ Complete or incomplete variations of this structure can
be seen and one often has to use their imagination to
make the identification
■ Finding spoke-wheel structures might be the only clue to
the correct diagnosis

PEARLS
■ A nonhealing area in an adult that bleeds spontaneously
is a basal cell carcinoma until proven otherwise
Dermatofibroma
CENTRAL WHITE PATCH
■ Most typical presentation of this criterion is
■ Centrally located

■ Scar-like Figure 1-15 Atypical Dermatofibroma. Regressive melanoma is in the


■ Bony or milky-white
dermoscopic differential diagnoses of this atypical dermatofibroma. There is
asymmetry of color and structure, the multicomponent global pattern, irreg-
■ Homogeneous area (scarring in this fibrohistiocytic tumor)
ular pigment network (box), irregular globules (red arrows), and irregular
■ Several variations such as white network-like structures (white blotches (yellow arrows). This warrants a histopathologic diagnosis.
network, negative pigment network, reticular depigmenta-
tion), which can also be seen in Spitz nevi and melanoma
■ Telangiectatic vessels with different shapes can also be ■ Atypical dermatofibromas with the following features are
found anywhere in the lesion melanoma mimics that warrant a histopathologic diagnosis
■ Not all dermatofibromas have a central white patch ■ Irregular pigment network

■ The clinically firm feel and “dimple” sign should be used ■ Irregular dots/globules/blotches

to help make the diagnosis ■ Pink color

■ Irregular regression-like white color


PIGMENT NETWORK
■ Polymorphous (different shapes) vascular structures
■ Dermatofibromas are one of the nonmelanocytic lesions that
(Figure 1-15)
can have a pigment network; solar lentigines are another
■ In most cases, a fine peripheral pigment network with
Vascular Lesions
thin brown lines is seen LACUNAE/LAGOONS/SACCULES
■ Ring-like structures that are a variation of a hyperpig- ■ Sharply demarcated bright red to bluish round or oval
mented network (Figure 1-14) structures (dialated vascular spaces in the dermis)
■ Not all dermatofibromas have a pigment network (Figure 1-16)

Figure 1-14 Dermatofibroma. A classic central white patch (black arrow) Figure 1-16 Hemangioma. Well-demarcated dark red lacunae (arrows)
and pigment network (black boxes) characterize this dermatofibroma. In this and blue-white color (stars) representing fibrous septa characterize this classic
instance, ring-like structures (white arrows) make up the pigment network. hemangioma.
Chapter 1 Dermoscopy from A to Z 11

■ Melanoma
■ In situ

■ Superficial spreading

■ Nodular

■ Amelanotic

■ Lentigo maligna/Lentigo maligna melanoma

■ Nail apparatus

■ Acral

■ Mucosal

■ Even though pattern analysis is considered a


melanocytic algorithm, the same principles are used to
diagnose all of the lesions that can be identified with
the technique
■ Melanocytic

■ Nonmelanocytic

■ Benign
Figure 1-17 Cutaneous Metastatic Melanoma. This is one of many ■ Malignant
generalized cutaneous metastatic lesions in a 27-year-old white male with a
■ Inflammatory
history of a 7mm melanoma on his back. There are well-demarcated
lacunae-like areas (arrows) and atypical vessels (boxes). A collision lesion,
hemangioma, and amelanotic melanoma are in the dermoscopic differential Pattern Analysis Method
diagnosis.
Step 1
■ Determine symmetry or asymmetry of color and/or
■ Different shades of red and blue colors can be seen in a structure using the mirror image technique
single hemangioma ■ Contour of the lesion is not important with this
■ Lacunae should not be mistaken for the milky-red color algorithm
seen in pigmented and amelanotic melanoma, which ■ The lesion is bisected by two lines that are placed 90° to
can have “out-of-focus” reddish globular-like structures each other
■ Black homogeneous structureless areas represent ■ One does not physically place lines over the lesion.
thrombosis The lines are visually imagined
■ Significant scale or dryness (hyperkeratosis) can be seen ■ The lines should be placed to create as much symmetry
in angiokeratomas as possible
■ Patchy white color or bluish-white color (fibrous septa) ■ Are the color and/or the structure on the left half of the
are commonly seen in hemangiomas lesion a mirror image of the right half
■ It should not be mistaken for the scar-like white color ■ Repeat the analysis for the upper and lower half of the lesion
of regression or the blue color that can be found in ■ Perfect symmetry of color and/or structure is not often
melanomas found in nature, and interobserver agreement may not be
■ Cutaneous metastatic melanoma and hemagioma-like good with this assessment even among experienced
melanoma can be indistinguishable from a hemangioma dermoscopists
■ A history of a previous melanoma will help make the ■ Symmetry or asymmetry can also be determined along
diagnosis (Figure 1-17) any axis through the center of the lesion (Menzies
method)
Step Two: Analysis of a Melanocytic Lesion ■ Significant asymmetry of color and/or structure is a very
Pattern Analysis Defined important clue that you might be dealing with high risk
■ Identify as many criteria in the lesion as possible and see pathology
if they fit into the known patterns associated with the ■ Raise a red flag for concern and proceed with focused
variants of attention to what else you might find
■ Melanocytic Nevi Step 2
■ Congenital ■ Determine the global/overall pattern of the lesion
■ Acquired ■ The predominant criteria seen throughout the lesion
■ Recurrent could be
■ Halo ■ Reticular

■ Combined ■ Globular

■ Blue ■ Cobblestone

■ Dysplastic ■ Homogeneous

■ Spitz ■ Parallel
12 DERMOSCOPY: AN ILLUSTRATED SELF-ASSESSMENT GUIDE

■ Starburst ■ Due to the different characteristics of the skin in these


■ Multicomponent locations the criteria are different on
■ Nonspecific ■ Trunk and extremities

■ There can be combinations of criteria in a single lesion ■ Face, nose, and ears

such as reticular and homogeneous or reticular and ■ Palms, soles, nail apparatus, and mucosa

globular ■ Thinner skin on the face, nose, ears versus the trunk and
■ The “reticular homogeneous pattern” or “reticular extremities, and thicker skin on the palms and soles with
globular pattern” fissures and ridges
Step 3 ■ The criteria found on the face, nose, ears, palms, soles, nails
■ Identify the local criteria in the lesion and mucosa are referred to as site-specific criteria (Table 1-6)
■ Pigment network

■ Dots and globules Global Patterns


■ Streaks (also called pseudopods and radial streaming) Reticular
■ Blotches ■ Pigment network filling most of the lesion
■ Blue-white veil Globular
■ Regression ■ Dots and globules filling most of the lesion
■ Colors
Cobblestone
■ Vascular structures
■ Larger angulated globules resembling street cobblestones
Step 4 filling most of the lesion (Figure 1-18)
■ Determine if the criteria seen are Homogeneous
■ Regular or irregular
■ Diffuse pigmentation in the absence of local criteria such
■ Good or bad
as pigment network, dots, and globules
■ Low or high risk
Starburst (Spitzoid)
■ Melanoma-specific criteria are defined as criteria that can
■ Streaks and/or dots and globules at the periphery of the
be seen in benign and malignant lesions but are more
lesion
specific for high risk pathology such as
■ Dysplastic nevi Multicomponent
■ Spitzoid lesions ■ Three or more different areas within a lesion
■ Melanoma ■ Each zone can be composed of a single criterion or
■ All of the high risk criteria can be seen in benign multiple criteria
pathology, and one should never tell patients that they Nonspecific
have melanoma 100% ■ None of the above global patterns can be identified

Table 1-6 MELANOMA-SPECIFIC CRITERIA IN DIFFERENT BODY REGIONS


TRUNK AND EXTREMITITES FACE, NOSE, AND EARS PALM AND SOLES NAIL APPARATUS

GLOBAL CRITERIA Asymmetrical pigmentation Parallel ridge pattern Diffuse irregular pigmentation
Asymmetry of color and/ around follicular openings Diffuse variegate Irregular pigmented
or structure Annular- granular structures pigmentation bands/melanonychia striata
Multicomponent pattern Rhomboid structures Multicomponent Loss of parallelism of pigmented
Nonspecific pattern Circle within a circle pattern bands
LOCAL CRITERIA Dark homogeneous areas Nonspecific pattern Hutchinson and micro-Hutchinson
Irregular pigment network Absence of the fingerprint sign
Irregular dots and globules pattern
Irregular streaks (pseudopods/
radial streaming)
Irregular blotches
Blue-white color
Regression
5 or 6 colors
Polymorphous vascular pattern
Chapter 1 Dermoscopy from A to Z 13

Regular Streaks
■ Black or brown linear projections of pigment
■ Can be free standing or associated with a pigment
network or dark blotches
■ At all points along the periphery of the lesion
■ Pseudopods and radial streaming are similar structures
dermoscopically and histopathologically (aggregates of
tumor cells running parallel to the epidermis that can be
seen in Spitz nevi or represent the radial growth phase of
melanoma), which are difficult to differentiate from each
another
■ To simplify the identification, the term “streaks” is now
used by many but not all experienced dermoscopists to
encompass all variations of this criterion
■ The shape of the linear projections does not determine if
they are regular or irregular, rather their distribution at
Figure 1-18 Acquired Nevus. Small dots and globules (boxes) and larger the periphery of the lesion
angulated globules (arrows) characterize this benign nevus. The mountain
and valley pattern seen in seborrheic keratosis is in the dermoscopic differ- Irregular Streaks
ential diagnosis. A positive wobble sign in which the soft nevus moves from ■ Black or brown linear projections
side to side with movement of instrumentation versus a stiff immoveable ■ Irregularly distributed at the periphery of a lesion
seborrheic keratosis helps to make the differentiation. ■ Some but not all points at the periphery, “foci of streaks”
Regular Blotches
■ Dark shades of black, brown or gray
Local Criteria ■ Structureless (ie, absence of network, dots, or globules)
Regular Pigment Network areas of color
■ Various shades of brown ■ Bigger than dots and globules
■ Honeycomb-like (web-like, reticular) line segments ■ Uniform shape and color symmetrically located in the lesion
(elongated hyperpigmented rete ridges) (aggregates of melanin in the epidermis and/or dermis)
■ Uniform color, thickness, and holes (dermal papilla) Irregular Blotches
Irregular Pigment Network ■ Dark shades of black, brown or gray structureless areas of
■ Shades of black or brown color
■ Line segments that are thickened, branched, and broken ■ Irregular in size and shape asymmetrically located in the
up (enlarged, fused rete ridges) lesion
■ There may be a diffuse distribution or foci of irregular Blue-White Veil
pigment network ■ Irregular, structureless area of confluent blue color
Regular Dots and Globules ■ Overlying whitish ground glass appearance
■ Brown roundish structures ■ Orthokeratosis

■ Usually clustered ■ Acanthosis

■ Dots (0.1mm) are smaller than globules (greater than ■ Hypergranulosis

0.1mm) ■ Can represent heavily pigmented tumor cells in the

■ Size, shape, and color are similar with an even distribution dermis
in the lesion (nest of melanocytes at the dermoepidermo Regression
junction) ■ Bony or milky-white scar-like depigmentation (fibrosis)
■ Dots and/or globules only found at the periphery can be ■ With or without gray pepper-like granules (free melanin
seen in dysplastic, Spitz, or actively changing nevi and/or melanophages in the dermis)
■ Actively changing means if followed digitally the nevus ■ The white color should be lighter than the surrounding skin
will invariably enlarge over time ■ The bony-white color should be differentiated from tan
■ Peripheral dots and globules are usually seen in younger hypopigmentation commonly found in melanocytic
patients with benign pathology lesions
■ Beware of this pattern in an adult ■ Blue color can also be seen in areas of regression

Irregular Dots and Globules ■ Regression by itself is an independently potentially high


■ Black, brown, gray, or red roundish structures risk criterion
■ Different sizes, shapes, and shades of color ■ The more the regression seen, the greater the chance the
■ Usually but not always asymmetrically located in the lesion lesion is a melanoma
14 DERMOSCOPY: AN ILLUSTRATED SELF-ASSESSMENT GUIDE

Blue-White Color
■ It is not always possible to identify classic regression or
blue-white veil
■ Blue and/or white colors of any intensity, shape or distribution
■ A red flag for concern should be raised that the lesion
could be a melanoma
1 2 3
Hypopigmentation
■ Commonly seen featureless areas of light brown color in all
types of melanocytic lesions both benign and malignant
■ Inexperienced dermoscopists can have trouble differenti-
ating hypopigmentation from the bony-white color seen
with true regression
Colors Seen with Dermoscopy
■ Eumelanin’s location in the skin will determine the colors
one sees with dermoscopy
■ In a flat or slightly raised lesion, black indicates that
Figure 1-20 Melanoma. This is a melanocytic lesion because there are
melanin is superficially located in the epidermis globules (circle). There is asymmetry of color and structure (+) plus the
■ Black is not always an ominous color but can be seen in multicomponent global pattern (1,2,3). Local criteria include: irregular dots
benign pathology as well as in invasive melanoma and globules (circle), blue-white color (stars), and peppering (boxes). The
■ Light and dark brown indicates pigment is at the dermo- classic blue-white veil is not seen. More than five colors including red are
another melanoma-specific criterion. (Reproduced, with permission, from
epidermal junction
Journal of Drugs in Dermatology. New Methods and Technologies. Sep 2007- Vol
■ Gray in the papillary dermis represents free melanin 6-Issue 9. Fig 3b.)
and/or melanophages (“peppering”)
■ As the pigment gets into the deeper dermis it looks blue
(the Tyndall effect)
Atypical Vascular Pattern/Polymorphous Vascular Pattern
■ Red and/or pink colors can be created by inflammation or
■ Vessels that can be seen in melanoma are nonspecific;
neovascularization
they can also be commonly found in other lesions,
■ Sebaceous material and hyperkeratosis can look yellow
including
■ The more colors seen, the greater the chance that one is
■ Benign
dealing with high risk pathology (Figures 1-19, 1-20, 1-21)
■ Malignant

■ Inflammatory

1
4

Figure 1-19 Melanoma. This is a melanocytic lesion because there is a pig-


ment network (red arrows) and globules (circles). There is asymmetry of color
and structure (+) plus the multicomponent global pattern (1,2,3,4). Local cri- Figure 1-21 Melanoma. This is a melanocytic lesion because there are
teria include: irregular pigment network (red arrows), irregular dots and glob- globules (circles). There is an atypical starburst (Spitzoid) global pattern with
ules (circles), irregular dark blotches (black arrows), and blue-white color foci of streaks at the periphery (boxes). Local criteria include: irregular dots
(stars). The classic blue-white veil is not seen. Peppering (yellow box) and gray and globules (circles), irregular streaks (boxes), and regression. The white
blotches (yellow arrows) are part of the regression. More than five colors are and gray colors (yellow arrows) make up the regression. The black arrows
seen including red. (Reproduced, with permission, from Journal of Drugs in point out where there are no streaks. Five colors including red round off the
Dermatology. New Methods and Technologies. May 2008-Vol 7-Issue 5. Fig 4b.) melanoma-specific criteria.
Chapter 1 Dermoscopy from A to Z 15

■ When identified, they should raise a red flag for concern ■ The color does not completely encircle the openings
including (early proliferation of atypical melanocytes)
■ Dotted/pinpoint ■ A melanoma-specific criterion, used to diagnose lentigo
■ Irregular linear maligna or lentigo maligna melanoma
■ Irregular torturous/corkscrew (irregular, thick, coiled) Annular-Granular Pattern/Structures
■ Irregular hairpin (irregular and thick hairpin-shaped) ■ Seen only on the face, nose, and ears (“site-specific”)
■ Glomerular ■ Brown or gray fine dots that surround follicular openings
■ One must focus their attention to make out the shapes of (melanophages and/or atypical melanocytes)
these small vessels ■ This criterion and asymmetrical follicular pigmentation
can be seen in
■ Lentigo maligna, lentigo maligna melanoma
PEARL
■ Pigmented actinic keratosis
■ Minimal pressure with polarizing instrumentation and the ■ Posttraumatic
use of ultrasound gel, hand cleaning gel or other fluid is ■ Late-stage lichen planus-like keratosis (Figures 1-23, 1-24)
the best technique to visualize small vessels ■ Melasma
Milky-Red Areas Rhomboid Structures
■ Localized or diffuse (amelanotic melanoma) pinkish-white ■ Seen only on the face, nose, and ears (“site-specific”)
color ■ Rhomboid is a parallelogram with unequal angles and sides
■ With or without reddish and/or bluish out-of-focus/ ■ Black, brown, or gray thickening around the follicular
fuzzy globular structures (neovasculariztion) openings
■ Not to be confused with the in focus lacunae seen in ■ In reality true rhomboids are not regularly formed
hemangiomas ■ Any pigmented thickening around follicular openings is
Glomerular Vessels worrisome
■ Diffuse or clustered fine coiled vessels that can be seen in ■ A melanoma-specific criterion used to diagnose lentigo
■ Bowen disease (Figure 1-22) maligna or lentigo maligna melanoma
■ Melanoma Dark Homogeneous Areas
■ Pink lichen planus-like keratosis ■ On the face, nose, and ears
■ Stasis and nonspecific dermatitis ■ Irregular in size and shape
■ Psoriasis ■ Black or brown homogeneous blotches of color
Asymmetrical Pigmentation Around Follicular Openings ■ Complete occlusion and disappearance of follicular open-
■ Seen only on the face, nose, and ears (“site-specific”) ings due to invasive melanoma (lentigo maligna melanoma)
■ Irregular brown color outlining parts of the round
follicular openings

Figure 1-23 Lichen Planus-like Keratosis. Remnants of a fingerprint


pattern (yellow boxes) of a flat seborrheic keratosis or solar lentigo and the
gray annular-granular pattern (black boxes) around follicular openings
Figure 1-22 Bowen Disease. Typical glomerular vessels (black box) and (arrows) are the clues that this is not lentigo maligna. The gray dots repre-
large dotted vessels (yellow box) help diagnose this nonspecific pink scaly sent melanophages and free melanin in the papillary dermis, not atypical
patch. (Reproduced, with permission, from Journal of Drugs in Dermatology. melanocytes. A sub-set of lichen planus-like keratosis is thought to represent
New Methods and Technologies. May 2008-Vol 7-Issue 5. Fig 2b.) an immunologic event against flat seborrheic keratosis of solar lentigines.
16 DERMOSCOPY: AN ILLUSTRATED SELF-ASSESSMENT GUIDE

Figure 1-24 Lentigo Maligna (ear lobe). This case demonstrates varia- Figure 1-25 Acrolentiginous/Acral Melanoma/Nail-Apparatus
tions of the classic criteria. The lesion is suspicious clinically but has a differ- Melanoma. The pigmented bands are not uniform in color and thickness
ential diagnosis that includes a seborrheic keratosis. The dermoscopic criteria (black arrows). Broken up line segments (box) represent loss of parallelism.
for a seborrheic keratosis are not present. There is asymmetry of color and There is also Hutchinson sign (yellow arrows).
structure, asymmetrical pigmentation (black arrows) around follicular open-
ings (red arrows), annular-granular structures (circles), and irregular dark
blotches (boxes). One should have a mental checklist of the melanoma-specific
criteria for this diagnosis because they are not always easy to find and identify.
Micro-Hutchinson Sign: Hutchinson Sign
■ Pigmentation of the cuticle that can only be seen clearly
with dermoscopy (micro-Hutchinson sign)
■ Pigmentation of the cuticle easily seen without dermoscopy
PEARLS (Hutchinson sign)
■ Actinic keratosis and actinic/solar lentigines can be asso- ■ Both are often associated with nail-apparatus melanoma
ciated with lentigo maligna/lentigo maligna melanoma yet not diagnostic (pseudo-Hutchinson sign)
■ Use the areas where the high risk/melanoma-specific cri- Nonmelanocytic Nail-Apparatus Bands
teria are to perform an incisional biopsy, or the malignant ■ Uniform grayish bands can be seen with
diagnosis could be missed ■ Lentigo
■ If you think the lesion is lentigo maligna yet the pathology ■ Post-inflammatory
report does not make the diagnosis, seek another ■ Post-traumatic
histopathologic opinion or biopsy another area of the lesion ■ Drug-induced pigmentation multiple nails involved
Benign Pigmented Nail Bands (Melanonychia Striata) ■ Darker skinned races multiple nails involved
■ Single or multiple nail involvement with brown longitudinal Nail-Apparatus Blood/Subungual Hematoma
parallel lines ■ The color of blood seen in the nail apparatus depends on
■ Uniform color, spacing, and thickness how long the blood has been there (breakdown of heme)
■ A single band in a lighter skinned person with these findings ■ Fresh blood looks red or purple/violaceous
is still worrisome ■ Older blood can look yellowish brown or black
Malignant Pigmented Nail Bands (Atypical Melanonychia Striata) ■ A well-demarcated homogeneous area with parallel lines
■ Loss of parallelism (broken-up line segments) with at the distal edge and/or globule-like blood spots/pebbles
brown, black, or gray parallel lines that demonstrate (Figure 1-26)
different shades of color, irregular spacing, and thickness ■ Digital dermoscopy is helpful to follow nail-apparatus
(Figure 1-25) blood that should slowly move distally over several months
■ High risk dermoscopic criteria at this location in adults
are usually not associated with high risk pathology when
seen in children
PEARLS
■ Disfiguring nail-matrix biopsies can usually be avoided ■ Presence of blood in a nail does not rule out melanoma
in children ■ Search carefully for high risk criteria that might also be
■ Any rapidly changing scenario warrants a histopathologic present
diagnosis no matter how old or young the patient ■ Finding the Hutchinson sign and the parallel ridge pat-
■ Digital monitoring is helpful to monitor pigmentation in the tern on the surrounding skin adjacent to the nail can help
nail apparatus to document stability or change over time make the diagnosis of nail apparatus melanoma
Chapter 1 Dermoscopy from A to Z 17

Figure 1-26 Sub-ungual Hemantoma. Different colors plus blood peb- Figure 1-27 Congenital Melanocytic Nevus. Terminal hairs with per-
bles (boxes) characterize this post-traumatic lesion. The white color (star) is ifollicular hypopigmentation (boxes), irregular pigment network (circles), and
secondary to trauma not regression. The brown (red arrows) and purple regular dots (arrows) characterize this small banal congenital melanocytic
blotches (white arrows) result from the breakdown of blood/heme. No nevus.
melanoma-specific criteria are seen.

Longitudinal Erythronychia ■ Symmetry of color and structure


■ An inconsistent finding in single or multiple finger nails ■ Comma-shaped blood vessels
■ Single or multiple longitudinal pinkish blanching bands ■ Hypopigmentation
of variable thickness (normal vasculature) ■ Milia-like cysts and pseudofollicular openings can be seen
■ Multiple nail involvement is a benign sign ■ Pink nevi can be featureless (an absence of local criteria)
■ Single nail involvement can be seen in amelanotic nail- or feature-poor (a few poorly developed local criteria)
apparatus melanoma, Bowen disease and onychomatricoma ■ A solitary flat pink lesion, with different shades of
■ Associated with inflammatory skin diseases (ie, lichen pink color, is more worrisome than multiple soft and
planus, Darier’s disease) compressible pink lesions
Common Dermoscopic Patterns PEARL
Congenital Nevi
■ Dermoscopy might not be helpful to diagnose pink macules
■ Diffuse homogeneous brown color
and papules, which can be melanocytic, nonmelanocytic,
■ Patchy or diffuse pigment network (target network may
benign, malignant, or inflammatory (Figure 1-28)
or may not be seen as network holes each with a small
centrally located brown dot or pinpoint vessel) Blue Nevi
■ Globular and/or cobblestone pattern (target globules may ■ Blue, blue-gray, or blue-black homogeneous color
or may not be seen as globules with a smaller centrally (Figure 1-3)
located brown dot or vessel) ■ Variable number of subtle blue globular-like structures
■ Islands of normal skin and islands of criteria such as may or may not be present
pigment network, dots and globules ■ Regression with white or gray areas commonly seen
■ Multicomponent pattern with three or more distinct areas ■ Radiation tattoo, nodular and cutaneous metastatic
of criteria melanoma are in the dermoscopic differential diagnosis
■ Dark coarse terminal hairs (hypertrichosis) with or with- ■ The history is essential to make the diagnosis
out surrounding hypopigmentation (perifollicular Combined Nevi
hypopigmentation) (Figure 1-27) ■ Light or dark brown homogeneous color +/− other local
■ Milia-like cysts and pseudofollicular openings, most often criteria (regular nevus) and a blue blotch (blue nevus) with
found in seborrhic keratosis, can be seen a “fried egg” clinical appearance
Acquired Nevi ■ Diffuse brown homogeneous color with a blue border
■ Light/dark brown or pink color ■ Diffuse blue homogeneous color with brown border
■ Regular pigment network ■ Variable combinations of blue and brown colors
■ Lacks sharp demarcation at the borders Recurrent Nevi/Pseudomelanoma
■ Globular or cobblestone pattern (the most common pattern ■ Sharp border
seen in children) ■ Irregular pigment network; irregular streaks
18 DERMOSCOPY: AN ILLUSTRATED SELF-ASSESSMENT GUIDE

Figure 1-28 Pink Lichen Planus-like Keratosis. This small papule was Figure 1-30 Dysplastic Nevus. There are foci of irregular dots and glob-
only found after a complete skin examination. There are different shades of ules (boxes), irregular dark blotches (black arrows), and multifocal hypopig-
pink color, pinpoint (boxes), and comma-shaped vessels (yellow arrows) mentation (red arrows).
plus a milky-red area (black arrow). (Reproduced, with permission, from
Journal of Drugs in Dermatology. New Methods and Technologies. Sep 2007- Vol
6-Issue 9. Fig 4b.)
■ E for evolving/changing might be the only clue that a
■ Irregular dots and globules lesion is high risk (ie, dysplastic nevus, melanoma)
■ White scar-like areas with linear and/or arborizing vessels ■ Asymmetry of color and structure
■ Any combination of criteria can be seen ■ A multicomponent global pattern
■ Pigmentation centrally located within the scar ■ Irregular pigment network
■ If the pigmentation goes beyond the borders of the scar, ■ Irregular dots and globules
raise a red flag for concern that the lesion might be a ■ Irregular blotches
melanoma ■ Multifocal hypopigmentation (Figure 1-30)
■ A history of previous surgery and histopathology is ■ Regression, blue-white color/blue-white veil, atypical vessels,
important to help rule out melanoma and streaks are not usually seen
■ If possible, review the pathology from the lesion that ■ Atypical pattern similar to superficial spreading melanoma
was excised (Figure 1-29) ■ Patients with multiple dysplastic nevi clinically, usually do
Dysplastic Nevi not have many that look very atypical with dermoscopy
■ ABCDE clinical lesions can be indistinguishable from ■ Pink dysplastic nevi can be feature-poor or featureless
melanoma and look banal or high risk with dermoscopy with low or high-grade histopathology

PEARLS
■ “Anything pink, stop and think!”
■ Look for the clinical and/or dermoscopic “ugly duckling”
to consider for biopsy or digital follow-up
Spitz Nevi
1
■ There are six patterns seen in Spitz nevi
3 ■ Starburst

■ Globular

■ Homogeneous
2
■ Pink

■ Black pigment network

■ Atypical

■ Spitzoid is the term used when any of the different six


patterns is seen
■ Starburst is the most common pattern (Figure 1-31)
■ Streaks and/or dots and globules at the periphery
Figure 1-29 Recurrent Nevus. Asymmetry of color and structure (+), the
■ Light/dark brown, black, or blue color centrally
multicomponent global pattern (1,2,3) irregular globules (boxes), irregular
dark blotch (yellow arrow), brown color (white arrows), scar tissue (stars) ■ Regular or irregular pattern depends on the location of

with linear vessels (black arrows) characterize this recurrent nevus. the streaks
Chapter 1 Dermoscopy from A to Z 19

■ A Spitzoid melanoma histopathologically does not always


have a Spitzoid dermoscopic features
In Situ Melanoma (Trunk and Extremities)
■ Can be associated with a dysplastic nevus or develop de novo
■ May or may not demonstrate the clinical ABCD clinical
criteria
■ Flat or slightly raised lesion
■ Asymmetry of color and structure
■ Multicomponent global pattern
■ Black and/or dark brown irregular pigment network
■ Irregular dots and globules
■ Irregular dark blotches
■ Regression and/or hypopigmentation
■ Lacks the criteria associated with deeper melanomas (pink,
red, or blue color, atypical vessels) (Figure 1-32)
Lentigo Maligna/Lentigo Maligna Melanoma (Face, Nose, Ears)
Figure 1-31 Spitz Nevus. A central regular dark blotch (stars) plus glob-
ules (black arrows) and a few streaks (red arrows) at all points along the ■ Asymmetrical follicular pigmentation
periphery characterize this classic symmetrical starburst/Spitzoid pattern. ■ Annular-granular structures
■ Rhomboid structures
■ Circle within a circle
■ Regular starburst pattern has symmetrical streaks ■ Gray color and/or gray granules (atypical melanocytes)
and/or dots and globules around the lesion surrounding the periphery of follicular units with the
■ Irregular starburst pattern has foci of streaks and/or
presence of a centrally located hair shaft
dots and globules at the periphery ■ Absence of the fingerprint pattern
■ Beware! Symmetrical and asymmetrical starburst patterns
■ With invasion (lentigo maligna melanoma) one sees
can be seen in melanoma irregular black blotches with or without the disappearance/
■ Globular is the second most common Spitzoid pattern destruction of follicular openings
■ Filled with regular or irregular dots and/or globules
■ Melanoma-specific criteria as seen in other locations with
■ Blue color seen centrally is a clue that the lesion might
more advanced disease
be a Spitz nevus
Superficial Spreading Melanoma
■ Homogeneous pattern
■ Starts in an existing nevus or de novo
■ Featureless brown color
■ Demonstrates the clinical ABCD criteria
■ Pink pattern
■ Featureless pink papule

■ Can have small vessels

■ Black network pattern


■ The lesion is composed totally of a prominent black

pigment network
■ Ink-spot lentigo and melanoma are in the differential

diagnosis
■ Atypical pattern 3
■ This can have any combination of melanoma-specific
1 2
criteria
■ The histopathologic diagnosis is usually a surprise
3
■ White pigment network/white network/negative pigment
network/reticular depigmentation can be seen within the
lesion
■ This is an important clue that the lesion is Spitzoid

■ Can also be seen in dermatofibromas and in melanoma

Figure 1-32 In Situ Melanoma. This is a melanocytic lesion because there is


PEARLS a pigment network (black box) and globules (circles). There is asymmetry of
■ Most but not all Spitzoid patterns require a histopathologic color and structures (+), the multicomponent global pattern (1,2,3), irregular
diagnosis especially in adults pigment network (black box), irregular dots and globules (circles), and reticu-
lar depigmentation (white box). The hypopigmentation (black stars) should
■ The pink variant could actually be amelanotic melanoma not be confused with regression. There is diffuse erythema (red stars) and only
■ A melanoma with a Spitzoid pattern dermoscopically is three other colors. (Reproduced, with permission, from Journal of Drugs in
not always Spitzoid histopathologically Dermatology. New Methods and Technologies. May 2008-Vol 7-Issue 5. Fig 3b.)
20 DERMOSCOPY: AN ILLUSTRATED SELF-ASSESSMENT GUIDE

■ Contains a variable number of the melanoma-specific


criteria found on the trunk and extremities (Figures 1-19,
1-20, 1-21)
Nodular Melanoma
■ Starts in an existing nevus or de novo
■ May or may not be fast growing
■ Pigmented, hypomelanotic, or amelanotic
■ Can be mistaken for benign nevus or squamous cell
carcinoma
■ Usually lacks the clinical ABCD criteria
■ Due to the absence of the radial growth phase there is a
scarcity of local criteria (pigment network, globules, streaks)
■ Remnants of local criteria may or may not be present at
the periphery of the lesion
■ Large intense irregular dark blotches
■ Multiple deeper skin colors seen such as blue, white, pink,
Figure 1-33 Amelanotic Melanoma (face). This is a melanocytic lesion
milky-red
because it has globules (boxes). Histopathologically it was an amelanotic
■ Atypical vessels melanoma even though brown color is seen with dermoscopy. There is an
absence of melanoma-specific criteria found on the face with different shades
PEARLS of pink and brown color plus ulceration (yellow arrows). Follicular openings
(black arrows) should not be confused with the milia-like cysts of a seborrheic
■ The clinical appearance of a lesion (flat, palpable or nodular, keratosis.
presence or absence of the ABCD criteria) plus the colors
and structures seen with dermoscopy can help estimate if
you are dealing with a thin, intermediate or thick melanoma Featureless Melanoma
■ Flat melanomas are usually in situ or early invasive,with ■ Melanoma without dermoscopic criteria at all
black and brown colors plus well-developed local criteria Melanoma Incognito/False Negative Melanoma
■ Thick melanomas tend to be elevated or nodular and can ■ Clinically and/or dermoscopically the lesion does not look
have a paucity or absence of local criteria such as pigment like melanoma
network, plus blue-white veil/color, multiple colors, and ■ With dermoscopy there can be obvious or subtle clues to
the atypical vascular pattern make the diagnosis
Amelanotic Melanoma ■ Clues to help make the diagnosis
■ Flat, palpable, or nodular ■ History of dermoscopic change over time
■ Hypopigmented, pink or red ■ A Spitzoid pattern in a lesion that does not look Spitzoid
■ May or may not have the melanoma-specific criteria clinically
typically seen in pigmented melanomas ■ Areas of regression as the major high risk criterion
■ Different shades of pink color and polymorphous ■ Polymorphous vessels in a pink lesion
vascular pattern ■ The “little red riding hood sign” is when the lesion looks
■ Milky-red areas are important clues to the correct diagnosis clinically benign from a distance but not when seen close
■ Pediatric patients have a high proportion of amelanotic up with dermoscopy
melanomas
■ Amelanotic melanoma should always be in the differential PEARL
diagnosis of a pyogenic granuloma (Figure 1-33) ■ Dermoscopy should not only be used on clinically suspi-
Cutaneous Metastatic Melanoma cious lesions if one wants to diagnose melanoma incognito
■ Dermoscopy might not be as helpful to make the diagno- Nail-Apparatus Melanoma
sis as the history of a melanoma being previously excised ■ Amelanotic diffuse reddish color/amelanotic tumor nodule
■ Single or multiple ■ Longitudinal erythronychia in a single nail
■ Pigmented or nonpigmented ■ Diffuse melanonychia with different shades of black,
■ All different sizes, shapes, and colors can be seen in each brown, or gray color
patient with or without atypical vessels ■ Irregular pigmented bands (Figure 1-25)
■ Any combination of criteria can be seen ■ A single uniform band does not rule out melanoma
■ Benign patterns such as a hemangioma-like cutaneous ■ Irregular dots and globules
metastatic melanoma (Figure 1-17) ■ Blood identified in 25% of lesions
Feature-poor Melanoma ■ Variable amounts of nail-plate destruction with advanced
■ Melanoma with subtle nondiagnostic criteria (Figure 1-33) disease
Chapter 1 Dermoscopy from A to Z 21

■ +/− Hutchinson sign PEARL


■ The parallel ridge pattern can be seen on the adjacent skin
■ The yellowish clinical appearance and globules are the
Ink-spot Lentigo main features used to differentiate sebaceous gland
■ Black macule or macules on sun-exposed areas hyperplasia from basal cell carcinoma.
■ Prominent thick black pigment network
Collision Tumor
■ Usually a very easy clinical and dermoscopic diagnosis
■ Lesion with the dermoscopic criteria for two different
■ Melanoma could be in the clinical and dermoscopic
pathologies
differential diagnosis
■ One can find a triple collision lesion with three different
■ Look for melanoma-specific criteria (ie, blue-white
pathologies
color, irregular blotches) that should not be present in
■ Collision tumors are commonly seen
an ink-spot lentigo
■ Diagnostic criteria can be side by side or one can be seen
Solar Lentigo/Actinic Lentigo within the other
■ Macules and/or patches ■ Examples include
■ Different shades of homogeneous brown color ■ Seborrheic keratosis, basal cell carcinoma
■ Sharply demarcated moth-eaten concave borders ■ Seborrheic keratosis, in situ or invasive squamous cell
■ Fingerprint pattern with straight and/or wavy linear line carcinoma
segments ■ Seborrheic keratosis, amelanotic or pigmented
Actinic Keratosis melanoma
■ Nonpigmented actinic keratosis ■ Seborrheic keratosis, eccrine porocarcinoma
■ Scaly surface ■ Basal cell carcinoma, seborrheic keratosis, clear cell
■ Pinkish-red pseudopigment network acanthoma
■ Pigmented actinic keratosis ■ Any combination is possible (Figure 1-34)
■ Mimicks lentigo maligna

■ Asymmetrical follicular pigmentation

■ Annular-granular structures
Other Diagnoses Made with Dermoscopy
■ Rhomboid structures Scabies
■ Burrows appear as discrete yellowish scaly linear areas

■ Mites can be seen as a small triangle/gray delta structure


PEARL that corresponds to the front section of the body with its
■ Multiple scaly lesions favor the diagnosis of actinic mouth/biting apparatus and legs
keratosis over lentigo maligna. Both can have ■ Higher magnification and oil/fluid increases the visibility
pigmented and nonpigmented variants. of the mite, stool, and eggs
Bowen Disease (In Situ Squamous Cell Carcinoma)
■ Pink or reddish scaly macules, papules, patches, plaques
■ Pinpoint and/or glomerular vessels
■ Clusters or diffuse distribution of vessels throughout the
lesion
■ With or without foci of pigmentation
Keratoacanthoma
■ A rapidly growing reddish scaly nodule of short duration
on sun exposed areas
■ Centrally located yellowish keratinous material
■ Peripheral whitish background
■ Hairpin vessels with or/without a white halo at the
periphery
Sebaceous Gland Hyperplasia
■ Delled yellow papules seen clinically
■ Multiple grouped white or yellow globules
■ Small caliber basal cell carcinoma-like vessels
■ The vessels have been termed crown or wreath-like vessels
■ Supposedly never to reach the center of the lesion
Figure 1-34 Collison Tumor—Squamous Cell Carcinoma and
Seborrheic Keratosis. A rapidly growing nodule (arrows) representing a
■ This is a misnomer because in reality the vessels
squamous cell carcinoma and the mountain and valley pattern of a sebor-
rarely meet this criterion and can be found anywhere rheic keratosis (box) characterizes this lesion. The cobblestone pattern of a
in the lesion nevus is in the dermoscopic differential diagnosis.
22 DERMOSCOPY: AN ILLUSTRATED SELF-ASSESSMENT GUIDE

Pediculosis Capitis Scleroderma Pattern


■ Direct visualization of the parasite and nits ■ The triad of
■ It is possible to see if the nits are full (vital nits) or empty, ■ Rarefied capillaries (less than 6 loops per mm)

which helps determine the success or failure of treatment ■ Thin loops

■ Megacapillaries
Pediculosis Pubis ■ Pearly shining sclerosis “cotton balls”
■ It is possible to easily see the parasite attached to adjacent

pubic hairs or hairs at other sites Dermatomyositis


■ Mega, twisted, branched loops, microhemorrhage

Lichen Planus
■ Peppering Lupus Erythematosus
■ Brown blotches ■ Considerable variation of loops, branching, twisted,

■ White reticular areas (Wickham striae) megacapillaries, microhemorrhage


■ Negative pigment network/white pigment network/reticular

depigmentation is in the dermoscopic differential diagnosis Trichoscopy


■ The use of dermoscopy to evaluate scalp skin and hair
of Wickham striae
follicles
■ Structures that can be visualized include
Warts
■ Hair shafts
■ Red and/or black dots (thrombosed capillaries)
■ Hair-follicle openings
■ With or without a white halo
■ Perifollicular epidermis

Psoriasis ■ Cutaneous microvasculature

■ Red scaly plaque or plaques ■ Higher magnifications (20-70 fold) with digital systems

■ Diffuse distribution of pinpoint and/or glomerular vessels and fluid (70% ethanol) are preferred
identical to Bowen disease ■ Hand-held instrumentation with lower magnification and

■ Distribution of lesions will help differentiate Psoriasis other fluids such as emersion oil or gels can also be used
from Bowen disease
■ Both can have single or multiple lesions
Genetic Hair-Shaft Abnormalities
Monilethrix
Nail Folds ■ Multiple constrictions of the hair shaft alternating with
■ Normal capillary loops are hairpin-shaped and run parallel elliptical nodosities that look like a “pearl necklace”
to the axis of the nail ■ A high tendency to fracture, which gives hair a stubble-
■ The main value of nail fold dermoscopy is the early like appearance
diagnosis of scleroderma before there are positive ■ Hair shafts bend regularly in multiple places and curving
clinical and serologic findings (Figure 1-35) in different directions “regularly bended ribbon sign”

Figure 1-35 Nail Fold Capillaries in Cutaneous Lupus. Uniform hairpin-shaped capillary loops are replaced with
large (mega) irregularly-shaped capillary loops that still have a hairpin shape. The surrounding skin is atrophic with diffuse
telangietatic vessels.
Chapter 1 Dermoscopy from A to Z 23

Netherton Syndrome ■ Terminal to vellus hair ratio can be calculated without


■ Trichorrhexis invaginata/bamboo hair/golf tee type skin biopsies
characterized by invagination of the distal portion of the ■ Increased percentage of thin hairs
hair shaft into its proximal portion forming a ball-in-cup ■ Decreased average hair diameter
appearance ■ Predominance of hair-follicle units with single hairs
■ Diagnosis can easily be made without the need for hair ■ Hyperkeratotic plugs
sampling and microscopic examination ■ Perifollicular pigmentation
Pili Annulati ALOPECIA AREATA
■ Alternating light and dark bands are seen clinically ■ Regularly distributed hyperkeratotic plugs in hair follicles
■ Light bands represent cavities within the cortex (yellow dots)
■ Cavities appear as whitish areas within a darker hair shaft ■ Cadaverized hairs (black dots)
■ The opposite is true with light microscopy ■ Dystrophic hairs
Acquired Hair Diseases ■ Micro-exclamation point hairs
ANDROGENIC ALOPECIA ■ Fibrosis with white dots in long-standing cases
■ Variable hair-shaft diameter CICATRICIAL ALOPECIA
■ Digital systems allow the precise measurement and ■ Scarring alopecia of different etiologies looks the same
monitoring of hair-shaft thickness with fibrosis of follicular ostia visible as white dots
■ Identify and count vellus hairs (thin hairs less than ■ With more advanced disease, the dots coalesce to form
0.03mm in width) bony white areas without visible ostia

QUESTIONS

1. Which criteria can be used to diagnose a 3. Which criteria can be used to diagnose a sebor-
melanocytic lesion? rheic keratosis?
a. Milia-like cysts and pigmented pseudofollicular a. Milky-red areas, irregular streaks, and pigmented
openings. pseudofollicular openings.
b. Arborizing vessels, ulceration, and pigmentation. b. Streaks, irregular blotches, and regression.
c. A central white patch plus fine peripheral pigment c. Fissures, ridges, sharp border demarcation, milia-
network. like cysts, pseudofollicular openings, fat fingers,
d. Lacunae and black homogenous blotches. and hairpin vessels.
e. Pigment network, brown globules, homogeneous d. Rhomboid structures and/or circle within a circle.
blue color, or parallel patterns. e. Diffuse brown color, glomerular vessels, and
milia-like cysts.
2. Diagnosing a melanocytic lesion by default means?
a. There are high risk criteria at the periphery of 4. Which criteria can be used to diagnose a basal
the lesion that are hard to identify. cell carcinoma?
b. There are criteria for a seborrheic keratosis or a. Pigment network and arborizing vessels.
basal cell carcinoma associated with pigment b. Arborizing and pinpoint vessels plus multifocal
network and brown globules. hypopigmentation.
c. There is an absence of criteria to diagnose a c. The absence of a pigment network, arborizing
melanocytic lesion, seborrheic keratosis, der- vessels, pigmentation, ulceration, and spoke-wheel
matofibroma, pyogenic granuloma, or ink-spot structures.
lentigo, therefore the lesion should be considered d. Glomerular vessels, ulceration, and blue ovoid
melanocytic. nests of pigmentation.
d. There is an absence of criteria to diagnose a e. Islands of black blotches, arborizing vessels, and
melanocytic lesion, seborrheic keratosis, basal cell moth-eaten borders.
carcinoma, dermatofibroma, or hemangioma, there-
fore the lesion should be considered melanocytic.
e. None of the above.
24 DERMOSCOPY: AN ILLUSTRATED SELF-ASSESSMENT GUIDE

5. Vascular lesions can contain the following 8. Dysplastic nevi typically have the following
criteria: combination of criteria:
a. Out of focus lacunae-like globules. a. Symmetry of color and structure and no melanoma-
b. A variable number of red, sharply demarcated specific criteria.
vascular spaces called lacunae and fibrous b. Asymmetry of color and structure, irregular pig-
septae. ment network, regular blotches, and regular streaks.
c. Ten to twenty major and minor lacunae and c. Multifocal regression, peppering, regular pigment
thromboses. network, regular dots, and globules.
d. A minimal of two well-developed glomerular d. Pinpoint, arborizing, and glomerular vessels plus
vessels. several melanoma-specific criteria.
e. Fibrous septae, peppering, and blue dark e. Asymmetry of color and structure plus several
lacunae. melanoma-specific criteria.

6. Dermatofibromas can be associated with the 9. Which of the following statements is true
following criteria: about Spitz nevi?
a. Pigment network, arborizing vessels, and central a. They can have ten different patterns.
white patch. b. A Spitzoid lesion only refers to the starburst or
b. A central white patch that is never located at the pink patterns.
periphery. c. Melanoma is not in the differential diagnosis of
c. A central white patch and peripheral pigment regular starburst pattern.
network. d. In an adult, most Spitzoid lesions do not need to
d. A complete absence of blood vessels and a few be excised.
milia-like cysts. e. Symmetrical and asymmetrical starburst patterns
e. Multifocal hypopigmentation, arborizing vessels, can be seen in melanoma.
and a central bluish-white veil.
10. Which of the following statements best describes
7. Melanoma-specific criteria on the trunk and the criteria seen in superficial spreading
extremities can contain the following combina- melanomas?
tion of criteria: a. Criteria associated with a benign nevus are never seen.
a. Asymmetry of color and structure, a cobble- b. They contain several well-developed melanoma-
stone global pattern, and regular globules or specific criteria such as symmetry of color and
blotches. structure and one prominent color.
b. A multicomponent global pattern, symmetry of c. Usually they have several well-developed melanoma-
color and structure, regular network, regular specific criteria such as asymmetry of color and
globules, and regression. structure, multicomponent global pattern, regular
c. Polymorphous vessels, arborizing vessels, two network, regular globules, and regular streaks.
colors, and regular streaks. d. They contain a variable number of melanoma-
d. Irregular pigment network, irregular globules, specific criteria such as asymmetry of color and
irregular blotches, and regression. structure, multicomponent global pattern, irregu-
e. Rhomboid structures and the parallel ridge lar local criteria, five or six colors, and polymor-
pattern. phous vessels.
e. Are usually feature-poor or featureless.

ANSWERS

1. E. Criteria to diagnose a melanocytic lesion include a melanocytic lesion. Milia-like cysts and pseudofollicular
any variation of pigment network (regular and/or irregu- openings can be seen in melanocytic lesions but are not
lar), multiple brown dots and/or globules, homogeneous primary criteria to make the diagnosis. Answers a, b, c and d
blue color of a blue nevus, and parallel patterns seen on diagnose a seborrheic keratosis, basal cell carcinoma,
acral skin. The default category is the last way to diagnose dermatofibroma, and hemangioma.
Chapter 1 Dermoscopy from A to Z 25

2. D. Diagnosing a melanocytic lesion by default means criteria are identified. There are innumerable ways that the
that one does not see criteria for a melanocytic lesion, seb- central white patch can appear, and in many cases it is not
orrheic keratosis, basal cell carcinoma, dermatofibroma, or centrally located. Telectangietatic vessels with polymorphous
hemangeoma. Default is an absence of criteria. One has to shapes are commonly seen but basal cell carcinoma-like
memorize all of the criteria from each specific potential arborizing vessels would make the diagnosis of a dermatofi-
diagnosis to be able to diagnose a melanocytic lesion by broma unlikely.
default. Dermoscopy cannot be mastered by osmosis. It is
7. D. Irregularity is the name of the game if criteria are
essential to study and practice the technique routinely in
to be considered melanoma-specific. Melanoma-specific
ones daily practice. Ink-spot lentigo and pyogenic granu-
criteria can be seen in both benign and malignant pathol-
loma are not in this algorithm.
ogy but are more sensitive and specific for melanoma.
3. C. All of the criteria used to diagnose seborrheic There is not a single melanoma-specific criterion that is
kerotosis are commonly seen in daily practice. Melanoma- pathognomonic for melanoma. One should learn their
specific criteria can also be seen in atypical seborrheic definitions, and study as many classic textbook examples
keratosis. Beware of seborrheic keratosis—like melanomas. as possible. Rhomboid structures help diagnose melanoma
Milky-red areas, irregular streaks, regression, rhomboid on the face and the parallel ridge pattern can be seen in
structures, and circle within a circle are all melanoma- acral melanomas.
specific criteria that are more sensitive and specific for
8. E. Dysplastic nevi are ubiquitous in the light-skinned
melanoma but could be found in seborrheic keratosis.
population and can be indistinguishable clinically and der-
Glomerular vessels are a primary criterion to diagnose
moscopically from melanoma. They usually look more
Bowen disease and are not seen in seborrheic keratosis.
benign than malignant with dermoscopy; however, there
4. C. Basal cell carcinomas are usually a clinical diagno- are melanomas that do not have well-developed
sis and dermoscopy is used to confirm ones clinical melanoma-specific criteria. Vessels of any kind are not
impression. Dermoscopy gives one an on the spot second typically seen except in pink feature-poor dysplastic nevi.
opinion. By definition, if one sees pigment network the They can have a variable number of melanoma-specific
lesion could not be a basal cell carcinoma. A sub-set of criteria (eg, irregular pigment network, irregular dots,
melanomas can be difficult to differentiate from basal cell and/or globules, irregular blotches) that are not as well
carcinoma with pigmentation and arborizing vessels. developed as those seen in melanoma. Streaks, regression,
Pinpoint and glomerular vessels could be seen but they and many colors are not usually seen and should raise a
would be out shadowed by arborizing vessels. If not, one red flag for concern that the lesion might be a melanoma.
could be dealing with a basal cell-like melanoma. Moth-
9. E. Spitzoid lesions are always a red flag for concern.
eaten borders are seen in lentigines and flat seborrheic
Even symmetrical patterns can be seen in melanoma. There
keratosis, never in basal cell carcinomas.
are only six patterns (starburst, globular, homogeneous,
5. B. The hallmark of vascular lesions are lacunae, vas- pink, black network, and atypical). One often has to use their
cular spaces with well-demarcated sharp borders. There is imagination to diagnose a Spitzoid lesion. Since symmetrical
no set number of lacunae needed to make the diagnoses. At and asymmetrical Spitzoid patterns can be found in
times one has to use their imagination to decide if the mar- melanoma, they should all be excised in children as well as in
gins fit the criteria for vascular spaces. Different shades of adults. A dermatopathologist that specializes in melanocytic
red, blue, and even black are typically seen. Black homoge- lesions is good, while one that has expertise in Spitzoid
neous color usually represents thombosis. Major and minor lesions is ideal. Even experienced dermatopathologists have
lacunae do not exist. Fibrous whitish septa and/or bluish- trouble differentiating atypical Spitzoid lesions from
white color are routinely seen in typical hemangiomas. At melanoma, and atypical Spitzoid lesions have the potential
times it is not possible to differentiate lacunae and red to metastasize because in reality they are melanomas.
color of a hemangioma from the milky-red areas that can
10. D. Superficial spreading melanoma can have it all
contain out-of-focus reddish globules seen in melanoma.
as far as the spectrum of melanoma-specific criteria goes.
6. C. Dermatofibromas are ubiquitous benign tumors The criteria can be well developed or difficult to identify.
and in most cases dermoscopy is not needed to make the Criteria associated with benign melanocytic lesions can
diagnosis. A central white patch and pigment network, the also be seen. The more the high risk criteria identified in
primary criteria to make the diagnosis, may or may not be the lesion, the greater the chance that one is dealing with a
present. It might not be possible to differentiate an atypical melanoma. Nodular and amelanotic melanoma are more
dermatofibroma from a melanoma if melanoma-specific likely to be feature-poor or featureless.
26 DERMOSCOPY: AN ILLUSTRATED SELF-ASSESSMENT GUIDE

SUGGESTED READINGS Johr RH. Pink lesions. Clin Dermatol. 2002; 20:289-296.
Kittler H, Guitera P, Riedl E et al. Identification of clinically featureless
Altamura D, Avramidis M, Menzies SW. Assessment of the optimal interval incipient melanoma using sequential dermoscopy imaging. Arch
for and sensitivity of short-term sequential digital dermoscopy monitor- Dermatol. 2006; 142(9):1113-1119.
ing for the diagnosis of melanoma. Arch Dermatol. 2008; 144(4):502-506. Malvehy J, Puig S, Braun R, Marghoob A, Kopf AW. Handbook of
Argenziano G, Zalaudek I, Johr RH et al. Dermoscopy features of Dermoscopy. London (UK): Taylor and Francis; 2006.
melanoma incognito: indications for biopsy. J Am Acad Dermatol. 2007; Malvehy J, Puis S et al. Dermoscopy report: proposal for standardization.
56(3):508-513. Results of a consensus meeting of the International Dermoscopy Society.
Argenziano G, Soyer HP, Chimenti S et al. Dermoscopy of pigmented skin J Am Acad Dermatol. 2007; 57:84-95.
lesions: results of a consensus meeting via the internet. J Am Acad Marghoob AA, Braun PR, Kopf AW eds. Atlas of Dermoscopy. London (UK):
Dermatol. 2003; 48:679-693. Taylor and Francis; 2005
Argenziano G, Zalundek I et al. Vascular structures in skin tumors: a der- Menzies S, Zalaudek I. Why perform dermoscopy? The evidence for its role
moscopy study. Arch Dermatol. 2004; 140:1485-1489. in the routine management of pigmented skin lesions. Arch Dermatol.
Bowling J, Agenziano G et al. Dermoscopy key points: recommenda- 2006; 142(9):1211-1212.
tions from the International Dermoscopy Society. Dermatol. 2007; Stolz W, Braun–Falco O et al. Color Atlas of Dermatoscopy. 2nd ed. Berlin:
214:3-5. Blackwell Publishing; 2002.
Cari P, DeGiorgi V, Crocetti E et al. Improvement of the malignant/benign Vestergaard ME, Macaskill P et al. Dermoscopy compared with naked eye
ratio in excised melanocytic lesions in the dermoscopy era: a retrospec- examination for the diagnosis of primary melanoma: a meta-analysis of
tive study 1997-2001. Br J Dermatol. 2004; 150(4):687-692. studies performed in a clinical setting. Br J Dermatol. 2008; 159:669-676.
Johr RH, Soyer HP, Argenziano G. Dermoscopy: The Essentials. Edinburgh:
Mosby; 2004.
Chapter 2

Scalp, Face, Nose, and Ears


.

General Instructions: You will find a list of True/False statements following each case
history. Select any statements, which you believe to be true. There may be one, more
than one or no true statements for any given case. Choose the correct risk, diagnosis
and disposition for each case. Then, turn the page to find a detailed discussion and
pearls for each case.
Chapter 2 Scalp, Face, Nose, and Ears 29

RISK
❑ Low
❑ Intermediate
❑ High

DIAGNOSIS
❑ Nevus
❑ Seborrheic keratosis
❑ Basal cell carcinoma
❑ Vascular

2-1a
❑ Dermatofibroma
❑ Squamous cell carcinoma
❑ Melanoma
❑ Other

DISPOSITION
❑ No intervention
❑ Follow-up
❑ Histopathologic diagnosis
2-1b

CASE 1
HISTORY
An 85-year-old man noticed the significant darkening of a long time facial lesion.
1. Clinically and dermoscopically this could be lentigo maligna.
2. Multiple milia-like cysts and pseudofollicular openings diagnose a seborrheic keratosis.
3. It is not always possible to differentiate follicular openings from pseudofollicular openings and
milia-like cysts.
4. The dark blotch with the bluish tinge is always high risk.
5. The dark blotch has histopathologic correlates that include atypical melanocytes.
30 DERMOSCOPY: AN ILLUSTRATED SELF-ASSESSMENT GUIDE

RISK
✔ Low

❑ Intermediate
❑ High

DIAGNOSIS
❑ Nevus

❑ Seborrheic keratosis
❑ Basal cell carcinoma
❑ Vascular
❑ Dermatofibroma
❑ Squamous cell carcinoma
❑ Melanoma
❑ Other

2-1c
DISPOSITION
ANSWERS
✔ No intervention

Answers: 2,3,5
❑ Follow-up
❑ Histopathologic diagnosis Discussion:
■ Multiple milia-like cysts and pseudofollicular openings characterize this seb-
orrheic keratosis.
■ Both criteria can also be found in benign nevi and melanoma.
■ Dark colors (hyperpigmentation) are commonly found in seborrheic keratosis.
DERMOSCOPIC CRITERIA ■ Concave “moth-eaten” borders are one criterion seen in flat seborrheic ker-
atosis and solar lentigines.
■ Follicular openings (boxes)
■ Blue and/or white color in any intensity should raise a red flag for concern.
■ Pigmented pseudofollicular ■ Bluish color is not always high risk.
openings (yellow arrows) ■ In a flat lesion subtle blue and/or white color usually is low risk
■ Milia-like cysts (black arrows) ■ In a raised lesion intense blue and/or white color is potentially high risk.
■ Hyperpigmentation
(white arrows) PEARLS
■ Moth-eaten borders
■ It is essential to differentiate the pseudofollicular openings and milia-like
(red arrows)
cysts of a seborrheic keratosis from the follicular openings seen in lentigo
maligna or lentigo maligna melanoma.
■ One should not stop and diagnose a seborrheic keratosis if milia-like cysts and
pseudofollicular openings are identified before examining the entire lesion.
■ High risk criteria pointing to the correct diagnosis might be found elsewhere
and then they would be missed.
Chapter 2 Scalp, Face, Nose, and Ears 31

RISK
❑ Low
❑ Intermediate
❑ High

DIAGNOSIS
❑ Nevus
❑ Seborrheic keratosis
❑ Basal cell carcinoma

2-2a-1

Vascular

2-2a-2
❑ Dermatofibroma
❑ Squamous cell carcinoma
❑ Melanoma
❑ Other

DISPOSITION
❑ No intervention
❑ Follow-up
❑ Histopathologic diagnosis
2-2b

CASE 2
HISTORY
The mother of this 7-year-old female is concerned about a blue spot on her daughter’s face. She
wants it treated immediately because the patient’s uncle was recently diagnosed with a melanoma.
The lesion has been present for two years and is not changing.
1. Historically, clinically, and dermoscopically this clearly falls within the range of a blue nevus.
2. This could be a combined nevus.
3. This polychromatic lesion with white, blue, and shades of brown color indicates that it is high
risk.
4. Different shades of blue, brown, black, and white can be seen in blue nevi.
5. The multiple follicular openings seen here are as expected on the face.
32 DERMOSCOPY: AN ILLUSTRATED SELF-ASSESSMENT GUIDE

RISK
✔ Low

❑ Intermediate
❑ High

DIAGNOSIS

❑ Nevus
❑ Seborrheic keratosis
❑ Basal cell carcinoma
❑ Vascular
❑ Dermatofibroma
❑ Squamous cell carcinoma
❑ Melanoma

2-2c
❑ Other

ANSWERS
Answers: 1,2,4,5
DISPOSITION
✔ No intervention
❑ Discussion:
■ This blue nevus has a dermoscopic differential diagnosis that includes a
❑ Follow-up
combined nevus.
❑ Histopathologic diagnosis
■ In most cases blue nevi are easy to diagnose.
■ The classic blue nevus is characterized by homogenous featureless blue color
with well-demarcated borders.
■ Hypopigmentation often scar-like is commonly seen.
DERMOSCOPIC CRITERIA ■ Globular-like structures can be seen in blue nevi.
■ Combined nevi contain two different cell populations.
■ Follicular openings (circles)
■ Blue nevi and compound nevi make up most combined nevi.
■ Brown color (arrows) ■ Combined nevi can contain various shades and distributions of blue and
■ Bluish-white color brown colors.
■ “Black” blue nevi are created by the black lamella (pigmented parakeratosis).
■ The black lamella looks like a jet black homogeneous blotch with a

glazed appearance.
■ Tape stripping can remove the black lamella, which is a positive clinical sign
pointing toward low risk pathology.
■ The same holds true in whatever lesions that have a black lamella

(ie, junctional nevus).


■ Polychromatic blue nevi should raise a red flag for concern.
■ The concept of “raising a red flag for concern” means that there is a clue

or clues that a lesion might be high risk. Proceed with focused attention
with the complete analysis of the lesion.

PEARLS
■ The history is essential to help make the correct diagnosis of a blue nevus.
■ Melanoma and cutaneous metastatic melanoma can be identical clinically
and dermoscopically with a blue nevus.
■ A “blue” nevus developing in an adult should always raise a red flag for concern.
■ To allay the anxiety of the parents that this could be a melanoma, a punch
excision of the lesion was performed which confirmed that this was in fact a
benign blue nevus.
Chapter 2 Scalp, Face, Nose, and Ears 33

RISK
❑ Low
❑ Intermediate
❑ High

DIAGNOSIS
❑ Nevus
❑ Seborrheic keratosis
❑ Basal cell carcinoma
❑ Vascular
❑ Dermatofibroma
❑ Squamous cell carcinoma
2-3a

❑ Melanoma
❑ Other

DISPOSITION
❑ No intervention
❑ Follow-up
❑ Histopathologic diagnosis
2-3b

CASE 3
HISTORY
This young patient was referred to you for a second opinion because their dermatologist recom-
mended excision of this “fried egg” appearing nevus. It was never examined with dermoscopy and
the referring pediatric dermatologist thought that dermoscopic examination would be beneficial
to help avoid surgery on the child’s face.
1. “Fried egg” appearing nevi are always at high risk, histopathologically, and, therefore, dermo-
scopic evaluation is not indicated.
2. In this age group “fried egg” appearing nevi are usually not high risk and dermoscopy can help
to prove this.
3. Regular dots, globules, and a regular blotch characterize this combined nevus.
4. The dark blotch in an adult should raise a red flag for concern.
5. Sequential digital clinical and dermoscopic monitoring is an alternative to skin biopsy or
excision.
34 DERMOSCOPY: AN ILLUSTRATED SELF-ASSESSMENT GUIDE

RISK
✔ Low

❑ Intermediate
❑ High

DIAGNOSIS

❑ Nevus
❑ Seborrheic keratosis
❑ Basal cell carcinoma
❑ Vascular
❑ Dermatofibroma
❑ Squamous cell carcinoma
❑ Melanoma

2-3c
Other

DISPOSITION
❑ No intervention
✔ Follow-up
❑ ANSWERS
❑ Histopathologic diagnosis Answers: 2,3,4,5

Discussion:
■ “Fried egg” appearing nevi should be evaluated with dermoscopy before

more invasive procedures are performed.


DERMOSCOPIC CRITERIA: ■ In this age group the chances of high risk pathology are extremely low.
■ This congenital combined nevus has two cell populations: a blue nevus (the
■ Regular dots and globules
bluish-brown blotch) and compound nevus.
(circles) ■ Combined nevi do not always have a “fried egg” appearance.
■ Islands of normal skin (arrows) ■ If found in a teenager or an adult, the bluish-brown blotch is potentially at
■ Regular bluish-brown blotch more high risk.
(star) ■ The bluish-brown blotch has a differential diagnosis that includes
melanocytic atypia or melanoma.
■ Islands of normal skin and islands of criteria are one of the patterns commonly
seen in congenital melanocytic nevi.

PEARLS
■ Sequential digital monitoring and confident reassurance to the parents that
an excision is not needed would be the optimal cutting-edge approach to
handle this case.
■ Whatever global patterns and local criteria are identified, one has to decide
if they are regular or irregular, good or bad, low or high risk.
Chapter 2 Scalp, Face, Nose, and Ears 35

RISK
❑ Low
❑ Intermediate
❑ High

DIAGNOSIS
❑ Nevus
❑ Seborrheic keratosis
❑ Basal cell carcinoma
❑ Vascular

2-4a
❑ Dermatofibroma
❑ Squamous cell carcinoma
❑ Melanoma
❑ Other

DISPOSITION
❑ No intervention
❑ Follow-up
❑ Histopathologic diagnosis
2-4b

CASE 4
HISTORY
A 76-year-old man came for a consultation of a slowly enlarging growth on his nose that started
one year ago.
1. Milia-like cysts and pigmented pseudofollicular openings diagnose this pigmented seborrheic
keratosis.
2. Irregular globules and blotches, polymorphous vessels, and regression characterize this
melanoma.
3. The absence of pigment network, arborizing vessels, plus pigmentation diagnoses this basal
cell carcinoma.
4. This pattern of pigmentation and vessels can be found in a basal cell carcinoma or melanoma.
5. Leaf-like structures are present throughout this lesion.
36 DERMOSCOPY: AN ILLUSTRATED SELF-ASSESSMENT GUIDE

RISK
❑ Low
❑ Intermediate
✔ High

DIAGNOSIS
❑ Nevus
❑ Seborrheic keratosis

❑ Basal cell carcinoma
❑ Vascular
❑ Dermatofibroma
❑ Squamous cell carcinoma
❑ Melanoma
❑ Other

2-4c
DISPOSITION
❑ No intervention
❑ Follow-up
ANSWERS
✔ Histopathologic diagnosis
❑ Answers: 3,4

Discussion:
■ An experienced dermoscopist can easily make the diagnosis.
■ A less-experienced dermoscopist should evaluate all of the criteria found in
DERMOSCOPIC CRITERIA the lesion with full knowledge of their definitions.
■ Melanoma is in the differential diagnosis because of the irregular blackish-
■ Arborizing vessels (red arrows) gray blotches and diffuse bony- white color (regression).
■ Irregular grayish-white blotches ■ White color is commonly found in basal cell carcinomas and does not represent
(black arrows) true regression.
■ Follicular openings inside the ■ Pinpoint; irregular linear and other irregularly-shaped telangectatic vessels
lesion (yellow arrows) (polymorphous vascular pattern) are absent.
■ Follicular openings outside the ■ The dermoscopic differential diagnosis of the large pseudofollicular opening
lesions (yellow boxes) includes ulceration.
■ Pseudofollicular openings ■ Ulceration is much more commonly found in basal cell carcinoma than in
(white arrows) melanoma.
■ Arborizing vessels are not found in seborrheic keratosis.
■ Bony-white color (stars)

PEARLS
■ There are innumerable shapes and sizes of pigmentation that can be found
in basal cell carcinomas.
■ One has to only determine if pigment is present or absent.
■ Adjectives (ie, ovoid, leaf-like) describing the shape of pigmentation are not
needed.
■ The irregular grayish-black blotch at the superior pole fits the definition of a
“leaf-like” structure, yet it has no resemblance to a leaf.
■ This is an inaccurate concept that is written and taught, which should be
abandoned.
Chapter 2 Scalp, Face, Nose, and Ears 37

RISK
❑ Low
❑ Intermediate
❑ High

DIAGNOSIS
❑ Nevus
❑ Seborrheic keratosis
❑ Basal cell carcinoma
❑ Vascular

2-5a
❑ Dermatofibroma
❑ Squamous cell carcinoma
❑ Melanoma
❑ Other

DISPOSITION
❑ No intervention
❑ Follow-up
❑ Histopathologic diagnosis
2-5b

(Reprinted, with permission, from Stolz W, Braun-Falco O, Bilek P


et al. (2002) Color Atlas of Dermatoscopy. Second Edition. Blackwell
Publishing, Oxford.)

CASE 5
HISTORY
This previously long-standing asymptomatic lesion suddenly became raised and tender, but only
for a few days. The patient’s primary care physician was concerned about the sudden change and
thought it was a skin cancer.
1. This is the classic clinical and dermoscopic picture of an inflamed molluscum contagiosum.
2. This collision tumor consists of a squamous and basal cell carcinoma.
3. Arborizing vessels and regression characterize this melanoma.
4. Arborizing “basal cell-like” vessels and whitish-yellow globules characterize this typical sebaceous
gland hyperplasia.
5. Basal cell carcinoma is in the clinical and dermoscopic differential diagnosis of sebaceous
gland hyperplasia.
38 DERMOSCOPY: AN ILLUSTRATED SELF-ASSESSMENT GUIDE

RISK
✔ Low

❑ Intermediate
❑ High

DIAGNOSIS
❑ Nevus
❑ Seborrheic keratosis
❑ Basal cell carcinoma
❑ Vascular
❑ Dermatofibroma
❑ Squamous cell carcinoma
❑ Melanoma

2-5c
Other

DISPOSITION
✔ No intervention

ANSWERS
❑ Follow-up
❑ Histopathologic diagnosis Answers: 4,5

Discussion:
■ This is a classic clinical and dermoscopic sebaceous gland hyperplasia.
■ The yellowish-white globules would not be seen in a basal cell carcinoma.
■ By definition “crown” or “wreath-like” vessels said to characterize sebaceous
DERMOSCOPIC CRITERIA gland hyperplasia surround and penetrate the lesion, but never reach the
■ Basal cell-like vessels center.
(black arrows) ■ It is not necessary to adhere to this strict definition because it is rarely found
■ Sebaceous globules in typical lesions.
(yellow arrows) ■ “Crown” or “wreath-like” vessels are poor terms that should be abandoned.
■ Follicular openings (red arrows) ■ Basal cell-like vessels in any distribution plus sebaceous globules make the
diagnosis.
■ Molluscum contagiosum typically presents with a clear papule that has a
central dell.
■ Irregular whitish globules filling a lesion and peripheral linear vessels

along the border have been reported.

PEARLS
■ The dramatic and worrisome short-lived change was created by an isolated
pimple that coincidentally developed under the long-standing sebaceous
gland hyperplasia.
■ In general, it is not always necessary to rush into making a histopathologic
diagnosis in a changing lesion when the change is of a short duration and
the etiology of the change not clear.
Chapter 2 Scalp, Face, Nose, and Ears 39

RISK
❑ Low
❑ Intermediate
❑ High

DIAGNOSIS
❑ Nevus
❑ Seborrheic keratosis
❑ Basal cell carcinoma
❑ Vascular
❑ Dermatofibroma

2-6a
Squamous cell carcinoma
❑ Melanoma
❑ Other

DISPOSITION
❑ No intervention
❑ Follow-up
❑ Histopathologic diagnosis
2-6b

CASE 6
HISTORY
A 64-year-old ski enthusiast with a history of many sunburns presented with a nonhealing lesion
on his nose.
1. This featureless lesion is of no concern.
2. This feature-poor lesion does not have enough criteria to diagnose a basal cell carcinoma.
3. Foci of asymmetrical follicular pigmentation are an important clue that this is a melanoma.
4. Milia-like cysts, pink and gray color diagnose this irritated seborrheic keratosis.
5. This feature poor melanoma has asymmetry of color and structure, asymmetrical follicular
pigmentation, pink and gray color, polymorphous vessels, and ulceration.
40 DERMOSCOPY: AN ILLUSTRATED SELF-ASSESSMENT GUIDE

RISK
❑ Low
❑ Intermediate
✔ High

DIAGNOSIS
❑ Nevus
❑ Seborrheic keratosis
❑ Basal cell carcinoma
❑ Vascular
❑ Dermatofibroma
❑ Squamous cell carcinoma

❑ Melanoma
❑ Other

2-6c
DISPOSITION
❑ No intervention ANSWERS
❑ Follow-up Answers: 2,3,5
✔ Histopathologic diagnosis

Discussion:
■ This lesion is feature-poor not featureless.
■ Feature-poor means that the important criteria are present but not well-
developed and easy to identify.
■ Clinically one suspects a basal cell carcinoma but the ulceration is not
DERMOSCOPIC CRITERIA enough to make that diagnosis.
■ Asymmetry of color and ■ Asymmetrical follicular pigmentation is the most specific criteria to diagnose
structure a melanoma.
■ Follicular openings (arrows) ■ The polymorphous vessels (ie, pinpoint, linear) could easily be missed if too
■ Asymmetrical follicular much pressure is applied with instrumentation.
pigmentation (boxes)
■ Diffuse pinkish color PEARLS
■ Grayish-white color (black stars) ■ Pink color should always be a red flag for concern in any lesion.
■ Polymorphous vessels (circle) ■ “If there’s pink, stop and think.”
■ Ulceration (yellow star) ■ If one has to spend too much time thinking about what is going on, it is
time to make a histopathologic diagnosis.
■ This diagnosis was a big surprise!
Chapter 2 Scalp, Face, Nose, and Ears 41

RISK
❑ Low
❑ Intermediate
❑ High

DIAGNOSIS
❑ Nevus
❑ Seborrheic keratosis
❑ Basal cell carcinoma
❑ Vascular

2-7a
❑ Dermatofibroma
❑ Squamous cell carcinoma
❑ Melanoma
❑ Other

DISPOSITION
❑ No intervention
❑ Follow-up
❑ Histopathologic diagnosis
2-7b

CASE 7
HISTORY
A 39-year-old woman was concerned about a strange-looking spot on her face that did not wash off
with soap.
1. This is a typical make-up tattoo that should be treated with a laser.
2. This could only be a melanocytic lesion because it has a pigment network.
3. Lentigines and dermafibromous are nonmelanocytic lesions that can have a pigment network.
4. Branched streaks and irregular globules diagnose this in situ melanoma.
5. The black stellate macule seen clinically and thickened, branched black pigment network
diagnose this classic ink-spot lentigo.
42 DERMOSCOPY: AN ILLUSTRATED SELF-ASSESSMENT GUIDE

RISK
✔ Low

❑ Intermediate
❑ High

DIAGNOSIS
❑ Nevus
❑ Seborrheic keratosis
❑ Basal cell carcinoma
❑ Vascular
❑ Dermatofibroma
❑ Squamous cell carcinoma
❑ Melanoma

❑ Other

2-7c
DISPOSITION
ANSWERS
✔ No intervention

❑ Follow-up Answers: 3,5
❑ Histopathologic diagnosis Discussion:
■ The diagnosis of an ink-spot lentigo can usually be made clinically.
■ Look for high risk criteria so that you do not miss an ink-spot lentigo-like
melanoma.
■ The criteria seen here are not the ones used to diagnose in situ melanoma
on the face (ie, asymmetrical follicular pigmentation, annular-granular
DERMOSCOPIC CRITERIA
structures, rhomboid structures, circle within a circle).
■ Thick, branched pigment ■ Follicular openings that one expects to find on the face are not found in
network (arrows) every patient.
■ Regular dots (circle) ■ The dot-like structures represent remnants of the pigment network.
■ Not all skin lesions with a pigment network are melanocytic (ie, lentigo,
dermatofibroma).

PEARLS
■ Ink-spot lentigines can be associated with a history of excessive sun exposure.
■ A complete skin examination should be performed to look for more high risk
pathology (ie, skin cancer).
■ Complete skin examinations should be performed on all patients at the time
one deems appropriate.
Chapter 2 Scalp, Face, Nose, and Ears 43

RISK
❑ Low
❑ Intermediate
❑ High

DIAGNOSIS
❑ Nevus
❑ Seborrheic keratosis
❑ Basal cell carcinoma
❑ Vascular

2-8a
❑ Dermatofibroma
❑ Squamous cell carcinoma
❑ Melanoma
❑ Other

DISPOSITION
❑ No intervention
❑ Follow-up
❑ Histopathologic diagnosis
2-8b

CASE 8
HISTORY
A 45-year-old man noticed the appearance of a new spot on his face that was getting darker.
1. Clinically and dermoscopically this could be an ink-spot lentigo.
2. The absence of a thick black branched pigment network should raise a red flag for concern
that his might not be an ink-spot lentigo.
3. The irregular black blotch, irregular globules, and foci of blue-white color characterize this in
situ melanoma.
4. This is too small to be a melanoma; therefore the high risk dermoscopic picture should be
ignored.
5. The isolated pseudofollicular opening, milia-like cysts, and sharp demarcation diagnose this
small seborrheic keratosis.
44 DERMOSCOPY: AN ILLUSTRATED SELF-ASSESSMENT GUIDE

RISK
❑ Low
✔ Intermediate

❑ High

DIAGNOSIS
❑ Nevus
❑ Seborrheic keratosis
❑ Basal cell carcinoma
❑ Vascular
❑ Dermatofibroma
❑ Squamous cell carcinoma

❑ Melanoma
❑ Other

2-8c
DISPOSITION
❑ No intervention ANSWERS
❑ Follow-up Answers: 1,2,3
✔ Histopathologic diagnosis

Discussion:
■ The history and dermoscopic picture are worrisome.
■ An ink-spot lentigo might look like this dermoscopically. However, without
the prominent pigment network it falls out of the range of a classic lesion.
DERMOSCOPIC CRITERIA ■ One has to create a dermoscopic differential diagnosis for the round white
structures (ie, milia-like cysts vs follicular openings).
■ Irregular black blotch
■ Milia-like cysts and pseudofollicular openings can be seen in melanoma.
(white star)
■ This entire picture warrants a histopathologic diagnosis even if it is in a
■ Follicular openings cosmetically sensitive area.
(red arrows) ■ Melanomas can start as a speck and remain less than 6mm for long periods
■ Pseudofollicular opening of time.
(white arrow)
■ Irregular dots (white circles) PEARLS
■ Blue-white color (black arrows)
■ Destructive therapy without a positive histopathologic diagnosis could be
disastrous.
■ If after destructive therapy without a previous histopathologic diagnosis a
lesion like this recurs, a histopathologic diagnosis should now be made
posthaste.
■ “Melanoma incognito” aka false negative melanoma can mimic benign or
other malignant lesions.
■ “Melanoma incognito” can clinically and/or dermoscopically mimic sebor-
rheic keratosis, vascular lesions, basal cell carcinoma combined, recurrent,
Spitz, blue and dysplastic nevi, actinic keratosis, or solar lentigines.
Chapter 2 Scalp, Face, Nose, and Ears 45

RISK
❑ Low
❑ Intermediate
❑ High

DIAGNOSIS
❑ Nevus
❑ Seborrheic keratosis
❑ Basal cell carcinoma
❑ Vascular
❑ Dermatofibroma
❑ Squamous cell carcinoma
❑ Melanoma
❑ Other
2-9a

DISPOSITION
❑ No intervention
❑ Follow-up
❑ Histopathologic diagnosis
2-9b

CASE 9
HISTORY
There is only a two-month-history of this asymptomatic pigmented skin lesion on the ear of a
30-year-old man.
1. This is a melanocytic lesion because there are multiple brown globules.
2. This is the typical dermoscopic appearance of a secondarily infected wound.
3. This is an atypical Spitzoid global pattern because there are foci of irregular streaks.
4. Asymmetry of color and structure, irregular globules, irregular streaks, an irregular black
blotch plus regression characterizes this melanoma.
5. An atypical Spitzoid pattern seen with dermoscopy does not always correlate with a “Spitzoid”
melanoma histopathologically.
46 DERMOSCOPY: AN ILLUSTRATED SELF-ASSESSMENT GUIDE

RISK
❑ Low
❑ Intermediate
✔ High

DIAGNOSIS
❑ Nevus
❑ Seborrheic keratosis
❑ Basal cell carcinoma
❑ Vascular
❑ Dermatofibroma
❑ Squamous cell carcinoma

❑ Melanoma

2-9c
Other

DISPOSITION
❑ No intervention ANSWERS
❑ Follow-up Answers: 1,3,4,5
✔ Histopathologic diagnosis

Discussion:
■ A regular/good/low risk Spitzoid pattern, aka “Starburst” pattern, can have
streaks and/or dots and globules at most points along the periphery of the
lesion.
DERMOSCOPIC CRITERIA ■ An atypical Spitzoid pattern can have foci of streaks and/or dots and globules
■ Asymmetry of color and at some points along the periphery of the lesion.
structure ■ There are only streaks connected to the irregular black blotch which fits the
■ Irregular dots and globules definition of irregular streaks.
(circles) ■ Inter-observer agreement among experienced dermoscopists is not always
good and another clinician might not agree that this is an atypical Spitzoid
■ Irregular streaks (black arrows)
global pattern.
■ Irregular black blotch
■ The grayish-white regression area fills half of the lesion.
(white star, white arrows) ■ “Spitzoid” dermoscopic patterns are not always Spitzoid histopathologically.
■ Regression (black star)
PEARLS
■ Even regular Spitzoid patterns can be seen in melanoma.
■ All lesions with this pattern of criteria should be removed no matter how
old or young the patient.
■ There would not be a good dermoscopic–pathologic correlation if the
histopathologic report was that of a benign lesion.
■ Speak with your pathologist about the asymmetrical dermoscopic appearance
of the criteria.
■ A reevaluation of the specimen might be indicated.
■ Do not hesitate to ask for another histopathologic opinion with a
pigmented-lesion expert if there is not a good dermoscopic–pathologic
correlation.
Chapter 2 Scalp, Face, Nose, and Ears 47

RISK
❑ Low
❑ Intermediate
❑ High

DIAGNOSIS
❑ Nevus
❑ Seborrheic keratosis
❑ Basal cell carcinoma

2-10a
Vascular
❑ Dermatofibroma
❑ Squamous cell carcinoma
❑ Melanoma
❑ Other

DISPOSITION
❑ No intervention
❑ Follow-up
❑ Histopathologic diagnosis
2-10b

CASE 10
HISTORY
During a routine total body skin examination, this pigmented skin lesion was found on the scalp of
an 83-year-old woman.
1. The clinical and dermoscopic differential diagnosis includes a seborrheic keratosis.
2. The clinical and dermoscopic differential diagnosis includes an ulcerated basal cell carcinoma.
3. The clinical and dermoscopic differential diagnosis includes invasive melanoma.
4. The dermoscopic differential diagnosis of the irregular brown color includes irregular blotch
vs ulceration.
5. The absence of arborizing vessels rules out a basal cell carcinoma.
48 DERMOSCOPY: AN ILLUSTRATED SELF-ASSESSMENT GUIDE

RISK
❑ Low
❑ Intermediate
✔ High

1
2
DIAGNOSIS 1
3
❑ Nevus
❑ Seborrheic keratosis

❑ Basal cell carcinoma
❑ Vascular
❑ Dermatofibroma

2-10c
❑ Squamous cell carcinoma
❑ Melanoma
❑ Other ANSWERS
Answers: 2,3,4
Discussion:
■ Strict analysis of the criteria favors the diagnosis of a melanoma.
DISPOSITION
■ Globules identify a melanocytic lesion:
❑ No intervention ■ Basal cell carcinomas can have pigmented globular-like structures that
❑ Follow-up cannot be differentiated from the dots and globules of a melanocytic
✔ Histopathologic diagnosis
❑ lesion.
■ Asymmetry of color and structure

■ Multicomponent global pattern

■ Irregular dots and globules

■ Irregular blotches
DERMOSCOPIC CRITERIA ■ Regression

■ Asymmetry of color and ■ Basal cell carcinomas do not always have arborizing vessels.
structure ■ Basal cell carcinomas do not always have pigment.
■ Some of the grayish-black blotches fit the definition of “leaf like” structure,
■ Multicomponent global
yet none look like any type of leaf!
pattern (1,2,3)
■ Spoke-wheel structures, which are not seen here, might be the only clue to
■ Irregular dots and globules diagnose a basal cell carcinoma.
(white boxes) ■ It is not always possible to be 100% sure of global patterns and local criteria.
■ Irregular grayish-black blotches ■ One has to be able to create a dermoscopic differential diagnosis for global
(yellow arrows) patterns and local criteria.
■ Ulceration (white arrows) ■ The multicomponent global pattern (three or more different areas within a
■ Regression (yellow stars) lesion) is not diagnostic of melanoma.
■ The multicomponent global pattern can be seen in melanocytic,
nonmelanocytic, benign, and malignant lesions.

PEARLS
■ In this case, dermoscopy has been clinically helpful. Clearly this is not a
seborrheic keratosis that needs no intervention. Even though one cannot be
sure if this is a basal cell carcinoma or melanoma, we know that a
histopathologic diagnosis is indicated.
■ It is essential to be familiar with all of the classic examples of global patterns
and local criteria.
■ One will need that knowledge to be able to create a dermoscopic differential
diagnosis when the occasion arises.
■ Regularly one has to think in terms of dermoscopic differential diagnosis.
Chapter 2 Scalp, Face, Nose, and Ears 49

RISK
❑ Low
❑ Intermediate
❑ High

DIAGNOSIS
❑ Nevus
❑ Seborrheic keratosis
❑ Basal cell carcinoma
❑ Vascular

2-11a
❑ Dermatofibroma
❑ Squamous cell carcinoma
❑ Melanoma
❑ Other

DISPOSITION
❑ No intervention
❑ Follow-up
❑ Histopathologic diagnosis
2-11b

CASE 11
HISTORY
While washing this seven-year-old girl’s hair, her mother found this asymptomatic spot on her scalp.
1. More than 50% of this lesion demonstrates regressive changes pathognomonic of a melanoma.
2. There are no clues to differentiate hypopigmentation from regression in this lesion.
3. Gray color and foci of “peppering” are the criteria to diagnose the regression in this lesion.
4. The more regression in a lesion, the greater the chance it is a melanoma.
5. Irregular dots and globules, irregular blotches, plus regression characterize this dysplastic nevus.
50 DERMOSCOPY: AN ILLUSTRATED SELF-ASSESSMENT GUIDE

RISK
❑ Low
✔ Intermediate

❑ High

DIAGNOSIS

❑ Nevus
❑ Seborrheic keratosis
❑ Basal cell carcinoma
❑ Vascular
❑ Dermatofibroma
❑ Squamous cell carcinoma
❑ Melanoma

2-11c
❑ Other

DISPOSITION
❑ No intervention ANSWERS
❑ Follow-up Answers: 3,4,5
✔ Histopathologic diagnosis

Discussion:
■ Regression is not diagnostic of melanoma.
■ Regression-like areas can be seen in melanocytic, nonmelanocytic, benign, and
malignant lesions.
DERMOSCOPIC CRITERIA ■ The more regression, the greater the chance of high risk pathology (ie,
dysplastic nevi, melanoma).
■ Irregular dots and globules
■ It is not possible to differentiate this severely dysplastic nevus from a
(boxes)
melanoma using dermoscopy.
■ Irregular blotches (black arrows) ■ One might not be able to differentiate hypopigmentation from true regression.
■ Regression (star) ■ The white color of regression should be lighter than the surrounding skin.
■ Gray color (white arrows) ■ Gray color indicates that melanin is in the papillary dermis.
■ Peppering (circles) ■ The fine granules of “peppering” represent free melanin and/or
melanophages in the papillary dermis.
■ Gray color and “peppering” are found in regression not in hypopigmentation.

PEARLS
■ The identification of regression is a red flag for concern.
■ Proceed with focused attention and look for other high risk criteria.
■ In this age group, and in this location, one would expect to find a globular
pattern with regular dots and globules.
■ There is not a good clinico-dermoscopic correlation—another red flag for
concern.
■ Not all scalp nevi in children need to be excised, especially if they have a
banal clinical and dermoscopic appearance.
■ The scalp is a common location to find dyspalstic nevi in children.
■ It is essential to include the scalp in a complete skin examination.
Chapter 2 Scalp, Face, Nose, and Ears 51

RISK
❑ Low
❑ Intermediate
❑ High

DIAGNOSIS
❑ Nevus
❑ Seborrheic keratosis
❑ Basal cell carcinoma
❑ Vascular

2-12a
❑ Dermatofibroma
❑ Squamous cell carcinoma
❑ Melanoma
❑ Other

DISPOSITION
❑ No intervention
❑ Follow-up
❑ Histopathologic diagnosis
2-12b

CASE 12
HISTORY
A first-year-dermatology resident found this small pigmented lesion after a complete skin examination
in a pediatric dermatology clinic. The parents had not been aware of it.
1. Regular dots and globules characterize this classic cobblestone pattern.
2. Irregular dots and globules are one important feature of this dysplastic nevus.
3. The multifocal hypopigmentation seen here is a site-specific criterion just found in the scalp.
4. Asymmetry of color and structure, irregular dots and globules, and hypopigmentation charac-
terize this dysplastic nevus.
5. Asymmetry of color and structure, irregular dots and globules, regression, and a milky-red area
characterize this dysplastic nevus.
52 DERMOSCOPY: AN ILLUSTRATED SELF-ASSESSMENT GUIDE

RISK
❑ Low
✔ Intermediate

❑ High

DIAGNOSIS

❑ Nevus
❑ Seborrheic keratosis
❑ Basal cell carcinoma
❑ Vascular
❑ Dermatofibroma
❑ Squamous cell carcinoma
❑ Melanoma

2-12c
❑ Other

DISPOSITION
❑ No intervention
ANSWERS
❑ Follow-up Answers: 2,5
✔ Histopathologic diagnosis
❑ Discussion:
■ As in Case 11, there is not a good clinico-dermoscopic correlation here
■ A red flag for concern.
■ One would expect to see the globular or cobblestone global pattern with
DERMOSCOPIC CRITERIA regular dots and globules or larger, angulated globlules.
■ Dots and globules with different sizes and shapes, scattered throughout the
■ Asymmetry of color and lesion are considered irregular by definition.
structure ■ The grayish-white area of regression is widespread.
■ Irregular dots and globules ■ Multifocal hypopigmentations, not found here, would be one criterion
(circles) commonly found in dysplastic nevi.
■ Regression/whitish gray color ■ There are no site-specific criteria found only on the scalp that are not found
(white arrows) elsewhere on the body.
■ Milky-red area/pink color (box) ■ The milky-red area does not contain out-of-focus, globular-like structures.
■ In this case, the milky-red area is nonspecific and is not indicative of
melanoma.

PEARLS
■ Do not be surprised to find nevi in the scalp of children.
■ Do not be surprised to find atypical dermoscopic criteria in this location.
■ In this case if the pathology report was not at least a dysplastic nevus, there
would not be a good dermoscopic–pathologic correlation.
■ When one does not have a good dermoscopic–pathologic correlation, speak
with your pathologist.
■ When one does not have a good dermoscopic–pathologic correlation,
consider getting a pigmented-lesion expert pathologist’s opinion.
Chapter 2 Scalp, Face, Nose, and Ears 53

RISK
❑ Low
❑ Intermediate
❑ High

2-13a
DIAGNOSIS
❑ Nevus
❑ Seborrheic keratosis
❑ Basal cell carcinoma
❑ Vascular
❑ Dermatofibroma
❑ Squamous cell carcinoma
❑ Melanoma
❑ Other

DISPOSITION
❑ No intervention
❑ Follow-up
❑ Histopathologic diagnosis
2-13b

CASE 13
HISTORY
A 78-year-old female had a basal cell carcinoma excised 8 years ago from her scalp. There is now a
new lesion adjacent to the surgery scar.
1. This recurrent basal cell carcinoma is filled with typical arborizing vessels.
2. This melanoma is filled with pinpoint and cork screw vessels.
3. This lesion is filled with hairpin vessels.
4. Hairpin vessels and gray color characterize this irritated seborrheic keratosis.
5. At times, it may be challenging to accurately categorize small vessels but an attempt always
helps.
54 DERMOSCOPY: AN ILLUSTRATED SELF-ASSESSMENT GUIDE

RISK
✔ Low

❑ Intermediate
❑ High

DIAGNOSIS
❑ Nevus

❑ Seborrheic keratosis
❑ Basal cell carcinoma
❑ Vascular
❑ Dermatofibroma
❑ Squamous cell carcinoma
❑ Melanoma

2-13c
❑ Other

DISPOSITION ANSWERS
✔ No intervention
❑ Answers: 3,4,5
❑ Follow-up
❑ Histopathologic diagnosis Discussion:
■ Clinically, but not dermoscopically, this appears to be a basal cell carcinoma.
■ Arborizing vessels are not seen anywhere.
■ Basal cell carcinomas do not always have classic arborizing vessels.
■ This lesion is filled with classic and not-so-classic hairpin vessels.
DERMOSCOPIC CRITERIA ■ There are no thick, irregular hairpin-shaped vessels that can be found in
■ Classic hairpin vessels melanoma.
(black boxes) ■ The grayish color is the clue that there is inflammation.
■ Nonperfect-shaped hairpin ■ The brown color and pinpoint vessels have no diagnostic significance.
vessels (yellow boxes)
■ Pinpoint vessels (circles) PEARLS
■ Linear vessels (red boxes) ■ A recurrent basal cell carcinoma after so many years would be unusual.
■ Gray color (black arrows) ■ A new primary basal cell carcinoma may happen to arise coincidentally in
■ Brown color (yellow arrows) the same area.
■ None of the specific shape designations given to vessels are pathonomonic
of any specific pathology.
■ Inter-observer agreement on the exact shape of blood vessels within a given
lesion might vary with the personal experience of the individual.
■ Polarizing instrumentation, ultrasound gel/hand cleaning gel, and minimal
pressure should be applied to better visualize small vessels.
Chapter 2 Scalp, Face, Nose, and Ears 55

RISK
❑ Low
❑ Intermediate
❑ High

DIAGNOSIS
❑ Nevus
❑ Seborrheic keratosis
❑ Basal cell carcinoma
❑ Vascular

2-14a
❑ Dermatofibroma
❑ Squamous cell carcinoma
❑ Melanoma
❑ Other

DISPOSITION
❑ No intervention
❑ Follow-up
❑ Histopathologic diagnosis
2-14b

CASE 14
HISTORY
A mother was worried about an enlarging growth by the right ear of her six-year-old child.
1. Clinically and dermoscopically this could be an amelanotic melanoma.
2. Clinically and dermoscopically this could be a Spitz nevus.
3. Different shades of pink color and polymorphous vessels are diagnostic of this amelanotic
melanoma.
4. One can see pinpoint, linear, and hairpin vessels.
5. A vascular pattern in a pink lesion is one of the sub-types of Spitz nevi.
56 DERMOSCOPY: AN ILLUSTRATED SELF-ASSESSMENT GUIDE

RISK
❑ Low
❑ Intermediate
✔ High

DIAGNOSIS

❑ Nevus
❑ Seborrheic keratosis
❑ Basal cell carcinoma
❑ Vascular
❑ Dermatofibroma
❑ Squamous cell carcinoma
❑ Melanoma

2-14c
❑ Other

DISPOSITION
❑ No intervention ANSWERS
❑ Follow-up
Answers: 1,2,4,5
✔ Histopathologic diagnosis

Discussion:
■ Amelanotic melanoma is a common subtype in this age group.
■ Different shades of pink color and centrally-located vessels are commonly
found in amelanotic melanoma, yet are not diagnostic.
DERMOSCOPIC CRITERIA
■ There are six dermoscopic patterns seen in Spitz nevi: starburst, globular,
■ Different shades of pink color homogeneous, reticular/black pigment network, vascular, and atypical.
■ White color ■ All of the patterns can be seen in children.
■ Linear vessels (black arrows) ■ This lesion demonstrates the vascular pattern.
■ Pinpoint vessels (boxes) ■ Absence of pigment and vessels creates the pink color.
■ Hairpin vessel (yellow arrow) ■ The milia-like cyst has no diagnostic significance.
■ Milia-like cyst (circle) ■ Structures identical to milia-like cysts can represent mucoid degeneration
and can be seen in melanomas.

PEARLS
■ Dermoscopy is more sensitive and specific to diagnose pigmented lesions.
■ Classic patterns and their variations are commonly found in Spitz nevi.
■ Pink lesions in any age group should raise a red flag for concern.
Chapter 2 Scalp, Face, Nose, and Ears 57

RISK
❑ Low
❑ Intermediate
❑ High

DIAGNOSIS
❑ Nevus
❑ Seborrheic keratosis
❑ Basal cell carcinoma
❑ Vascular
2-15a

❑ Dermatofibroma
❑ Squamous cell carcinoma
❑ Melanoma
❑ Other

DISPOSITION
❑ No intervention
❑ Follow-up
❑ Histopathologic diagnosis
2-15b

CASE 15
HISTORY
An 80-year-old female with a history of recurrent lentigo maligna on the right side of her face
developed a new blue papule by her left ear.
1. This could be a de novo melanoma characterized by irregular blotches, blue-white color, and
regression.
2. This could be a heavily pigmented basal cell carcinoma without arborizing vessels and ulceration.
3. Commonly, basal cell carcinomas have such extensive pigmentation.
4. Uncommonly, basal cell carcinomas have such extensive pigmentation.
5. Without arborizing vessels and spoke-wheel structures, one cannot diagnose a basal cell
carcinoma.
58 DERMOSCOPY: AN ILLUSTRATED SELF-ASSESSMENT GUIDE

RISK
❑ Low
❑ Intermediate
✔ High

DIAGNOSIS
❑ Nevus
❑ Seborrheic keratosis

❑ Basal cell carcinoma
❑ Vascular
❑ Dermatofibroma
❑ Squamous cell carcinoma
❑ Melanoma

2-15c
❑ Other

DISPOSITION
❑ No intervention ANSWERS
❑ Follow-up
Answers: 1,2,4
✔ Histopathologic diagnosis

Discussion:
■ With strict adherence to the definitions of the dermoscopic criteria, it is not
possible to differentiate a basal cell carcinoma from melanoma.
■ The bluish-white color has a dermoscopic differential diagnosis.
DERMOSCOPIC CRITERIA ■ Found in basal cell carcinomas and melanomas.
■ Ulceration (yellow arrows) ■ Islands of pigmented basal cell carcinoma cells.
■ Bluish-white color (stars) ■ Deep dermal heavily pigmented atypical melanocytes.
■ Purplish color (red arrows) ■ The irregular dark color has a dermoscopic differential diagnosis.
■ Ulceration vs irregular blotches.
■ Blotches representing heavily pigmented atypical melanocytes.
■ The reddish color represents vascularization.
■ Most basal cell carcinomas are not so heavily pigmented.
■ The minority of basal cell carcinomas (10 %) have spoke-wheel structures.
■ Basal cell carcinomas do not always have arborizing vessels.

PEARLS
■ In general it is not always possible to be 100% sure of a given diagnosis.
■ Be prepared to create a dermoscopic differential diagnosis.
■ Err on the side of the most aggressive pathology when planning your
surgical approach.
Chapter 2 Scalp, Face, Nose, and Ears 59

RISK
❑ Low
❑ Intermediate
❑ High

DIAGNOSIS
❑ Nevus
❑ Seborrheic keratosis
❑ Basal cell carcinoma
❑ Vascular

2-16a
❑ Dermatofibroma
❑ Squamous cell carcinoma
❑ Melanoma
❑ Other

DISPOSITION
❑ No intervention
❑ Follow-up
❑ Histopathologic diagnosis
2-16b

CASE 16
HISTORY
A barber found this lesion on the scalp of a 61-year-old man.
1. Compared to Case 15, this basal cell carcinoma is easier to diagnose because there are classic
arborizing vessels.
2. A bluish-white veil and irregular blotches plus corkscrew vessels characterize this melanoma.
3. Arborizing vessels are always thick and branch at 90° angles.
4. Typically the arborizing vessels seen in basal cell carcinomas are in-focus because they are
located on the surface of the lesion.
5. In or out-of-focus arborizing vessels can also be found in melanomas.
60 DERMOSCOPY: AN ILLUSTRATED SELF-ASSESSMENT GUIDE

RISK
❑ Low
❑ Intermediate
✔ High

DIAGNOSIS
❑ Nevus
❑ Seborrheic keratosis

❑ Basal cell carcinoma
❑ Vascular
❑ Dermatofibroma
❑ Squamous cell carcinoma
❑ Melanoma

2-16c
❑ Other

DISPOSITION ANSWERS
❑ No intervention Answers: 1,4,5
❑ Follow-up
✔ Histopathologic diagnosis
❑ Discussion:
■ There are no criteria to diagnose a melanocytic lesion (ie, pigment network,
globules), unless one considers the default category.
■ The foci of ovoid brown color are not a criterion that can be used to

diagnose a melanocytic lesion.


DERMOSCOPIC CRITERIA ■ Arborizing vessels plus ulceration are the two major criteria to help make
■ Arborizing vessels (black arrows) the diagnosis.
■ Ulceration (yellow arrows) ■ Thick and thin bored arborizing vessels are classic.
■ The vessels are very thin in this lesion.
■ Bluish-white color (stars)
■ Arborizing vessels can also be found in melanomas but much less frequently
■ Brown color (white arrows)
than in basal cell carcinomas.
■ Out-of-focus vessels should raise a red flag for concern.
■ Out-of-focus arborizing vessels favor a diagnosis of melanoma.
■ It is not necessary to use a protractor to determine the angles of branching
vessels.
■ Angles are not of diagnostic significance.
■ The “classic” bluish-white veil is not present.
■ Blue and white colors are commonly found in basal cell carcinomas.

PEARLS
■ The stool quaic test can be used to differentiate an ulceration with blood
from an irregular melanocytic blotch, which would not contain heme.
■ At any location on the body, do not hesitate to cut off some hair to get a
better view.
Chapter 2 Scalp, Face, Nose, and Ears 61

RISK
❑ Low
❑ Intermediate
❑ High

DIAGNOSIS
❑ Nevus
❑ Seborrheic keratosis
❑ Basal cell carcinoma
❑ Vascular

2-17a
❑ Dermatofibroma
❑ Squamous cell carcinoma
❑ Melanoma
❑ Other

DISPOSITION
❑ No intervention
❑ Follow-up
❑ Histopathologic diagnosis
2-17b

CASE 17
HISTORY
This 88-year-old woman has a biopsy proven basal cell carcinoma under a dark plaque. Clinically,
both lesions seem to be connected.
1. Leaf-like structures fill the pigmented component of this basal cell carcinoma.
2. Asymmetrically pigmented follicular openings are plentiful.
3. A few rhomboid structures can be identified.
4. Irregular blotches with and without follicular openings indicate dermal invasion in this
melanoma.
5. Extensive regression characterizes this heavily pigmented basal cell carcinoma.
62 DERMOSCOPY: AN ILLUSTRATED SELF-ASSESSMENT GUIDE

RISK
❑ Low
❑ Intermediate
✔ High

DIAGNOSIS
❑ Nevus
❑ Seborrheic keratosis
❑ Basal cell carcinoma
❑ Vascular
❑ Dermatofibroma
❑ Squamous cell carcinoma

❑ Melanoma

2-17c
❑ Other

DISPOSITION ANSWERS
❑ No intervention Answers: 2,3,4
❑ Follow-up
✔ Histopathologic diagnosis
❑ Discussion:
■ Clinically and dermoscopically, this is an obvious invasive melanoma.
■ The first step in the progression of melanoma on the face is represented by
asymmetrical follicular pigmentation.
■ The follicular openings should not be confused with milia-like cysts seen

DERMOSCOPIC CRITERIA in seborrheic keratosis.


■ The next step in the progression of melanoma on the face is the formation
■ Follicular openings of rhomboid structures.
(yellow arrows) ■ True “rhomboid” forms (parallelogram with unequal angles and sides) are
■ Asymmetrical follicular not necessary.
pigmentation (black arrows) ■ Any pigmented thickening that surrounds follicular openings should be
■ Rhomboid structures (yellow considered “rhomboid” structures.
box) ■ The irregular black blotch with follicular openings is the first sign of dermal
■ Irregular blotch with follicular invasion.
openings (black box) ■ The irregular black blotch without follicular openings represents complete
■ Irregular blotch without obliteration of follicular openings indicating further dermal invasion.
follicular openings (stars) ■ By definition, this is a collision tumor, representing a basal cell carcinoma
and melanoma.
■ Irregular dots and globules
(yellow circles)
■ Areas of regression (white PEARLS
arrows) ■ One does not need dermoscopy to realize that this is high risk pathology.
■ Polymorphous vessels ■ The diagnosis of melanoma on the face is not always this easy.
(red circles) ■ Dermoscopy is essential to help make the diagnosis in most cases of early
lentigo maligna, which offers the patients their best chance for survivial.
Chapter 2 Scalp, Face, Nose, and Ears 63

RISK
❑ Low
❑ Intermediate
❑ High

DIAGNOSIS
❑ Nevus
❑ Seborrheic keratosis
❑ Basal cell carcinoma

2-18a
Vascular
❑ Dermatofibroma
❑ Squamous cell carcinoma
❑ Melanoma
❑ Other

DISPOSITION
❑ No intervention
❑ Follow-up
❑ Histopathologic diagnosis
2-18b

CASE 18
HISTORY
Over the past few months this asymptomatic pigmented skin lesion on a 74-year-old man has been
increasing in size and getting darker.
1. Milia-like cysts and pseudofollicular openings characterize this seborrheic keratosis.
2. The irregular blotch and areas of regression are important clues that this could be a
melanoma.
3. The multicomponent global pattern, asymmetry of color and structure, irregular dots and
globules, and an irregular blotch plus regression characterize this melanoma.
4. Multiple follicular openings are not identifiable.
5. Based on the history, clinical, and dermoscopic appearance, sequential digital dermoscopic
monitoring is indicated rather than a biopsy.
64 DERMOSCOPY: AN ILLUSTRATED SELF-ASSESSMENT GUIDE

RISK
1
❑ Low
3
❑ Intermediate
✔ High
❑ 2

DIAGNOSIS
❑ Nevus
❑ Seborrheic keratosis
❑ Basal cell carcinoma
❑ Vascular
❑ Dermatofibroma
❑ Squamous cell carcinoma

❑ Melanoma

2-18c
❑ Other

DISPOSITION
❑ No intervention ANSWERS
❑ Follow-up Answers: 2,3,4
✔ Histopathologic diagnosis

Discussion:
■ The history, clinical, and dermoscopic picture are worrisome.
■ There are no criteria to diagnose a seborrheic keratosis.
■ The irregular black blotch could be seen in a heavily pigmented seborrheic
DERMOSCOPIC CRITERIA keratosis.
■ Asymmetry of color and ■ The typical site-specific and melanoma-specific criteria from the face are not
seen.
structure
■ Asymmetrical follicular pigmentation
■ Multicomponent global
■ Annular-granular structures
pattern (1,2,3) ■ Rhomboid structures
■ Irregular dots and globules ■ Circle within a circle
(circles) ■ The regression area is not seen clinically.
■ Irregular black blotch (yellow ■ The regression area consists of blue and white colors plus peppering.
arrows) ■ One can expect to see multiple follicular openings on the face which are not
■ Regression (white arrows) seen here.
■ Peppering (box) ■ Follicular openings can be found outside of the lesion on normal skin.
■ Follicular openings
(black arrows) PEARLS
■ A large dark pigmented lesion on the face should always raise a red flag for
concern.
■ A large facial lesion could be a congenital melanocytic nevus, solar lentigo,
or melanoma.
■ The absence of criteria that favor low risk pathology (ie, fingerprint pattern)
should raise a red flag for concern.
■ If the expected site-specific criteria are absent, use the pattern analysis
algorithm for the trunk and extremities or an algorithm of your choice to
evaluate the lesion.
Chapter 2 Scalp, Face, Nose, and Ears 65

RISK
❑ Low
❑ Intermediate
❑ High

DIAGNOSIS
❑ Nevus
❑ Seborrheic keratosis
❑ Basal cell carcinoma
❑ Vascular

2-19a

Dermatofibroma
❑ Squamous cell carcinoma
❑ Melanoma
❑ Other

DISPOSITION
❑ No intervention
❑ Follow-up
❑ Histopathologic diagnosis
2-19b

CASE 19
HISTORY
A total body skin examination was performed on this 70-year-old man at his first visit with a new
dermatologist. The patient was not aware of the lesion on his right ear lobe that the dermatologist found.
1. This is a classic pigmented seborrheic keratosis that does not need dermoscopic evaluation.
2. In order not to miss melanoma incognito, aka false negative melanomas, one should routinely
examine lesions that do not look atypical clinically.
3. Asymmetry of color and structure, irregular dots, globules and blotches, asymmetrical follicu-
lar pigmentation and regression characterize this melanoma.
4. Symmetry of color and structure, regular dots and globules, and blotches characterize this
small congenital melanocytic nevus.
5. The dramatic asymmetric follicular pigmentation should raise a red flag for concern.
66 DERMOSCOPY: AN ILLUSTRATED SELF-ASSESSMENT GUIDE

RISK
❑ Low
❑ Intermediate
✔ High

DIAGNOSIS
❑ Nevus
❑ Seborrheic keratosis
❑ Basal cell carcinoma
❑ Vascular
❑ Dermatofibroma
❑ Squamous cell carcinoma

❑ Melanoma

2-19c
❑ Other

DISPOSITION ANSWERS
❑ No intervention Answers: 2,3,5
❑ Follow-up
Discussion:
✔ Histopathologic diagnosis
❑ ■ The clinical differential diagnosis includes a seborrheic keratosis or
melanoma.
■ This would be an unusual location for both pathologies.
■ The fingerprint pattern, milia-like cysts, and pseudofollicular openings used
to diagnose a seborrheic keratosis are absent.
DERMOSCOPIC CRITERIA ■ The asymmetrical follicular pigmentation units are dramatic.
■ Asymmetry of color and ■ One should not confuse the follicular openings with milia-like cysts of a
structure seborrheic keratosis.
■ Follicular openings ■ One infers that there is regression based on the clinical appearance of the
(white arrows) lesion.
■ Asymmetrical follicular ■ The regression areas are not well developed because the color is not whiter
pigmentation (black arrows) than the surrounding skin.
■ Asymmetrical follicular ■ Gray color and “peppering” are absent.
■ Criteria can be present yet do not always fit into the exact definitions given
pigmentation units (white
to them.
boxes)
■ Irregular dots and globules
(circles)
PEARLS
■ Irregular dark blotches (black ■ On the face, nose, and ears, it is not always easy to identify the high risk
boxes) criteria.
■ Regression (stars) ■ It is essential to learn the definitions of the site-specific criteria.
■ It is essential to have a color atlas of dermoscopy images, in your mind, of
the important criteria.
■ Focus your attention and make a careful search for the melanoma-specific
criteria.
Chapter 2 Scalp, Face, Nose, and Ears 67

RISK
❑ Low
❑ Intermediate
❑ High

DIAGNOSIS
❑ Nevus
❑ Seborrheic keratosis
❑ Basal cell carcinoma
❑ Vascular

2-20a
❑ Dermatofibroma
❑ Squamous cell carcinoma
❑ Melanoma
❑ Other

DISPOSITION
❑ No intervention
❑ Follow-up
❑ Histopathologic diagnosis
2-20b

CASE 20
HISTORY
A 74-year-old man has this spot on his face, which has not changed for many years.
1. “Moth-eaten” convex borders and pseudopigment network, aka pseudonetwork, characterize
this solar lentigo.
2. “Moth-eaten” concave borders, pseudonetwork, remnants of a fingerprint pattern, and homog-
enous color characterize this solar lentigo.
3. The pigment network and pseudonetwork are both created by elongated hyperpigmented rete
ridges.
4. The pseudonetwork is found only on the face, nose, and ears, and diagnoses melanocytic
lesions.
5. The pseudonetwork can be seen in melanocytic and nonmelanocytic lesions.
68 DERMOSCOPY: AN ILLUSTRATED SELF-ASSESSMENT GUIDE

RISK
✔ Low

❑ Intermediate
❑ High

DIAGNOSIS
❑ Nevus
❑ Seborrheic keratosis
❑ Basal cell carcinoma
❑ Vascular
❑ Dermatofibroma
❑ Squamous cell carcinoma
❑ Melanoma

2-20c
❑ Other

DISPOSITION
✔ No intervention

❑ Follow-up ANSWERS
❑ Histopathologic diagnosis Answers: 2,5

Discussion:
■ The criteria to diagnose melanoma are not seen.
■ Typically, solar lentigines have sharp borders.
DERMOSCOPIC CRITERIA ■ Typically, solar lentigines have concave borders, said to resemble a moth-eaten
garment, known as “moth-eaten borders.”
■ Sharp border demarcation ■ The pigment network found on the trunk and extremities is created by
■ Concave “moth-eaten” borders elongated hyperpigmented rete ridges and is not usually seen on the face,
(black arrows) nose, or ears.
■ Pseudonetwork (boxes) ■ Follicular and other appendageal openings that penetrate areas of pigmentation
■ Homogeneous color (stars) create the pseudonetwork.
■ Remnants of the fingerprint ■ The terms pseudonetwork and pseudopigment network are used
pattern (white arrows) interchangeably.
■ The fingerprint pattern made up of fine, light brown, parallel line segments
resembling finger prints differs from the honeycomb-like line segments of
the pigment network.
■ Well developed or fragments of the fingerprint pattern are commonly seen.
■ In this case the fragments of the fingerprint pattern are very difficult to

see and could be easily overlooked.

PEARLS
■ Despite common teaching, there is no evidence to substantiate that the
pseudonetwork is a primary criterion to diagnose melanocytic lesions.
■ The pseudonetwork is site-specific and can be seen in melanocytic and
nonmelanocytic lesions.
■ One has to identify other criteria to make a diagnosis once the pseudonetwork
is identified.
Chapter 2 Scalp, Face, Nose, and Ears 69

RISK
❑ Low
❑ Intermediate
❑ High

DIAGNOSIS
❑ Nevus
❑ Seborrheic keratosis
❑ Basal cell carcinoma
❑ Vascular

2-21a
❑ Dermatofibroma
❑ Squamous cell carcinoma
❑ Melanoma
❑ Other

DISPOSITION
❑ No intervention
❑ Follow-up
❑ Histopathologic diagnosis
2-21b

CASE 21
HISTORY
The pigmented skin lesion on the face of this 8-year-old boy was scheduled for excision by a plastic
surgeon, under general anesthesia, after the patient was referred by a pediatric dermatologist. The
dermatologist thought that it looked atypical clinically and there was a history of change. The
lesion was never examined with dermoscopy.
1. Asymmetry of color and structure, irregular dots and globules characterizes this dysplastic
nevus.
2. Nodular melanoma is in the clinical and dermoscopic differential diagnosis.
3. Minimal asymmetry of color and structure, foci of pseudopigment network, and heavy
pigmentation characterize this small congenital melanocytic nevus.
4. Asymmetry of color and structure are commonly seen in congenital melanocytic nevi.
5. A history of change in a nevus is not always high risk, especially in a child.
70 DERMOSCOPY: AN ILLUSTRATED SELF-ASSESSMENT GUIDE

RISK
✔ Low

❑ Intermediate
❑ High

DIAGNOSIS

❑ Nevus
❑ Seborrheic keratosis
❑ Basal cell carcinoma
❑ Vascular
❑ Dermatofibroma
❑ Squamous cell carcinoma
❑ Melanoma

2-21c
❑ Other

DISPOSITION
✔ No intervention
❑ ANSWERS
❑ Follow-up Answers: 3,4,5
❑ Histopathologic diagnosis
Discussion:
■ Clinically and dermoscopically this is not high risk.
■ Asymmetry of color and structure are commonly found in congenital
melanocytic nevi of all sizes.
DERMOSCOPIC CRITERIA ■ Other experienced dermoscopists might not agree that there is asymmetry
■ Asymmetry of color and of color and structure.
structure ■ The pseudopigment network is easy to identify and expected in this location.
■ Appendageal openings ■ The terms appendageal and follicular openings can be used interchangeably.
■ At first blush, this lesion is similar to Case 18 however, there are no well-
(arrows)
developed melanoma-specific criteria.
■ Pseudopigment network com-
plex (homogeneous color and
appendageal openings)
PEARLS
■ Regular dots and globules ■ The majority of pigmented skin lesions in children are not high risk.
(boxes) ■ Unnecessary surgery can be avoided in most children if dermoscopy is used.
■ No child should ever have a pigmented skin lesion biopsied or removed
before it is examined with dermoscopy.
Chapter 2 Scalp, Face, Nose, and Ears 71

RISK
❑ Low
❑ Intermediate
❑ High

DIAGNOSIS
❑ Nevus
❑ Seborrheic keratosis

2-22a
❑ Basal cell carcinoma
❑ Vascular
❑ Dermatofibroma
❑ Squamous cell carcinoma
❑ Melanoma
❑ Other

DISPOSITION
❑ No intervention
❑ Follow-up
❑ Histopathologic diagnosis
2-22b

CASE 22
HISTORY
A 65-year-old man was referred by another dermatologist to evaluate this lesion with dermoscopy
to determine if it should be biopsied. This history of the lesion is not known by the patient.
1. Clinically and dermoscopically a solar lentigo and in situ melanoma are in the differential
diagnosis.
2. There are no high risk dermoscopic features in this solar lentigo.
3. The linear tracks and globules diagnose a melanoma.
4. Linear tracks and linear globules can be seen in benign and malignant pathology.
5. The “peppering” seen in this lesion indicates that there is a component of trauma.
72 DERMOSCOPY: AN ILLUSTRATED SELF-ASSESSMENT GUIDE

RISK
❑ Low
✔ Intermediate

❑ High
1

DIAGNOSIS 4
2
❑ Nevus
3
❑ Seborrheic keratosis
❑ Basal cell carcinoma
❑ Vascular
❑ Dermatofibroma
❑ Squamous cell carcinoma
❑ Melanoma

2-22c
❑ Other

DISPOSITION ANSWERS
❑ No intervention Answers: 1,4,5
❑ Follow-up
✔ Histopathologic diagnosis
❑ Discussion:
■ Most solar lentigines on the lips have a banal dermoscopic appearance
(ie, homogeneous brown color, fine parallel lines).
■ It is not always possible to differentiate a solar lentigo from in situ
melanoma clinically and with dermoscopy.
DERMOSCOPIC CRITERIA ■ In such situations a biopsy is indicated.
■ Several distinct patterns of criteria can be seen on the lips and in genital
■ Asymmetry of color and
mucosa (nonglabrous skin):
structure ■ Regular or irregular linear globules
■ Multicomponent global pattern ■ Parallel lines of globules sometimes creating a “fingerprint” pattern
(1,2,3,4) ■ Single thin or thick lines
■ Irregular linear globules ■ Two parallel lines the “train track” pattern
(yellow box) ■ Multiple parallel line segments another “fingerprint” pattern

■ Irregular linear lines aka train ■ Thin or thick network-like structures

tracks (arrows) ■ “Peppering” indicates trauma unless other high risk criteria are seen.
■ Irregular thick network ■ Asymmetry of color and structure, a multicomponent global pattern, and
(black box) irregular criteria should raise a red flag for concern.
■ Peppering (white box) ■ Blue-white color, dark irregular blotches, ulceration, and polymorphous
vessels not identified here can be seen in invasive melanoma.

PEARLS
■ In this location surgery can be avoided in most cases after a banal picture
is seen with dermoscopy.
■ Avoiding surgery and the formation of potentially raised scars that can be
very annoying to the patient, is an important benefit of dermoscopy.
■ In gray-zone lesions sequential digital clinical and dermoscopic monitoring
can be used.
■ The most atypical dermoscopic area should be used for an incisional biopsy.
Chapter 2 Scalp, Face, Nose, and Ears 73

RISK
❑ Low
❑ Intermediate
❑ High

DIAGNOSIS
❑ Nevus
❑ Seborrheic keratosis
❑ Basal cell carcinoma
❑ Vascular

2-23a
❑ Dermatofibroma
❑ Squamous cell carcinoma
❑ Melanoma
❑ Other

DISPOSITION
❑ No intervention
❑ Follow-up
❑ Histopathologic diagnosis
2-23b

CASE 23
HISTORY
This 57-year-old man’s main complaint was a slowly enlarging spot on his nose.
1. Arborizing telangiectases must always be present to diagnose a basal cell carcinoma on the
nose.
2. The vessels in this lesion fall within the range of vessels that can be seen in basal cell carcino-
mas.
3. Blue, black, gray, brown, red, and white colors can be seen in basal cell carcinomas.
4. Asymmetry of color and structure, multiple colors, dots and globules, plus telangiectatic vessels
characterize this pigmented basal cell carcinoma.
5. Pigment network, ovoid nets of blue pigment, ulceration, spoke-wheel structures, plus arborizing
vessels are the criteria used to diagnose basal cell carcinomas.
74 DERMOSCOPY: AN ILLUSTRATED SELF-ASSESSMENT GUIDE

RISK
❑ Low
❑ Intermediate
✔ High

DIAGNOSIS
❑ Nevus
❑ Seborrheic keratosis

❑ Basal cell carcinoma
❑ Vascular
❑ Dermatofibroma
❑ Squamous cell carcinoma
❑ Melanoma
❑ Other

DISPOSITION
❑ No intervention

2-23c
❑ Follow-up
✔ Histopathologic diagnosis

ANSWERS
Answers: 2,3,4
DERMOSCOPIC CRITERIA Discussion:
■ Asymmetry of color and ■ Classic thick and thin arborizing vessels are not always found.
structure ■ On occasion, pinpoint, hairpin, linear, kinked, and other shaped telang-
■ Nonarborizing telangiectatic iectatic vessels can be seen with or without arborizing vessels.
“basal cell–like” vessels (black ■ The classic description of pigmentation that can be found in basal cell
arrows) carcinomas includes: large gray-blue ovoid nests, gray-blue dots and globules,
■ Blue globules (yellow arrows) and leaf-like structures.
■ Brown dots and globules (circle) ■ Be aware that there are innumerable variations of size and shape of the
pigmentation found in basal cell carcinomas.
■ Multiple colors—brown, blue,
■ The absence of a pigment network helps to differentiate this from a
white
melanoma.
■ Appendageal openings ■ Pigment network is not seen in basal cell carcinomas.
(red arrows) ■ In order to diagnose a basal cell carcinoma there must not be a pigment
network.

PEARLS
■ It is important to learn the definitions of the types of pigment that can be
found in basal cell carcinomas.
■ It is not essential to perfectly fit pigmentation into the specifically defined
shapes and colors.
■ If one follows the definition of how leaf-like structures should look, one will
realize that they never look like any type of leaf!
■ The concept of leaf-like structures is a misnomer that should be abandoned.
Chapter 2 Scalp, Face, Nose, and Ears 75

RISK
❑ Low
❑ Intermediate
❑ High

DIAGNOSIS
❑ Nevus
❑ Seborrheic keratosis
❑ Basal cell carcinoma

2-24a
Vascular
❑ Dermatofibroma
❑ Squamous cell carcinoma
❑ Melanoma
❑ Other

DISPOSITION
❑ No intervention
❑ Follow-up
❑ Histopathologic diagnosis
2-24b

CASE 24
HISTORY
A 71-year-old man’s barber found this asymptomatic lesion on his scalp.
1. Multiple brown globules categorize this as a melanocytic lesion.
2. The multicomponent global pattern and asymmetry of color and structure are significant red
flags for concern.
3. Irregular dots and globules and a large irregular black blotch are more reason for concern.
4. The blue-white and gray colors of regression points in the direction of a melanoma.
5. Even though streaks and polymorphous vessels are absent, this still could be a melanoma.
76 DERMOSCOPY: AN ILLUSTRATED SELF-ASSESSMENT GUIDE

RISK
❑ Low
❑ Intermediate 1
✔ High

DIAGNOSIS
❑ Nevus 2
❑ Seborrheic keratosis 3
❑ Basal cell carcinoma
❑ Vascular 4

❑ Dermatofibroma
❑ Squamous cell carcinoma

❑ Melanoma

2-24c
Other

DISPOSITION ANSWERS
❑ No intervention Answers: 1,2,3,4,5
❑ Follow-up
✔ Histopathologic diagnosis
❑ Discussion:
■ It is essential to include the scalp in a total body skin examination.
■ Criteria used to diagnose benign pathology are not seen (ie, cobblestone
pattern, milia-like cysts, brain-like pattern).
■ Superficial spreading and nodular melanoma are typically found on the scalp.
DERMOSCOPIC CRITERIA ■ One should have a systematic check list of criteria that need to be identified.
■ Asymmetry of color and ■ Go over the checklist with each lesion:
■ Step 1: Is the lesion melanocytic or nonmelanocytic?
structure (+)
■ Multicomponent global pattern If the lesion is melanocytic:
■ Step 2: To determine if it is low, intermediate, or high risk using the
(1,2,3,4)
algorithm of your choice.
■ Irregular dots and globules
If using pattern analysis:
(circles) ■ Step 3: What is the global pattern?
■ Irregular black blotch (stars) ■ Step 4: Is there symmetry or asymmetry of color and structure?
■ Regression (arrows) ■ Step 5: Identify the local criteria and determine if they are regular or

irregular, good or bad, low or high risk.


■ Step 6: Put everything together and make a dermoscopic diagnosis.

■ Do not make a dermoscopic diagnosis before identifying all of the criteria in


a lesion.

PEARLS
■ Bright light is essential for a good scalp examination.
■ The entire scalp should be carefully checked when doing a complete skin
examination.
■ The sooner the checklists are memorized, the sooner one is on their way to
becoming a well-organized dermoscopist.
Chapter 2 Scalp, Face, Nose, and Ears 77

RISK
❑ Low
❑ Intermediate
❑ High

DIAGNOSIS
❑ Nevus
❑ Seborrheic keratosis
❑ Basal cell carcinoma
❑ Vascular
2-25a

❑ Dermatofibroma
❑ Squamous cell carcinoma
❑ Melanoma
❑ Other

DISPOSITION
❑ No intervention
❑ Follow-up
❑ Histopathologic diagnosis
2-25b

CASE 25
HISTORY
A 77-year-old man was not aware of the pigmented lesion on his ear.
1. There are multiple asymmetrically pigmented follicular openings.
2. There are foci of rhomboid structures.
3. The irregular black blotches with and without follicular openings indicates dermal invasion of
this lentigo maligna melanoma.
4. The irregular black blotches without follicular openings represents the “black lamella” indicat-
ing this pigmented lesion is benign.
5. If one has good eyesight and can perform a biopsy, dermoscopy is not helpful to diagnose
lentigo maligna and its variants.
78 DERMOSCOPY: AN ILLUSTRATED SELF-ASSESSMENT GUIDE

RISK
❑ Low
❑ Intermediate
✔ High

DIAGNOSIS
❑ Nevus
❑ Seborrheic keratosis
❑ Basal cell carcinoma
❑ Vascular
❑ Dermatofibroma
❑ Squamous cell carcinoma

❑ Melanoma

2-25c
❑ Other

DISPOSITION
ANSWERS
❑ No intervention
Answers: 1,2,3
❑ Follow-up
✔ Histopathologic diagnosis
❑ Discussion:
■ Clinically, but not dermoscopically, this could be a seborrheic keratosis or
melanoma.
■ There are no dermoscopic criteria to diagnose a seborrheic keratosis (ie,
milia-like cysts, pseudofollicular openings, fingerprint pattern).
DERMOSCOPIC CRITERIA ■ The “black lamella” could look like this—a black homogenous blotch with a
■ Asymmetry of color and glazed look.
structure ■ The “black lamella” created by pigmented parakeratosis is not seen in
■ Follicular openings (red melanoma.
arrows) ■ Tape stripping often removes the “black lamella.”
■ Asymmetrical follicular pig- ■ Tape stripping will not remove the irregular black blotch in this melanoma.
■ By now the diagnosis should be relatively easy.
mentation (black arrows)
■ There are many faces of melanoma and the diagnosis will not always be so
■ Irregular black blotch with
easy.
follicular openings (white
arrows)
PEARLS
■ Irregular black blotch without
follicular openings (yellow ■ One should memorize the criteria that are needed to diagnose lentigo
arrows) maligna and lentigo maligna melanoma.
■ One should have images of the classic examples of the high risk criteria
■ Rhomboid structures (white
cemented in their minds.
boxes)
■ Focus ones attention to make the diagnosis.
Chapter 2 Scalp, Face, Nose, and Ears 79

RISK
❑ Low
❑ Intermediate
❑ High

DIAGNOSIS
❑ Nevus
❑ Seborrheic keratosis
❑ Basal cell carcinoma
❑ Vascular

2-26a
❑ Dermatofibroma
❑ Squamous cell carcinoma
❑ Melanoma
❑ Other

DISPOSITION
❑ No intervention
❑ Follow-up
❑ Histopathologic diagnosis
2-26b

CASE 26
HISTORY
A 76-year-old man has several gray scaly spots on his face that are sensitive to touch.
1. The clinical impression of a melanoma with regression is supported by the dermoscopic
picture.
2. The gray pseudonetwork seen here is created by confluent annular-granular structures.
3. Annular-granular structures can be seen in pigmented actinic keratosis, lichen-planus keratosis,
and lentigo maligna.
4. Annular-granular structures are the only criterion that is 100% diagnostic of lentigo maligna.
5. Multiple scaly lesions favor a diagnosis of pigmented actinic keratosis.
80 DERMOSCOPY: AN ILLUSTRATED SELF-ASSESSMENT GUIDE

RISK
❑ Low
✔ Intermediate

❑ High

DIAGNOSIS
❑ Nevus
❑ Seborrheic keratosis
❑ Basal cell carcinoma
❑ Vascular
❑ Dermatofibroma
❑ Squamous cell carcinoma
❑ Melanoma

2-26c

❑ Other

DISPOSITION
❑ No intervention ANSWERS
❑ Follow-up Answers: 2,3,5
✔ Histopathologic diagnosis

Discussion:
■ Annular-granular structures are created by fine gray or brown dots that
surround follicular openings.
■ When gray they can represent melanophages and/or free melanin or atypical
DERMOSCOPIC CRITERIA melanocytes.
■ Follicular openings (arrows) ■ Annular-granular structures can be localized or diffuse.
■ Annular-granular structures ■ Confluent annular-granular structure creates the gray pseudonetwork.
(circles) ■ The differential diagnosis in this case includes: pigmented actinic keratosis,
■ Gray pseudonetwork (box) lichen planus keratosis, and lentigo maligna.
■ Clinical sensitivity often expressed by patients favors the diagnosis of
■ Brown color (star)
pigmented actinic keratosis.
■ Diffuse “peppering” and remnants of an actinic/solar lentigo or flat sebor-
rheic keratosis (fingerprint pattern) are characteristic of a chronic long
standing lichen planus-like keratosis.

PEARLS
■ Pigmented actinic keratosis and solar lentigines can be associated with lentigo
maligna.
■ Look for other criteria associated with lentigo maligna before making a final
diagnosis of a pigmented actinic keratosis.
■ Use the most atypical area for an incisional biopsy.
■ A rough scaly pigmented area is a clue that a lesion with this dermoscopic
picture is a pigmented actinic keratosis.
Chapter 2 Scalp, Face, Nose, and Ears 81

RISK
❑ Low
❑ Intermediate
❑ High

DIAGNOSIS
❑ Nevus
❑ Seborrheic keratosis
❑ Basal cell carcinoma
❑ Vascular

2-27a
❑ Dermatofibroma
❑ Squamous cell carcinoma
❑ Melanoma
❑ Other

DISPOSITION
❑ No intervention
❑ Follow-up
❑ Histopathologic diagnosis
2-27b

CASE 27
HISTORY
A 68-year-old man has an asymptomatic spot on his nose that was discovered by his wife.
1. This barely visible tan macule does not warrant dermoscopic examination.
2. Multiple milia-like cysts and pseudofollicular openings diagnose a seborrheic keratosis.
3. Asymmetry of color and structure are a red flag for concern.
4. Asymmetrical follicular pigmentation is another red flag for concern.
5. For cosmetic reasons this lesion could be treated with liquid nitrogen without a biopsy.
82 DERMOSCOPY: AN ILLUSTRATED SELF-ASSESSMENT GUIDE

RISK
❑ Low
❑ Intermediate
✔ High

DIAGNOSIS
❑ Nevus
❑ Seborrheic keratosis
❑ Basal cell carcinoma
❑ Vascular
❑ Dermatofibroma
❑ Squamous cell carcinoma

❑ Melanoma

2-27c
❑ Other

DISPOSITION
❑ No intervention ANSWERS
❑ Follow-up Answers: 3,4
✔ Histopathologic diagnosis

Discussion:
■ Compared with all of the cases with the same diagnosis, the dermoscopic
picture is not dramatic.
■ One should not confuse the follicular openings with milia-like cysts of a
DERMOSCOPIC CRITERIA seborrheic keratosis.
■ The overall picture is that of an irregular pseudonetwork.
■ Asymmetry of color and
■ A uniform pseudonetwork would be made up of light brown homogeneous
structure color studded with monomorphous round follicular openings. There would
■ Different shades of brown and not be asymmetrical follicular pigmentation.
gray color ■ One must focus attention to find the asymmetrically pigmented follicular
■ Follicular openings (black openings.
circles) ■ Use the area with asymmetrically pigmented follicular openings if an incisional
■ Asymmetrical follicular biopsy is performed.
pigmentation (boxes) ■ The criteria for a solar lentigo (fingerprint pattern) are absent.
■ Irregular globules (yellow circle) ■ Poorly defined borders seen here do not favor a solar lentigo.
■ Poorly defined border (black ■ Sharp concave “moth-eaten” border favors a solar lentigo.
arrows)
PEARLS
■ Banal appearing pigmentation can represent lentigo maligna.
■ Dermoscopy should not only be used to examine atypically pigmented skin
lesions in order not to miss melanoma incognito.
■ In this clinical scenario, an important clue is the absence of the fingerprint
pattern. Immediately one should think this could be lentigo maligna and
carefully search for the diagnostic criteria.
■ This case points out the benefits of dermoscopy to diagnose early lentigo
maligna.
■ Use a Woods light to determine the extent of the pigmentation.
Chapter 2 Scalp, Face, Nose, and Ears 83

RISK
❑ Low
❑ Intermediate
❑ High

DIAGNOSIS
❑ Nevus
❑ Seborrheic keratosis
❑ Basal cell carcinoma
❑ Vascular

2-28a
❑ Dermatofibroma
❑ Squamous cell carcinoma
❑ Melanoma
❑ Other

DISPOSITION
❑ No intervention
❑ Follow-up
❑ Histopathologic diagnosis
2-28b

CASE 28
HISTORY
A 36-year-old woman has this brown spot on her face for a few years. It is asymptomatic and there
is no history of change.
1. Different shades of brown and gray colors plus asymmetry of color and structure are a red flag
for concern.
2. The uniform pseudonetwork rules out lentigo maligna.
3. There are only two asymmetrically pigmented follicular openings.
4. The light areas have a differential diagnosis that includes hypopigmentation, areas of
inflammation, or regression.
5. Well developed “moth-eaten” borders and fingerprint structures diagnose a solar lentigo.
84 DERMOSCOPY: AN ILLUSTRATED SELF-ASSESSMENT GUIDE

RISK
❑ Low
❑ Intermediate
✔ High

DIAGNOSIS
❑ Nevus
❑ Seborrheic keratosis
❑ Basal cell carcinoma
❑ Vascular
❑ Dermatofibroma
❑ Squamous cell carcinoma

❑ Melanoma

2-28c
❑ Other

DISPOSITION
❑ No intervention ANSWERS
❑ Follow-up Answers: 1,3,4
✔ Histopathologic diagnosis

Discussion:
■ The dermoscopic picture is not perfectly banal appearing.
■ A uniform pseudonetwork is not seen.
■ One cannot determine if there are classic “moth-eaten” borders and sharp
demarcation.
DERMOSCOPIC CRITERIA ■ There are only two asymmetrically pigmented follicular openings (a clue
■ Asymmetry of color and that this could be lentigo maligna) that even experienced dermoscopist might
structure not see.
■ The hypopigmented areas have a grayish tinge, which suggests that there is
■ Different shades of brown and
inflammation.
gray color
■ True regression is not seen.
■ Follicular openings (yellow ■ Well-formed site-specific melanoma- specific criteria are absent.
boxes) ■ The irregular dots and globules are a minor criterion that has no diagnostic
■ Asymmetrical follicular significance.
pigmentation (black boxes)
■ Irregular dots and globules PEARLS
(circles)
■ Sequential short-term (3 months) digital clinical and dermoscopic monitor-
■ Multifocal hypopigmentation
ing would be one alternative to a biopsy in this cosmetically important area.
(stars) ■ One must be as knowledgeable as possible with difficult cases like this, in
■ Moth-eaten borders (arrows) order to form a meaningful differential diagnosis.
■ Look carefully for high risk criteria before making a benign dermoscopic
diagnosis.
■ Only subtle dermoscopic clues helped us diagnose this melanoma incognito
in the form of lentigo maligna!
Chapter 2 Scalp, Face, Nose, and Ears 85

RISK
❑ Low
❑ Intermediate
❑ High

DIAGNOSIS
❑ Nevus
❑ Seborrheic keratosis
❑ Basal cell carcinoma
❑ Vascular
❑ Dermatofibroma
❑ Squamous cell carcinoma
❑ Melanoma
❑ Other

DISPOSITION
2-29a
❑ No intervention
❑ Follow-up
❑ Histopathologic diagnosis

CASE 29
HISTORY
A less-experienced colleague e-mails you this image and wants to know if you think a biopsy is
indicated. A history and clinical image were not supplied.
1. There are annular-granular structures.
2. There is asymmetry of color and structure.
3. There is asymmetrical follicular pigmentation.
4. The borders are poorly defined.
5. The differential diagnosis includes a pigmented actinic keratosis vs lentigo maligna.
86 DERMOSCOPY: AN ILLUSTRATED SELF-ASSESSMENT GUIDE

RISK
❑ Low
❑ Intermediate
✔ High

DIAGNOSIS
❑ Nevus
❑ Seborrheic keratosis
❑ Basal cell carcinoma
❑ Vascular
❑ Dermatofibroma
❑ Squamous cell carcinoma

❑ Melanoma

2-29b
❑ Other

DISPOSITION
ANSWERS
❑ No intervention
Answers: 1,2,3,4,5
❑ Follow-up
✔ Histopathologic diagnosis
❑ Discussion:
■ The striking dermoscopic findings are the annular-granular structures and
asymmetrical follicular pigmentation.
■ Once again there is an irregular pseudonetwork indicating the lesion is on
the face.
DERMOSCOPIC CRITERIA ■ The annular-granular structures and asymmetrical follicular pigmentation
■ Asymmetry of color and are together in this lesion and are enough criteria to diagnose lentigo
maligna.
structure
■ Interobserver agreement might be in conflict regarding the presence or
■ Different shades of tan and
absence of “moth-eaten” borders.
gray colors
■ Your advice to your colleague would include the differential diagnosis:
■ Follicular openings (black pigmented actinic keratosis vs lentigo maligna.
arrows) ■ Your colleague should biopsy the area with annular-granular structures and
■ Asymmetrical follicular asymmetrical follicular pigmentation.
pigmentation (boxes)
■ Annular-granular structures PEARLS
(circles)
■ With knowledge and experience one can come up with a diagnosis, differen-
■ “Moth-eaten” borders tial diagnosis, and plan without a history or clinical image just based on the
(red arrows) dermoscopic picture.
■ Make sure that your pathologist is a pigmented lesion expert.
■ The histopathology of an actinic keratosis could be confused with lentigo
maligna.
■ If one thinks the lesion is a lentigo maligna but the biopsy reports an actinic
keratosis biopsy another area.
Chapter 2 Scalp, Face, Nose, and Ears 87

RISK
❑ Low
❑ Intermediate
❑ High

DIAGNOSIS
❑ Nevus
❑ Seborrheic keratosis
❑ Basal cell carcinoma
❑ Vascular

2-30a
❑ Dermatofibroma
❑ Squamous cell carcinoma
❑ Melanoma
❑ Other

DISPOSITION
❑ No intervention
❑ Follow-up
❑ Histopathologic diagnosis
2-30b

CASE 30
HISTORY
Two dermatologists saw this lesion and said it was nothing to worry about. They did not examine it
with dermoscopy.
1. The roundish white structures could be milia-like cysts.
2. The roundish white structures could be follicular openings.
3. “Moth-eaten” borders diagnose a solar lentigo.
4. Irregular dots and globules are a red flag for concern.
5. Foci of asymmetrical follicular pigmentation are present.
88 DERMOSCOPY: AN ILLUSTRATED SELF-ASSESSMENT GUIDE

RISK
❑ Low
❑ Intermediate
✔ High

DIAGNOSIS
❑ Nevus
❑ Seborrheic keratosis
❑ Basal cell carcinoma
❑ Vascular
❑ Dermatofibroma
❑ Squamous cell carcinoma

❑ Melanoma Lentigo Maligna

2-30c
❑ Other Solar Lentigo

DISPOSITION ANSWERS
❑ No intervention Answers: 1,2,4,5
❑ Follow-up
✔ Histopathologic diagnosis
❑ Discussion:
■ A large solitary lesion on the face with different shades of brown color are
the first red flags for concern.
■ Now is the time to focus one’s attention and look for high risk criteria

associated with lentigo maligna.


DERMOSCOPIC CRITERIA ■ The differential diagnosis of the round structures includes follicular open-
■ Asymmetry of color and ings versus milia-like cysts.
structure ■ The entire picture is that of an irregular pseudonetwork filled with follicular
■ Different shades of brown openings.
color ■ With foci of criteria associated with lentigo maligna, the “moth-eaten”
borders are not enough to diagnose a large solar lentigo.
■ Follicular openings
■ Knowing that there is an association between solar lentigo and lentigo
(black arrows)
maligna was important in this case.
■ Annular-granular structures ■ Two biopsies were performed for academic reasons.
(circle) ■ The right side without any high risk criteria diagnosed a solar lentigo.
■ Asymmetrical follicular ■ The left side diagnosed lentigo maligna.
pigmentation (boxes) ■ The biopsy was performed in the darker area where asymmetrical follicular
■ “Moth-eaten” borders pigmentation and annular-granular structures were seen with dermoscopy.
(red arrows)
PEARLS
■ If one cannot differentiate a seborrheic keratosis from a melanocytic lesion
based on the dermoscopy, look and palpate the lesion to help make the dif-
ferentiation.
■ Putting as much of the clinical picture together as possible is the best way to
make the correct diagnosis.
Chapter 2 Scalp, Face, Nose, and Ears 89

RISK
❑ Low
❑ Intermediate
❑ High

DIAGNOSIS
❑ Nevus
❑ Seborrheic keratosis
❑ Basal cell carcinoma
❑ Vascular
❑ Dermatofibroma
❑ Squamous cell carcinoma
2-31a

❑ Melanoma
❑ Other

DISPOSITION
❑ No intervention
❑ Follow-up
❑ Histopathologic diagnosis
2-31b

CASE 31
HISTORY
A 55-year-old man thought that a lesion on his forehead was getting darker.
1. Pseudofollicular openings, peppering, and remnants of the fingerprint pattern characterize this
irritated seborrheic keratosis.
2. The gray pseudonetwork diagnoses a pigmented actinic keratosis.
3. Multiple circles within a circle are a red flag for concern and diagnose lentigo maligna.
4. Asymmetrical follicular pigmentation could be confused with circle within a circle structures.
5. Multiple milia-like cysts are enough to diagnose a seborrheic keratosis.
90 DERMOSCOPY: AN ILLUSTRATED SELF-ASSESSMENT GUIDE

RISK
❑ Low
❑ Intermediate
✔ High

DIAGNOSIS
❑ Nevus
❑ Seborrheic keratosis
❑ Basal cell carcinoma
❑ Vascular
❑ Dermatofibroma
❑ Squamous cell carcinoma

❑ Melanoma

2-31c
❑ Other

DISPOSITION
❑ No intervention
ANSWERS
❑ Follow-up Answers: 3,4
✔ Histopathologic diagnosis

Discussion:
■ At first blush, the striking asymmetry of color and structure and different
shades of brown and gray color should be a red flag for concern.
■ Annular-granular structures (atypical melanocytes) vs peppering
DERMOSCOPIC CRITERIA (melanophages and/or free melanin) seen throughout the lesion is the
differential diagnosis for the gray color—another red flag for concern
■ Asymmetry of color and ■ Circle within a circle is a poorly studied unique structure composed of a
structure central hair shaft and gray pigmentation (atypical melanocytes and/or
■ Different shades of brown and peppering) that surround the hair shaft.
gray color ■ More work needs to be done to study this criterion to better delineate the
■ Hair shafts (black arrows) histopathologic correlation.
■ Follicular pigmentation (yellow ■ The differential diagnosis for the circle within a circle would be asymmetri-
arrows) cal follicular pigmentation, pseudofollicular openings, and milia-like cysts.
■ Circle within a circle units ■ The central hair shaft differentiates the circle within a circle from follicular
(black boxes) openings with pigmentation.
■ Asymmetrical follicular ■ A focus of well-developed fingerprint pattern (parallel line segments) makes
the dermoscopic association of lentigo maligna and solar lentigo.
pigmentation (white box)
■ Fingerprint pattern (red box)
■ Peppering (yellow box) PEARLS
■ Gray color (stars) ■ The German philosopher Johann Wolfgang Goethe once wrote “One cannot
see what one does not know.” (In German of course!)
■ The concept of circle within a circle is not well known.
■ Circle within a circle might be the only criterion seen to diagnose lentigo
maligna.
■ Mastering dermoscopy by “osmosis” cannot be done.
■ Put in the time to learn as much dermoscopy as possible.
■ The learning curve is steep!
Chapter 2 Scalp, Face, Nose, and Ears 91

RISK
❑ Low
❑ Intermediate
❑ High

DIAGNOSIS
❑ Nevus
❑ Seborrheic keratosis
❑ Basal cell carcinoma
❑ Vascular

2-32a
❑ Dermatofibroma
❑ Squamous cell carcinoma
❑ Melanoma
❑ Other

DISPOSITION
❑ No intervention
❑ Follow-up
❑ Histopathologic diagnosis
2-32b1

CASE 32
HISTORY
These two lesions were found on the forehead of a 70-year-old man with a history of actinic
keratosis, basal, and squamous cell carcinoma.

LESION 1
1. Globules diagnose a melanocytic lesion.
2. Irregular pigmented pseudofollicular openings and milia-like cysts diagnose a seborrheic
keratosis.
3. On close inspection there is significant asymmetry of color and structure.
4. The blue-white color and irregular dark blotches are a red flag for concern.
5. This could not be a melanoma because the typical site-specific and melanoma-specific criteria
are not seen.
92 DERMOSCOPY: AN ILLUSTRATED SELF-ASSESSMENT GUIDE

RISK
❑ Low
❑ Intermediate
✔ High
❑ 1

2
DIAGNOSIS
❑ Nevus
❑ Seborrheic keratosis
❑ Basal cell carcinoma
3
❑ Vascular
❑ Dermatofibroma
❑ Squamous cell carcinoma

❑ Melanoma

2-32b2
❑ Other

DISPOSITION
❑ No intervention
ANSWERS
❑ Follow-up
✔ Histopathologic diagnosis
❑ Answers: 1,3,4

Discussion:
■ At first blush this looks more like a seborrheic keratosis than melanoma.
■ The irregular dots and globules could be confused with pigmented
pseudofollicular openings characteristic of a seborrheic keratosis.
DERMOSCOPIC CRITERIA ■ A melanoma on the face with a cobblestone-like pattern is very unusual.
■ Asymmetry of color and ■ There are no well-developed site-specific and melanoma-specific criteria
structure to diagnose this in situ melanoma.
■ Multicomponent global pattern ■ The bizarre-shaped blotches (yellow arrows) favor the diagnosis of irregular
(1,2,3) vs pseudofollicular openings of a seborrheic keratosis.
■ Irregular cobblestone global ■ The bluish-white color and large dark blotch could be found in a seborrheic
pattern (irregular, large keratosis.
angulated globules) ■ This difficult case could lead to a misdiagnosis of a seborrheic keratosis
with the initiation of inappropriate destructive therapy before making a
■ Irregular dots and globules
histopathologic diagnosis.
(circles)
■ Irregular blotches (star and
yellow arrows)
PEARLS
■ Blue-white color (white arrows) ■ This lesion points out the need for a differential diagnosis for global patterns,
■ Asymmetrical follicular local criteria, and the potential diagnosis.
pigmentation (red arrows) ■ If destructive therapy in a lesion like this is not successful, it is better to
make a histopathologic diagnosis as soon as possible rather than carrying
out additional blind destructive therapies.
■ Dermoscopy is not a perfect technique. Don’t expect it to be!
Chapter 2 Scalp, Face, Nose, and Ears 93

RISK
❑ Low
❑ Intermediate
❑ High

DIAGNOSIS
❑ Nevus
❑ Seborrheic keratosis
❑ Basal cell carcinoma
❑ Vascular
❑ Dermatofibroma
❑ Squamous cell carcinoma
❑ Melanoma
2-32c1 ❑ Other

DISPOSITION
❑ No intervention
❑ Follow-up
❑ Histopathologic diagnosis

LESION 2
1. The classic pseudonetwork diagnoses a solar lentigo.
2. Ulceration and pigmentation characterize this basal cell carcinoma.
3. Different shades of pink and brown are a red flag for concern.
4. Several asymmetrically pigmented follicular openings are an important clue that this could be
another melanoma.
5. This could be a feature-poor melanoma.
94 DERMOSCOPY: AN ILLUSTRATED SELF-ASSESSMENT GUIDE

RISK
❑ Low
❑ Intermediate
✔ High

DIAGNOSIS
❑ Nevus
❑ Seborrheic keratosis
❑ Basal cell carcinoma
❑ Vascular
❑ Dermatofibroma
❑ Squamous cell carcinoma

❑ Melanoma

2-32c2
❑ Other

DISPOSITION
❑ No intervention
❑ Follow-up ANSWERS
✔ Histopathologic diagnosis
❑ Answers: 3,4,5

Discussion:
■ The pink color is the first clue of potentially high risk pathology.
■ “If there’s pink, stop and think !”

DERMOSCOPIC CRITERIA ■ Two completely different, melanomas in the same patient appearing side by
side, is extremely unusual.
■ Different shades of pink and ■ Complete clinical and histopathologic separation rules out a single
brown color melanoma.
■ Regression (stars) ■ The lesion is feature-poor because there are not well-defined high risk criteria.
■ Asymmetrical follicular pig- ■ This feature-poor pink lesion turned out to be an early invasive
mentation (yellow boxes) melanoma!
■ Irregular dots and globules ■ The asymmetrically pigmented follicular openings could be missed even
(blaick box) by the most experienced dermoscopist.
■ Scale (red arrows) ■ Different shades of brown and especially pink color are clues to the seriousness
■ Ulceration (black arrows) of this lesion.
■ The irregular dots and globules are a clue that the lesion is melanocytic.
■ The ulceration was created by trauma.

PEARLS
■ Even the most experienced dermoscopists routinely have difficult cases in
which dermoscopy is inconclusive.
■ It is a fact of life that everyone misses or misdiagnoses melanoma.
■ Dermoscopists have a much better chance to avoid this unpleasant situation.
Chapter 3

Trunk and Extremities


General Instructions: You will find a list of True/False statements following each case
history. Select any statements, which you believe to be true. There may be one, more
than one or no true statements for any given case. Choose the correct risk, diagnosis
and disposition for each case. Then, turn the page to find a detailed discussion and
pearls for each case.
Chapter 3 Trunk and Extremities 97

RISK
❑ Low
❑ Intermediate
❑ High

DIAGNOSIS

3-1a
Nevus
❑ Seborrheic keratosis
❑ Basal cell carcinoma
❑ Vascular
❑ Dermatofibroma
❑ Squamous cell carcinoma
❑ Melanoma
❑ Other

DISPOSITION
❑ No intervention
❑ Follow-up
❑ Histopathologic diagnosis
3-1b

CASE 1
HISTORY
A 32-year-old man has many small banal appearing nevi on his trunk and extremities. A nevus on
his left arm looked different than the others.
1. This is a melanocytic lesion because there is a pigment network.
2. It has a reticular global pattern.
3. There is symmetry of color and structure.
4. A uniform, regular pigment network fills most of the lesion.
5. Multifocal hypopigmentation and irregular pigment network diagnose a moderately dysplastic
nevus.
98 DERMOSCOPY: AN ILLUSTRATED SELF-ASSESSMENT GUIDE

RISK
✔ Low

❑ Intermediate
❑ High

DIAGNOSIS

❑ Nevus
❑ Seborrheic keratosis
❑ Basal cell carcinoma
❑ Vascular
❑ Dermatofibroma
❑ Squamous cell carcinoma
❑ Melanoma
❑ Other

3-1c
DISPOSITION ANSWERS
✔ No intervention
❑ Answers: 1,2,3,4
❑ Follow-up Discussion:
❑ Histopathologic diagnosis ■ Melanoma is in the clinical differential diagnosis because there are slightly
irregular borders, several dark colors, and it is an “ugly duckling.”
■ One will not see the entire picture until the lesion is examined with
dermoscopy. It could have looked much worse!
DERMOSCOPIC CRITERIA ■ The “ugly duckling” is a pigmented skin lesion that clinically and/or dermo-
scopically is different from the other lesions on a patient.
■ Regular pigment network ■ An “ugly duckling” lesion is a red flag for concern; however, it is not always
(black boxes) high risk.
■ Irregular pigment network ■ The dermoscopic picture seen here should give one reassurance that this is
(white boxes) not a melanoma.
■ Multifocal hypopigmentation ■ By definition this is the reticular global pattern because the lesion is made
(black arrows) up of pigment network.
■ There are foci of irregular pigment network with broadened, branched, and
broken up line segments.
■ Irregular pigment network is not always associated with high risk pathology.
■ Multifocal hypopigmentation and irregular pigment network can be found
in dysplastic nevi.
■ The hypopigmentaion could represent normal skin that commonly is found
in melanocytic nevi.
■ There are no well-developed melanoma-specific criteria, aka high risk criteria.

PEARLS
■ If in doubt cut it out.
■ With experience one will become less and less concerned with this entire
clinical scenario.
■ Digital clinical and dermoscopic monitoring for changes over time is a
cutting-edge way to handle this case.
■ An experienced dermoscopist would find this lesion perfectly banal appearing.
Chapter 3 Trunk and Extremities 99

RISK
❑ Low
❑ Intermediate
❑ High

DIAGNOSIS
❑ Nevus
❑ Seborrheic keratosis
❑ Basal cell carcinoma
❑ Vascular

3-2a
❑ Dermatofibroma
❑ Squamous cell carcinoma
❑ Melanoma
❑ Other

DISPOSITION
❑ No intervention
❑ Follow-up
❑ Histopathologic diagnosis
3-2b

CASE 2
HISTORY
This asymptomatic pigmented skin lesion was found by a dermatologist while performing total body
skin examination.
1. Compared to Case 1, the global dermoscopic picture is more worrisome.
2. There is definite asymmetry of color and structure.
3. Asymmetry of color and structure, irregular blotches, and foci of regression characterize this
in situ melanoma.
4. This could be a dysplastic nevus characterized by asymmetry of color and structure, irregular
pigment network, and multifocal hypopigmentation.
5. Skin markings traversing the lesion are creating the linear hypopigmentation.
100 DERMOSCOPY: AN ILLUSTRATED SELF-ASSESSMENT GUIDE

RISK
✔ Low

❑ Intermediate
❑ High

DIAGNOSIS

❑ Nevus
❑ Seborrheic keratosis
❑ Basal cell carcinoma
❑ Vascular
❑ Dermatofibroma
❑ Squamous cell carcinoma
❑ Melanoma
❑ Other

3-2c
DISPOSITION
✔ No intervention

ANSWERS
❑ Follow-up
❑ Histopathologic diagnosis Answers: 1,2,4,5

Discussion:
■ Compared to Case 1 the global/overall dermoscopic picture is more
worrisome.
DERMOSCOPIC CRITERIA ■ The irregular pigment network is more widespread than in Case 1.
■ Linear hypopigmentation is commonly seen and is created by the normal
■ Asymmetry of color and skin markings that traverse the lesion.
structure (+) ■ The bony-white color of regression is not seen.
■ Irregular pigment network ■ A banal acquired nevus and mildly dysplastic nevus are in this differential
(black boxes) diagnosis.
■ Multifocal hypopigmentation ■ Melanoma is not in the clinical and dermoscopic differential diagnosis.
(black arrows) ■ Multiple similar appearing nevi are less worrisome than a single lesion that
looks like this.

PEARLS
■ For the novice dermoscopist, digital clinical and dermoscopic monitoring
for changes over time would be a cutting-edge way to handle this case.
■ An experienced dermosopist might find this lesion perfectly banal appearing
and find no need for follow-up.
Chapter 3 Trunk and Extremities 101

RISK
❑ Low
❑ Intermediate
❑ High

DIAGNOSIS
❑ Nevus
❑ Seborrheic keratosis
❑ Basal cell carcinoma
❑ Vascular

3-3a
❑ Dermatofibroma
❑ Squamous cell carcinoma
❑ Melanoma
❑ Other

DISPOSITION
❑ No intervention
❑ Follow-up
❑ Histopathologic diagnosis
3-3b

CASE 3
HISTORY
The daughter of this 52-year-old woman noticed an “irregular mole” on her mother’s back.
1. Pigment network and globules identify a melanocytic lesion.
2. There is significant asymmetry of color and structure.
3. A regular pigment network, regular dots and globules plus regular dark blotches diagnose a
congenital melanocytic nevus.
4. Asymmetry of color and structure, irregular dots and globules, irregular dark blotches, plus the
suggestion of regression diagnose a melanoma.
5. One can tell by the colors and structures that this is not a deep melanoma.
102 DERMOSCOPY: AN ILLUSTRATED SELF-ASSESSMENT GUIDE

RISK
❑ Low
❑ Intermediate 1
✔ High

2
DIAGNOSIS 1
❑ Nevus
2
❑ Seborrheic keratosis 2
❑ Basal cell carcinoma 3
❑ Vascular
❑ Dermatofibroma
❑ Squamous cell carcinoma 3

❑ Melanoma
❑ Other

3-3c
DISPOSITION
❑ No intervention ANSWERS
❑ Follow-up Answers: 1,2,4,5
✔ Histopathologic diagnosis

Discussion:
■ Compared to Cases 1 and 2 the dermoscopic picture is worse.
■ There are no irregular dark blotches in Cases 1 and 2.
■ The presence of irregular dark blotches is a major red flag for concern.
DERMOSCOPIC CRITERIA ■ The asymmetry of color and structure, and irregular dark blotches but not
the irregular dots and globules are the main high risk features in this lesion.
■ Asymmetry of color and
■ It is debatable if true regression is present.
structure (+)
■ The grayish color favors regression vs. hypopigmentation.
■ Multicomponent global ■ The global pattern could also be described as reticular-homogeneous with
pattern (1, 2, 3) foci of pigment network and homogeneous featureless dark color.
■ Regular pigment network ■ In situ melanoma and a dysplastic nevus are in the differential diagnosis.
(black boxes) ■ In a flat or slightly raised lesion, the presence of local criteria (ie, pigment net-
■ Irregular dots and globules work, dots and globules) and light plus dark brown plus gray colors are all clues
(circles) that this will not be a deep melanoma. These are all superficially located criteria.
■ Irregular dark blotches ■ Black color represents pigment in the epidermis.

(black arrows) ■ In a nodular lesion, black color can be seen in deeply invasive melanoma

■ Light and dark brown color represents pigment at the dermal-epidermal


■ Regression (stars)
junction.
■ Gray represents pigment in the papillary dermis.

■ As a melanoma gets more invasive, local criteria tend to be less prominent


and one see colors associated with a deeper location in the dermis (ie, blue,
steel blue, pink, red).

PEARLS
■ Clinically and dermoscopically, one can diagnose an in situ melanoma.
■ With this information in hand, one can plan an excision with a 5mm border
that would be adequate for an in situ melanoma, thereby getting the job
done with one rather than typical two surgical procedures needed for most
melanomas.
Chapter 3 Trunk and Extremities 103

RISK
❑ Low
❑ Intermediate
❑ High

DIAGNOSIS
❑ Nevus
❑ Seborrheic keratosis
❑ Basal cell carcinoma
❑ Vascular

3-4a
❑ Dermatofibroma
❑ Squamous cell carcinoma
❑ Melanoma
❑ Other

DISPOSITION
❑ No intervention
❑ Follow-up
❑ Histopathologic diagnosis
3-4b

CASE 4
HISTORY
Two lesions catch your eye as potential melanomas when examining this patient.
1. Pigment network and globules identify a melanocytic lesion.
2. There is asymmetry of color and structure and the multicomponent global pattern.
3. The pigment network, globules, and dark blotches are irregular.
4. Regression fills the lower half of the lesion.
5. Blue and/or white color represents another high risk criterion.
104 DERMOSCOPY: AN ILLUSTRATED SELF-ASSESSMENT GUIDE

RISK
❑ Low
❑ Intermediate
✔ High
❑ 2
1

DIAGNOSIS
❑ Nevus 4
❑ Seborrheic keratosis
❑ Basal cell carcinoma
❑ Vascular
❑ Dermatofibroma 3
❑ Squamous cell carcinoma

❑ Melanoma
❑ Other

3-4c
DISPOSITION ANSWERS
❑ No intervention
Answers: 1,2,3,4,5
❑ Follow-up
✔ Histopathologic diagnosis
❑ Discussion:
■ Step 1 is to determine if the lesion is melanocytic or nonmelanocytic.
■ If melanocytic, Step 2 is to determine if it is high risk (ie, dysplastic nevus or
melanoma).
■ Pattern analysis:
DERMOSCOPIC CRITERIA ■ What is the global pattern?

■ Asymmetry of color and ■ Is there symmetry or asymmetry of color and structure?

structure (+) ■ Identify all of the local criteria.

■ Are the local criteria regular or irregular?


■ Multicomponent global
■ Put everything together and make a dermoscopic diagnosis.
pattern (1, 2, 3, 4)
■ This invasive melanoma has foci with overlapping of criteria (ie, irregular
■ Irregular pigment network
pigment network and irregular dark blotches, regression and remnants of
(boxes)
fine pigment network, bluish-white color over brown color).
■ Irregular dots and globules ■ Bluish-white color but not the classic “bluish-white veil” is present → a
(circles) high risk criterion.
■ Irregular dark blotches (white ■ The irregular dark blotches could be considered hyperpigmentation.
arrows)
■ Blue-white color (stars) PEARL
■ Regression (black arrows)
■ If one wants to be thorough with the analysis of each lesion:
■ Multiple colors
■ Memorize a checklist of things to look for.

■ Go over the checklist with every lesion.

■ Analyze all of the criteria in a lesion before making a diagnosis.


Chapter 3 Trunk and Extremities 105

RISK
❑ Low
❑ Intermediate
❑ High

DIAGNOSIS

3-5a2
❑ Nevus
❑ Seborrheic keratosis
3-5a1

❑ Basal cell carcinoma


❑ Vascular
❑ Dermatofibroma
❑ Squamous cell carcinoma
❑ Melanoma
❑ Other

DISPOSITION
❑ No intervention
❑ Follow-up
❑ Histopathologic diagnosis
3-5b

CASE 5
HISTORY
This was the other suspicious lesion found on back of the patient from the last case.
1. Clinically but not dermoscopically this could be diagnosed as a seborrheic keratosis.
2. Gray dots and globules diagnose a melanocytic lesion.
3. Brown dots and globules diagnose a melanocytic lesion.
4. Pinkish-white color and peppering characterize the area of regression.
5. There are foci of polymorphous vessels; however, they are hard to differentiate from the peppering.
106 DERMOSCOPY: AN ILLUSTRATED SELF-ASSESSMENT GUIDE

RISK
❑ Low 1

❑ Intermediate
✔ High

2
DIAGNOSIS
❑ Nevus
❑ Seborrheic keratosis
❑ Basal cell carcinoma
❑ Vascular 3
3
❑ Dermatofibroma
❑ Squamous cell carcinoma

❑ Melanoma
❑ Other

3-5c
ANSWERS
DISPOSITION
Answers: 1,3,4,5
❑ No intervention
❑ Follow-up Discussion:
✔ Histopathologic diagnosis
❑ ■ Brown, not gray, dots and globules are one criterion used to diagnose a
melanocytic lesion.
■ There are no criteria to suggest that this is a seborrheic keratosis.
■ Regression fills most of the lesion.
■ The gray dots and globules “peppering” plus the pinkish-white color make
DERMOSCOPIC CRITERIA up the area of regression.
■ Asymmetry of color and structure plus the multicomponent global pattern
■ Asymmetry of color and
are easy to identify if one knows how they are defined.
structure (+)
■ Foci of pinpoint and linear vessels (aka the polymorphous vascular pattern)
■ Multicomponent global are hard to differentiate from the peppering.
pattern (1, 2, 3) ■ Gray color favors peppering vs. red color favoring pinpoint vessels.
■ Irregular brown dots and ■ Inter-observer agreement even among expert dermoscopists might differ
globules (black boxes) on the best name for all of the small dots.
■ Regression (stars) ■ This brings up the concept of dermoscopic differential diagnosis.

■ Peppering (red boxes) ■ Peppering with foci of vessels vs pure peppering without vessels.

■ Polymorphous vessels (circles) ■ The hyperkeratosis has no significance.


■ Hyperkeratosis (arrows) ■ Putting everything together one could not diagnose anything other than a
regressive melanoma.

PEARLS
■ One is obligated to examine most of the pigmented skin lesions in a patient
with so many lesions.
■ A good plan with such a difficult case is to have the patient return another
day for a re-examination.
■ Final conclusions should not be made hastily.
■ This patient would benefit from total body photography.
■ This patient would benefit from mapping out her nevi dermoscopically for
sequential monitoring to look for significant changes over time.
Chapter 3 Trunk and Extremities 107

RISK
❑ Low
❑ Intermediate
❑ High

DIAGNOSIS
❑ Nevus
❑ Seborrheic keratosis
❑ Basal cell carcinoma
❑ Vascular

3-6a
❑ Dermatofibroma
❑ Squamous cell carcinoma
❑ Melanoma
❑ Other

DISPOSITION
❑ No intervention
❑ Follow-up
❑ Histopathologic diagnosis
3-6b

CASE 6
HISTORY
This clinically banal looking nevus was found on the back of a 7-year-old girl.
1. There isn’t a good clinico-dermoscopic correlation.
2. Brown peripheral globules identify a melanocytic lesion.
3. The asymmetry of color and structure is a red flag for concern.
4. There are foci of markedly irregular pigment network.
5. Asymmetry of color and structure, irregular pigment network, irregular dots and globules,
multifocal hypopigmentation, and diffuse erythema characterize this dysplastic nevus.
108 DERMOSCOPY: AN ILLUSTRATED SELF-ASSESSMENT GUIDE

RISK
❑ Low
✔ Intermediate

1
❑ High

2
DIAGNOSIS

❑ Nevus 3
❑ Seborrheic keratosis
❑ Basal cell carcinoma
❑ Vascular
❑ Dermatofibroma
❑ Squamous cell carcinoma
❑ Melanoma

3-6c
Other

DISPOSITION ANSWERS
❑ No intervention Answers: 1,2,3,4,5
❑ Follow-up
✔ Histopathologic diagnosis
❑ Discussion:
■ In this age group, a nevus that looks like this clinically should have a globular
or cobblestone pattern with dermoscopy.
■ Since it does not look so, there is not a good clinico-dermoscopic correlation
→ a red flag for concern.
DERMOSCOPIC CRITERIA ■ When there is not a good clinico-dermoscopic correlation, one should
■ Asymmetry of color and proceed with caution and look for high risk criteria.
■ Peripheral dots and globules are routinely found in children and indicate an
structure (+)
actively changing nevus.
■ Multicomponent global
■ The peripheral dots and globules that can be seen in Spitz nevi would be
pattern (1, 2, 3) more numerous.
■ Irregular pigment network ■ Sequential monitoring of a nevus with peripheral dots and globules will
(boxes) show that it enlarges over time.
■ Irregular dots and globules ■ The pigment network is markedly irregular.
(circles) ■ Foci are asymmetrically located.

■ Multifocal hypopigmentation ■ They are irregularly pigmented, broken-up, branched and thickened.

(stars) ■ Diffuse erythema is commonly found in dysplastic nevi.


■ Diffuse erythema ■ Pressure with instrumentation blanched away the erythema revealing
multifocal hypopigmentation.
■ Multifocal hypopigmenation, not to be confused with regression, is
commonly found in dysplastic nevi.
■ This lesion is more benign than malignant looking with a typical pattern
seen in dysplastic nevi.

PEARL
■ In order not to miss incognito lesions (ie, lesions that look banal clinically
but not dermoscopically) such as dysplastic nevi, Spitz nevi, and melanoma:
■ Routinely examine with dermoscopy both banal and atypical appearing

skin lesions.
Chapter 3 Trunk and Extremities 109

RISK
❑ Low
❑ Intermediate
❑ High

DIAGNOSIS
❑ Nevus
❑ Seborrheic keratosis
❑ Basal cell carcinoma
❑ Vascular

3-7a
❑ Dermatofibroma
❑ Squamous cell carcinoma
❑ Melanoma
❑ Other

DISPOSITION
❑ No intervention
❑ Follow-up
❑ Histopathologic diagnosis
3-7b

CASE 7
HISTORY
This lesion was found on the hairy chest of a 56-year-old man.
1. Clinically and dermoscopically this is a typical benign appearing nevus spilus.
2. Clinically and dermoscopically this could be a benign agminated nevus.
3. Pigment network and globules identifies a melanocytic lesion.
4. Irregular pigment network, irregular dots and globules, hyperpigmentation and areas of
regression are the main features of this melanoma.
5. Islands of normal skin plus foci of regular pigment network and regular blotches characterize
this benign congenital melanocytic nevus.
110 DERMOSCOPY: AN ILLUSTRATED SELF-ASSESSMENT GUIDE

RISK
❑ Low
❑ Intermediate
✔ High

DIAGNOSIS
❑ Nevus
❑ Seborrheic keratosis
❑ Basal cell carcinoma
❑ Vascular
❑ Dermatofibroma
❑ Squamous cell carcinoma

❑ Melanoma
❑ Other

3-7c
DISPOSITION ANSWERS
❑ No intervention Answers: 3, 4
❑ Follow-up
✔ Histopathologic diagnosis
❑ Discussion:
■ The tan lentiginous background characteristic of a nevus spilus is not seen.
■ Agminated nevi consisting of multiple grouped similar pigmented lesions
remain in the clinical and dermoscopic differential diagnosis.
■ Agminated nevi have been reported in association with congenital,

DERMOSCOPIC CRITERIA acquired, blue, and Spitz nevi.


■ The prominent irregular pigment network and areas of regression are the
■ Asymmetry of color and main diagnostic features of this early invasive melanoma.
structure ■ The areas of regression are not whiter than the surrounding skin and do not
■ Irregular pigment network contain “peppering.”
(boxes) ■ Criteria do not always conform to exact definitions.

■ Irregular dots and globules ■ Clinically regression is clearly apparent.

(circles) ■ The irregular dots and globules and hyperpigmentation are more high risk
■ Irregular brown pigmentation criteria that help make the diagnosis.
(hyperpigmentation)
■ Multifocal hypopigmentation PEARLS
(white arrows) ■ Dysplastic nevi and melanoma can rarely develop in a nevus spilus.
■ Regression (stars) ■ As a general rule, everything that one evaluates with dermoscopy should be
categorized as being regular or irregular, good or bad, low or high risk.
■ There are no exceptions to this rule.

■ It is essential to study images with the classic examples of all the important
dermoscopic principles.
■ Routinely one encounters variations of the classic morphology of global
patterns and local criteria.
■ Do not hesitate to cut away some hair if you need to get a clearer look in a
hairy area.
Chapter 3 Trunk and Extremities 111

RISK
❑ Low
❑ Intermediate
❑ High

DIAGNOSIS
❑ Nevus
❑ Seborrheic keratosis
❑ Basal cell carcinoma
❑ Vascular

3-8a
❑ Dermatofibroma
❑ Squamous cell carcinoma
❑ Melanoma
❑ Other

DISPOSITION
❑ No intervention
❑ Follow-up
❑ Histopathologic diagnosis
3-8b

CASE 8
HISTORY
A 70-year-old woman was concerned about this changing lesion on her left shin.
1. Historically and clinically, more so than dermoscopically, this is suspicious for a melanoma.
2. There are no dramatic high risk dermoscopic criteria.
3. The asymmetry of color and structure and a focus of an irregular pigment network are not
high risk criteria.
4. Asymmetry of color and structure, irregular pigment network, and multifocal
hypopigmentation are diagnostic of this dysplastic nevus.
5. Different shades of brown color are a red flag for concern.
112 DERMOSCOPY: AN ILLUSTRATED SELF-ASSESSMENT GUIDE

RISK
❑ Low
❑ Intermediate
✔ High

DIAGNOSIS
❑ Nevus
❑ Seborrheic keratosis
❑ Basal cell carcinoma
❑ Vascular
❑ Dermatofibroma
❑ Squamous cell carcinoma

❑ Melanoma
❑ Other

3-8c
DISPOSITION
ANSWERS
❑ No intervention
❑ Follow-up Answers: 1,2,5
✔ Histopathologic diagnosis
❑ Discussion:
■ This is a gray zone lesion that looks more benign than malignant.
■ The history of change is very important.
■ The clinical appearance is more worrisome than the overall dermoscopic
picture.
DERMOSCOPIC CRITERIA ■ There are no dramatic melanoma- specific criteria.
■ Asymmetry of color and ■ Clinically and dermoscopically this looks more like a dysplastic nevus.
structures (+) ■ Different shades of brown are a red flag for concern.
■ Different shades of any color are a red flag for concern.
■ Irregular pigment network
■ Do not confuse areas of hypopigmentation with the bony-white color of
(boxes)
regression.
■ Irregular globules (circle)
■ Multifocal hypopigmentation
(stars)
PEARLS
■ Different shades of brown color ■ The most experienced clinician could miss this in situ melanoma.
■ Sequential clinical and dermoscopic monitoring can be used with gray zone
lesions.
■ Short-term monitoring → check again for changes in 3 to 4 months.
■ Long-term monitoring → check again for changes in 6 months to a year.
Chapter 3 Trunk and Extremities 113

RISK
❑ Low
❑ Intermediate
❑ High

DIAGNOSIS
❑ Nevus
❑ Seborrheic keratosis
❑ Basal cell carcinoma
❑ Vascular

3-9a
❑ Dermatofibroma
❑ Squamous cell carcinoma
❑ Melanoma
❑ Other

DISPOSITION
❑ No intervention
❑ Follow-up
❑ Histopathologic diagnosis
3-9b

CASE 9
HISTORY
A 64-year-old woman had a pigmented lesion on her left lower leg for several years that recently
began to get bigger.
1. Pigment network and brown globules diagnose a melanocytic lesion.
2. There is significant asymmetry of color and structure as well as the multicomponent global
pattern.
3. There is an irregular pigment network and irregular brown not red dots and globules.
4. The nodular component of this melanoma is amelanotic.
5. In a nodular lesion suspicious for melanoma, sequential digital monitoring is contraindicated.
114 DERMOSCOPY: AN ILLUSTRATED SELF-ASSESSMENT GUIDE

1
RISK
❑ Low
❑ Intermediate
✔ High

2

DIAGNOSIS
❑ Nevus
❑ Seborrheic keratosis
❑ Basal cell carcinoma
❑ Vascular
❑ Dermatofibroma
2
❑ Squamous cell carcinoma

❑ Melanoma
❑ Other

DISPOSITION 3
❑ No intervention
❑ Follow-up
✔ Histopathologic diagnosis

3-9c
DERMOSCOPIC CRITERIA ANSWERS
■ Asymmetry of color and Answers: 1,2,3,4,5
structure (+)
■ Multicomponent global Discussion:
pattern (1,2,3) ■ Compared with Case 8, melanoma-specific criteria are much easier to
■ Regular pigment network identify.
■ This invasive melanoma is not mistakable for a benign lesion.
(yellow box)
■ Ask your pathologist to examine additional sections and reevaluate this case,
■ Irregular pigment network
if a benign lesion is reported.
(black boxes) ■ Consider the opinion of a dermato-pathologist with special expertise in
■ Irregular brown dots and pigmented skin lesions if this is not reported to be a melanoma.
globules (circles) ■ There should always be a good dermoscopic-pathologic correlation!
■ Milky-red area (arrows) ■ This pinkish-white area has a dermoscopic differential diagnosis that
■ Amelanotic nodule (stars) includes regression or the amelanotic component of the melanoma.
■ The milky-red area represents neovascularization.

PEARLS
■ A nodular lesion suggestive of a melanoma should be fully excised posthaste,
allowing for pathologic examination of the entire lesion as well as tumor
staging.
■ Sequential monitoring is contraindicated in a nodular lesion that could be a
melanoma.
■ Even a short delay in making the diagnosis of a nodular melanoma could
worsen the patient’s prognosis.
Chapter 3 Trunk and Extremities 115

RISK
❑ Low
❑ Intermediate
❑ High

DIAGNOSIS
❑ Nevus
❑ Seborrheic keratosis
❑ Basal cell carcinoma
❑ Vascular

3-10a
❑ Dermatofibroma
❑ Squamous cell carcinoma
❑ Melanoma
❑ Other

DISPOSITION
❑ No intervention
❑ Follow-up
❑ Histopathologic diagnosis
3-10b

CASE 10
HISTORY
This asymptomatic lesion on a 24-year-old man was discovered by the patient who wanted to know
what it was.
1. This is a nodular amelanotic melanoma arising in a nevus.
2. Milky-red areas and markedly atypical pigment network characterize this melanoma.
3. This is a typical benign collision tumor consisting of a melanocytic nevus and hemangioma.
4. Dermoscopy in general is not helpful to diagnose collision tumors.
5. Benign and malignant pathology in any combination is possible in collision tumors.
116 DERMOSCOPY: AN ILLUSTRATED SELF-ASSESSMENT GUIDE

RISK
✔ Low

❑ Intermediate
❑ High

DIAGNOSIS

❑ Nevus
❑ Seborrheic keratosis
❑ Basal cell carcinoma

❑ Vascular
❑ Dermatofibroma
❑ Squamous cell carcinoma
❑ Melanoma

3-10c
❑ Other

DISPOSITION
ANSWERS
Answers: 3,5
✔ No intervention

❑ Follow-up Discussion:
❑ Histopathologic diagnosis ■ Collision tumors consist of two distinct pathologies within the same lesion.
■ Collision tumors are commonly encountered.
■ The most common tumors found in collision are: seborrheic keratoses,
melanocytic nevi, hemangiomas, basal cell carcinomas, in situ and invasive
squamous cell carcinomas, in situ, invasive amelanotic or pigmented
DERMOSCOPIC CRITERIA melanoma.
■ Regular pigment network ■ Triple collision tumors exist yet are not common (eg, basal cell carcinoma,
(boxes) seborrheic keratosis and clear cell acanthoma).
■ Any combination is possible.
■ Homogeneous red color (stars)
■ Even though the pigment network is thickened and branched, it is uniform
■ Lacunae (circles)
throughout → it is not considered an irregular pigment network.
■ The lacuanae are visible but not well developed.
■ Milky-red areas are in the differential diagnosis and consist of pinkish-white
homogeneous color that can contain red and/or blue globular-like structures.
■ Hemanagiomas typically have sharp in focus vascular spaces.

■ Homogeneous red color along with lacunae can be seen.

■ Milky-red areas typically have out-of-focus/blurry globular-like

structures.
■ At times this distinction cannot be made.

PEARLS
■ As a general rule, identify the criteria in a collision tumor and determine if
they are regular or irregular just as you would do with a noncollision tumor.
■ If there is any suggestion of high risk pathology → make a histopathologic
diagnosis ASAP.
Chapter 3 Trunk and Extremities 117

RISK
❑ Low
❑ Intermediate
❑ High

DIAGNOSIS
❑ Nevus
❑ Seborrheic keratosis
❑ Basal cell carcinoma

3-11a
Vascular
❑ Dermatofibroma
❑ Squamous cell carcinoma
❑ Melanoma
❑ Other

DISPOSITION
❑ No intervention
❑ Follow-up
❑ Histopathologic diagnosis
3-11b

CASE 11
HISTORY
This lesion was found on the arm of a 52-year-old man.
1. This is a benign halo nevus characterized by a cobblestone pattern and surrounding white halo.
2. This is a benign halo hemangioma characterized by well-dermarcated lacunae and
hypopigmentation.
3. The brown color of the lacunae-like structures is a clue that this might be a melanocytic lesion.
4. Clinically and dermoscopically, this could be a melanoma with halo nevus-like regression.
5. Brown, blue, gray, and white colors plus irregular vessels are clues that this is a melanoma.
118 DERMOSCOPY: AN ILLUSTRATED SELF-ASSESSMENT GUIDE

RISK 1
❑ Low
❑ Intermediate
✔ High
❑ 1

DIAGNOSIS 2 1
❑ Nevus
❑ Seborrheic keratosis
❑ Basal cell carcinoma 3
❑ Vascular
❑ Dermatofibroma
❑ Squamous cell carcinoma 1

❑ Melanoma

3-11c
❑ Other

DISPOSITION ANSWERS
❑ No intervention Answers: 3,4,5
❑ Follow-up
✔ Histopathologic diagnosis
❑ Discussion:
■ A halo nevus is in the clinical and dermoscopic differential diagnosis.
■ With an eccentrically located lesion and several high risk dermoscopic criteria,
an invasive melanoma with halo nevus-like regression is a more likely
diagnosis.
DERMOSCOPIC CRITERIA ■ The differential diagnosis of the tumor includes an irregular cobblestone pattern
■ Asymmetry of color and of a melanocytic nevus or lacunae-like vascular structures of a hemangioma.
■ The brown color rules out a vascular lesion.
structure
■ Halo phenomenon can occur around melanocytic and nonmelanocytic
■ Multicomponent global
lesions:
pattern (1,2,3) ■ Congenital, acquired, dysplastic, blue, Spitz nevi, and melanoma.
■ Irregular brown globules ■ Seborrheic keratosis, neurofibroma, basal cell carcinoma.
(white boxes) ■ But not around hemangiomas.
■ Regression (stars) ■ ascular
V lesions typically have white color but within the lesion
■ Gray color (black arrows) (fibrous septae).
■ Irregular blue blotch (yellow ■ Cutaneous metastatic melanoma could have this dermoscopic picture with
box) lacunae-like structures.
■ Irregular vessels (red arrows) ■ Bony–white, blue, and gray color make up the area of regression.
■ The fine gray granules (“peppering”) that can be seen in regression are not
identified.

PEARLS
■ A halo nevus-like lesion in an adult is melanoma until proven otherwise.
■ Halo nevi can be found in children and in most cases are not high risk
lesions.
■ Halo phenomenon in a patient with a history of melanoma can be a clue
that the patient has metastatic melanoma.
■ One should search for irregular criteria before diagnosing a benign halo nevus.
Chapter 3 Trunk and Extremities 119

RISK
❑ Low
❑ Intermediate
❑ High

DIAGNOSIS
❑ Nevus
❑ Seborrheic keratosis
❑ Basal cell carcinoma
❑ Vascular

3-12a
❑ Dermatofibroma
❑ Squamous cell carcinoma
❑ Melanoma
❑ Other

DISPOSITION
❑ No intervention
❑ Follow-up
❑ Histopathologic diagnosis
3-12b

CASE 12
HISTORY
This isolated lesion was thought to be high risk by the referring general practitioner, and was
located on the back of a 34-year-old woman.
1. Milky-red areas and a blue-white veil characterize this nodular-melanoma.
2. Well-dermarcated vascular spaces, fibrous septa, and the absence of criteria seen in a
melanocytic lesion characterize this classic hemangioma.
3. By definition, hemangiomas must have tightly spaced lacunae.
4. Bright red, violaceous, blue, and black colors can be found in hemangiomas.
5. Pyogenic granulomas can have similar lacunae-like structures and a white peripheral border.
120 DERMOSCOPY: AN ILLUSTRATED SELF-ASSESSMENT GUIDE

RISK
✔ Low

❑ Intermediate
❑ High

DIAGNOSIS
❑ Nevus
❑ Seborrheic keratosis
❑ Basal cell carcinoma

❑ Vascular
❑ Dermatofibroma
❑ Squamous cell carcinoma
❑ Melanoma

3-12c
❑ Other

DISPOSITION ANSWERS
✔ No intervention
❑ Answers: 2,4
❑ Follow-up Discussion:
❑ Histopathologic diagnosis ■ Clinically and dermoscopically this is a classic hemangioma.
■ There are no criteria for a melanocytic → there should be no criteria for a
melanocytic lesion.
■ The main criteria are lacunae (aka saccules, lagoons).
■ Lacunae → sharply demarcated, vary in size and distribution round to oval
DERMOSCOPIC CRITERIA structures.
■ Lacunae (boxes) ■ Lacunae can be tightly packed or loosely distributed throughout the lesion.
■ Fibrous septa (white arrows) ■ The color is determined by the location of vascular spaces in the dermis.
■ Fibrous border (black arrows) ■ Bright red (cherry/senile hemangiomas) the vascular spaces are high up

in the papillary dermis.


■ The deeper the vascular spaces, the darker the color (ie, darker shades of

red, violaceous, blue).


■ Black color indicates thrombosis.

■ Individual hemangiomas can have single or multiple colors.


■ Bluish-white or white fibrous septa are commonly seen and should not be
confused with the bluish-white or bony- white color found in melanomas.
■ Milky-red areas in melanomas can have lacunae-like structures but the
borders are not sharply demarcated.
■ Pyogenic granulomas can have septa-like white lines in the lesion and at the
periphery but lack lacunae.
■ Angiokeratomas are hemangioma-like with varying amounts of scale or crust.
■ Lymphangiomas have clear vascular-like spaces.
■ Lymph-hemangiomas have a combination of clear and red vascular spaces.

PEARLS
■ It is essential to determine if the vascular spaces have sharp well-demarcated
borders.
■ It is essential to look for the criteria associated with melanocytic lesions
(eg, melanoma).
Chapter 3 Trunk and Extremities 121

RISK
❑ Low
❑ Intermediate
❑ High

DIAGNOSIS
❑ Nevus
❑ Seborrheic keratosis
❑ Basal cell carcinoma
❑ Vascular

3-13a
❑ Dermatofibroma
❑ Squamous cell carcinoma
❑ Melanoma
❑ Other

DISPOSITION
❑ No intervention
❑ Follow-up
❑ Histopathologic diagnosis
3-13b

CASE 13
HISTORY
This is from the abdomen of a 19-year-old patient, and has been getting “scabby” over a 3-month
period.
1. This nodular melanoma is characterized by an irregular black blotch, ulceration, and
amelanotic milky-red areas.
2. A hemorrhagic crust, remnants of lacunae, and peripheral erythema characterize this
thrombosed hemangioma.
3. This could be a partially thrombosed angiokeratoma.
4. A hemorrhagic crust is never found in nodular melanoma.
5. Black color in a vascular lesion represents thrombosis.
122 DERMOSCOPY: AN ILLUSTRATED SELF-ASSESSMENT GUIDE

RISK
❑ Low
✔ Intermediate

❑ High

DIAGNOSIS
❑ Nevus
❑ Seborrheic keratosis
❑ Basal cell carcinoma

❑ Vascular
❑ Dermatofibroma
❑ Squamous cell carcinoma
❑ Melanoma

3-13c
❑ Other

DISPOSITION ANSWERS
❑ No intervention
Answers: 2,3,5
❑ Follow-up
✔ Histopathologic diagnosis
❑ Discussion:
■ This is a worrisome gray zone lesion that could be a nodular melanoma.
■ The lacunae and homogenous reddish color represent what is left of the
hemangioma.
■ Milky-red areas are in the differential diagnosis of the hemangioma component.
DERMOSCOPIC CRITERIA ■ The peripheral erythema could represent the amelanotic component of a
nodular melanoma.
■ Hemorrhagic crust (aka ■ The criteria seen here, point out the need for one to create a dermoscopic
thrombosis) (stars) differential diagnosis.
■ Lacunae (white arrows) ■ A pyogenic granuloma would not have such a large hemorrhagic crust.
■ Peripheral erythema ■ A thrombosed angiokeratoma could look like this especially since there are
(black arrows) areas of yellow color that represents hyperkeratosis.
■ Melanoma can have ulceration and crust formation.

PEARLS
■ If one has to spend abundant time thinking about what is going on, it is
time to make a histopathologic diagnosis especially if nodular melanoma is
in the differential diagnosis.
■ A positive stool guaiac test might help differentiate the hemorrhagic crust
(guaiac )+from a nonulcerated black blotch in a melanoma (guaiac −)
■ The often dramatic jet-black color in thrombosed vascular lesions is as black
as ever can be seen when using-dermoscopy, with the exception of the black
lamella.
Chapter 3 Trunk and Extremities 123

RISK
❑ Low
❑ Intermediate
❑ High

DIAGNOSIS
❑ Nevus
❑ Seborrheic keratosis
❑ Basal cell carcinoma
❑ Vascular
3-14a

❑ Dermatofibroma
❑ Squamous cell carcinoma
❑ Melanoma
❑ Other

DISPOSITION
❑ No intervention
❑ Follow-up
❑ Histopathologic diagnosis
3-14b

CASE 14
HISTORY
You were consulted because it was felt that this 16-year-old girl has a melanoma.
1. Pigment network and globules diagnose a melanocytic lesion.
2. The dark round structures could be irregular globules or thrombosed lacunae.
3. The reddish oval structures could represent lacunae of a hemangioma or the poorly defined
globules seen in milky-red areas.
4. This melanoma is characterized by asymmetry of color and structure, a multicomponent
global pattern, irregular pigment network, irregular globules, and a blue-white veil.
5. This thrombosed hemangioma is characterized by lacunae, thrombosed lacunae, and the
bluish-white color created by hyperkeratosis.
124 DERMOSCOPY: AN ILLUSTRATED SELF-ASSESSMENT GUIDE

RISK
❑ Low
✔ Intermediate
❑ 3
❑ High
1
2

DIAGNOSIS
❑ Nevus
❑ Seborrheic keratosis
❑ Basal cell carcinoma

❑ Vascular 2
❑ Dermatofibroma 3
❑ Squamous cell carcinoma
❑ Melanoma

3-14c
❑ Other

DISPOSITION ANSWERS
❑ No intervention Answers: 2,3,5
❑ Follow-up
✔ Histopathologic diagnosis Discussion:

■ As in Case 13, the dermoscopic picture is not clear.
■ The global pattern and local criteria have a differential diagnosis → vascular
vs melanocytic.
■ Several well-demarcated lacunae are an important clue that this is a vascular
lesion.
DERMOSCOPIC CRITERIA ■ The presence of the lacunae suggests that the black round structures could be
■ Asymmetry of color and thrombosed lacunae rather than irregular globules of a melanocytic lesion.
structure (+) ■ The milky-red globular-like structures are still lacunae→ lacunae do not
■ Multicomponent global always have sharp borders.
pattern (1,2,3) ■ The histopathologic correlate of the network-like structure is not known.
■ Clinically, the lesion is dry and scaly → this creates the blue color seen by
■ Lacunae (black boxes)
dermoscopy.
■ Thrombosed lacunae (arrows)
■ Blue and/or white color is not always high risk and has a differential diagnosis.
■ Milky-red globular-like ■ A multicomponent global pattern with three or more different areas within
structures (white boxes) a lesion is not diagnostic of a melanoma and can be seen in benign and
■ Network-like structures malignant pathologies (ie, congenital and dysplastic nevi or basal
(yellow box) cell carcinomas).
■ Bluish-white color (stars)
PEARLS
■ One cannot see what one does not know.
■ Study hard to better serve your patients whose lives can depend on your
correct dermoscopic diagnosis.
■ There is a significant learning curve to become a competent dermoscopist.
Chapter 3 Trunk and Extremities 125

RISK
❑ Low
❑ Intermediate
❑ High

DIAGNOSIS
❑ Nevus
❑ Seborrheic keratosis
❑ Basal cell carcinoma
❑ Vascular

3-15a
❑ Dermatofibroma
❑ Squamous cell carcinoma
❑ Melanoma
❑ Other

DISPOSITION
❑ No intervention
❑ Follow-up
❑ Histopathologic diagnosis
3-15b

CASE 15
HISTORY
A 63-year-old man has a growing lesion on his chest.
1. Pigment network and globules identify this melanocytic lesion.
2. There is asymmetry of color and structure plus a multicomponent global pattern.
3. There are foci of irregular pigment network, irregular dots and globules, irregular streaks,
irregular pigmentation, and blue-white color.
4. Regression, milky-red areas, and polymorphous vessels are important high risk criteria that are
not seen.
5. The differential diagnosis includes a dysplastic nevus or melanoma.
126 DERMOSCOPY: AN ILLUSTRATED SELF-ASSESSMENT GUIDE

RISK
❑ Low
❑ Intermediate
✔ High

1

3
DIAGNOSIS
❑ Nevus
❑ Seborrheic keratosis
2
❑ Basal cell carcinoma
❑ Vascular
❑ Dermatofibroma
❑ Squamous cell carcinoma

❑ Melanoma

3-15c
❑ Other

DISPOSITION ANSWERS
Answers: 1,2,3,4,5
❑ No intervention
❑ Follow-up Discussion:
✔ Histopathologic diagnosis
❑ ■ The order of the questions is the order in which one should evaluate each
lesion.
■ Step 1 → Is the lesion melanocytic or not?

■ Step 2 → Is there symmetry or asymmetry of color and/or structure?

■ Step 3 → What is the global pattern?

DERMOSCOPIC CRITERIA ■ Step 4 → Find all of the local criteria (pigment network, dots and

globules, streaks, blotches, regression, colors, vessels).


■ Asymmetry of color and
■ Step 5 → Determine if the local criteria are regular or irregular.
structure (+) ■ Step 6 → Double check to make sure you have not missed anything.
■ Multicomponent global ■ Step 7 → Could there be a dermoscopic differential diagnosis for the
pattern (1,2,3) global pattern or any of the local criteria?
■ Irregular pigment network ■ Step 8 → Put the entire clinical scenario together and make a diagnosis.
(black boxes) ■ Step 9 → What is the best disposition for the lesion?

■ Irregular dots and globules ■ The foci of irregular streaks are not that easy to identify in this invasive
(circles) melanoma.
■ Irregular streaks (yellow box) ■ One could consider the foci of irregular pigment network foci or irregular
■ Irregular brown pigmentation streaks.
■ Bluish-white color (stars) ■ Different shades of brown and/or any other colors are a high risk criterion.

PEARLS
■ Memorize all of the features one should look for in each case.
■ Do not make a final diagnosis before you find and analyze all of the features
in a lesion.
Chapter 3 Trunk and Extremities 127

RISK
❑ Low
❑ Intermediate
❑ High

DIAGNOSIS
❑ Nevus
❑ Seborrheic keratosis
❑ Basal cell carcinoma
❑ Vascular

3-16a
❑ Dermatofibroma
❑ Squamous cell carcinoma
❑ Melanoma
❑ Other

DISPOSITION
❑ No intervention
❑ Follow-up
❑ Histopathologic diagnosis
3-16b

CASE 16
HISTORY
A 48-year-old man has a nonchanging brownish lesion in his left axilla for many years.
1. Clinically but not dermoscopically this looks similar to the melanoma in Case 15.
2. There are no criteria to diagnose a melanocytic lesion.
3. There are no criteria to diagnose a basal cell carcinoma.
4. Milia-like cysts and pigmented pseudofollicular openings characterize this seborrheic keratosis.
5. Asymmetry of color and structures, a multicomponent global pattern, irregular globules, blue-
white veil, and regression characterize this melanoma.
128 DERMOSCOPY: AN ILLUSTRATED SELF-ASSESSMENT GUIDE

RISK
❑ Low
✔ Intermediate

❑ High

DIAGNOSIS
❑ Nevus

❑ Seborrheic keratosis
❑ Basal cell carcinoma
❑ Vascular
❑ Dermatofibroma

3-16c
❑ Squamous cell carcinoma
❑ Melanoma
❑ Other
ANSWERS
Answers: 1,2,3,4

Discussion:
DISPOSITION
■ The history, if it is accurate, and clinical appearance are not high risk.
❑ No intervention ■ The global dermoscopic appearance on the other hand is worrisome.
❑ Follow-up ■ Multiple milia-like cysts and pigmented pseudofollicular openings favor the
✔ Histopathologic diagnosis
❑ diagnosis of a seborrheic keratosis.
■ The differential diagnosis of the pigmented pseudofollicular openings
includes irregular dots and globules of a melanocytic lesion.
■ If one considers that this could be melanocytic then the asymmetry of color
and structure, multicomponent global pattern, irregular dots and globules,
DERMOSCOPIC CRITERIA blue-white and other colors are very worrisome.
■ Milia-like cysts (black boxes) ■ Milia-like cysts and pigmented pseudofollicular openings can be found in
■ Pigmented pseudofollicular melanoma.
■ Typically seborrheic keratosis can be heavily pigmented.
openings (arrows)
■ The blue color in this benign lesion can be created by pigment deeper in the
■ Bluish-white colors (stars)
dermis similar to a blue nevus → the Tyndall effect.
■ Peppering (yellow box) ■ Peppering → gray color with fine gray dots indicates that there may be an
inflammatory component (irritated seborrheic keratosis).
■ The asymmetry of color and structure and multicomponent global pattern
has no significance in nonmelanocytic lesions.
■ A collision tumor, (eg, melanoma or nevus and a seborrheic keratosis), is in
the differential diagnosis.
■ This lesion should be very worrisome for the novice dermoscopist.
■ One should always listen to the patient’s history. However, patient histories are
not always reliable → in this case there was no history of change for many years.

PEARLS
■ Commonly, pathology reports of melanomas have seborrheic keratosis as
the primary diagnosis or seborrheic keratosis is in the differential diagnosis.
■ Even the most experienced clinician could miss a seborrheic keratosis-like
melanoma aka melanoma incognito → false negative melanoma.
■ Atypical seborrheic keratosis can be impossible to differentiate from melanoma.
■ Ask a more experienced dermosopist for their opinion with difficult cases.
■ A consensus of minds is often the way to go with difficult cases.
■ If in doubt, cut it out.
Chapter 3 Trunk and Extremities 129

RISK
❑ Low
❑ Intermediate
❑ High

DIAGNOSIS
❑ Nevus
❑ Seborrheic keratosis
❑ Basal cell carcinoma
❑ Vascular

3-17a
❑ Dermatofibroma
❑ Squamous cell carcinoma
❑ Melanoma
❑ Other

DISPOSITION
❑ No intervention
❑ Follow-up
❑ Histopathologic diagnosis
3-17b

CASE 17
HISTORY
A 63-year-old man was in for a yearly skin examination, and this solitary pigmented lesion was
found on his upper back. There is no other history.
1. Pigment network identifies a melanocytic lesion.
2. There is asymmetry of color and structure.
3. There is a homogeneous reticular global pattern.
4. The pigment network is irregular.
5. The irregular pigmentation is a red flag for concern.
130 DERMOSCOPY: AN ILLUSTRATED SELF-ASSESSMENT GUIDE

RISK
❑ Low
✔ Intermediate

❑ High

DIAGNOSIS

❑ Nevus
❑ Seborrheic keratosis
❑ Basal cell carcinoma
❑ Vascular
❑ Dermatofibroma
❑ Squamous cell carcinoma
❑ Melanoma

3-17c
❑ Other

DISPOSITION ANSWERS
❑ No intervention Answers: 1,2,3,4,5
✔ Follow-up

❑ Histopathologic diagnosis Discussion:
■ One would expect to see this lesion filled with a regular pigment network →
there is not a good clinico–dermoscopic correlation—a red flag for concern.
■ Whatever the global pattern (eg, homogeneous—reticular; reticular—globular)
one has to determine if the criteria are regular or irregular.
DERMOSCOPIC CRITERIA ■ By definition, the pigment network (reticular component of the global
■ Asymmetry of color and pattern) is irregular → thick, branched, broken up line segments.
structure (+) ■ Irregular pigment network can be found in low risk nevi (eg, junctional nevi).
■ The homogeneous darker color (homogeneous component of the global
■ Homogeneous—reticular
pattern) has a differential diagnosis that includes epidermal and/or dermal
global pattern
pigmentation with or without atypical melanocytes.
■ Irregular pigment network ■ The differential diagnosis of this lesion includes a nevus vs dysplastic nevus.
(boxes) ■ Melanoma is not in the differential diagnosis.
■ Irregular hyperpigmentation ■ There would not be a good dermoscopic–pathologic correlation if this was
(stars) thought to be a melanoma by the pathologist.
■ Background normal skin ■ Normal background skin is commonly found within a lesion and has no
(arrows) diagnostic significance → it must be differentiated from hypopigmentation
or regression.

PEARLS
■ Good communication with one’s pathologist is essential when there is not a
good dermoscopic–pathologic correlation.
■ A single lesion with this dermoscopic appearance is more worrisome than
several lesions that look like this.
■ For the novice dermoscopist → consider making histopathologic diagnosis.
■ For the experienced dermoscopist → sequential digital monitoring.
Chapter 3 Trunk and Extremities 131

RISK
❑ Low
❑ Intermediate
❑ High

DIAGNOSIS
❑ Nevus
❑ Seborrheic keratosis
❑ Basal cell carcinoma
❑ Vascular
❑ Dermatofibroma
❑ Squamous cell carcinoma
❑ Melanoma
❑ Other

DISPOSITION
3-18a
❑ No intervention
❑ Follow-up
❑ Histopathologic diagnosis

CASE 18
HISTORY
A colleague e-mailed this image for your opinion of whether this could be a melanoma. It was
located on the back of a 72-year-old man.
1. Pigment network identifies a melanocytic lesion.
2. Asymmetry of color and structure, a multicomponent global pattern, irregular pigment
network, and irregular globules characterize this melanoma.
3. This is a classic fingerprint pattern often found in flat seborrheic keratosis.
4. A fingerprint pattern and nonpigmented pseudofollicular openings characterize this
seborrheic keratosis.
5. Parallel line segments rather than honeycomb-like line segments differentiate the fingerprint
pattern of a seborrheic keratosis from the pigment network of a melanocytic lesion.
132 DERMOSCOPY: AN ILLUSTRATED SELF-ASSESSMENT GUIDE

RISK
✔ Low

❑ Intermediate
❑ High

DIAGNOSIS
❑ Nevus

❑ Seborrheic keratosis
❑ Basal cell carcinoma
❑ Vascular
❑ Dermatofibroma
❑ Squamous cell carcinoma
❑ Melanoma

3-18b
❑ Other

DISPOSITION ANSWERS
✔ No intervention

Answers: 3,4,5
❑ Follow-up
❑ Histopathologic diagnosis Discussion:
■ This is a classic fingerprint pattern with brown parallel line segments that
resemble the dermatoglyphics of the hand → fingerprints.
■ The differential diagnosis includes an early brain-like pattern also seen in

seborrheic keratosis or an atypical pigment network.


DERMOSCOPIC CRITERIA ■ This pattern can be found in flat seborrheic keratosis or solar lentigines.
■ Fingerprint pattern (boxes) ■ There are multiple nonpigmented pseudofollicular openings that would not
■ Nonpigmented pseudofollicu- be seen in a solar lentigo.
lar openings (yellow arrows) ■ The nonpigmented pseudofollicular openings should not be confused with
■ Concave moth-eaten borders globules of a melanocytic lesion.
(black arrows) ■ The line segments of the fingerprint pattern can be seen throughout the lesion.
■ There may only be remnants of a fingerprint pattern found in a lesion.
■ There are no melanoma-specific criteria in this lesion.
■ Once the definition of the fingerprint pattern is learned and one sees images
of the pattern, it usually is easy to recognize.

PEARLS
■ Caution! The fingerprint pattern can be associated with lentigo maligna.
■ Look for high risk criteria throughout a lesion before diagnosing seborrheic
keratosis or solar lentigo.
Chapter 3 Trunk and Extremities 133

RISK
❑ Low
❑ Intermediate
❑ High

DIAGNOSIS
❑ Nevus
❑ Seborrheic keratosis
❑ Basal cell carcinoma
❑ Vascular

3-19a
❑ Dermatofibroma
❑ Squamous cell carcinoma
❑ Melanoma
❑ Other

DISPOSITION
❑ No intervention
❑ Follow-up
❑ Histopathologic diagnosis
3-19b

CASE 19
HISTORY
A 45-year-old man with signs of extensive sun damage has this pigmented skin lesion on his chest.
1. This could be a false negative dermoscopic melanoma because the clinical but not the
dermoscopic appearance suggests a melanoma.
2. This is a solar lentigo with the fingerprint pattern and moth-eaten borders.
3. This is a melanocytic lesion because there is a pigment network.
4. Hypopigmentation filling the lesion rules out a melanoma.
5. Asymmetry of color and structure, a multicomponent global pattern, irregular pigment
network, and regression characterize this invasive melanoma.
134 DERMOSCOPY: AN ILLUSTRATED SELF-ASSESSMENT GUIDE

RISK
1 2 2
❑ Low
❑ Intermediate
✔ High

DIAGNOSIS
1
❑ Nevus
❑ Seborrheic keratosis
❑ Basal cell carcinoma
3
❑ Vascular
❑ Dermatofibroma 3
❑ Squamous cell carcinoma

❑ Melanoma

3-19c
❑ Other

DISPOSITION
❑ No intervention ANSWERS
❑ Follow-up Answers: 1,3,5
✔ Histopathologic diagnosis

Discussion:
■ The lesion looks worse clinically than dermoscopically.
■ It is not completely banal appearing dermoscopically.

■ It looks more benign than malignant dermoscopically.

DERMOSCOPIC CRITERIA ■ The clinical appearance overrides the dermoscopic picture.


■ Regular pigment network with uniform line segments and holes (dermal
■ Asymmetry of color and papilla) can be contrasted with an irregular pigment network.
structure (+) ■ The parallel lines of a fingerprint pattern are not seen.
■ The multicomponent global ■ The areas of regression look more like hypopigmentation → they are inferred
pattern (1,2,3) to be present because one sees regression clinically.
■ Regular pigment network ■ Hypopigmentation has no specific diagnostic significance.

(yellow boxes) ■ There are no dramatic melanoma- specific criteria.


■ Irregular pigment network ■ The differential diagnosis includes a dysplastic nevus or melanoma.
(black boxes) ■ This is a false negative dermoscopic melanoma.
■ Regression (stars)
PEARLS
■ When a lesion looks low risk clinically, a high risk dermoscopic appearance
should not be ignored.
■ When a lesion looks high risk clinically but not dermoscopically, the clinical
appearance should not be ignored.
■ In most cases, high risk looking lesions clinically will have a low risk
dermoscopic appearance.
■ With experience one will feel more comfortable handling all of these clinical
scenarios.
■ Gut feelings about the entire clinical scenario should never be ignored → if
one has an uncomfortable feeling about a lesion it should probably be excised.
Chapter 3 Trunk and Extremities 135

RISK
❑ Low
❑ Intermediate
❑ High

DIAGNOSIS
❑ Nevus
❑ Seborrheic keratosis
❑ Basal cell carcinoma
❑ Vascular

3-20a
❑ Dermatofibroma
❑ Squamous cell carcinoma
❑ Melanoma
❑ Other

DISPOSITION
❑ No intervention
❑ Follow-up
❑ Histopathologic diagnosis
3-20b

CASE 20
HISTORY
A 32-year-old man with a slowly growing pigmented lesion on his chest for many years is referred
for your evaluation.
1. This medium sized congenital melanocytic nevus is characterized by regular pigment network,
regular dots and globules, plus islands of normal skin.
2. The parallel fingerprint pattern helps to diagnose a traumatized flat seborrheic keratosis.
3. The clinical appearance, asymmetry of color and structure, regular pigment network, regular
dots and globules, plus regression characterize this melanoma.
4. The clinical appearance, asymmetry of color and structure, a multicomponent global pattern,
irregular pigment network, irregular dots and globules, plus regression characterize this
invasive melanoma.
5. Clinically this could only be a melanoma and dermoscopic evaluation would not add useful
information.
136 DERMOSCOPY: AN ILLUSTRATED SELF-ASSESSMENT GUIDE

RISK
❑ Low
❑ Intermediate
✔ High

DIAGNOSIS
❑ Nevus
❑ Seborrheic keratosis
❑ Basal cell carcinoma
❑ Vascular
❑ Dermatofibroma
❑ Squamous cell carcinoma

❑ Melanoma

3-20c
❑ Other

DISPOSITION ANSWERS
❑ No intervention
Answers: 4
❑ Follow-up
✔ Histopathologic diagnosis
❑ Discussion:
■ A medium-sized congenital melanocytic nevus and seborrheic keratosis are
in the clinical, but not the dermoscopic differential diagnosis.
■ It would be unlikely that a 32-year-old would have a solitary large seborrheic
keratosis.
DERMOSCOPIC CRITERIA ■ There are no criteria to suggest this is a seborrheic keratosis.
■ Asymmetry of color and ■ Congenital melanocytic nevi can have atypical dermoscopic features.
structure ■ Irregular criteria in a congenital melanocytic nevus should always raise a red
■ Multicomponent global pattern flag for concern.
■ Irregular pigment network ■ The dramatic foci of parallel pigment network resembling the fingerprint
pattern are only one criterion and not enough to diagnose a solar lentigo or
(black boxes)
seborrheic keratosis.
■ Fingerprint-like pigment
■ All of the other melanoma-specific criteria must be taken into consideration.
network (yellow boxes) ■ Asymmetry of color and structure
■ Irregular dots and globules ■ Multicomponent global pattern
(circles) ■ Irregular pigment network
■ Regression (arrows) ■ Irregular dots and globules

■ Regression

■ Dermoscopy adds essential information to confirm that this is a very


high risk lesion.
■ Dermoscopy rules out low risk pathology.

PEARLS
■ One should use the areas with most atypical features for an incisional biopsy.
■ Based on the clinical appearance and high risk dermoscopic criteria, one
would be justified to completely excise this lesion without a preliminary
biopsy even though it would create a big scar.
Chapter 3 Trunk and Extremities 137

RISK
❑ Low
❑ Intermediate
❑ High

DIAGNOSIS
❑ Nevus
❑ Seborrheic keratosis
❑ Basal cell carcinoma
❑ Vascular

3-21a
❑ Dermatofibroma
❑ Squamous cell carcinoma
❑ Melanoma
❑ Other

DISPOSITION
❑ No intervention
❑ Follow-up
❑ Histopathologic diagnosis
3-21b

CASE 21
HISTORY
A 20-year-old woman was worried about a mole that was getting darker.
1. Globules identify a melanocytic lesion.
2. This could be the “mountain and valley” pattern of a seborrheic keratosis.
3. This seborrheic keratosis is filled with pigmented pseudofollicular openings.
4. Irregular dots and globules suggest that this is a changing dysplastic nevus or melanoma.
5. This is the classic cobblestone global pattern of a banal nevus.
138 DERMOSCOPY: AN ILLUSTRATED SELF-ASSESSMENT GUIDE

RISK
✔ Low

❑ Intermediate
❑ High

DIAGNOSIS

❑ Nevus
❑ Seborrheic keratosis
❑ Basal cell carcinoma
❑ Vascular
❑ Dermatofibroma
❑ Squamous cell carcinoma
❑ Melanoma

3-21c
❑ Other

DISPOSITION ANSWERS
✔ No intervention
❑ Answers: 1,2,5
❑ Follow-up
❑ Histopathologic diagnosis Discussion:
■ The history of a changing nevus is not always high risk.
■ One expects nevi to change in children.
■ Clinically and dermoscopically this is a banal lesion.
■ This is the classic cobblestone global pattern:
DERMOSCOPIC CRITERIA ■ Filled with large angulated brown globules resembling street

cobblestones.
■ Symmetry of color and
■ Smaller dots and globules can also be found.
structure (+) ■ The clinical and dermoscopic differential diagnosis includes a seborrheic
■ Cobblestone global pattern keratosis with a “mountain and valley” pattern.
■ Regular dots and globules ■ Globules→ mountains
(circles) ■ Fissures → valleys
■ Cobblestone globules ■ There are no other criteria to diagnose a seborrheic keratosis → milia-like

(red arrows) cysts, pseudofollicular openings, “fat fingers”, or hairpin vessels.


■ Hypopigmented fissures ■ An actively changing nevus typically has dots and globules only at the
(yellow arrows) periphery of the lesion.

PEARLS
■ When the clinical and dermoscopic differential diagnosis is between a nevus
and a seborrheic keratosis, look at and feel the lesion:
■ Soft and compressible→ nevus

■ Immovable dry and scaly → seborrheic keratosis

■ A positive “wobble” sign (applying pressure with instrumentation [ie,


Dermlite] and gentle side-to-side motion) helps diagnose a nevus →
■ A nevus will have a positive “wobble” sign with side-to-side movement of

the lesion.
■ A seborrheic keratosis will have a negative “wobble” sign without any

movement.
Chapter 3 Trunk and Extremities 139

RISK
❑ Low
❑ Intermediate
❑ High

DIAGNOSIS
❑ Nevus
❑ Seborrheic keratosis
❑ Basal cell carcinoma
❑ Vascular

3-22a
❑ Dermatofibroma
❑ Squamous cell carcinoma
❑ Melanoma
❑ Other

DISPOSITION
❑ No intervention
❑ Follow-up
❑ Histopathologic diagnosis
3-22b

CASE 22
HISTORY
A 59-year-old woman with a history of melanoma returned for a routine skin examination. You
notice that in general the color of her skin is unusual.
1. This is a classic example of cutaneous metastatic melanoma.
2. Globules identify a melanocytic lesion.
3. The jaundiced color of the lesion is a clue that the patient has metastatic melanoma to the
liver.
4. The unusual color of the lesion is created by a self-tanner.
5. This is the cobblestone pattern of a benign nevus.
140 DERMOSCOPY: AN ILLUSTRATED SELF-ASSESSMENT GUIDE

RISK
✔ Low

❑ Intermediate
❑ High

DIAGNOSIS

❑ Nevus
❑ Seborrheic keratosis
❑ Basal cell carcinoma
❑ Vascular
❑ Dermatofibroma
❑ Squamous cell carcinoma
❑ Melanoma

3-22c
❑ Other

DISPOSITION ANSWERS
✔ No intervention

Answers: 2,4,5
❑ Follow-up
❑ Histopathologic diagnosis Discussion:
■ Commonly, one encounters patients with a history of melanoma that still
want to be tan.
■ Patients that use self tanners can usually be easily recognized.
■ Self tanners do affect what one sees with dermoscopy, although the changes
DERMOSCOPIC CRITERIA are not significant.
■ Cobblestone global pattern ■ Once again as in Case 21 the differential diagnosis is between a nevus with a
■ Symmetry of color and cobblestone pattern and a seborrheic keratosis with a “mountain and valley”
structure pattern.
■ Cobblestone globules (black ■ Foci of small “fat fingers” favor a seborrheic keratosis.
■ This case points out that “fat fingers” are not diagnostic of seborrheic
arrows)
keratosis and can be seen in melanocytic lesions.
■ Fissures (yellow arrows)
■ “Fat fingers” (white arrows)
PEARLS
■ Suggest to a patient that uses self-tanners that its use might adversely affect
your dermoscopic examination.
■ Try to get a feel if your patient has psychological issues related to the history
of melanoma that might benefit from a psychological evaluation.
Chapter 3 Trunk and Extremities 141

RISK
❑ Low
❑ Intermediate
❑ High

DIAGNOSIS
❑ Nevus
❑ Seborrheic keratosis
❑ Basal cell carcinoma
❑ Vascular

3-23a
❑ Dermatofibroma
❑ Squamous cell carcinoma
❑ Melanoma
❑ Other

DISPOSITION
❑ No intervention
❑ Follow-up
❑ Histopathologic diagnosis
3-23b

CASE 23
HISTORY
A 37-year-old pregnant woman developed multiple pigmented skin lesions during her 7th month
of pregnancy. She was especially worried about this one.
1. This could be the cobblestone pattern of a melanocytic nevus.
2. This could be the fissure and ridge pattern of a seborrheic keratosis.
3. Highly irregular hypopigmented globules and regression characterize this nodular melanoma.
4. Without milia-like cysts one cannot diagnose a seborrheic keratosis.
5. “Fat fingers,” nonpigmented pseudofollicular openings, fissures, crypts, and sharp border
demarcation diagnose a seborrheic keratosis.
142 DERMOSCOPY: AN ILLUSTRATED SELF-ASSESSMENT GUIDE

RISK
✔ Low

❑ Intermediate
❑ High

DIAGNOSIS
❑ Nevus

❑ Seborrheic keratosis
❑ Basal cell carcinoma
❑ Vascular
❑ Dermatofibroma
❑ Squamous cell carcinoma

3-23c
❑ Melanoma
❑ Other

ANSWERS
Answers: 1,2,5
DISPOSITION
Discussion:
✔ No intervention
❑ ■ This seborrheic keratosis is characterized by fissures and ridges.
❑ Follow-up ■ Several nonpigmented pseudofollicular openings should not be confused
❑ Histopathologic diagnosis with globules of a melanocytic lesion.
■ When the typical pseudofollicular openings expand into larger irregular
keratin filled areas they are referred to as crypts.
■ Crypts can be found in papillomatous nevi.
DERMOSCOPIC CRITERIA ■ Hypo or hyperpigmented ridges can be digit-like in shape and are referred
to as “fat fingers.”
■ Sharp border demarcation ■ “Fat fingers” are commonly encountered.
(red arrows) ■ “Fat fingers” can be straight, kinked, circular, or branched.
■ Nonpigmented pseudofollicu- ■ “Fat fingers” might be the only clue to diagnose a seborrheic keratosis.
lar openings (yellow arrows) ■ Seborrheic keratosis can be devoid of milia-like cysts.
■ Fissures (white arrows) ■ A melanocytic nevus could look exactly like this.
■ Crypts (stars) ■ Palpate the lesion to help differentiate:
■ Ridges aka “fat fingers” ■ A nevus will be soft and compressible while a seborrheic keratosis will be

(black arrows) firm and not compressible.


■ Incipient seborrheic keratosis ■ There is an independent small seborrheic keratosis seen clinically and with
dermoscopy.
(blue arrow)
PEARLS
■ There are several misnomers in the dermoscopic language, which we think
should not be used.
■ The “crown vessels” seen in sebaceous gland hyperplasia are defined as basal

cell-like vessels at the border of the lesion that penetrate the lesion but
never reach the center. Rarely will you see this distribution of vessels in
typical sebaceous gland hyperplasia.
■ “Leaf-like” structures represent one type of pigmentation found in basal

cell carcinomas that have finger-like projections said to look like a “maple
leaf ” or be “leaf-like.” If one finds pigmentation in a basal cell carcinoma
that fits this description it will never look like any type of a leaf.
■ “Fat fingers” are not a misnomer because they are an excellent description of
one morphologic variation of ridges.
Chapter 3 Trunk and Extremities 143

RISK
❑ Low
❑ Intermediate
❑ High

DIAGNOSIS
❑ Nevus
❑ Seborrheic keratosis
❑ Basal cell carcinoma
❑ Vascular

3-24a
❑ Dermatofibroma
❑ Squamous cell carcinoma
❑ Melanoma
❑ Other

DISPOSITION
❑ No intervention
❑ Follow-up
❑ Histopathologic diagnosis
3-24b

CASE 24
HISTORY
On routine skin examination, this lesion was found on the back of a 28-year-old man.
1. Peripheral globules are seen clinically suggesting this could be a Spitz nevus.
2. This could be a dysplastic nevus characterized by irregular dots and globules and different
shades of brown color.
3. Peripheral dots and globules indicate this could be an actively changing nevus.
4. Clinically and dermoscopically this could be a banal acquired nevus.
5. A symmetrical “Spitzoid” pattern may be found in melanoma.
144 DERMOSCOPY: AN ILLUSTRATED SELF-ASSESSMENT GUIDE

RISK
❑ Low
✔ Intermediate

❑ High

DIAGNOSIS

❑ Nevus
❑ Seborrheic keratosis
❑ Basal cell carcinoma
❑ Vascular
❑ Dermatofibroma

3-24c
Squamous cell carcinoma
❑ Melanoma
❑ Other
ANSWERS
Answers: 1,2,3,4,5

Discussion:
DISPOSITION
■ This is a melanocytic lesion because there are brown globules.
❑ No intervention ■ These are irregular dots and globules because they are of different sizes and shapes.
❑ Follow-up ■ Globally this has a “Spitzoid” pattern because it resembles the “starburst” pattern.
✔ Histopathologic diagnosis
❑ ■ There are peripheral globules without streaks.

■ Any of the six dermoscopic pictures representing Spitz nevi are also referred
to as being “Spitzoid.”
■ All “Spitzoid” lesions do not have a “starburst” pattern.

■ A symmetrical “Spitzoid” pattern is less worrisome than an irregular


DERMOSCOPIC CRITERIA “Spitzoid” pattern.
■ Symmetry of color and ■ This is a symmetrical “Spitzoid” pattern because the dots and globules surround
structure (+) the entire lesion.
■ “Spitzoid” global pattern ■ The differential diagnosis includes:
■ Nevus
■ Irregular dots and globules
■ Spitz nevus (correct diagnosis)
(circles)
■ Dysplastic nevus
■ Hypopigmentation (arrows)
■ Melanoma

■ Even a symmetrical “Spitzoid” pattern could be found in melanoma.


■ All “Spitzoid” lesions should be removed, especially in adults.
■ When present, different shades of dark color are a red flag for concern.
■ One should search carefully for other high risk criteria that might not be
that easy to find.
■ Peripheral dots and globules can be a clue of an actively changing nevus →
it will enlarge over time.
■ Inter-observer disagreement and decision-making in a lesion like this exist:
■ This is a perfectly banal appearing Spitz nevus.

■ This could be a symmetrical “Spitzoid” melanoma.

■ This is a banal lesion with peripheral globules that will change over time.

PEARLS
■ The history and clinical appearance of the lesion are very helpful to determine
the disposition in this particular case.
■ For the experienced dermoscopist, sequential digital follow-up is one
alternative to excision.
Chapter 3 Trunk and Extremities 145

RISK
❑ Low
❑ Intermediate
❑ High

DIAGNOSIS
❑ Nevus
❑ Seborrheic keratosis
❑ Basal cell carcinoma
❑ Vascular

3-25a
❑ Dermatofibroma
❑ Squamous cell carcinoma
❑ Melanoma
❑ Other

DISPOSITION
❑ No intervention
❑ Follow-up
❑ Histopathologic diagnosis
3-25b

CASE 25
HISTORY
A 45-year-old woman with a history of lentigo maligna on her face presented with this new
pigmented skin lesion on her left anterior thigh.
1. Globules identify a melanocytic lesion.
2. There is significant asymmetry of color and structure plus a multicomponent global pattern.
3. Irregular pigment network, irregular globules, a focus of purplish-white color, and regression
are all red flags for concern.
4. The lesion was excised and reported to be a dysplastic nevus which is a good
dermoscopic–pathologic correlation.
5. Diagnosing this histopathologically as a dysplastic nevus is not a good dermoscopic–
pathologic
correlation.
146 DERMOSCOPY: AN ILLUSTRATED SELF-ASSESSMENT GUIDE

RISK
❑ Low 1

❑ Intermediate
✔ High

2

DIAGNOSIS 3

❑ Nevus
❑ Seborrheic keratosis

3-25c
Basal cell carcinoma
❑ Vascular
❑ Dermatofibroma
ANSWERS
❑ Squamous cell carcinoma

❑ Melanoma Answers: 1,2,3,5
❑ Other Discussion:
■ This new, small pigmented skin lesion looks high risk clinically and with
dermoscopy.
■ The asymmetry of color and structure is striking and is an important red
DISPOSITION flag for concern → proceed with focused attention and look for other
high risk criteria.
❑ No intervention
■ The globules are also unusually large and asymmetrically located at the
❑ Follow-up periphery, another red flag for concern.
✔ Histopathologic diagnosis
❑ ■ The focus of pigment network is unusually thick and irregular.
■ The pigment network and irregular globules could be considered to be
irregular streaks.
■ Whatever one decides these structures are, they are still atypical and high risk.

■ The light color seen throughout has a differential diagnosis →


DERMOSCOPIC CRITERIA
hypopigmentation vs regression.
■ Asymmetry of color and ■ Different shades of homogeneous brown color is another high risk criterion
structure (+) to put into the entire pattern analysis algorithm.
■ Multicomponent global ■ The purplish -white color does not fit the description of a blue-white veil
pattern (1,2,3) and could represent neovacularization.
■ Irregular pigment network ■ There would be no point to do an incisional biopsy on such a small lesion
(box) that clinically and dermoscopically could be a melanoma.
■ Irregular globules (circles) ■ A good dermoscopic–pathologic correlation:
■ The histopathologic diagnosis should correspond with your dermoscopic
■ Different shades of homoge-
diagnosis.
neous brown color
■ When there is not a good dermoscopic– pathologic correlation:
■ Purplish-white color
■ Speak with your pathologist.
(black arrows) ■ Consider getting another dermato-pathologists opinion.
■ Regression (stars)
PEARLS
■ There should always be a good dermoscopic– pathologic correlation.
■ Having digital dermoscopic images of a potentially high risk lesion is the
best way to go back and make sure that there is a good dermoscopic–
pathologic correlation.
■ Addendum: Initially the lesion was excised with clear margins. Based on a
poor dermoscopic–pathologic correlation (histopathologic diagnosis was a
dysplastic nevus), another histopathologic opinion was obtained with a
pigmented lesion expert. The second opinion was that of an early invasive
melanoma, which led to an appropriate re-excision. The patient is doing
well 5 years post diagnosis.
Chapter 3 Trunk and Extremities 147

RISK
❑ Low
❑ Intermediate
❑ High

DIAGNOSIS
❑ Nevus
❑ Seborrheic keratosis
❑ Basal cell carcinoma
❑ Vascular

3-26a
❑ Dermatofibroma
❑ Squamous cell carcinoma
❑ Melanoma
❑ Other

DISPOSITION
❑ No intervention
❑ Follow-up
❑ Histopathologic diagnosis
3-26b

CASE 26
HISTORY
A 47-year-old woman with a history of melanoma and multiple seborrheic keratosis developed this
new lesion in the epigastric area.
1. Clinically and dermoscopically with “fat fingers” this looks like a seborrheic keratosis.
2. Globules identify a melanocytic lesion.
3. Large peripheral globules create a “Spitzoid” global pattern.
4. Clinically and dermoscopically the lesion appears to be a seborrheic keratosis which is a good
clinical–
dermoscopic correlation.
5. A “Spitzoid” lesion in an adult is not a red flag for concern.
148 DERMOSCOPY: AN ILLUSTRATED SELF-ASSESSMENT GUIDE

RISK
❑ Low
✔ Intermediate

❑ High

DIAGNOSIS

❑ Nevus
❑ Seborrheic keratosis
❑ Basal cell carcinoma
❑ Vascular
❑ Dermatofibroma
❑ Squamous cell carcinoma
❑ Melanoma

3-26c
❑ Other

DISPOSITION ANSWERS
❑ No intervention Answers: 2,3
❑ Follow-up
✔ Histopathologic diagnosis
❑ Discussion:
■ This new lesion was less than 6 mm and looked like a seborrheic keratosis
clinically.
■ It was a surprise to see this dermoscopic picture.
■ If it were not for the history that this was a new lesion, it might not have been
DERMOSCOPIC CRITERIA examined with dermoscopy.
■ Symmetry of color and ■ To avoid missing high risk incognito lesions, one should routinely examine
structure (+) clinically banal appearing lesions.
■ This was a clinically false negative high risk moderately dysplastaic nevus with
■ “Spitzoid” global pattern
atypical “Spitzoid” features histopathologically.
■ Irregular globules (arrows)
■ This is an excellent example of a bad clinico-dermoscopic correlation → a red
■ Multifocal hypopigmentation flag for concern.
(stars) ■ The clinical and dermoscopic pictures do not match up well.

■ Even though the globules are large, they do not look like “fat fingers.”
■ Inter-observer agreement even among expert dermoscopists is not always

good.
■ Another dermoscopist might think that the globules do resemble “fat fingers.”

■ Symmetry of color and structure can still be found in high risk pathology.
■ The hypopigmentation has no significance and should not be confused with
regression, which would be whiter.

PEARLS
■ It can be difficult for even an experienced dermato-pathologist to differentiate
a benign “Spitzoid” lesion from a “Spitzoid” melanoma.
■ Clarify with the dermato-pathologist if a benign “Spitzoid” lesion with atypical
features is really not a melanoma.
■ Several experienced dermato-pathologists might not be able to correctly
diagnose a “Spitzoid” melanoma.
■ Consider sentinel node biopsy with atypical “Spitzoid” lesions if they have

sufficient depth of invasion.


Chapter 3 Trunk and Extremities 149

RISK
❑ Low
❑ Intermediate
❑ High

DIAGNOSIS
❑ Nevus
❑ Seborrheic keratosis
❑ Basal cell carcinoma
❑ Vascular

3-27a
❑ Dermatofibroma
❑ Squamous cell carcinoma
❑ Melanoma
❑ Other

DISPOSITION
❑ No intervention
❑ Follow-up
❑ Histopathologic diagnosis
3-27b

CASE 27
HISTORY
The pediatrician of this 5-year-old referred the patient for evaluation of a congenital melanocytic
nevus that was getting bigger, and recently developed a few new dark spots. The patient’s mother
was very worried.
1. Dots and globules identify a melanocytic lesion.
2. The dots and globules are more or less similar in size and shape, and symmetrically located
throughout the lesion.
3. Asymmetry of color and structure, irregular dots and globules, and irregular blotches
characterize this dysplastic congenital nevus.
4. Islands of normal skin and the globular pattern characterize this banal congenital nevus.
5. The presence of high risk criteria (eg, a multicomponent global pattern) in a congenital
melanocytic nevus is always high risk.
150 DERMOSCOPY: AN ILLUSTRATED SELF-ASSESSMENT GUIDE

RISK
✔ Low

❑ Intermediate
❑ High

DIAGNOSIS

❑ Nevus
❑ Seborrheic keratosis
❑ Basal cell carcinoma
❑ Vascular
❑ Dermatofibroma
❑ Squamous cell carcinoma

3-27c
Melanoma
❑ Other

ANSWERS
DISPOSITION Answers: 1,2,4
❑ No intervention Discussion:
✔ Follow-up
❑ ■ Clinically and dermoscopically this is a banal congenital melanocytic nevus.
❑ Histopathologic diagnosis ■ There is symmetry of color and structure → this pattern is seen throughout
the entire lesion.
■ There are uniform, regular dots and globules.
■ Foci of darker color (hyperpigmentation) are routinely seen in congenital
nevi and are not a sign of atypia.
DERMOSCOPIC CRITERIA ■ Islands of normal skin may or may not be found around hairs typically
■ Symmetry of color and found in congenital nevi.
■ Perifolicular hypopigmentation
structure
■ Fine lanugo or thick dark terminal hairs are commonly but not always seen
■ Globular global pattern
in congenital nevi.
■ Regular dots and globules
■ There are no high risk criteria in this lesion.
(circles) ■ High risk criteria (eg, irregular dark blotches or irregular bluish globules)
■ Hyperpigmented globules would be a red flag for concern and a histopathologic diagnosis should be
(black boxes) considered.
■ Islands of normal skin ■ It is expected that congenital melanocytic nevi will enlarge as this child grows.
(black arrows)
■ Perifollicular hypopigmentation PEARLS
(yellow boxes)
■ The management of congenital melanocytic nevi is controversial.
■ Lanugo hairs (yellow arrows) ■ Most authorities believe that they should either be excised or monitored to
identify early changes of malignancy.
■ Digital gross and digital dermoscopic imaging for sequential monitoring
would be the cutting-edge way to follow any congenital melanocytic
nevus → small, medium, or large.
■ Side-by-side comparisons of baseline and follow-up images could be

made to look for important changes over time such as:


■ Asymmetrical enlargement

■ The presence of new high risk criteria

■ The presence of new colors

■ Regression

■ Disappearance of criteria
Chapter 3 Trunk and Extremities 151

RISK
❑ Low
❑ Intermediate
❑ High

DIAGNOSIS
❑ Nevus
❑ Seborrheic keratosis
❑ Basal cell carcinoma
❑ Vascular

3-28a
❑ Dermatofibroma
❑ Squamous cell carcinoma
❑ Melanoma
❑ Other

DISPOSITION
❑ No intervention
❑ Follow-up
❑ Histopathologic diagnosis
3-28b

CASE 28
HISTORY
Over the past 3 months this acquired pigmented skin lesion on the back of a 31-year-old woman
got bigger.
1. A central white patch and peripheral pigment network characterize this dermatofibroma.
2. Regular pigment network, regular globules, and hypopigmentation characterize this small
congenital nevus.
3. The irregular black blotch and regression are clues that this is a high risk lesion.
4. Regular pigment network, regular globules, symmetry of color and structure, plus a “Spitzoid”
global pattern characterize this Spitz nevus.
5. Asymmetry of color and structure, a multicomponent global pattern, irregular pigment
network, irregular dots and globules, an irregular black blotch and regression characterize this
melanoma.
152 DERMOSCOPY: AN ILLUSTRATED SELF-ASSESSMENT GUIDE

RISK 1
❑ Low
❑ Intermediate
✔ High

2

3
DIAGNOSIS
❑ Nevus
❑ Seborrheic keratosis
❑ Basal cell carcinoma
❑ Vascular
❑ Dermatofibroma
❑ Squamous cell carcinoma

❑ Melanoma

3-28c
❑ Other

DISPOSITION ANSWERS
❑ No intervention
Answers: 3,5
❑ Follow-up
✔ Histopathologic diagnosis
❑ Discussion:
■ Clinically, one sees a persistent mark on the skin which is created by pressure
from the instrumentation. This sign could mean:
■ This is an interesting case that many people looked at.

■ This is a difficult case that a novice dermoscopist spent a lot of time looking at.
DERMOSCOPIC CRITERIA ■ This is a melanocytic lesion because there is pigment network and globules.
■ Asymmetry of color and ■ There are foci where the network and globules seem to be a unit → this is
structure an unusual union.
■ Multicomponent global ■ The area of regression is widespread and well developed.
pattern (1,2,3) ■ The lesion is more malignant then benign looking, and has a differential
■ Irregular pigment network diagnosis → dysplastic nevus.
(black boxes) ■ The central patch of a dermatofibroma is not always centrally located but it
is always bony– white, not gray, in color.
■ Irregular dots and globules
■ This could not be a congenital nevus because it recently developed and the
(circles)
overall/global pattern would not be seen in a congenital lesion.
■ Irregular black blotch
(yellow box)
PEARL
■ Regression (stars)
■ “Peppering” (white boxes) ■ If one needs too much time to figure it out, then it’s time to cut it out!
Chapter 3 Trunk and Extremities 153

RISK
❑ Low
❑ Intermediate
❑ High

DIAGNOSIS
❑ Nevus
❑ Seborrheic keratosis
❑ Basal cell carcinoma
❑ Vascular

3-29a
❑ Dermatofibroma
❑ Squamous cell carcinoma
❑ Melanoma
❑ Other

DISPOSITION
❑ No intervention
❑ Follow-up
❑ Histopathologic diagnosis
3-29b

CASE 29
HISTORY
A 50-year-old man was found to have this asymptomatic lesion on his left lower leg.
1. Pigment network and globules identify a melanocytic lesion.
2. Pigment network and central white patch diagnose a dermatofibroma.
3. The central white patch of this dermatofibroma is characterized by a negative pigment
network (ie, white pigment network, reticular depigmentation) and homogenous white color.
4. Reticular depigmentation (white pigment network) can be found in dermatofibromas, Spitz
nevi, and melanoma.
5. A white pigment network might be the only clue that a lesion is a melanoma.
154 DERMOSCOPY: AN ILLUSTRATED SELF-ASSESSMENT GUIDE

RISK
✔ Low

❑ Intermediate
❑ High

DIAGNOSIS
❑ Nevus
❑ Seborrheic keratosis
❑ Basal cell carcinoma
❑ Vascular

❑ Dermatofibroma
❑ Squamous cell carcinoma
❑ Melanoma

3-29c
❑ Other

DISPOSITION ANSWERS
✔ No intervention
❑ Answers: 2,3,4,5
❑ Follow-up
❑ Histopathologic diagnosis
Discussion:
■ The pink color seen clinically and with dermoscopy represents diffuse
erythema.
■ Diffuse erythema and/or small vessels with different shapes (eg, pinpoint,
linear, branched) are commonly found in dermatofibromas and are not
DERMOSCOPIC CRITERIA high risk.
■ Presence of a pigment network is not always diagnostic of a melanocytic
■ Regular pigment network lesion because it can also be seen in lentignes and dermatofibromas.
(black boxes) ■ The central patch of a dermatofibroma should be milky/bony–white.
■ Central white patch ■ The central white patch in this lesion has two components:
(black arrows) ■ Homogenous white color
■ Reticular depigmentation ■ Reticular white color→ reticular depigmentation, white network, negative

(white boxes) pigment network, white pigment network.


■ Globular-like structures ■ The globular-like structures are not true globules of a melanocytic lesion
(yellow arrows) (ie, nests of melanocytes) but are created by the reticular depigmentation
■ Pink color over a background of brown color.
■ A reticulated central white patch is commonly found in dermatofibromas.
■ A reticulated white pigment network can also be found in banal acquired
nevi, Spitz nevi, and melanoma.

PEARLS
■ Typically, dermatofibromas have a characteristically firm feel on palpitation
→ use your fingers and touch the lesion in question.
■ Look carefully for melanoma-specific criteria when there is a negative
pigment network, so as not to miss a dermatofibroma-like melanoma.
■ A reticulated white pigment network might be the only clue that a lesion is
a melanoma.
Chapter 3 Trunk and Extremities 155

RISK
❑ Low
❑ Intermediate
❑ High

DIAGNOSIS
❑ Nevus
❑ Seborrheic keratosis
❑ Basal cell carcinoma
❑ Vascular

3-30a
❑ Dermatofibroma
❑ Squamous cell carcinoma
❑ Melanoma
❑ Other

DISPOSITION
❑ No intervention
❑ Follow-up
❑ Histopathologic diagnosis
3-30b

CASE 30
HISTORY
A 38-year-old woman noticed a color change over a few week period in this pigmented lesion located
on her left arm.
1. A central white patch and reticular depigmentation characterize this dermatofibroma.
2. An eccentric focus of reticular depigmentation is a clue that this night not be a
dermatofibroma but a melanoma.
3. Globules identify a melanocytic lesion.
4. Asymmetry of color and structure, a multicomponent global pattern, multifocal
hypopigmentation, and regular dots and globules characterize this mildly dysplastic nevus.
5. White and gray homogeneous colors plus “peppering” characterize the regression in this
melanoma.
156 DERMOSCOPY: AN ILLUSTRATED SELF-ASSESSMENT GUIDE

RISK
❑ Low
❑ Intermediate
✔ High
❑ 1

DIAGNOSIS
❑ Nevus 2
❑ Seborrheic keratosis
❑ Basal cell carcinoma 3
❑ Vascular
❑ Dermatofibroma
❑ Squamous cell carcinoma

❑ Melanoma

3-30c
❑ Other

DISPOSITION ANSWERS
❑ No intervention
Answers: 2,3,5
❑ Follow-up
✔ Histopathologic diagnosis
❑ Discussion:
■ This is a melanocytic lesion because there are brown globules.
■ There is asymmetry of color and structure because:
■ The gray homogeneous color on the right side is not seen on the left side.

■ The gray homogeneous color seen in the lower half is not seen in the upper half.
DERMOSCOPIC CRITERIA ■ Reticular depigmentation is mostly on the right side.

■ Asymmetry of color and ■ Dots and globules are just on the right side.

structure ■ The central white patch of a dermatofibroma is in the differential diagnosis


■ Multicomponent global pattern of the bony-white color of regression.
(1,2,3) ■ The white color should not be confused with hypopigmentation → It is too
■ Irregular dots and globules white.
■ An absence of pigment network does not rule out a dermatofibroma →
(black boxes)
there are dermatofibromas without pigment network or central white patch.
■ Bony-white color of regression
■ Homogenous gray color and the fine gray dots “peppering” are part of the
(black arrows) regression.
■ Gray homogeneous color of ■ The large area of regression is much easier to see than the foci of reticular
regression (yellow arrows) depigmentation.
■ “ Peppering “ (yellow box)
■ Reticular depigmentation PEARLS
(white arrows)
■ Typically there is no history of change with dermatofibromas.
■ The entire clinical scenario is important:
■ Personal and family history

■ History of the lesion

■ Clinical and dermoscopic features.

■ Dermoscopy in a vacuum is not as helpful as knowing the whole story.


■ Think differential diagnosis for local criteria and global patterns.
Chapter 3 Trunk and Extremities 157

RISK
❑ Low
❑ Intermediate
❑ High

DIAGNOSIS
❑ Nevus
❑ Seborrheic keratosis
❑ Basal cell carcinoma
❑ Vascular

3-31a
❑ Dermatofibroma
❑ Squamous cell carcinoma
❑ Melanoma
❑ Other

DISPOSITION
❑ No intervention
❑ Follow-up
❑ Histopathologic diagnosis
3-31b

CASE 31
HISTORY
A 33-year-old woman recently noticed an irregularly shaped skin lesion on her left lower leg. There
was no history of any changes.
1. Globules diagnose a melanocytic lesion.
2. Multifocal hypopigmentation, symmetry of color and structure, and regular dots and globules
diagnose a benign nevus.
3. Clinically and dermoscopically, there is the suggestion of regression, a red flag for concern.
4. The absence of reticular depigmentation rules out a melanoma.
5. A multicomponent global pattern, asymmetry of color and structure, irregular dots and
globules, regression and reticular depigmentation characterize this invasive melanoma.
158 DERMOSCOPY: AN ILLUSTRATED SELF-ASSESSMENT GUIDE

RISK
❑ Low
❑ Intermediate
✔ High
❑ 1

DIAGNOSIS
❑ Nevus
❑ Seborrheic keratosis 2
❑ Basal cell carcinoma
❑ Vascular
❑ Dermatofibroma
❑ Squamous cell carcinoma

❑ Melanoma

3-31c
Other 3

DISPOSITION
❑ No intervention ANSWERS
❑ Follow-up Answers: 1,3,5
✔ Histopathologic diagnosis

Discussion:
■ The reticular depigmentation is more widespread than in Case 30.
■ Compared to Case 30 the regression is not bony-white.
■ Multifocal hypopigmentation would be in the differential diagnosis of the
areas of regression.
DERMOSCOPIC CRITERIA ■ Gray color favors regression over hypopigmentation.
■ Asymmetry of color and ■ Hypopigmentation is light brown color and does not contain gray color.
structure ■ Regression is observed clinically, which is a point in favor of regression

■ Multicomponent global over hypopigmentation.


pattern (1,2,3)
■ Irregular dots and globules PEARLS
(circles) ■ The important features could be missed if one is in a hurry.
■ Regression (stars) ■ If there are clues that a lesion might be high risk, think to yourself slow
■ Reticular depigmentation down, focus your attention, and look carefully at the lesion.
(boxes) ■ High risk criteria are not always easy to find (ie, reticular depigmentation)
but might be the only clues to suggest the seriousness of a lesion.
Chapter 3 Trunk and Extremities 159

RISK
❑ Low
❑ Intermediate
❑ High

DIAGNOSIS
❑ Nevus
❑ Seborrheic keratosis
❑ Basal cell carcinoma
❑ Vascular

3-32a
❑ Dermatofibroma
❑ Squamous cell carcinoma
❑ Melanoma
❑ Other

DISPOSITION
❑ No intervention
❑ Follow-up
❑ Histopathologic diagnosis
3-32b

CASE 32
HISTORY
A 63-year-old man had this lesion on his chest that has been getting bigger and darker over a
1 year period.
1. Clinically and dermoscopically, the eccentric pigmentation is diagnostic of a melanoma.
2. Peripheral pigment network and a black central white patch characterize a senile
dermatofibroma.
3. The irregular pigment network and irregular black blotch are melanoma-specific criteria.
4. Symmetry of color and structure, regular pigment network, a regular black blotch, and
regression characterize this melanoma.
5. Asymmetry of color and structure, a multicomponent global pattern, irregular pigment
network, irregular black blotch, and hypopigmentation characterize this melanoma.
160 DERMOSCOPY: AN ILLUSTRATED SELF-ASSESSMENT GUIDE

RISK
❑ Low
❑ Intermediate
✔ High

DIAGNOSIS
❑ Nevus
❑ Seborrheic keratosis
❑ Basal cell carcinoma
1
❑ Vascular
❑ Dermatofibroma 3
❑ Squamous cell carcinoma

❑ Melanoma

3-32c
❑ Other

DISPOSITION
ANSWERS
❑ No intervention
Answers: 3, 5
❑ Follow-up
✔ Histopathologic diagnosis
❑ Discussion:
■ Pigment network identifies a melanocytic lesion.
■ A black central white patch does not exist in dermatofibromas.
■ To date senile dermatofibromas have not been reported.
■ Eccentric criteria (eg, pigment network, dots and globules, blotches) are a
DERMOSCOPIC CRITERIA red flag for concern.
■ Asymmetry of color and ■ Eccentric criteria may or may not be high risk → melanoma-specific criteria.
structure (+) ■ One must determine if eccentric criteria are regular or irregular, low or

■ Multicomponent global high risk, good or bad.


pattern (1,2,3) ■ The black blotch is irregular because:
■ Irregular pigment network ■ It is asymmetrically located.

■ There are different shades of dark color.


(boxes)
■ The borders are irregular.
■ Irregular black blotch (yellow
■ The pigment network is irregular because:
arrows)
■ There are only a few foci of network.
■ Hypopigmentation (stars)
■ The line segments are thick and branched.

■ The hypopigmentation should not be confused with regression.


■ It is not bony-white

■ The hypopigmentation has no diagnostic significance.

PEARLS
■ Eccentric criteria are a red flag for concern.
■ Proceed with focused attention and analyze all of the criteria in the lesion.
Chapter 3 Trunk and Extremities 161

RISK
❑ Low
❑ Intermediate
❑ High

DIAGNOSIS
❑ Nevus
❑ Seborrheic keratosis
❑ Basal cell carcinoma
❑ Vascular

3-33a
❑ Dermatofibroma
❑ Squamous cell carcinoma
❑ Melanoma
❑ Other

DISPOSITION
❑ No intervention
❑ Follow-up
❑ Histopathologic diagnosis
3-33b

CASE 33
HISTORY
A 24-year-old, fair-skinned, blue-eyed, red head with multiple banal appearing nevi was found to
have this lesion on his abdomen.
1. This is a melanocytic lesion by default.
2. There is asymmetry of color and structure and a multicomponent global pattern.
3. The differential diagnosis of the light area includes hypopigmentation or regression.
4. The eccentrically located regular blue blotch is a red flag for concern.
5. The dermoscopic differential diagnosis includes a banal nevus, combined nevus, or regressive
melanoma.
162 DERMOSCOPY: AN ILLUSTRATED SELF-ASSESSMENT GUIDE

RISK
❑ Low
✔ Intermediate
❑ 1
❑ High

DIAGNOSIS

❑ Nevus
3
❑ Seborrheic keratosis
❑ Basal cell carcinoma
❑ Vascular
❑ Dermatofibroma 4
❑ Squamous cell carcinoma
❑ Melanoma

3-33c
❑ Other

DISPOSITION ANSWERS
❑ No intervention Answers: 1,2,3,4,5
❑ Follow-up
✔ Histopathologic diagnosis
❑ Discussion:
■ Compared to Case 32, the entire picture is not so clear.
■ This is a melanocytic lesion by default because there are no clear cut criteria
for a melanocytic lesion, seborrheic keratosis, basal cell carcinoma,
dermatofibroma, or vascular lesion.
DERMOSCOPIC CRITERIA ■ White color is always a red flag for concern.
■ Asymmetry of color and ■ The white and gray colors favor regression over hypopigmentation.
■ Histopathologically, there was no regression found in this lesion.
structure
■ One cannot tell if there are true dots and globules or they are created by the
■ Multicomponent global
irregularity of the dark color.
pattern (1,2,3,4) ■ The blue color is considered a blotch because it is larger than dots or
■ Regular blue blotch (box) globules.
■ Hypopigmentation (stars) ■ The blue blotch has a histopathologic differential diagnosis that includes:
■ Irregular dot and globular-like ■ The blue nevus component of a combined nevus
structures (circles) ■ Melanocytic atypia

■ Irregular purplish-brown color ■ Malignant melanocytes

■ Gray homogenous color


(yellow arrows) PEARLS
■ This case points out the need for one to be as knowledgeable as possible and
the need to think in terms of dermoscopic and histopathologic differential
diagnosis.
■ Dermoscopic criteria are not always what they appear to be (eg, there is no
regression in this lesion).
■ Dermoscopy is not a perfect technique. Don’t expect it to be!
Chapter 3 Trunk and Extremities 163

RISK
❑ Low
❑ Intermediate
❑ High

DIAGNOSIS
❑ Nevus
❑ Seborrheic keratosis
❑ Basal cell carcinoma
❑ Vascular
3-34a

❑ Dermatofibroma
❑ Squamous cell carcinoma
❑ Melanoma
❑ Other

DISPOSITION
❑ No intervention
❑ Follow-up
❑ Histopathologic diagnosis
3-34b

CASE 34
HISTORY
A 17-year-old presented with multiple small congenital nevi. There was no history of change in any of
the lesions. While examining all of the nevi with dermoscopy, this “ugly duckling” nevus with an
irregular black blotch was found. Initially, the blotch was not seen clinically because it was very small.
1. This is a banal appearing congenital combined nevus.
2. Dermoscopically and clinically this is an “ugly duckling” lesion because it was the only nevus
with a black blotch.
3. On close inspection, cobblestone-like globules are seen asymmetrically located in the lesion.
4. The irregular black blotch could represent transepidermal elimination of melanin, melanocytic
atypia, or melanoma arising in a small congenital melanocytic nevus.
5. To avoid a large keloid with a complete excision, one could biopsy the black blotch to make the
diagnosis.
164 DERMOSCOPY: AN ILLUSTRATED SELF-ASSESSMENT GUIDE

RISK 1

❑ Low
❑ Intermediate
✔ High

2

DIAGNOSIS

❑ Nevus
❑ Seborrheic keratosis
❑ Basal cell carcinoma
❑ Vascular 3

❑ Dermatofibroma
❑ Squamous cell carcinoma

3-34c
Melanoma
❑ Other

ANSWERS
DISPOSITION
Answers: 2,3,4,5
❑ No intervention
❑ Follow-up Discussion:
✔ Histopathologic diagnosis
❑ ■ It was a surprise to find the black blotch in what appeared to be a banal
small congenital melanocytic nevus.
■ All of the other nevi had a classic cobblestone global pattern.
■ Uniform large angulated globules filling the lesion.

■ Retrospectively, a black speck was seen clinically but only after it was found
DERMOSCOPIC CRITERIA with dermoscopy.
■ Asymmetry of color and ■ One should always try to examine clinically banal as well as atypical lesions
structure no matter how many nevi a patient has.
■ Have the patient come back another time if there is not enough time to
■ Multicomponent global
fully examine most if not all of the lesions.
pattern (1,2,3)
■ An irregular cobblestone pattern is in the differential diagnosis of the
■ Irregular black blotch
multicomponent global pattern.
(yellow arrows) ■ The irregular black blotch is a red flag for concern and the histopathologic
■ Bluish-white color (yellow star) correlates include:
■ Hypopigmentation ■ Transepidermal elimination of melanin
(black stars) ■ Melanocytic atypia
■ Irregular cobblestone ■ Malignant melanocytes

globules (white box) ■ The black color rules out a combined nevus in which the color should be bluish.
■ Irregular dots and globules ■ There is significant scaliness, which creates areas that look like reticular
(circles) depigmentation.
■ A quick swipe with an alcohol prep will eliminate the scaliness to confirm
■ Scale (yellow boxes)
that there is no reticular depigmentation.

PEARLS
■ Always eliminate dryness over a lesion to get a better dermoscopic view.
■ Melanoma arising in a small satellite congenital melanocytic nevus is a rarity.
■ Beauty is in the eye of the beholder. The “ugly duckling” is not always ugly
especially to its mother.
■ A clinical and/or dermoscopically “ugly duckling” lesion is always a red flag
for concern but it is not always high risk.
Chapter 3 Trunk and Extremities 165

RISK
❑ Low
❑ Intermediate
❑ High

DIAGNOSIS
❑ Nevus
❑ Seborrheic keratosis
❑ Basal cell carcinoma
❑ Vascular

3-35a
❑ Dermatofibroma
❑ Squamous cell carcinoma
❑ Melanoma
❑ Other

DISPOSITION
❑ No intervention
❑ Follow-up
❑ Histopathologic diagnosis
3-35b

CASE 35
HISTORY
A 55-year-old man developed a dark papule in a pre-existing pigmented skin lesion on his
abdomen.
1. Globules identify a melanocytic lesion.
2. Milia-like cysts and pigmented pseudofollicular openings diagnose a seborrheic keratosis.
3. The large blue ovoid nest of pigment is enough to diagnose a pigmented basal cell carcinoma.
4. Different shades of brown color are clues that this could be a melanoma.
5. If it were not for the history of change, this could be a combined nevus.
166 DERMOSCOPY: AN ILLUSTRATED SELF-ASSESSMENT GUIDE

RISK 1

❑ Low
❑ Intermediate
✔ High

1 2

DIAGNOSIS
❑ Nevus
3
❑ Seborrheic keratosis 1

3-35c
❑ Basal cell carcinoma
❑ Vascular
❑ Dermatofibroma ANSWERS
❑ Squamous cell carcinoma Answers: 1,4,5

❑ Melanoma
❑ Other Discussion:
■ The development of a dark papule, in a pre-existing melanocytic lesion, is
melanoma until proven otherwise.
■ The skin markings, seen clinically, do not favor a benign lesion.
■ Skin markings and hairs can be found in melanomas.
DISPOSITION ■ The irregular brown dots and globules could be easily overlooked.
❑ No intervention ■ The blotch is irregular because:
❑ Follow-up ■ There is bluish-white color.

✔ Histopathologic diagnosis
❑ ■ There is a focus of irregular blue dots and globules.

■ It is asymmetrically located within the lesion.

■ The blue dots and globules could be created by nests of melanocytes deeper in
the dermis or by irregularities in the homogeneity of the bluish-white blotch.
■ Different shades of homogenous brown color are a red flag for concern.
DERMOSCOPIC CRITERIA ■ Different shades of any homogeneous color should always be a red flag for

■ Asymmetry of color and concern.


structure ■ There are a few milia-like cysts.
■ To diagnose a seborrheic keratosis there should be multiple milia-like
■ Multicomponent global
pattern (1,2,3) cysts plus other criteria.
■ The irregular brown dots and globules do not look like the pigmented
■ Irregular brown dots and
pseudofollicular openings seen in seborrheic keratosis.
globules (black circles)
■ The only resemblance this melanoma has to a pigmented basal cell carci-
■ Irregular blue globules noma is the dark bluish-white blotch.
(white circle) ■ A pigmented basal cell carcinoma would not have diffuse brown homoge-
■ Irregular bluish-white blotch nous color.
(white arrows) ■ Arborizing vessels are not needed to diagnose a basal cell carcinoma.
■ Hypopigmentation (stars) ■ A combined nevus is in the dermoscopic but not in the historical or clinical
■ Different shades of brown color differential diagnosis.
■ Milia-like cysts (boxes) ■ The areas of hypopigmentation are not white enough to be considered regression.

PEARLS
■ One does not need dermoscopy to realize that this could be a melanoma.
■ Dermoscopy increases the diagnosis of melanoma by at least 16% when

compared to skin examination without it.


■ Dermoscopy is an enjoyable tool that allows one to see colors and structures not
visible with the naked eye or with the typical magnification that clinicians use.
■ Dermoscopy helps confirm with an on the spot second opinion clinical
impressions of high risk pathology (eg, dysplastic, Spitz nevi, squamous and
basal cell carcinoma, melanoma).
Chapter 3 Trunk and Extremities 167

RISK
❑ Low
❑ Intermediate
❑ High

DIAGNOSIS
❑ Nevus
❑ Seborrheic keratosis
❑ Basal cell carcinoma
❑ Vascular

3-36a
❑ Dermatofibroma
❑ Squamous cell carcinoma
❑ Melanoma
❑ Other

DISPOSITION
❑ No intervention
❑ Follow-up
❑ Histopathologic diagnosis
3-36b

CASE 36
HISTORY
This was the only pigmented lesion on the back of a 45-year-old man. The rest of his skin was
devoid of melanocytic nevi, lentigines, hemangiomas, seborrheic, or actinic keratosis, typically
found on people that live in Florida.
1. Globules identify a melanocytic lesion.
2. There is asymmetry of color and structure and a multicomponent global pattern.
3. There are irregular dots and globules, irregular streaks, irregular blackish-gray blotches, and
blue-white color.
4. The dermoscopic differential diagnosis includes a dysplastic nevus and melanoma.
5. One could speculate that this will not be an in situ melanoma based on an evaluation of the
entire dermoscopic picture.
168 DERMOSCOPY: AN ILLUSTRATED SELF-ASSESSMENT GUIDE

RISK
❑ Low
❑ Intermediate
✔ High

1

DIAGNOSIS
❑ Nevus
❑ Seborrheic keratosis
3
❑ Basal cell carcinoma 4
❑ Vascular
❑ Dermatofibroma

3-36c
Squamous cell carcinoma

❑ Melanoma
❑ Other
ANSWERS
Answers: 1,2,3,5
DISPOSITION Discussion:
❑ No intervention ■ Clinically, the lesion is not as dramatic as the dermoscopic picture.
❑ Follow-up ■ All of the important melanoma-specific criteria are easy to identify.

✔ Histopathologic diagnosis
❑ ■ One should go over the entire checklist of points that need to be evaluated:
■ Melanocytic vs nonmelanocytic.

■ Symmetry vs asymmetry of color and structure.

■ What is the global pattern?

■ Identify and evaluate all of the local criteria.


DERMOSCOPIC CRITERIA ■ Make a dermoscopic diagnosis.

■ Asymmetry of color and ■ There is blue and white color but not the blue-white veil.
structure ■ Blue and/or white color in any form is a high risk criterion.

■ Multicomponent global ■ One can tell by the multiple colors and paucity of local criteria that this is
pattern (1,2,3,4) not an in situ melanoma.
■ Clinically and dermoscopically this looks more like a superficial spreading
■ Irregular dots and globules
rather than a nodular melanoma.
(circles)
■ Histopathologically it was a nodular melanoma.
■ Irregular streaks (black arrows) ■ This is not a perfect dermoscopic-pathologic correlation even though it is
■ Irregular blackish-gray blotches still a melanoma.
(white arrows) ■ One should confirm with the dermatopathologist that this is really a
■ Irregular blue blotch nodular melanoma.
(yellow arrow) ■ A dysplastic nevus would not have so much diversity of color and structure.
■ Bluish-white color (stars) ■ Foci of streaks at the periphery not their shape determines that they are
irregular.
■ Regular streaks would be found symmetrically around the lesion at the

periphery.
■ As in this case, streaks can be an extension of the pigment network or

come off a dark blotch (tumor body).

PEARLS
■ A dramatic dermoscopic picture like this one can come as a surprise.
■ When that happens, one must refrain from outbursts of surprise with statements
such as “Oh my God” or “This looks very bad.” Patients don’t like that!
Chapter 3 Trunk and Extremities 169

RISK
❑ Low
❑ Intermediate
❑ High

DIAGNOSIS
❑ Nevus
❑ Seborrheic keratosis
❑ Basal cell carcinoma
❑ Vascular

3-37a
❑ Dermatofibroma
❑ Squamous cell carcinoma
❑ Melanoma
❑ Other

DISPOSITION
❑ No intervention
❑ Follow-up
❑ Histopathologic diagnosis
3-37b

CASE 37
HISTORY
The patient’s neighbor noticed this skin lesion on the upper thigh of a 42-year-old man.
1. A central white patch and blue blotch characterize this small dermatofibroma.
2. Multifocal hypopigmentation, regular dots and globules, and a regular blue blotch characterize
this dysplastic nevus.
3. Several globules identify a melanocytic lesion.
4. The blue blotch in the context of a melanocytic lesion diagnoses this combined nevus.
5. Asymmetry of color and structure, a multicomponent global pattern, irregular globules, an
irregular bluish-white blotch, regression, and pinpoint vessels characterize this nodular
melanoma.
170 DERMOSCOPY: AN ILLUSTRATED SELF-ASSESSMENT GUIDE

RISK
❑ Low
❑ Intermediate
✔ High
❑ 1

DIAGNOSIS
❑ Nevus
2
❑ Seborrheic keratosis
❑ Basal cell carcinoma
❑ Vascular
❑ Dermatofibroma 3
❑ Squamous cell carcinoma

❑ Melanoma

3-37c
❑ Other

DISPOSITION ANSWERS
❑ No intervention Answers: 3,5
❑ Follow-up
✔ Histopathologic diagnosis
❑ Discussion:
■ As compared to Case 36, clinically and dermoscopically, this looks more like
a nodular melanoma because there is:
■ A nodule not flat lesion

■ Paucity of local criteria


DERMOSCOPIC CRITERIA ■ Pinpoint vessels

■ Asymmetry of color and ■ The gray homogenous color favors regression over hypopigmentation.
structure ■ Purplish color differentiates the pinpoint vessels from dots and globules of a
■ Multicomponent global melanocytic lesion, which should be brown.
■ One cannot always tell the difference.
pattern (1,2,3)
■ A combined nevus would not have so much diversity of criteria.
■ Irregular brown globules (black
■ An atypical dermatofibroma could look like this.
boxes) ■ Melanoma is in the differential diagnosis of atypical dermatofibromas.
■ Irregular bluish-white blotch ■ Atypical dermatofibromas should be removed.
(yellow arrows)
■ White component of PEARLS
regression (stars)
■ Any papule or nodule with even a suggestion that it could be a melanoma
■ Gray component of regression
should be excised posthaste.
(white arrows)
■ Sequential digital monitoring is contraindicated in any lesion that could be a
■ Pinpoint vessels (yellow boxes) nodular melanoma.
■ Any delay in making the diagnosis could affect the patient’s survival chances.
Chapter 3 Trunk and Extremities 171

RISK
❑ Low
❑ Intermediate
❑ High

DIAGNOSIS

3-38a
❑ Nevus
❑ Seborrheic keratosis
❑ Basal cell carcinoma
❑ Vascular
❑ Dermatofibroma
❑ Squamous cell carcinoma
❑ Melanoma
❑ Other

DISPOSITION
❑ No intervention
❑ Follow-up
❑ Histopathologic diagnosis
3-38b

CASE 38
HISTORY
The daughter of this 81-year-old man found this lesion on the arm of her father and brought it to
the attention of his dermatologist.
1. Dermoscopy is not needed to diagnose this melanoma.
2. Globules identify a melanocytic lesion.
3. There is significant asymmetry of color and structure and a multicomponent global pattern.
4. The dots and globules are irregular.
5. An irregular dark blotch, bluish-white color, and regression are other melanoma-specific criteria.
172 DERMOSCOPY: AN ILLUSTRATED SELF-ASSESSMENT GUIDE

RISK
❑ Low
❑ Intermediate 4
✔ High

DIAGNOSIS
1
❑ Nevus
❑ Seborrheic keratosis 3
❑ Basal cell carcinoma
5
❑ Vascular
❑ Dermatofibroma

3-38c
❑ Squamous cell carcinoma

❑ Melanoma
❑ Other
ANSWERS
Answers: 1,2,3,4,5

DISPOSITION Discussion:
■ It is not necessary to examine the entire lesion to diagnose this melanoma.
❑ No intervention
■ Examine the entire lesion before making a dermoscopic diagnosis.
❑ Follow-up
■ Asymmetry of color and structure.
✔ Histopathologic diagnosis
❑ ■ The left and right sides are not mirror images of each other.

■ The lower and upper halves are not mirror images of each other.

■ A multicomponent global pattern.


■ Three or more different areas within the lesion.

■ A single criterion or multiple criteria can make up each zone.


DERMOSCOPIC CRITERIA
■ Irregular dots and globules.
■ Asymmetry of color and ■ Different sizes and shapes
structure ■ Asymmetrical location
■ Multicomponent global ■ Irregular black blotch.
pattern (1,2,3,4,5) ■ Irregular in size and shape
■ Irregular dots and globules ■ Asymmetrical location

(circles) ■ Regression.
■ Irregular black blotch ■ Bony/milky-white color

(yellow arrows) ■ Peppering → fine gray dots

■ Regression (stars) ■ Bluish-white color.


■ This is not a blue-white veil → irregular homogenous blue color with an
■ Peppering (boxes)
overlying ground glass appearance.
■ Bluish-white color
■ Blue and/or white color is any form is a high risk criterion.
(white arrows)
■ The bluish-white color lies over the irregular black blotch.
■ Irregular blue blotch ■ Irregular blue blotch.
(black arrow) ■ Larger than a dot or globule

■ Irregular border

■ Asymmetrical location

PEARLS
■ Seeing examples of well-developed melanoma-specific criteria is essential if
one has a chance to identify them when they are not so well developed.
■ Look carefully for subtle criteria before determining that they are absent.
■ All of the melanoma-specific (high risk) criteria can be found in benign lesions.
■ No single criterion is pathognomonic of a melanoma.
Chapter 3 Trunk and Extremities 173

RISK
❑ Low
❑ Intermediate
❑ High

DIAGNOSIS
❑ Nevus
❑ Seborrheic keratosis
❑ Basal cell carcinoma
❑ Vascular

3-39a
❑ Dermatofibroma
❑ Squamous cell carcinoma
❑ Melanoma
❑ Other

DISPOSITION
❑ No intervention
❑ Follow-up
❑ Histopathologic diagnosis
3-39b

CASE 39
HISTORY
A 27-year-old man moved from another city to Miami. He came in for a routine skin examination
because he has a history of dysplastic nevi. This lesion on his very hairy chest was being followed in
another pigmented lesion clinic with regular photography and not dermoscopy. There was no
history of change.
1. One’s first impression is that this is a melanoma.
2. The bluish-white veil clinches the melanoma diagnosis.
3. The milky-red globules are diagnostic of a melanoma.
4. The lesion was excised with a diagnosis of a dysplastic nevus. That is a good
dermoscopic–pathologic correlation.
5. Asymmetry of color and structure, a homogenous global pattern, regular dots and globules,
regular blotch, veil, and globular vessels characterize this melanoma.
174 DERMOSCOPY: AN ILLUSTRATED SELF-ASSESSMENT GUIDE

1
RISK
❑ Low
❑ Intermediate 2

✔ High

3

2
4
DIAGNOSIS

❑ Nevus
1 5

❑ Seborrheic keratosis
❑ Basal cell carcinoma
❑ Vascular
❑ Dermatofibroma 6

3-39c
❑ Squamous cell carcinoma
❑ Melanoma
❑ Other
ANSWERS
Answers: 1

Discussion:
DISPOSITION ■ The dermoscopic picture is dramatic compared to the not so high risk looking
❑ No intervention clinical lesion.
❑ Follow-up ■ The histopathologic diagnosis of a dysplastic nevus was not a good
✔ Histopathologic diagnosis
❑ dermoscopic-pathologic correlation.
■ Dermoscopically it looked like a melanoma but it was not a

histopathologic melanoma.
■ When there is not a good dermoscopic–pathologic correlation, seek other

dermatopathologists’ opinions.
DERMOSCOPIC CRITERIA ■ This is a false positive dermoscopic melanoma.

■ In this case, none of the three dermatopathologists diagnosed a melanoma.


■ Asymmetry of color and
structure ■ There is blue and white color but not the blue-white veil as it is defined.
■ The bluish-white color is over poorly defined irregular dark color.
■ Multicomponent global
■ Milky-red globules are not diagnostic of a melanoma but are a red flag for
pattern (1,2,3,4,5,6)
concern.
■ Irregular dots and globules ■ It is hard to tell if some of the globules are reddish or brown.
(black circles) ■ The melanoma-specific criteria include:
■ Bluish -white color (stars) ■ Asymmetry of color and structure
■ Milky-red globules (red circle) ■ Multicomponent global pattern

■ Irregular dots and globules

■ Bluish-white color

■ Milky-red globules

■ Multiple colors

PEARLS
■ Never tell a patient that they have melanoma 100%. The worst clinical
scenario including the dermoscopic picture might still be benign.
■ With difficult cases, one can send digital dermoscopic images to expert
dermoscopists for their opinion.
■ Take dermoscopy to a higher level by having equipment to take digital

dermoscopic images.
■ Do not hesitate to seek other dermatopathologists opinions with difficult cases.
■ At times it is necessary to cut hair away to get a better clinical and dermoscopic
look at a suspicious skin lesion.
Chapter 3 Trunk and Extremities 175

RISK
❑ Low
❑ Intermediate
❑ High

DIAGNOSIS
❑ Nevus
❑ Seborrheic keratosis
❑ Basal cell carcinoma

3-40a
Vascular
❑ Dermatofibroma
❑ Squamous cell carcinoma
❑ Melanoma
❑ Other

DISPOSITION
❑ No intervention
❑ Follow-up
❑ Histopathologic diagnosis
3-40b

CASE 40
HISTORY
A 55-year-old man came in for a skin examination only to make sure that he was okay. This lesion
was found on the upper back. He was not aware of its presence.
1. Globules identify a melanocytic lesion.
2. There is asymmetry of color and structure and a multicomponent global pattern.
3. One sees foci of irregular pigment network, irregular dots and globules, an irregular brown
blotch, regression, and bluish-white color.
4. The lesion was excised and reported to be a dysplastic nevus, which is not a good
dermoscopic-pathologic correlation.
5. The arborizing vessels, surrounding the lesion, indicate that there is significant sun-damaged skin.
176 DERMOSCOPY: AN ILLUSTRATED SELF-ASSESSMENT GUIDE

RISK
❑ Low
❑ Intermediate
✔ High

1

DIAGNOSIS 2
❑ Nevus
❑ Seborrheic keratosis
3
❑ Basal cell carcinoma
❑ Vascular
❑ Dermatofibroma

3-40c
❑ Squamous cell carcinoma

❑ Melanoma
❑ Other
ANSWERS
Answers: 1,2,3,4,5

Discussion:
DISPOSITION ■ The dermoscopic diagnosis but not the histopathologic diagnosis was
❑ No intervention thought to be a melanoma.
❑ Follow-up ■ That is not a good dermoscopic–pathologic correlation

✔ Histopathologic diagnosis
❑ ■ It was sent for review to a national pigment lesion expert dermatopathol-

ogist who reported this was “definitely” not a melanoma.


■ With free margins and a benign pathology report, was the patient in the clear?

■ No, this is a false negative pathologic, but not dermoscopic melanoma.

■ This case is the opposite of Case 39 in which the lesion was truly benign.
DERMOSCOPIC CRITERIA ■ Some of the melanoma-specific criteria are easy to see:
■ Asymmetry of color and structure
■ Asymmetry of color and
■ Multicomponent global pattern
structure
■ Regression
■ Multicomponent global
■ Bluish-white color
pattern (1,2,3)
■ Some of the melanoma-specific criteria are not easy to see:
■ Irregular pigment network ■ Irregular pigment network
(yellow arrows) ■ Irregular dots and globules
■ Irregular dots and globules ■ Irregular brown blotch
(circles) ■ Arborizing vessels are not diagnostic of a basal cell carcinoma and can also
■ Irregular brown blotch be found in/on:
(white arrow) ■ Nevi

■ Regression (stars) ■ Sebaceous gland hyperplasia

■ Bluish-white color (yellow box) ■ Scar tissue

■ Sun-damaged skin → individual mats or diffuse arborizing vessels


■ Arborizing vessels
■ It is not always possible to differentiate a solitary mat of arborizing vessels
(black arrows)
from a basal cell carcinoma→ if in doubt, cut it out!

PEARLS
■ Even the most experienced pigmented lesion expert dermatopathologists do
not always get it right.
■ A consensus of opinions might be better than a single opinion
(eg, dermoscopists, dermatopathologists).
■ Every one misses and/or misdiagnoses melanoma.
■ It is a sad fact of life!
Chapter 3 Trunk and Extremities 177

RISK
❑ Low
❑ Intermediate
❑ High

DIAGNOSIS
❑ Nevus
❑ Seborrheic keratosis
❑ Basal cell carcinoma
❑ Vascular

3-41a
❑ Dermatofibroma
❑ Squamous cell carcinoma
❑ Melanoma
❑ Other

DISPOSITION
❑ No intervention
❑ Follow-up
❑ Histopathologic diagnosis
3-41b

CASE 41
HISTORY
The patient of Case 40 returned for a 6-month follow-up skin examination and some spots were
seen in the melanoma excision site.
1. This is a typical combined nevus that coincidentally is near the melanoma excision site.
2. The brown, blue, and white colors could be seen in melanoma.
3. The history, clinical, and dermoscopic appearances of the new spots are consistent with
cutaneous metastatic melanoma.
4. The brown and bluish-white colors are irregular.
5. Without the history and visible surgical scar, the dermoscopic differential diagnosis includes:
a combined, blue, Spitz nevus, nodular, and cutaneous metastatic melanoma.
178 DERMOSCOPY: AN ILLUSTRATED SELF-ASSESSMENT GUIDE

RISK
❑ Low
❑ Intermediate
✔ High

DIAGNOSIS
❑ Nevus

3-41c
❑ Seborrheic keratosis
❑ Basal cell carcinoma
❑ Vascular
❑ Dermatofibroma
❑ Squamous cell carcinoma ANSWERS

❑ Melanoma Answers: 2,3,4,5
❑ Other Discussion:
■ The histopathology was said to be benign, however, the large blue and small
brown spots made clear that the original diagnosis was not correct. → It was
a melanoma!
DISPOSITION ■ Cutaneous and lymph node metastasis might be the only way to diagnose
❑ No intervention melanoma in clinically and histopathologically equivocal cases.
❑ Follow-up ■ The histopathologic diagnosis of the bluish-brown area was consistent with
✔ Histopathologic diagnosis nodular or cutaneous metastatic melanoma.

■ A sentinel lymph node biopsy was positive → metastatic melanoma.

■ The patient is disease free 6 years post diagnosis

■ Without the history, the dermoscopic differential diagnosis includes:


■ Combined nevus ■ Spitz nevus

DERMOSCOPIC CRITERIA ■ Blue nevus ■ Nodular melanoma

■ Cutaneous metastatic melanoma


■ Irregular bluish-white color
■ Dermoscopic features of cutaneous metastatic melanoma are not specific.
(stars)
■ Pigmented
■ Irregular brown color ■ Nonpigmented → hypo or amelanotic (pink or reddish in color),
(white arrows) ■ With or without polymorphous vessels → differently shaped small or
■ Surgical scar (black arrows) larger telangiectatic vessels.
■ The history of a past melanoma excision thick enough to metastasize is
usually more helpful than the dermoscopic features to diagnose cutaneous
metastatic melanoma.

PEARLS
■ This case points out how treacherous melanoma can be.
■ Luck might be as important as depth of invasion for patient survival.
■ The diagnosis of an in situ amelanotic melanoma in the scalp of a young

man with a lot of hair → good luck.


■ A young man with an “irritated” mole who thinks it is caused by his belt,

does nothing about it, and then dies at age 33 → bad luck.
Chapter 3 Trunk and Extremities 179

RISK
❑ Low
❑ Intermediate
❑ High

DIAGNOSIS
❑ Nevus
❑ Seborrheic keratosis
❑ Basal cell carcinoma
❑ Vascular

3-42a
❑ Dermatofibroma
❑ Squamous cell carcinoma
❑ Melanoma
❑ Other

DISPOSITION
❑ No intervention
❑ Follow-up
❑ Histopathologic diagnosis
3-42b

CASE 42
HISTORY
This lesion was found in the posterior upper arm of a 42-year-old woman.
1. Clinically and dermoscopically this could be a nodular melanoma.
2. Clinically and dermoscopically this could be a blue nevus.
3. Clinically and dermoscopically this could be cutaneous metastatic melanoma.
4. This is the classic homogenous blue global pattern characteristic of a blue nevus.
5. Irregular dots and globules plus different shades of blue and brown color are diagnostic of a
nodular melanoma.
180 DERMOSCOPY: AN ILLUSTRATED SELF-ASSESSMENT GUIDE

RISK
✔ Low

❑ Intermediate
❑ High

DIAGNOSIS

❑ Nevus
❑ Seborrheic keratosis
❑ Basal cell carcinoma
❑ Vascular
❑ Dermatofibroma
❑ Squamous cell carcinoma
❑ Melanoma

3-42c
❑ Other

DISPOSITION
✔ No intervention

❑ Follow-up ANSWERS
❑ Histopathologic diagnosis
Answers: 1,2,3,4

Discussion:
■ This is a classic blue nevus with a homogeneous global pattern.
■ Based on the history, there could be other diagnostic possibilities:
DERMOSCOPIC CRITERIA
■ Rapidly growing nodule → nodular melanoma.
■ Homogenous global pattern ■ A history of a melanoma in the area → cutaneous metastatic melanoma.
■ Irregular bluish dots and ■ Typically, one finds a solitary blue papule or nodule that the patient knows
globules (boxes) has been present for years without any changes.
■ Milia-like cyst (circle) ■ There are different shades of blue, bluish-white, and some suggestion of
■ Homogeneous brown color brown color.
(arrows) ■ This combination of colors could also be found in a combined nevus.

■ The irregular dots and globules (nests of melanocytes) are blue because they
are deeper in the dermis (ie, the “Tyndall” effect).
■ They could also be created by irregularities of the bluish-white color over

the background of diffuse blue color.


■ The milia-like cyst has no diagnostic significance.
■ Milia-like cysts are not exclusively found in seborrheic keratosis.

■ Milia-like cysts can be found in melanocytic nevi and in melanomas.

PEARLS
■ The history is essential to help diagnose a blue nevus.
■ In general, the family and personal history plus the history and clinical
appearance of a lesion will help one make a more accurate dermoscopic
diagnosis.
Chapter 3 Trunk and Extremities 181

RISK
❑ Low
❑ Intermediate
❑ High

DIAGNOSIS
❑ Nevus
❑ Seborrheic keratosis
❑ Basal cell carcinoma
❑ Vascular
❑ Dermatofibroma
❑ Squamous cell carcinoma
❑ Melanoma
❑ Other

3-43a DISPOSITION
❑ No intervention
❑ Follow-up
❑ Histopathologic diagnosis

CASE 43
HISTORY
This dermoscopic image was on the dermatology board examination without a history or clinical
image.
1. Globules suggest that this could be a melanocytic lesion.
2. Ulceration, an ovoid blue nest of pigmentation, and a spoke-wheel structure suggest this could
be a basal cell carcinoma.
3. Asymmetry of color and structure, a multicomponent global pattern, irregular brown globules,
irregular streaks, an irregular bluish-black blotch, and regression suggest that this could be a
melanoma.
4. Ulceration is present but does not help distinguish a basal cell carcinoma from a melanoma.
5. Spoke-wheel structures have not yet been reported in melanoma and are exclusively found in
basal cell carcinomas.
182 DERMOSCOPY: AN ILLUSTRATED SELF-ASSESSMENT GUIDE

RISK
❑ Low
1
❑ Intermediate
✔ High
❑ 2

DIAGNOSIS
❑ Nevus 3 4
❑ Seborrheic keratosis

❑ Basal cell carcinoma
❑ Vascular

3-43b
❑ Dermatofibroma
❑ Squamous cell carcinoma
❑ Melanoma ANSWERS
❑ Other
Answers: 1,2,3,4,5
Discussion:
■ This pigmented basal cell carcinoma can not be distinguished from melanoma.
■ There are no arborizing vessels.
DISPOSITION ■ Basal cell carcinomas can be without arborizing vessels.
❑ No intervention ■ Multicomponent global pattern is not exclusively found in melanomas.
❑ Follow-up ■ Ulceration can be found in basal cell carcinomas and in melanomas.
✔ Histopathologic diagnosis
❑ ■ Statistically, ulceration is found more often in basal cell carcinomas.
■ Ulceration in this case favors the diagnosis of a basal cell carcinoma.
■ There are globular-like and streak-like structures, which means:
■ These structures resemble globules and streaks, respectively, but
morphologically they are not true globules or streaks.
DERMOSCOPIC CRITERIA ■ The histopathologic correlation of these criteria is not known in this case

■ Asymmetry of color and because it was not specifically looked for by the dermatopathologist.
■ They are variations of pigmentation seen in basal cell carcinomas.
structure
■ The blue ovoid nest of pigmentation has a differential diagnosis that
■ Multicomponent global includes an irregular dark blotch of a melanocytic lesion.
pattern (1,2,3,4) ■ Finger-like projections coming off the blotch favor blue-ovoid basal cell nest.
■ Irregular streak-like structures ■ The differential diagnosis includes irregular streaks of a melanocytic lesion.
(black arrows) ■ This is also how “leaf-like” structures are said to look.

■ Irregular dot and globular-like ■ There is no resemblance to any type of leaf!


■ “Leaf-like” structures is a misnomer and the concept should be abandoned.
structures (circles)
■ Blue ovoid nest of ■ The whitish color suggests regression.
■ White color can be found in basal cell carcinomas.
pigmentation (white stars)
■ The spoke-wheel structure has a central brown globule (“hub”) with a
■ Finger-like projections suggestion of linear structures radiating from the center (spokes).
(yellow arrows) ■ Imagination may be needed to diagnose spoke-wheel structures.
■ Ulceration (yellow box) ■ Number of radial projections varies.

■ Spoke-wheel structure ■ One or more spoke-wheels can be found in a basal cell carcinoma.

(black box) ■ Spoke-wheel structures might be the only criterion to diagnose a basal
cell carcinoma → no arborizing vessels, pigmentation, or ulceration.
■ One cannot always differentiate melanoma from basal cell carcinoma.
■ Dermoscopy shows → histopathologic diagnosis is indicated.

PEARLS
■ Use the potentially most high risk diagnosis to plan a surgical approach:
■ Biopsy a basal cell carcinoma
■ Excision of a potential melanoma
■ This case points out the need to create a dermoscopic differential diagnosis.
Chapter 3 Trunk and Extremities 183

RISK
❑ Low
❑ Intermediate
❑ High

DIAGNOSIS
❑ Nevus
❑ Seborrheic keratosis
❑ Basal cell carcinoma
❑ Vascular

3-44a
❑ Dermatofibroma
❑ Squamous cell carcinoma
❑ Melanoma
❑ Other

DISPOSITION
❑ No intervention
❑ Follow-up
❑ Histopathologic diagnosis
3-44b

CASE 44
HISTORY
This asymptomatic dark blue lesion was found on the thigh of an 85-year-old man.
1. Globules identify a melanocytic lesion.
2. Multiple blue ovoid nests diagnose a basal cell carcinoma.
3. Large, well-demarcated blue vascular spaces diagnose a hemangioma.
4. Asymmetry of color and structure, a multicomponent global pattern, regular dots and
globules, regular blotches and regression diagnose a nodular melanoma.
5. Areas with purplish color, suggest that this is a vascular lesion.
184 DERMOSCOPY: AN ILLUSTRATED SELF-ASSESSMENT GUIDE

RISK
✔ Low
❑ 1
❑ Intermediate
❑ High

DIAGNOSIS 2

❑ Nevus
❑ Seborrheic keratosis
❑ Basal cell carcinoma

❑ Vascular
❑ Dermatofibroma
3
❑ Squamous cell carcinoma
❑ Melanoma

3-44c
❑ Other

DISPOSITION
✔ No intervention

ANSWERS
❑ Follow-up
❑ Histopathologic diagnosis Answers: 3,5

Discussion:
■ This vascular lesion points out that asymmetry of color and structure and a
multicomponent global pattern are not diagnostic of a melanoma.
DERMOSCOPIC CRITERIA ■ Vascular lesions do not always have small lacunae.
■ Vascular lesions can have large vascular spaces.
■ Asymmetry of color and ■ The vascular spaces have a differential diagnosis that includes:
structure ■ Ovoid nests of pigmentation → basal cell carcinoma
■ Multicomponent global ■ Irregular blotches → melanoma
pattern (1,2,3) ■ Agminated (grouped in the same area) blue nevi
■ Large blue vascular spaces ■ The purplish color is a clue that this could be a vascular lesion.
(white stars) ■ Vascular lesions can have different shades of red, blue, and/or black colors

■ Purplish color (arrows) (representing thrombosis).


■ Whitish color (black stars) ■ Vascular lesions typically have white color created by hyperkeratosis or by
■ Globular-like structures fibrous septa.
(circles) ■ The globular-like structures are artifacts created by oil placed over the lesion
to eliminate the dryness.
■ The less-experienced dermoscopist might consider this to be a melanoma.
■ If in doubt, cut it out!

PEARL
■ Always eliminate dryness when it exists to get a better dermoscopic view of a
lesion.
■ A quick swipe with an alcohol prep gets the job done but does not last

long and might need to be repeated.


■ Oil or gel (eg, ultrasound gel, hand washing gel) can be used if one wants

to take digital dermoscopic images because it lasts much longer than


alcohol to eliminate dryness and enhances digital image quality.
Chapter 3 Trunk and Extremities 185

RISK
❑ Low
❑ Intermediate
❑ High

DIAGNOSIS
❑ Nevus
❑ Seborrheic keratosis
❑ Basal cell carcinoma
❑ Vascular

3-45a
Dermatofibroma
❑ Squamous cell carcinoma
❑ Melanoma
❑ Other

DISPOSITION
❑ No intervention
❑ Follow-up
❑ Histopathologic diagnosis
3-45b

CASE 45
HISTORY
This was found on the left forearm arm of a 67-year-old woman.
1. Globules identify a melanocytic lesion.
2. There is asymmetry of color and structure and a multicomponent global pattern.
3. There are irregular dots and globules, bluish-white color, and regression.
4. A few milia-like cysts are not enough to diagnose a pigmented seborrheic keratosis.
5. A focus of milky-red globules helps diagnose this melanoma.
186 DERMOSCOPY: AN ILLUSTRATED SELF-ASSESSMENT GUIDE

RISK
❑ Low 4
❑ Intermediate
✔ High

1
3
DIAGNOSIS
❑ Nevus
❑ Seborrheic keratosis
2
❑ Basal cell carcinoma
2
❑ Vascular
❑ Dermatofibroma
❑ Squamous cell carcinoma

❑ Melanoma

3-45c
❑ Other

DISPOSITION ANSWERS
❑ No intervention
Answers: 1,2,3,4,5
❑ Follow-up

❑ Histopathologic diagnosis Discussion:
■ Melanoma arose in a nevus → area 2 of multicomponent global pattern.
■ Shows the importance of a systematic analysis of a suspicious lesion.
■ This is melanocytic because there are multiple globules.
■ There is obvious asymmetry of color and structure.
DERMOSCOPIC CRITERIA ■ The global pattern is multicomponent because there are four distinct areas
■ Asymmetry of color and with different criteria.
structure ■ Local criteria include:
■ Multicomponent global ■ Irregular dots and globules

pattern (1,2,3,4) ■ Bluish-white color → there is no “veil”

■ Areas of regression are questionably present


■ Irregular dots and globules
■ A small focus of milky-red area (neovascularization)
(circles)
■ Regression not easily seen with dermoscopy was present histopathologically.
■ Bluish-white color
■ The skin markings have no diagnostic significance.
(white arrows)
■ They crisscross the homogeneous brown color of the benign nevus com-
■ Regression (stars) ponent of the lesion.
■ Milky-red area (yellow arrows)
■ Normal skin markings
PEARLS
(red arrows)
■ Milia-like cysts (black arrows) ■ Memorizing a systematic plan of attack requires motivation to master the
technique, dedication, and some hard work.
■ Having a written cheat sheet of the important criteria may aid learning.
■ Better communication between the dermoscopist and pathologist could lead
to a more accurate histopathologic diagnosis.
■ Regression is seen with dermoscopy but not reported histopathologically.

■ Important criteria might be asymmetrically located in the lesion but not

reported histopathologically → more cuts might be indicated.


Chapter 3 Trunk and Extremities 187

RISK
❑ Low
❑ Intermediate
❑ High

DIAGNOSIS
❑ Nevus
❑ Seborrheic keratosis
❑ Basal cell carcinoma
❑ Vascular
3-46a

❑ Dermatofibroma
❑ Squamous cell carcinoma
❑ Melanoma
❑ Other

DISPOSITION
❑ No intervention
❑ Follow-up
❑ Histopathologic diagnosis
3-46b

CASE 46
HISTORY
A 36-year-old man was concerned about this darkening lesion on his right arm.
1. Clinically and dermoscopically, the blue and brown areas represent cutaneous metastatic
melanoma.
2. This is the classic “starburst” global pattern of a Spitz nevus.
3. There are irregular streaks at the periphery.
4. There are regular streaks at the periphery.
5. The majority of the lesion is filled with a regular black blotch.
188 DERMOSCOPY: AN ILLUSTRATED SELF-ASSESSMENT GUIDE

RISK
✔ Low

❑ Intermediate
❑ High

DIAGNOSIS

❑ Nevus
❑ Seborrheic keratosis
❑ Basal cell carcinoma
❑ Vascular
❑ Dermatofibroma
❑ Squamous cell carcinoma
❑ Melanoma

3-46c
❑ Other

DISPOSITION
❑ No intervention
ANSWERS
❑ Follow-up Answers: 2,4,5

❑ Histopathologic diagnosis Discussion:
■ This is a classic “starburst” global pattern:
■ Regular streaks

■ Homogeneous dark color

■ Bluish-white color
DERMOSCOPIC CRITERIA
■ The streaks are considered to be regular because they are located at most
■ “Starburst” global pattern points along the periphery.
■ Symmetry of color and ■ Streaks are considered irregular when they are not located at most points
structure along the periphery (ie, when there are only foci of streaks, they are
■ Regular streaks (arrows) considered irregular).
■ Regular black blotch (stars) ■ Location at the periphery, not the shape, determines if streaks are regular

■ Bluish-white color or irregular.


■ Nevus (box) ■ Dots and globules at the periphery can be components of the “starburst”
pattern → they are not present in this lesion.
■ Any of the six dermoscopic patterns representing Spitz nevi are referred to
as being “Spitzoid”:
■ Starburst

■ Globular

■ Homogeneous

■ Vascular (pink or reddish)

■ Black pigment network (reticular)

■ Nonspecific

■ A regular “Spitzoid” pattern is less worrisome than an irregular “Spitzoid” pattern.


■ Cutaneous metastatic melanomas could have a homogeneous dark blotch
but not a symmetrical ring of streaks at the periphery.
■ The brown color with a fine regular pigment network represents a nevus
that has nothing to do with the Spitz nevus.
■ It is not uncommon to have different pathologies adjacent to each other

(eg, seborrheic keratoses, nevi, and/or hemangiomas).


Chapter 3 Trunk and Extremities 189

■ A “black lamella” (ie, pigmented parakeratosis, histopathologically) is in the


differential diagnosis of the black blotch.
■ It is a jet black thin appearing blotch with an overall shiny appearance →

similar to the black color of a black widow spider.


■ Tape stripping can help to make the differentiation → specks from the

“black lamella” can become stuck to the tape.


■ The “black lamella” does not always come off with tape stripping.

PEARLS
■ An acquired “Spitzoid” lesion in an adult is always a red flag for concern.
■ Melanoma can have a symmetrical “Spitzoid” pattern.
■ All “Spitzoid” lesions should be considered for excision, not only in adults,
and regardless of the age of the patient.
This page intentionally left blank
Chapter 3 Trunk and Extremities 191

RISK
❑ Low
❑ Intermediate
❑ High

DIAGNOSIS
❑ Nevus
❑ Seborrheic keratosis
❑ Basal cell carcinoma

3-47a
Vascular
❑ Dermatofibroma
❑ Squamous cell carcinoma
❑ Melanoma
❑ Other

DISPOSITION
❑ No intervention
❑ Follow-up
❑ Histopathologic diagnosis
3-47b

CASE 47
HISTORY
Friends of this 44-year-old man were concerned about this darkening lesion on the upper arm.
1. Similar to Case 46, this is a symmetrical “Spitzoid” or “starburst” global pattern.
2. This is an atypical “Spitzoid” pattern composed of irregular dots and globules at the periphery,
an irregular black blotch, and bluish-white color.
3. The bluish-white color and irregular black blotch are melanoma-specific criteria.
4. “Spitzoid” melanomas always have a “Spitzoid” dermoscopic pattern.
5. One can have “Spitzoid” dermoscopic pattern without a “Spitzoid” histopathologic pattern.
192 DERMOSCOPY: AN ILLUSTRATED SELF-ASSESSMENT GUIDE

RISK
❑ Low
❑ Intermediate
✔ High

DIAGNOSIS
❑ Nevus
❑ Seborrheic keratosis
❑ Basal cell carcinoma
❑ Vascular
❑ Dermatofibroma
❑ Squamous cell carcinoma

❑ Melanoma

3-47c
❑ Other

DISPOSITION
❑ No intervention ANSWERS
❑ Follow-up Answers: 2,3,5

❑ Histopathologic diagnosis
Discussion:
■ This is an irregular “Spitzoid” global pattern.
■ There is asymmetry of color and structure.

■ There are foci of irregular dots and globules at the periphery → some of
DERMOSCOPIC CRITERIA the globules look like streaks.
■ Irregular “Spitzoid” global ■ There is bluish-white color typically found in “Spitz” nevi.

■ There is an irregular black blotch→ black color can also be found in


pattern
■ Asymmetry of color and “Spitz” nevi.
■ Any combination of dots, globules, and/or streaks at the periphery plus
structure (+)
light/dark brown, black, or blue color located within the lesion may be
■ Irregular dots and globules
found in “Spitz” nevi.
(circles) ■ A negative or white pigment network (reticular depigmentation) is a clue
■ Irregular black blotch that a lesion might be a “Spitz” nevus.
(white stars) ■ One often has to stretch their imagination to diagnose a “Spitzoid”
■ Bluish-white color (arrows) pattern.
■ Peripheral erythema ■ Experienced dermoscopists might not think this is a “Spitzoid” pattern.
(black stars) ■ It could be considered a multicomponent global pattern.

■ The melanoma-specific criteria are striking and enough to make the


diagnosis without further categorization as “Spitzoid.”
■ This was not a “Spitzoid” melanoma histopathologically.

■ The peripheral erythema has no diagnostic significance in this case.


■ One can see a corona of erythema clinically around “Spitz” nevi.

■ In general, a corona of peripheral erythema seen clinically is a clue that

the lesion might be a “Spitz” nevus.

PEARLS
■ “Spitzoid” lesions are very commonly encountered.
■ Any atypical “Spitzoid” dermoscopic and/or histopathologic diagnosis could
actually be a melanoma.
Chapter 3 Trunk and Extremities 193

RISK
❑ Low
❑ Intermediate
❑ High

DIAGNOSIS
❑ Nevus
❑ Seborrheic keratosis
❑ Basal cell carcinoma
❑ Vascular

3-48a
❑ Dermatofibroma
❑ Squamous cell carcinoma
❑ Melanoma
❑ Other

DISPOSITION
❑ No intervention
❑ Follow-up
❑ Histopathologic diagnosis
3-48b

(Reprinted, with permission, from Stolz W, Braun-Falco O, Bilek P et al. (2002) Color Atlas of
Dermatoscopy. Second Edition. Blackwell Publishing, Oxford.)

CASE 48
HISTORY
Over a 6-month period, this black spot on the back of a 5-year-old girl has been getting bigger.
1. Irregular streaks and black color characterize this de novo nodular melanoma.
2. In general, this is a “Spitzoid” global pattern with the sub-category of a “starburst” pattern.
3. Regular streaks are the major clue that this is a Spitz nevus.
4. The black color is diagnostic of a thrombosed hemangioma.
5. The homogenous black color could be created by a “black lamella.”
194 DERMOSCOPY: AN ILLUSTRATED SELF-ASSESSMENT GUIDE

RISK
✔ Low

❑ Intermediate
❑ High

DIAGNOSIS

❑ Nevus

3-48c
❑ Seborrheic keratosis
❑ Basal cell carcinoma
❑ Vascular
❑ Dermatofibroma
❑ Squamous cell carcinoma ANSWERS
❑ Melanoma
Answers: 2,3,5
❑ Other
Discussion:
■ In this age group, acquired black papules are most likely a Spitz nevus, also
called Reed nevi when dark.
DISPOSITION ■ Nodular melanoma is in clinical but not dermoscopic differential.
■ This is a classic “starburst” global pattern.
❑ No intervention
■ Melanomas do not usually demonstrate so much symmetry.
❑ Follow-up
■ The more the asymmetry of color and structure the greater chance the
✔ Histopathologic diagnosis

lesion is high risk (eg, dysplastic or atypical Spitz nevus, melanoma).
■ Regular streaks (sometimes termed “pseudopods” and “radial streaming”)
are finger-like projections along the periphery.
■ Streaks represent variations of the same criterion with the same
DERMOSCOPIC CRITERIA histopathologic correlates:
■ Intraepidermal or junctional confluent radial nest of melanocytes,
■ Symmetry of color and
which can be benign or malignant.
structure ■ The radial growth phase of melanoma.
■ “Starburst” global pattern ■ Even though jet black color may be seen in thrombosed hemangiomas,
■ Regular streaks (arrows) hemangiomas do not show symmetrical streaks at the periphery.
■ Regular black blotch (stars) ■ This lesion could have a “black lamella” created by pigmented parakeratoses.
■ Tape strip to see if fragments of pigmented “black lamella” adhere to tape.

■ Fragments of “black lamella” may or may not stick to the tape.

■ Spitz nevi have been shown to be dynamic, changing patterns over time
(eg, a globular pattern morphing into a “starburst” pattern).
■ Even though this lesion demonstrates striking symmetry of color and
structure, histopathologically it could still be an atypical “Spitzoid” lesion or
melanoma. For this reason a histopathologic diagnosis should be made.
■ Atpyical “Spitzoid” lesions are not uncommonly found in children.

■ “Spitzoid” melanomas can be found in children.

PEARLS
■ One of the major benefits of dermoscopy is patient reassurance.
■ One should take the opportunity to alleviate the anxiety most parents would have
in this situation by confidently making the diagnosis of a benign “Spitz” nevus.
■ Parents should be informed of a very small chance this could be higher risk.
Chapter 3 Trunk and Extremities 195

RISK
❑ Low
❑ Intermediate
❑ High

3-49a
DIAGNOSIS
❑ Nevus
❑ Seborrheic keratosis
❑ Basal cell carcinoma
❑ Vascular
❑ Dermatofibroma
❑ Squamous cell carcinoma
❑ Melanoma
❑ Other

DISPOSITION
❑ No intervention
❑ Follow-up
❑ Histopathologic diagnosis
3-49b

CASE 49
HISTORY
A 45-year-old man was worried about a dark spot on his chest, which started to itch.
1. This is a melanocytic lesion because there are brown globules.
2. There is asymmetry of color and structure and a multicomponent global pattern.
3. This could be considered an atypical “Spitzoid” global pattern with irregular globules and
bluish-white color.
4. Symmetry of color and structure, bluish-white color and regular globules rules out a
melanoma.
5. Significant asymmetry of color and structure and several melanoma-specific local criteria
suggest that this could be a dysplastic nevus or melanoma.
196 DERMOSCOPY: AN ILLUSTRATED SELF-ASSESSMENT GUIDE

RISK
❑ Low
❑ Intermediate
✔ High

1
2

DIAGNOSIS
❑ Nevus
❑ Seborrheic keratosis
❑ Basal cell carcinoma 3
❑ Vascular
❑ Dermatofibroma
❑ Squamous cell carcinoma

❑ Melanoma

3-49c
❑ Other

DISPOSITION ANSWERS
❑ No intervention Answers: 1,2,3,5
❑ Follow-up

❑ Histopathologic diagnosis Discussion:
■ The multicomponent global pattern could also be considered as atypical
“Spitzoid.”
■ Bluish-white color

■ Irregular globules → globular “Spitzoid” pattern


DERMOSCOPIC CRITERIA ■ Both global patterns are high risk and a significant red flag for concern.
■ Asymmetry of color and ■ This atypical “Spitzoid” lesion actually turned out to be a “Spitzoid”
structure (+) melanoma.
■ All atypical “Spitzoid” dermoscopic patterns are not histopathologically
■ Multicomponent global
“Spitzoid” melanomas.
pattern (1,2,3)
■ Blue or bluish-white color is a clue that the lesion might be “Spitzoid.”
■ Irregular dots and globules
■ There is a good clinico–dermoscopic correlation.
(circles) ■ Both clinical and dermoscopic pictures are potentially high risk.
■ Bluish-white color ■ There is a good dermoscopic-pathologic correlation.
(white arrows) ■ The dermoscopy looks bad enough to be a melanoma.
■ Different shades of brown ■ The hypopigmentation is light brown and should not be confused with the
color bony-white color of regression.
■ Hypopigmentation ■ This lesion is more malignant looking than benign but it is not definitively
(black arrows) malignant.
■ Case 39 looks similar to this and turned out to be a dysplastic nevus.

■ Always consider a dermoscopic differential diagnosis.

PEARLS
■ Never tell a patient that they have melanoma 100% because you might be
wrong. Your reputation in the community will suffer if that happens.
■ Always be as positive with the patient as possible even if there is a chance
that they have a melanoma.
■ Put yourself in their shoes and be empathetic.
Chapter 3 Trunk and Extremities 197

RISK
❑ Low
❑ Intermediate
❑ High

DIAGNOSIS
❑ Nevus
❑ Seborrheic keratosis
❑ Basal cell carcinoma
❑ Vascular

3-50a
❑ Dermatofibroma
❑ Squamous cell carcinoma
❑ Melanoma
❑ Other

DISPOSITION
❑ No intervention
❑ Follow-up
❑ Histopathologic diagnosis
3-50b

CASE 50
HISTORY
A 70-year-old Haitian man with Fitzpatrick skin type VI came in to treat some warts. After a total
body skin examination, this pigmented skin lesion was found hidden on his right buttock under
the waistband of his underwear.
1. Scaliness, pigmented pseudofollicular openings, and pseudostreaks are typical of pigmented
seborrheic keratosis in darker-skinned races.
2. Globules identify a melanocytic lesion.
3. The global pattern could be multicomponent or “Spitzoid.”
4. Asymmetry of color and structure, irregular pigment network, irregular dots and globules,
irregular streaks, bluish-white color, and regression put melanoma in the differential diagnosis.
5. An incisional biopsy was consistent with tumoral melanosis (TM) with no suggestion of
melanoma. This is a good dermoscopic–pathologic correlation.
198 DERMOSCOPY: AN ILLUSTRATED SELF-ASSESSMENT GUIDE

RISK 1

❑ Low
❑ Intermediate
✔ High

DIAGNOSIS 2

❑ Nevus 4

❑ Seborrheic keratosis
❑ Basal cell carcinoma 3
❑ Vascular
❑ Dermatofibroma
❑ Squamous cell carcinoma

3-50c

❑ Melanoma
❑ Other
ANSWERS
Answers: 2,3,4
Discussion:
DISPOSITION ■ This in situ melanoma was hidden under the waistband of this dark-skinned
❑ No intervention patient’s underwear.
❑ Follow-up ■ Melanoma is most typically found at acral sites in darker-skinned individuals.

✔ Histopathologic diagnosis
❑ ■ Initial clinical and dermoscopic impression was seborrheic keratosis.
■ A significant number of melanomas have seborrheic keratosis as the primary
diagnosis or is in the differential diagnosis on pathology request forms.
■ Irregular dots and globules thought to be pigmented pseudofollicular openings.
■ Streaks considered pseudostreaks of seborrheic keratosis.

DERMOSCOPIC CRITERIA ■ An Incisional biopsy consistent with tumoral melanosis with no sign of melanoma.
■ This was not a good dermoscopic–pathologic correlation.
■ Asymmetry of color and ■ Tumoral melanosis (TM) is a rare histopathological finding:
structure ■ A nodular or plaque-like accumulation of melanin-laden
■ Multicomponent global macrophages/melanophages in the dermis.
pattern (1,2,3,4) ■ Immunohistochemical stains are important to confirm no melanocytes.

■ Irregular pigment network ■ TM pigmented cells are negative for S-100 and HBM-45.
■ TM pigmented cells are positive for macrophage markers such as CD68.
(red boxes)
■ Most biopsies are to rule out melanoma.
■ Irregular dots and globules
■ There can be a history of melanoma and TM is found in a satellite nodule
(white boxes)
or lymph node mass → metastatic melanoma.
■ Irregular streaks (black arrows) ■ TM may be the residual finding after partial or complete melanoma regression.
■ Bluish-white color (red stars) ■ TM with complete regression should be considered a high risk melanoma.
■ Regression (black stars) ■ A metastatic work-up is indicated.

■ Regular pigment network on ■ Sentinel node biopsy and metastatic work-up negative in this case.

normal skin (circle) ■ Dramatic dermoscopy with well-developed melanoma-specific criteria.


■ An atypical starburst “Spitzoid” global pattern is in the differential diagnosis
since there is bluish-white color and irregular streaks.
■ This was not a “Spitzoid” melanoma histopathologically.

PEARLS
■ People of all skin types deserve total body skin examinations.
■ The incidence of melanoma is increasing in Latinos and African-Americans.
■ White Hispanics and African-Americans tend to have deeper melanomas and/
or metastatic melanoma at the time of diagnosis compared to Caucasians.
■ Dermoscopy is beneficial in all skin types.
■ Dermoscopic findings have been consistent in all skin types.
Chapter 3 Trunk and Extremities 199

RISK
❑ Low
❑ Intermediate
❑ High

DIAGNOSIS
❑ Nevus
❑ Seborrheic keratosis
❑ Basal cell carcinoma

3-51a
❑ Vascular
❑ Dermatofibroma
❑ Squamous cell carcinoma
❑ Melanoma
❑ Other

DISPOSITION
❑ No intervention
❑ Follow-up
❑ Histopathologic diagnosis
3-51b

CASE 51
HISTORY
Eight years ago, a 48-year-old Latin female had a pigmented lesion on her left anterior thigh
excised. Unfortunately, the lab report with the diagnosis was lost. Three years ago the lesion
recurred in the same spot and has been getting progressively darker. The patient did not remember
the name of the doctor who did the excision or the lab where the specimen was sent.
1. The lesion was excised with a histopathologic diagnosis of tumoral melanosis (TM). The
margins were free and there was no evidence of melanoma. No further surgery or work-up is
indicated.
2. Clinically and dermoscopically this should be considered a regressed melanoma even without
histopathologic proof.
3. Widespread regression is diagnostic of melanoma.
4. The irregular black blotch with foci of bluish-white color is diagnostic of melanoma.
5. Reticular depigmentation is seen within and outside of the lesion and has no diagnostic
significance.
200 DERMOSCOPY: AN ILLUSTRATED SELF-ASSESSMENT GUIDE

RISK
❑ Low 1
❑ Intermediate
✔ High

DIAGNOSIS
❑ Nevus
❑ Seborrheic keratosis
2
❑ Basal cell carcinoma
❑ Vascular
❑ Dermatofibroma 3
❑ Squamous cell carcinoma

❑ Melanoma 3

3-51c
❑ Other

DISPOSITION ANSWERS
❑ No intervention Answers: 2,5
❑ Follow-up
Discussion:
✔ Histopathologic diagnosis
❑ ■ Two cases of tumoral melanosis (TM), a rare diagnosis seen within a 2-week
period is remarkable.
■ Perhaps TM is under diagnosed.

■ With clinical and dermoscopic diagnosis of melanoma and histopathologic


diagnosis of TM with regression, contact dermatopathology posthaste!
DERMOSCOPIC CRITERIA ■ There was not a good dermoscopic-pathologic correlation.
■ Asymmetry of color and ■ The block was sent to a pigmented lesion expert for another histopatho-

structure (+) logic opinion which was the same diagnosis → TM, regression without
■ Multicomponent global evidence of melanoma consider melanoma to a depth of 1.8mm.
pattern (1, 2, 3) ■ The original pathology report could not be obtained.

■ Sentinel node biopsy and a metastatic work-up were negative.


■ Irregular dots and globules
(white boxes) ■ There is a differential diagnosis for the irregular black blotch.
■ Atypical melanocytes → melanoma.
■ Irregular black blotch (arrows)
■ TM → melanophages and free melanin in the dermis-> melanoma?
■ Bluish-white color (over the
■ The black globules have a differential diagnosis.
irregular black blotch) ■ Nests of melanocytes → melanoma.
■ Regression (stars) ■ Nests of melanophages → TM.
■ Peppering (circles) ■ The reticular depigmentation seen within and outside of the lesion
■ Reticular depigmentation represents normal skin markings and has no diagnostic significance.
(red boxes) ■ High risk criteria (eg, arborizing vessels and reticular depigmentation) are

significant only when seen within the lesion.

PEARLS
■ Consider excision at the time of the initial visit with any patient that might
have a melanoma to avoid loss to follow-up.
■ Patients can be unreliable and wait long periods of time before having

suspicious lesions, that turn out to be melanoma, removed!


■ TM is not an easy diagnosis to make and a consensus of educated, experienced
clinicians may be needed to decide the proper disposition.
Chapter 3 Trunk and Extremities 201

RISK
❑ Low
❑ Intermediate
❑ High

DIAGNOSIS
❑ Nevus
❑ Seborrheic keratosis
❑ Basal cell carcinoma
❑ Vascular
3-52a

❑ Dermatofibroma
❑ Squamous cell carcinoma
❑ Melanoma
❑ Other

DISPOSITION
❑ No intervention
❑ Follow-up
❑ Histopathologic diagnosis
3-52b

CASE 52
HISTORY
This lesion was found on the calf of a 72-year-old man who plays tennis three times a week in
Florida.
1. Globules identify a melanocytic lesion.
2. There is asymmetry of color and structure and a multicomponent global pattern.
3. There are regular dots, globules, streaks, blotches, and bluish- white color diagnosing a
dysplastic nevus.
4. This could be a collision tumor.
5. The melanoma-specific criteria are more than enough to diagnose a melanoma.
202 DERMOSCOPY: AN ILLUSTRATED SELF-ASSESSMENT GUIDE

RISK
❑ Low
1
❑ Intermediate
✔ High

2

DIAGNOSIS 1

❑ Nevus
❑ Seborrheic keratosis 3
❑ Basal cell carcinoma
❑ Vascular
❑ Dermatofibroma
❑ Squamous cell carcinoma
3

❑ Melanoma
❑ Other

3-52c
DISPOSITION
❑ No intervention
❑ Follow-up
✔ Histopathologic diagnosis

ANSWERS
Answers: 1,2,4,5

Discussion:
DERMOSCOPIC CRITERIA ■ The clinical suspicion of a melanoma is confirmed with dermoscopy.
■ Asymmetry of color and ■ There is a pigmented and amelanotic component to this melanoma.
structure ■ A collision tumor is in the differential diagnosis.

■ Multicomponent global ■ The melanoma-specific criteria are well developed:


pattern (1,2,3) ■ Asymmetry of color and structure → no mirror images of criteria.

■ Multicomponent global pattern → each section has several criteria.


● Irregular dots and globules
■ Irregular dots and globules → different sizes and shapes, asymmetrical
(circles)
location.
■ Irregular streaks (arrows)
■ Irregular streaks → only in one spot.
■ Irregular black blotch
■ Six colors.
(white stars) ■ Irregular black blotch with overlying bluish-white color → no “veil.”
■ Bluish-white color (over the ■ Regression with peppering.
irregular black blotch) ■ There is a differential diagnosis of the peppering.
■ Regression (black stars) ■ “Pinpoint” vessels if they look red to you.
■ Peppering (red box) ■ “Peppering” if they look gray.

■ Milky-red color in the lower half.

PEARLS
■ This is a straightforward case.
■ One can practice their memorized checklist of points that should be covered
in every case for completeness.
Chapter 3 Trunk and Extremities 203

RISK
❑ Low
❑ Intermediate
❑ High

DIAGNOSIS
❑ Nevus
❑ Seborrheic keratosis
❑ Basal cell carcinoma
❑ Vascular

3-53a
❑ Dermatofibroma
❑ Squamous cell carcinoma
❑ Melanoma
❑ Other

DISPOSITION
❑ No intervention
❑ Follow-up
❑ Histopathologic diagnosis
3-53b

CASE 53
HISTORY
This 40-year-old woman had a nevus adjacent to the areola on her left breast for years and was not
aware of any changes, but admits that she does not usually check her skin.
1. This is a melanocytic lesion by default.
2. The presence of a “black lamella” rules out a melanoma.
3. The black blotch is very irregular and it is the most obvious high risk criterion present in the lesion.
4. Remnants of a pigment network and homogenous light brown color are clues that this
melanoma arose in a pre-existing nevus.
5. The clinical lesion and dermoscopic criteria suggest that this is an in situ or thin invasive melanoma.
204 DERMOSCOPY: AN ILLUSTRATED SELF-ASSESSMENT GUIDE

1
RISK
❑ Low
❑ Intermediate
✔ High

2
2

2
DIAGNOSIS
❑ Nevus 3

❑ Seborrheic keratosis
❑ Basal cell carcinoma 3
❑ Vascular 3

3-53c
❑ Dermatofibroma
3
❑ Squamous cell carcinoma

❑ Melanoma
❑ Other
ANSWERS
Answers: 3,4,5

Discussion:
DISPOSITION
■ Pigment network and globules are the criteria that diagnose this as a
❑ No intervention melanocytic lesion. → It is not melanocytic by default.
❑ Follow-up ■ Consider a lesion to be melanocytic “by default” if there are no criteria to
✔ Histopathologic diagnosis
❑ diagnose a melanocytic lesion, seborrheic keratosis, basal cell carcinoma,
dermatofibroma, or vascular lesion.
■ The irregular black blotch is not a “black lamella.”
■ A “black lamella” is a homogeneous black blotch with a shiny appearance.

■ The irregular black blotch is inhomogeneous → broken up.


DERMOSCOPIC CRITERIA ■ Some of the criteria are hard to see:
■ Asymmetry of color and ■ Pigment network is questionably present.
structure ■ Focus of irregular streaks is questionably present.

■ Multicomponent global ■ Bluish-white color is very subtle and could be missed if one is in a hurry.

pattern (1,2,3) ■ Regression is questionably present and could represent normal

■ Irregular pigment network background skin color.


■ Dots and globules are easier to find scattered throughout the lesion.
(black boxes)
■ Irregular dots and globules ■ The irregular black blotch and asymmetry of color and structure are easy to
see and these high risk criteria suggest this is a melanoma.
(white boxes)
■ Section 2 represents the melanoma arising in a dysplastic nevus in Section 3.
■ Irregular streaks (black arrows)
■ There are no criteria to suggest the nevus is dysplastic yet it turned out to
■ Irregular black blotch be dysplastic histopathologically.
(white stars) ■ It was a feature-poor dysplastic nevus without high risk criteria.
■ Bluish-white color ■ One can tell that this is not a deep melanoma by following indications:
(yellow arrows) ■ Flat lesion clinically, not raised or nodular.
■ Regression (black stars) ■ Black and brown color indicating the pathology/pigment is in the epider-

mis and/or the upper dermis.


■ An absence of criteria and colors of a deeper melanoma

(eg, polymorphous vessels, prominent blue color).


■ The foci of bluish-white color are of no diagnostic significance in this lesion.

PEARLS
■ This case is easy to diagnose even for the novice provided they actively study
and practice the technique.
■ Dermoscopy cannot be learned by osmosis.
Chapter 3 Trunk and Extremities 205

RISK
❑ Low
❑ Intermediate
❑ High

DIAGNOSIS
❑ Nevus
❑ Seborrheic keratosis
❑ Basal cell carcinoma
❑ Vascular

3-54a
❑ Dermatofibroma
❑ Squamous cell carcinoma
❑ Melanoma
❑ Other

DISPOSITION
❑ No intervention
❑ Follow-up
❑ Histopathologic diagnosis
3-54b

CASE 54
HISTORY
A 35-year-old woman reported that this small lesion on her abdomen got darker over a 3-month period.
1. While not obvious, the suggestion that there are globules identifies this as a melanocytic lesion.
2. Symmetry of color and structure and regular blotches diagnose a banal acquired nevus.
3. The blotches are not as dramatic as the ones in Case 53, yet they are still irregular and a red
flag for concern.
4. The clinical and dermoscopic differential diagnosis includes a dysplastic nevus or in situ
melanoma.
5. Subtle bluish-white color and erythema diagnose an in situ melanoma.
206 DERMOSCOPY: AN ILLUSTRATED SELF-ASSESSMENT GUIDE

RISK
❑ Low
❑ Intermediate
✔ High

DIAGNOSIS

❑ Nevus
❑ Seborrheic keratosis
❑ Basal cell carcinoma
❑ Vascular

3-54c
❑ Dermatofibroma
❑ Squamous cell carcinoma
❑ Melanoma
❑ Other
ANSWERS
Answers: 1,3,4
Discussion:
DISPOSITION ■ This is a gray zone lesion in which the picture is not so clear.
❑ No intervention ■ The differential diagnosis includes a dysplastic nevus or in situ melanoma.
❑ Follow-up ■ Asymmetry of color and structure and irregular black blotches are the
✔ Histopathologic diagnosis
❑ melanoma-specific criteria that are easy to see.
■ Melanoma-specific criteria are more sensitive and specific for melanoma,

but could be found in benign lesions (eg, banal or dysplastic nevi).


■ The global pattern is homogeneous:
■ Most of the lesion is filled with homogeneous color.
DERMOSCOPIC CRITERIA ■ There are a few areas with local criteria (eg, dots and globules).

■ Asymmetry of color and ■ Could be a multicomponent global pattern with three distinct areas of

structure criteria.
■ Homogeneous global pattern ■ In general, whatever global pattern one finds, it is still essential to determine
■ Irregular dots and globules if it is regular or irregular, good or bad, low or high risk.
(boxes) ■ The irregular globules seen in one of the blotches might be created by
inhomogeneity of the blotch and not true globules.
■ Irregular black blotches
■ The hypopigmented area has an erythematous component:
(black arrows)
■ Pressure with instrumentation blanched away the erythema leaving
■ Bluish-white color (overlying residual hypopigmentation.
the irregular dark blotches) ■ Erythema is commonly found in dysplastic nevi.
■ Hypopigmentation (stars) ■ The high risk criteria that raise a red flag for concern include:
■ Even though it is small, a clinical lesion with the ABCD features.

■ E for evolving/changing is very worrisome and should be added → ABCDE.

■ Asymmetry of color and structure.

■ Irregular black blotches.

■ Diffuse erythema

■ Bluish-white color.

PEARLS
■ Well developed or not, melanoma-specific criteria should not be ignored.
■ Subtle foci of melanoma-specific criteria may be only clue of high risk.
■ Before making a benign diagnosis look for poorly developed high risk criteria.
■ Small melanomas (less than 6 mm) can be diagnosed with attention to history,
clinical appearance, and subtle melanoma-specific criteria.
Chapter 3 Trunk and Extremities 207

RISK
❑ Low
❑ Intermediate
❑ High

DIAGNOSIS
❑ Nevus
❑ Seborrheic keratosis
❑ Basal cell carcinoma
3-55a ❑ Vascular
❑ Dermatofibroma
❑ Squamous cell carcinoma
❑ Melanoma
❑ Other

DISPOSITION
❑ No intervention
❑ Follow-up
❑ Histopathologic diagnosis
3-55b

CASE 55
HISTORY
The husband of this 57-year-old woman found this lesion on her back.
1. A central white patch and peripheral network diagnose a dermatofibroma.
2. A central bluish-white blotch and regular streaks diagnose a “Spitz” nevus.
3. A large crypt, milia-like cysts, and pigmented pseudofollicular openings diagnose a pigmented
seborrheic keratosis.
4. There are no well-developed melanoma-specific criteria in this irritated congenital nevus.
5. Multifocal hypopigmentation, regular peripheral globules, and a regular blotch diagnose an
actively changing nevus.
208 DERMOSCOPY: AN ILLUSTRATED SELF-ASSESSMENT GUIDE

RISK
❑ Low
❑ Intermediate
✔ High

1

2
3
DIAGNOSIS
❑ Nevus
❑ Seborrheic keratosis
❑ Basal cell carcinoma
❑ Vascular

3-55c
❑ Dermatofibroma
❑ Squamous cell carcinoma

❑ Melanoma
❑ Other ANSWERS
Answers: None are correct
Discussion:
■ Critics of dermoscopy might say that one does not need the technique to
DISPOSITION realize this could be a melanoma and they are correct in this case.
❑ No intervention ■ There is no pigment network, and one might argue that the bluish-white color
❑ Follow-up is a variation of the morphology of the white color seen in dermatofibromas.
✔ Histopathologic diagnosis
❑ ■ There are no crypts → large crater-like invaginations filled with yellowish
keratin found in seborrheic keratosis and dermal nevi.
■ There is bluish-white color superimposed on areas of the irregular dark blotch.
■ The streaks are questionably present and irregular
■ Asymmetrically located at points along the periphery.
DERMOSCOPIC CRITERIA ■ With bluish-white color and irregular streaks this could be considered to be
■ Asymmetry of color and an irregular starburst “Spitzoid” global pattern.
structure ■ Differential of larger brown peripheral globules may be irregular streaks.
■ Inter-observer agreement among expert dermoscopists is not always ideal.
■ Multicomponent global
■ Some might consider the globules to be streaks.
pattern (1,2,3)
■ There is widespread regression
■ Irregular dots and globules
■ The milia-like cysts have a differential diagnosis:
(circles) ■ They are created by inhomogeneity of the whitish-colored regression.
■ Irregular streaks (black arrows) ■ True milia-like cysts → milia-like cysts can be found in melanomas.
■ Irregular dark blotch ■ The main dermoscopic features that diagnose this melanoma include:
(yellow arrows) ■ Asymmetry of color and structure
■ Bluish-white color (overlying ■ Multicomponent or irregular starburst “Spitzoid” global pattern

the irregular dark blotch) ■ Irregular dark blotch

■ Regression (stars) ■ Irregular streaks

■ Milia-like cysts (yellow boxes) ■ Bluish-white color → there is no “veil”

■ Multiple colors
■ Five colors

PEARLS
■ Dermoscopy is not needed to diagnose all high risk lesions.
■ Meta-analysis has shown that dermoscopy improves the diagnosis of
melanomas over clinical examination alone by as much as 16%.
■ Though not required in this case, not using dermoscopy risks:
■ Missing melanomas

■ Unnecessary surgery (malignant to benign ratio for dermoscopists is

much better than for nondermoscopists).


Chapter 3 Trunk and Extremities 209

RISK
❑ Low
❑ Intermediate
❑ High

DIAGNOSIS
❑ Nevus
❑ Seborrheic keratosis
❑ Basal cell carcinoma
❑ Vascular

3-56a
❑ Dermatofibroma
❑ Squamous cell carcinoma
❑ Melanoma
❑ Other

DISPOSITION
❑ No intervention
❑ Follow-up
❑ Histopathologic diagnosis
3-56b

CASE 56
HISTORY
This contentious 70-year-old man came in for some scaly spots on his face. He almost refused a
total body skin examination that we routinely offer new patients. It was a surprise to find this
lesion on his lower back.
1. Clinically and dermoscopically, this is highly suspicious for a melanoma.
2. Pigment network and globules identify a melanocytic lesion.
3. There are foci of irregular pigment network.
4. It does not matter if one considers the global pattern to be homogeneous or multicomponent;
it is still regular and low risk.
5. Asymmetry of color and structure, an irregular dark blotch and regression are major features
of this lesion.
210 DERMOSCOPY: AN ILLUSTRATED SELF-ASSESSMENT GUIDE

RISK
❑ Low
❑ Intermediate 1
✔ High

2
2
DIAGNOSIS
❑✔ Nevus
❑ Seborrheic keratosis
2
❑ Basal cell carcinoma
❑ Vascular 2

❑ Dermatofibroma
❑ Squamous cell carcinoma
❑ Melanoma 3

3-56c
❑ Other

DISPOSITION ANSWERS
❑ No intervention Answers: 1,2,3,5
❑ Follow-up
✔ Histopathologic diagnosis
❑ Discussion:
■ The histopathologic diagnosis was a dysplastic nevus:
■ This was not a good clinico–dermoscopic–pathologic correlation.

■ There was a very strong feeling that this was an in situ melanoma.

■ A second histopathologic opinion by a pigmented lesion expert concurred


DERMOSCOPIC CRITERIA with the diagnosis of a dysplastic nevus.
■ Asymmetry of color and ■ In situ melanoma arising in a nevus was in the differential diagnosis.

structure (+) ■ There are well-developed melanoma-specific criteria:


■ Multicomponent global ■ Asymmetry of color and structure

■ Irregular pigment network


pattern (1,2,3)
■ Regression
■ Irregular pigment network
■ Composed of white, gray, and blue color.
(boxes)
■ Regression was not seen clinically or histopathologically.
■ Irregular dots and globules
■ Hyperkeratosis and melanosis can cause bluish-white color → dermoscopic
(circles) differential diagnosis in play.
■ Irregular dark blotch (arrows) ■ The irregular dark blotch is not well developed compared to the ones in
■ Regression (stars) Cases 51, 52, and 53.
■ Bluish-white and gray colors ■ It might not be considered a true blotch but hyperpigmentation.
(within the regression) ■ This is a false positive dermoscopic melanoma.
■ Multiple colors
PEARLS
■ Dermoscopy helps one decide on a surgical approach.
■ Many colleagues routinely shave off atypically pigmented skin lesions.
■ When the suspicion for melanoma is high, we do not recommend a shave
excision in which the chances of getting free margins are not as great as with
a complete excision.
Chapter 3 Trunk and Extremities 211

RISK
❑ Low
❑ Intermediate
❑ High

DIAGNOSIS
❑ Nevus
❑ Seborrheic keratosis
❑ Basal cell carcinoma
❑ Vascular

3-57a
❑ Dermatofibroma
❑ Squamous cell carcinoma
❑ Melanoma
❑ Other

DISPOSITION
❑ No intervention
❑ Follow-up
❑ Histopathologic diagnosis
3-57b

CASE 57
HISTORY
A 19-year-old woman noticed this pigmented lesion on her upper arm a month ago.
1. Clinically and with a nonhomogeneous blue global pattern this could be a blue nevus.
2. Clinically and with bluish-white color this could be a nodular melanoma.
3. A few lacunae-like structures put a vascular lesion in the differential diagnosis.
4. A paucity of local criteria suggests this is a heavily pigmented nodular melanoma.
5. Clinically and dermoscopically this is suspicious for a nodular melanoma, however, one does
not see nodular melanoma in this young age group.
212 DERMOSCOPY: AN ILLUSTRATED SELF-ASSESSMENT GUIDE

RISK
❑ Low
❑ Intermediate
✔ High

DIAGNOSIS
❑ Nevus
❑ Seborrheic keratosis
❑ Basal cell carcinoma
❑ Vascular
❑ Dermatofibroma
❑ Squamous cell carcinoma
❑✔ Melanoma

3-57c
❑ Other

DISPOSITION ANSWERS
❑ No intervention Answers: 1,2,3,4
❑ Follow-up
✔ Histopathologic diagnosis
❑ Discussion:
■ Nodular melanoma can be the easiest diagnosis to make or miss.
■ Always maintain a high index of suspicion with a papule or nodule no

matter what the history, clinical, and/or dermoscopic appearance is.


■ Asymmetry of the shades of bluish-white color is a red flag for concern.
DERMOSCOPIC CRITERIA ■ This is an inhomogeneous homogeneous global pattern.

■ Homogeneous global pattern ■ An absence or paucity of local criteria is often found in nodular melanoma.
■ Bluish-white color (stars) ■ Check the periphery for foci of local criteria.

■ Reddish color (arrows) ■ The histopathologic correlate of the reddish color is not known:
■ It could be erythema.
■ Irregular dots and globules
■ It is not the milky-red color that can be seen in melanoma.
(yellow boxes)
■ The suggestion of a few lacunae-like structures and blue color could be seen
■ Lacunae-like structures
a vascular lesion.
(white boxes)
■ Nodular melanoma
■ Can be found in any age group, young and old alike.

■ Can develop overnight.

■ Grow rapidly or be present for a long time.

■ Be amelanotic, hypomelanotic, or heavily pigmented.

■ Patients may or may not be aware of changes in a long-standing lesion.

PEARLS
■ Include nodular melanoma in your systematic checklist of points to think
about in any nodular lesion.
■ It could be one of the great masqueraders!
■ Foci of local criteria are a red flag for concern in equivocal nodular lesions.
Chapter 3 Trunk and Extremities 213

RISK
❑ Low
❑ Intermediate
❑ High

DIAGNOSIS
❑ Nevus
❑ Seborrheic keratosis
❑ Basal cell carcinoma
❑ Vascular

3-58a
❑ Dermatofibroma
❑ Squamous cell carcinoma
❑ Melanoma
❑ Other

DISPOSITION
❑ No intervention
❑ Follow-up
❑ Histopathologic diagnosis
3-58b

CASE 58
HISTORY
This black nodule was clearly an “ugly duckling” lesion found on the back of an 81-year-old man
whose back was covered with typical brown seborrheic keratosis.
1. A sharply demarcated black nodule with very irregular black globules characterizes this
nodular melanoma.
2. Sharp border demarcation, multiple milia-like cysts, and pigmented pseudofollicular openings
characterize this classic seborrheic keratosis.
3. A black nodule with a paucity of local criteria help diagnose this nodular melanoma.
4. Diffuse bluish-white color favors the diagnosis of a melanoma.
5. There are no criteria to suggest that this is a melanoma.
214 DERMOSCOPY: AN ILLUSTRATED SELF-ASSESSMENT GUIDE

RISK
✔ Low

❑ Intermediate
❑ High

DIAGNOSIS
❑ Nevus

❑ Seborrheic keratosis
❑ Basal cell carcinoma
❑ Vascular
❑ Dermatofibroma
❑ Squamous cell carcinoma
❑ Melanoma

3-58c
❑ Other

DISPOSITION ANSWERS
✔ No intervention

Answers: 2,5
❑ Follow-up
❑ Histopathologic diagnosis Discussion:
■ There are no criteria to diagnose a melanocytic lesion, basal cell carcinoma,
dermatofibroma, vascular lesion, squamous cell carcinoma, or anything
other than a classic seborrheic keratosis.
■ Clinically this looks very similar to the nodular melanoma in Case 57, which
DERMOSCOPIC CRITERIA points out the benefits of dermoscopic examination vs clinical examination
■ Sharp border demarcation alone.
■ This was an “ugly duckling” lesion clinically → a red flag for concern.
(black arrows)
■ Nodular melanoma is clearly in the clinical differential diagnosis.
■ Milia-like cysts (boxes)
■ Nodular melanoma is clearly not in the dermoscopic differential diagnosis.
■ Pigmented pseudofollicular
■ Sharp border demarcation can be seen in seborrheic keratosis and melanoma.
openings (white arrows) ■ Multiple milia-like cysts and multiple-pigmented pseudofollicular openings
■ Bluish-white color are not typically found in melanoma.
(yellow arrows) ■ A few milia-like cysts and pseudofollicular openings can be found in

benign and malignant melanocytic lesions.


■ One does not need to find the full constellation of criteria to diagnose
seborrheic keratosis.
■ The novice dermoscopist might have a difficult time deciding if there are
irregular dots and globules or pigmented pseudofollicular openings.
■ The subtle bluish-white and black color are commonly found in seborrheic
keratosis and are not melanoma-specific criteria.
● Seborrheic keratosis can be heavily pigmented.

PEARLS
■ Clinical and/or dermoscopic “ugly duckling” lesions are not always high risk.
■ Just a red flag for concern that deserves your undivided attention.

■ Beware! There are seborrehic keratosis-like melanomas → melanoma incognito.


■ Collision tumors consisting of seborrehic keratosis and melanoma are a

common combination.
Chapter 3 Trunk and Extremities 215

RISK
❑ Low
❑ Intermediate
❑ High

DIAGNOSIS
❑ Nevus
❑ Seborrheic keratosis
❑ Basal cell carcinoma
❑ Vascular

3-59a
❑ Dermatofibroma
❑ Squamous cell carcinoma
❑ Melanoma
❑ Other

DISPOSITION
❑ No intervention
❑ Follow-up
❑ Histopathologic diagnosis
3-59b

CASE 59
HISTORY
A 60-year-old man has this violaceous nodule on his back for many years.
1. Milky-red areas and regression characterize this amelanotic nodular melanoma.
2. The white stellate areas represent reticular depigmentation-(negative/white network).
3. Homogeneous purple color, lacunae, and fibrous septa characterize this hemangioma.
4. One cannot diagnose a vascular lesion because the color is too dark.
5. At least 50% of a lesion should be filled with well-developed lacunae before one can diagnose a
vascular lesion.
216 DERMOSCOPY: AN ILLUSTRATED SELF-ASSESSMENT GUIDE

RISK
✔ Low

❑ Intermediate
❑ High

DIAGNOSIS
❑ Nevus
❑ Seborrheic keratosis
❑ Basal cell carcinoma

❑ Vascular
❑ Dermatofibroma
❑ Squamous cell carcinoma
❑ Melanoma

3-59c
❑ Other

DISPOSITION ANSWERS
✔ No intervention
❑ Answers: 3
❑ Follow-up
❑ Histopathologic diagnosis Discussion:
■ Clinically but not dermoscopically melanoma is in the differential diagnosis.
■ This lesion looks clinically similar to Cases 57 and 58.

■ The history favors of a benign lesion.


■ Patient’s histories are not always accurate but should always be listened to.
DERMOSCOPIC CRITERIA ■ This is not a melanocytic lesion by default because there are criteria for a
■ Homogeneous purple colors vascular lesion:
(stars) ■ Lacunae

■ Lacunae (boxes) ■ Homogeneous purple color

■ Fibrous septa
■ Fibrous septa (arrows)
■ Lacunae are:
■ Not always well developed.

■ May be few in number or diffusely fill a lesion.

■ There is no set number required to diagnose a vascular lesion.

■ Milky-red areas found in melanomas are pinkish-white not purplish in


color, with or without out-of-focus reddish globule-like structures.
■ Reticular depigmentation (white pigment network, negative pigment
network) found in melanoma typically is thinner, whiter, and in sharper
focus.
■ Fibrous septa (white color) are typically found in vascular lesions.
■ Any shade of red or purple color can be found in vascular lesions.
■ The deeper in the dermis that the vascular spaces are the darker the color

will be.

PEARLS
■ In equivocal cases search for foci of melanoma specific-criteria before
diagnosing a benign vascular lesion.
■ If in doubt, cut it out because there are hemangioma-like melanomas and
hemangioma-like cutaneous metastatic melanoma → melanoma incognito.
Chapter 3 Trunk and Extremities 217

RISK
❑ Low
❑ Intermediate
❑ High

DIAGNOSIS
❑ Nevus
❑ Seborrheic keratosis

3-60a
❑ Basal cell carcinoma
❑ Vascular
❑ Dermatofibroma
❑ Squamous cell carcinoma
❑ Melanoma
❑ Other

DISPOSITION
❑ No intervention
❑ Follow-up
❑ Histopathologic diagnosis
3-60b

CASE 60
HISTORY
A 47-year-old physician was very upset when he discovered this new lesion on his right lower
abdomen.
1. Globules identify a melanocytic lesion.
2. The global pattern is “Spitzoid.”
3. There are irregular dots and globules, irregular dark blotches, and reticular depigmentation.
4. An atypical Spitz nevus and “Spitzoid” melanoma are in the differential diagnosis.
5. It is not clear if there are globules or streaks at the periphery and does not matter because both
are high risk.
218 DERMOSCOPY: AN ILLUSTRATED SELF-ASSESSMENT GUIDE

RISK
❑ Low
❑ Intermediate
✔ High

DIAGNOSIS
❑✔ Nevus
❑ Seborrheic keratosis
❑ Basal cell carcinoma
❑ Vascular

3-60c
Dermatofibroma
❑ Squamous cell carcinoma
❑ Melanoma
❑ Other ANSWERS
Answers: 1,2,3,4,5

Discussion:
DISPOSITION ■ Even though the lesion does not demonstrate the ABCD criteria and even before
❑ No intervention it is examined with dermoscopy one should be thinking this could be high risk.
■ A new black lesion in an adult.
❑ Follow-up
■ The global pattern is atypical starburst:
✔ Histopathologic diagnosis

■ Asymmetry of color and structure.
■ Foci of dots, globules, and/or streaks at the periphery.
■ Centrally located reticular depigmentation (eg, negative pigment network).
■ The globules are irregular by definition:
■ Different sizes and shapes
DERMOSCOPIC CRITERIA
■ Asymmetrical location
■ Asymmetry of color and ■ The dark blotches are irregular by definition:
structure ■ Irregular outline
■ “Spitzoid “ global pattern ■ Asymmetrical location

■ Irregular dots and globules ■ Morphology is a variation of classic reticular depigmentation:


(circles) ■ It should be with thinner white lines and with more defined reticulation
■ At times one has to use their imagination to identify criteria that are not classic.
■ Irregular dark blotches
■ Reticular depigmentation can be found in dermatofibromas, dysplastic nevi,
(white arrows)
Spitz nevi and melanomas and is not a differentiating criterion.
■ Reticular depigmentation/ ■ There is not a good dermoscopic–pathologic correlation:
white network (yellow arrows) ■ The dermoscopic pattern in this Spitz nevus is atypical and “Spitzoid” yet
there was no mention of atypia histopathologically.
■ One should communicate with their dermatopathologist and make sure
atypical histopathologic features were not missed.
■ Consider another histopathologic opinion with a “Spitzod” lesion
expert → they are out there.

PEARLS
■ Some believe Spitz nevi are atypical histopathologically and not high risk.
■ Histopathologic criteria that subcategorize Spitz nevi as atypical, potentially
high risk→ melanoma.
■ Ensure pathologist is aware of the criteria in an atypical “Spitzoid” lesion.
■ Sending clinical and dermoscopic images to the pathologist improves
communication and can motivate them to search for high risk features that
could otherwise be missed.
Chapter 3 Trunk and Extremities 219

RISK
❑ Low
❑ Intermediate
❑ High

DIAGNOSIS
❑ Nevus
❑ Seborrheic keratosis
❑ Basal cell carcinoma
❑ Vascular

3-61a
❑ Dermatofibroma
❑ Squamous cell carcinoma
❑ Melanoma
❑ Other

DISPOSITION
❑ No intervention
❑ Follow-up
❑ Histopathologic diagnosis
3-61b

CASE 61
HISTORY
This solitary lesion was found on the anterior thigh of a 53-year-old woman.
1. The clinical appearance and presence of multiple milia-like cysts diagnose a pigmented
seborrheic keratosis.
2. Peripheral globules identify a melanocytic lesion.
3. Bluish-white color and an irregular black blotch filling most of the lesion are red flags for
concern.
4. Milia-like cysts are usually white or yellow and not bluish-white.
5. It is hard to decide if this is a heavily pigmented seborrheic keratosis or a melanoma which is
an indication to make a histopathologic diagnosis.
220 DERMOSCOPY: AN ILLUSTRATED SELF-ASSESSMENT GUIDE

RISK
❑ Low
❑ Intermediate
✔ High
❑ 1

DIAGNOSIS 2
❑ Nevus
❑ Seborrheic keratosis 4
❑ Basal cell carcinoma 3
❑ Vascular
❑ Dermatofibroma
❑ Squamous cell carcinoma

❑ Melanoma 5

3-61c
❑ Other

DISPOSITION ANSWERS
❑ No intervention
Answers: 2,3,4,5
❑ Follow-up
✔ Histopathologic diagnosis
❑ Discussion:
■ The irregular black blotch filling most of the lesion is an immediate cause
for concern.
■ Proceed with focused attention to completely analyze the lesion.

■ With a potentially difficult and/or high risk lesion go directly into your
DERMOSCOPIC CRITERIA organized checklist of features to look for.
■ Asymmetry of color and ■ Step one → melanocytic vs nonmelanocytic
structure ■ This is melanocytic because there are brown globules.

■ Multicomponent global ■ Step two → Global features


pattern (1,2,3,4,5) ■ Asymmetry of color and structure.

■ Multicomponent global pattern with four different areas of criteria.


■ Irregular black and brown dots
and globules (circles) ■ Step three → Local criteria
■ Poorly defined irregular black and brown dots and globules.
■ Irregular bluish-white dots
■ Irregular bluish-white dots and globules.
(boxes)
■ These are not milia-like cysts and probably represent hyperkeratosis
■ Irregular black blotch (arrows) (scale).
■ Bluish-white homogeneous ■ Irregular black blotch is filling most of the lesion.
color (stars) ■ Subtle foci of homogeneous bluish-white color → not a “veil.”

■ Step four → Diagnosis and/or differential diagnosis


■ One cannot be sure if this is a heavily pigmented seborrheic keratosis or

melanoma.
■ Step five → Lesion disposition
■ Make a histopathologic diagnosis since melanoma is in the differential

diagnosis.

PEARL
■ If one gets to the point of confusion, stop and think “If in doubt, cut it out.”
Problem solved!
Chapter 3 Trunk and Extremities 221

RISK
❑ Low
❑ Intermediate
❑ High

DIAGNOSIS
❑ Nevus
❑ Seborrheic keratosis
❑ Basal cell carcinoma
❑ Vascular

3-62a
❑ Dermatofibroma
❑ Squamous cell carcinoma
❑ Melanoma
❑ Other

DISPOSITION
❑ No intervention
❑ Follow-up
❑ Histopathologic diagnosis
3-62b

CASE 62
HISTORY
A 74-year-old woman has this nodule on her sternum for two years.
1. This is a beautiful heart-shaped benign seborrheic keratosis with multiple milia-like cysts and
large pigmented crypts.
2. A central white patch and hyperpigmentation characterize this dermatofibroma.
3. This is a melanocytic lesion by default.
4. Irregular black blotches and an absence of local criteria other than a few irregular vessels at the
periphery help to diagnose a nodular melanoma.
5. The whitish color could be created by hyperkeratosis (scale) or regression.
222 DERMOSCOPY: AN ILLUSTRATED SELF-ASSESSMENT GUIDE

1
RISK
❑ Low
❑ Intermediate 2
✔ High

DIAGNOSIS
❑ Nevus
3
❑ Seborrheic keratosis
❑ Basal cell carcinoma
❑ Vascular
❑ Dermatofibroma 4
❑ Squamous cell carcinoma

❑ Melanoma

3-62c
❑ Other

DISPOSITION ANSWERS
❑ No intervention Answers: 3,4,5
❑ Follow-up
✔ Histopathologic diagnosis
❑ Discussion:
■ An adorable heart-shaped lesion seen clinically is of no significance other
than the fact that it is a black nodule.
■ There is an absence of criteria to diagnose a melanocytic lesion, seborrheic
keratosis, basal cell carcinoma, dermatofibroma, or hemangioma →
DERMOSCOPIC CRITERIA melanocytic by default.
■ Asymmetry of color and ■ Scaliness creates the whitish color.
■ Dry skin significantly diminishes what one can see with dermoscopy.
structure
■ Mineral oil was used to eliminate the dryness → a glob of mineral oil
■ Multicomponent global
covers 90% of the lesion.
pattern (1,2,3,4)
■ The whitish dots are not milia-like cysts and are created by the dry scaly skin.
■ Irregular dark blotches (arrows)
■ Once a melanocytic lesion is diagnosed, the irregular dark blotches, absence
■ Hyperkeratosis (scale) (stars) of local criteria, and polymorphous vessels put nodular melanoma at the top
■ Whitish dots (white boxes) of the diagnostic list.
■ Polymorphous vessels ■ One has to look hard to identify the atypical vessels and for some they
(yellow boxes) would be questionably present.

PEARLS
■ It is essential to eliminate dry scaly skin to get a better dermoscopic view.
■ With a vivid imagination interesting shapes can be see dermoscopically!
■ Hearts, Yorkies, and even invasive melanoma that looks like Mickey

Mouse with two big ears.


■ An interesting observation that adds an aspect of frivolity to a very

serious subject.
Chapter 3 Trunk and Extremities 223

RISK
❑ Low
❑ Intermediate
❑ High

DIAGNOSIS
❑ Nevus
❑ Seborrheic keratosis
❑ Basal cell carcinoma
❑ Vascular

3-63a
❑ Dermatofibroma
❑ Squamous cell carcinoma
❑ Melanoma
❑ Other

DISPOSITION
❑ No intervention
❑ Follow-up
❑ Histopathologic diagnosis
3-63b

CASE 63
HISTORY
This solitary lesion was found on the back of a 40-year-old woman.
1. The irregular round to oval black structures could represent globules of a melanocytic lesion
or pigmented pseudofollicular openings of a seborrheic keratosis.
2. The irregular black blotch could represent invasive melanoma or hyperpigmentation in a
seborrheic keratosis.
3. The diffuse bluish-white color does not help to differentiate a melanoma from a seborrheic
keratosis.
4. The sharp border demarcation does not help differentiate a melanoma from a seborrheic
keratosis.
5. Multiple hairpin-shaped vessels suggest that this is a seborrheic keratosis.
224 DERMOSCOPY: AN ILLUSTRATED SELF-ASSESSMENT GUIDE

RISK
❑ Low 1
✔ Intermediate

❑ High
2

DIAGNOSIS
❑ Nevus 3

❑ Seborrheic keratosis
❑ Basal cell carcinoma 4
❑ Vascular
❑ Dermatofibroma
❑ Squamous cell carcinoma
❑ Melanoma

3-63c
Other

ANSWERS
DISPOSITION Answers: 1,2,3,4,5
❑ No intervention
❑ Follow-up Discussion:
✔ Histopathologic diagnosis ■ Asymmetry of color and structure and the multicomponent global pattern

are not exclusively found in high risk melanocytic lesions.
■ This case points out the need to think in terms of dermoscopic differential
diagnosis.
■ Irregular dark globules of a melanocytic lesion vs pigmented

DERMOSCOPIC CRITERIA pseudofollicular openings of a seborrheic keratosis.


■ The focus of small dots has no diagnostic significance.
■ Asymmetry of color and
■ Irregular black blotch of a melanoma vs hyperpigmentation of a
structure
seborrheic keratosis.
■ Multicomponent global
■ Bluish-white color is nonspecific and can be seen in seborrheic keratosis
pattern (1,2,3,4) and in melanoma.
■ Sharp border demarcation ■ The absence of milia-like cysts does not rule out a seborrheic keratosis.
(black arrows) ■ Polymorphous vessels with some hairpin shapes can be seen in melanoma
■ Pigmented pseudofollicular and seborrheic keratosis.
openings (yellow arrows) ■ One often has to use their imagination to see hairpin shapes.

■ Nonpigmented ■ The hairpin-shaped vessels and nonpigmented pseudofollicular opening


pseudofollicular opening favors the diagnosis of a seborrheic keratosis.
(white box) ■ Pigmented pseudofollicular openings can have a three-dimensional appear-
■ Irregular brown dots ance when seen in vivo that does not show up in digital dermoscopic
(yellow box) images.
■ For the experienced clinician, the diagnosis of an atypical seborrheic kerato-
■ Irregular black blotch
sis should be easier to make than by the novice dermoscopist.
(white arrows)
■ A stuck on clinical appearance seen here is commonly found in
■ Bluish-white color (stars) seborrheic keratosis.
■ Hairpin-shaped vessels
(black boxes)
PEARLS
■ Polymorphous vessels
(red boxes) ■ Beware! Melanomas can have a stuck on clinical appearance similar to
seborrheic keratosis.
■ A collision tumor, melanoma and seborrheic keratosis is in the differential
diagnosis.
Chapter 3 Trunk and Extremities 225

RISK
❑ Low
❑ Intermediate
❑ High

DIAGNOSIS
❑ Nevus
❑ Seborrheic keratosis
❑ Basal cell carcinoma

3-64a
Vascular
❑ Dermatofibroma
❑ Squamous cell carcinoma
❑ Melanoma
❑ Other

DISPOSITION
❑ No intervention
❑ Follow-up
❑ Histopathologic diagnosis
3-64b

CASE 64
HISTORY
You spot this innocuous looking pink macule on the upper back of a 69-year-old man.
1. Globules identify a melanocytic lesion.
2. Regression, polymorphous vessels, and irregular globules diagnose a regressive melanoma.
3. A pink scaly macule with glomerular vessels diagnoses Bowen disease.
4. An absence of pigment network, arborizing vessels, and pigmentation diagnose a pigmented
basal cell carcinoma.
5. A pink lesion seen clinically is nonspecific and the vessels seen with dermoscopy can in many
instances make a specific diagnosis.
226 DERMOSCOPY: AN ILLUSTRATED SELF-ASSESSMENT GUIDE

RISK
❑ Low 1
❑ Intermediate
✔ High

DIAGNOSIS 2
❑ Nevus
❑ Seborrheic keratosis

❑ Basal cell carcinoma
❑ Vascular 3
❑ Dermatofibroma
❑ Squamous cell carcinoma
❑ Melanoma

3-64c
❑ Other

ANSWERS
DISPOSITION Answers: 4,5
❑ No intervention Discussion:
❑ Follow-up ■ This is a nonmelanocytic lesion with asymmetry of color and structure and
✔ Histopathologic diagnosis
❑ a multicomponent global pattern.
■ Pink macules and papules are nonspecific and can be melanocytic,
nonmelanocytic, benign, malignant, or inflammatory.
■ Shiny semi-translucent clinical appearance suggests basal cell carcinoma.

■ Bowen disease (in situ squamous cell carcinoma) typically has:


DERMOSCOPIC CRITERIA ■ Pinkish scaly macules, patches or plaques
■ Asymmetry of color and ■ Not shiny or semi-translucent
structure ■ Contains diffuse or localized glomerular vessels → pinpoint or circular

■ Multicomponent global grouped telectangietatic vessels


pattern (1,2,3) ■ One sees classic arborizing vessels, both small and large bore, branching:
■ Arborizing vessels (arrows) ■ Sharply in focus due to surface location

■ Out-of-focus vessels are not on the surface of a lesion.


■ Brown dots and globules
■ Out-of-focus arborizing vessels are a “red flag” and could be melanoma.
(black circles)
■ Arborizing vessels are not exclusive to basal cell carcinoma. They can be in:
■ Blue globule (red circle)
■ Melanoma
■ Depigmentation (stars)
■ Sun-damaged skin

■ Scar tissue

■ Nevi

■ Sebaceous gland hyperplasia

■ Nonarborizing vessels can also be seen in basal cell carcinomas.


■ Pigmentation is commonly found in basal cell carcinomas.
■ The size, shape, and color are highly variable.

■ No specific pigmentation type is diagnostic of basal cell carcinoma.

■ White color is commonly found in basal cell carcinomas and could be


confused with the bony-white color of regression seen in melanomas.

PEARLS
■ Always keep in mind that the “classic” clinical and/or dermoscopic basal cell
carcinoma could be a melanoma.
■ Be vigilant and always maintain a high-melanoma index of suspicion.
Chapter 3 Trunk and Extremities 227

RISK
❑ Low
❑ Intermediate
❑ High

DIAGNOSIS
❑ Nevus
❑ Seborrheic keratosis
❑ Basal cell carcinoma
❑ Vascular

3-65a
❑ Dermatofibroma
❑ Squamous cell carcinoma
❑ Melanoma
❑ Other

DISPOSITION
❑ No intervention
❑ Follow-up
❑ Histopathologic diagnosis
3-65b

CASE 65
HISTORY
This suspicious lesion was found by an internist on the chest of an 80-year-old man.
1. Milky-red areas, pinpoint and arborizing vessels characterize this amelanotic melanoma.
2. The white area has arborizing vessels and ulceration diagnosing a basal cell carcinoma.
3. The brown area has globules and pigment network diagnosing a nevus.
4. This could be a collision tumor or a melanoma arising in a nevus.
5. A basal cell carcinoma and nevus are never seen in combination in collision tumors.
228 DERMOSCOPY: AN ILLUSTRATED SELF-ASSESSMENT GUIDE

RISK
❑ Low
❑ Intermediate
✔ High

DIAGNOSIS
❑ Nevus
❑ Seborrheic keratosis

❑ Basal cell carcinoma
❑ Vascular
❑ Dermatofibroma
❑ Squamous cell carcinoma
❑ Melanoma

3-65c
❑ Other

DISPOSITION ANSWERS
❑ No intervention
Answers: 2,3,4
❑ Follow-up
✔ Histopathologic diagnosis
❑ Discussion:
■ Strict dermoscopic evaluation diagnoses a collision tumor or melanoma
arising in a nevus.
■ There are classic arborizing vessels and a small ulceration that diagnoses a
basal cell carcinoma.
DERMOSCOPIC CRITERIA ■ Melanoma could also have similar appearing arborizing vessels and
■ Arborizing vessels (arrows) ulceration.
■ Polymorphous vessels ■ The diffuse bony-white color could be seen in a basal cell or melanoma and
(red boxes) is not a differentiating criterion.
■ Ulceration (black box) ■ The irregular brown globules and pigment network are not well developed
■ Irregular brown globules and could be easily overlooked.
(circle) ■ Any combination of benign and/or malignant tumors is theoretically
possible in collision tumors.
■ Regular fine pigment network
■ A basal cell carcinoma and nevus would not be unusual.
(yellow box)
■ There are foci of nonarborizing vessels (eg, polymorphous vessels).
■ Not uncommonly, small telectangietatic vessels with various shapes (eg,

linear, comma-shaped, pinpoint) are also found in basal cell carcinomas


along with arborizing vessels.
■ There are no milky-red globules.
■ The globules are brown and not milky-red in color.

■ There is not a pinkish background surrounding the globules.

PEARL
■ Be aware of the concept of collision tumors so that they are in the
differential diagnosis when the dermoscopic criteria present themselves.
Chapter 3 Trunk and Extremities 229

RISK
❑ Low
❑ Intermediate
❑ High

DIAGNOSIS
❑ Nevus
❑ Seborrheic keratosis
❑ Basal cell carcinoma

3-66a
Vascular
❑ Dermatofibroma
❑ Squamous cell carcinoma
❑ Melanoma
❑ Other

DISPOSITION
❑ No intervention
❑ Follow-up
❑ Histopathologic diagnosis
3-66b

CASE 66
HISTORY
This very small lesion was found on the posterior shoulder of a 45-year-old man.
1. There are classic arborizing vessels of a basal cell carcinoma.
2. The combination of arborizing vessels, white color, and pigment diagnose a pigmented basal
cell carcinoma.
3. The large brown irregular blotch with a focus of bluish-white color is a clue that this might be
a melanoma.
4. Pigment network seen outside of the lesion has no diagnostic significance.
5. In focus arborizing vessels are never seen in basal cell carcinomas.
230 DERMOSCOPY: AN ILLUSTRATED SELF-ASSESSMENT GUIDE

RISK
❑ Low
❑ Intermediate
✔ High

1
2

DIAGNOSIS
❑ Benign Nevus
❑ Seborrheic keratosis
❑ Basal cell carcinoma 2
❑ Vascular
❑ Dermatofibroma 3
❑ Squamous cell carcinoma

❑ Melanoma

3-66c
❑ Other

DISPOSITION ANSWERS
❑ No intervention Answers: 1,3,4
❑ Follow-up
✔ Histopathologic diagnosis
❑ Discussion:
■ Here it is, a melanoma that at first blush looks like a typical basal cell
carcinoma that has:
■ Classic arborizing vessels

■ White color
DERMOSCOPIC CRITERIA ■ Pigmentation

■ Asymmetry of color and ■ One sees fine pigment network surrounding the lesion:
structure ■ A normal skin finding when found outside of the lesion.

■ Multicomponent global ■ More commonly found in darker-skinned races.

pattern (1,2,3) ■ There are clues that this might be a melanoma:


■ Clinically it does not look like a basal cell carcinoma.
■ Irregular brown blotch
■ The large brown blotch with a focus of bluish-white color is not typically
(black arrows)
seen in basal cell carcinomas.
■ Bluish-white color (circle)
■ The white color has a differential diagnosis:
■ Arborizing vessels (red arrows) ■ Regression in a melanoma.
■ Regression (stars) ■ White color typically seen in basal cell carcinomas.
■ Pigment network (boxes) ■ In-focus classic arborizing vessels can also be found in melanomas.
■ Statistically they are much more commonly found in basal cell carcinomas.

PEARLS
■ Congratulate yourself if you diagnose a barely perceptable lesion like this.
■ You have taken dermatology and dermoscopy to a higher level.
■ Focus one’s attention on every patient at all times because patients’ lives are
on the line any time of the day.
Chapter 3 Trunk and Extremities 231

RISK
❑ Low
❑ Intermediate
❑ High

DIAGNOSIS
❑ Nevus
❑ Seborrheic keratosis
❑ Basal cell carcinoma
3-67a
❑ Vascular
❑ Dermatofibroma
❑ Squamous cell carcinoma
❑ Melanoma
❑ Other

DISPOSITION
❑ No intervention
❑ Follow-up
❑ Histopathologic diagnosis
3-67b

(Reproduced, with permission, from Journal of Drugs in Dermatology. New Methods


and Technologies. Nov 2008-Vol 7- Issue 11. Fig 1.)

CASE 67
HISTORY
This nodule on the back of a 63-year-old man was found by his wife while in the swimming pool.
1. Clinically and dermoscopically, this is a classic basal cell carcinoma.
2. Arborizing vessels, blue ovoid nests of pigment, and brown globules diagnose a basal cell
carcinoma.
3. Brown globules identify a melanocytic lesion.
4. Well-developed melanoma-specific criteria suggest that this could be a melanoma.
5. There is a large area of milky-red color with a few milky-red globules.
232 DERMOSCOPY: AN ILLUSTRATED SELF-ASSESSMENT GUIDE

RISK 1

❑ Low 2 2
❑ Intermediate
✔ High

3

DIAGNOSIS
❑ Nevus 3 1
❑ Seborrheic keratosis 1
❑ Basal cell carcinoma
❑ Vascular
❑ Dermatofibroma

3-67c
❑ Squamous cell carcinoma

❑ Melanoma
❑ Other ANSWERS
Answers: 3,4,5

Discussion:
DISPOSITION ■ A dark nodule with a multicolor macular component is not what one would
❑ No intervention see clinically with a basal cell carcinoma.
❑ Follow-up ■ Brown globules identify a melanocytic lesion.
■ They are difficult to see.
✔ Histopathologic diagnosis

■ The lesion is filled with well-developed melanoma-specific criteria.
■ Asymmetry of color and structure ■ Polymorphous vessels

■ Multicomponent global pattern ■ Milky-red area

■ Regression with peppering ■ Six colors

DERMOSCOPIC CRITERIA ■ Bluish-white color

■ Asymmetry of color and ■ Some of the criteria have a differential diagnosis.


■ Bluish-white color of a melanoma vs blue ovoid nests of a basal cell
structure (+)
carcinoma.
■ Multicomponent global
■ The blue color is not ovoid in shape, which goes against the diagnosis
pattern (1,2,3) of a basal cell carcinoma.
■ Irregular brown dots and ■ Arborizing vessels of a basal cell carcinoma vs polymorphous vessels of a
globules (black boxes) melanoma.
■ Bluish-white color (blue stars) ■ The vessels have very little branching, which goes against the diagnosis
■ Regression (black stars) of a basal cell carcinoma.
■ Peppering (yellow boxes) ■ The presence of other shapes (eg, pinpoint, linear) favors the diagnosis

■ Polymorphous vessels of a melanoma.


(black arrows) ■ There is a large milky-red area with a few out-of-focus reddish globules.
■ This favors the diagnosis of a melanoma.
■ Milky-red area (red arrows)
■ Not typically seen in basal cell carcinomas.
■ Six colors
■ This could easily be overlooked by the novice dermoscopist.

PEARLS
■ At first blush, one might focus on the basal cell-like linear vessels and blue
color, and diagnose a basal cell carcinoma.
■ It is essential to examine all of the criteria in a lesion before making a
dermoscopic diagnosis.
■ When it is not possible to differentiate a basal cell carcinoma from a
melanoma, consider the lesion to be a melanoma and arrange to make a
histopathologic diagnosis posthaste!
Chapter 3 Trunk and Extremities 233

RISK
❑ Low
❑ Intermediate
❑ High

DIAGNOSIS
❑ Nevus
❑ Seborrheic keratosis
❑ Basal cell carcinoma
❑ Vascular

3-68a
❑ Dermatofibroma
❑ Squamous cell carcinoma
❑ Melanoma
❑ Other

DISPOSITION
❑ No intervention
❑ Follow-up
❑ Histopathologic diagnosis
3-68b

(Reproduced, with permission, from Journal of Drugs in Dermatology. New


Methods and Technologies. Nov 2008-Vol 7- Issue 11. Fig 2.)

CASE 68
HISTORY
This case was e-mailed to you for your opinion. Is it a pigmented basal cell carcinoma or melanoma?
1. Clinically it looks like a melanoma.
2. Pigment network and globules identify a melanocytic lesion.
3. Asymmetry of color and structure and a multicomponent global pattern could be seen in both
pathologies.
4. Lacks the arborizing vessels suggestive of a basal cell carcinoma.
5. The large irregular black blotch, white color with polymorphous vessels favors the diagnosis of
a melanoma.
234 DERMOSCOPY: AN ILLUSTRATED SELF-ASSESSMENT GUIDE

RISK
❑ Low 3
❑ Intermediate
✔ High

1

DIAGNOSIS 2
❑ Nevus
❑ Seborrheic keratosis
❑ Basal cell carcinoma 3
❑ Vascular
2
❑ Dermatofibroma
❑ Squamous cell carcinoma

❑ Melanoma
❑ Other 3
3

3-68c
DISPOSITION
❑ No intervention ANSWERS
❑ Follow-up Answers: 1,2,3,4,5
✔ Histopathologic diagnosis

Discussion:
■ The lesion looks like a melanoma clinically:
■ Central dark blotch

■ ABCD criteria
DERMOSCOPIC CRITERIA ■ Absence of the semi-translucency typically seen in basal cell carcinoma

■ Asymmetry of color and ■ There are well-developed melanoma-specific criteria:


■ Asymmetry of color and structure
structure
■ Multicomponent global pattern
■ Multicomponent global
■ Both could be seen in basal cell carcinoma or melanoma.
pattern (1,2,3)
■ Irregular black globules ■ Polymorphous vessels
■ Irregular pigment network
■ Irregular black blotches ■ Six colors
(black boxes) ■ Regression
■ Irregular black globules ■ The diffuse white color has a differential diagnosis:
(yellow boxes) ■ Bony-white color of regression seen in melanoma
■ Irregular black blotches ■ White color of a basal cell carcinoma
(yellow arrows) ■ Polymorphous vessels seen in the bony-white color favors the diagnosis of a
■ Regression (stars) melanoma.
■ Polymorphous vessels ■ The absence of arborizing vessels goes against the diagnosis of a basal cell
■ Pinpoint (circles) carcinoma.
■ Linear (black arrows)

■ Six colors PEARLS


■ Hyperkeratosis (white box) ■ Even experienced dermoscopists might confuse this with a pigmented basal
cell carcinoma if one does not focus their attention and is in a hurry.
■ Minimal pressure with instrumentation and use of a gel (ultrasound or
hand sanitizer) applied to the skin will allow one to better visualize small
telangiectatic vessels.
■ Small vessels are better visualized with polarizing noncontact dermoscopy
(dermlite 3GEN) when not using gel.
Chapter 3 Trunk and Extremities 235

RISK
❑ Low
❑ Intermediate
❑ High

DIAGNOSIS
❑ Nevus
❑ Seborrheic keratosis
❑ Basal cell carcinoma
❑ Vascular

3-69a
❑ Dermatofibroma
❑ Squamous cell carcinoma
❑ Melanoma
❑ Other

DISPOSITION
❑ No intervention
❑ Follow-up
❑ Histopathologic diagnosis
3-69b

CASE 69
HISTORY
A 76-year-old man was seen by his dermatologist for a skin infection when this lesion was found
on his posterior left upper arm.
1. Globules identify a melanocytic lesion.
2. Milky-red color and reticular depigmentation are clues that this is a melanoma.
3. A central white patch, peripheral pigment network, and arborizing vessels characterize this
dermatofibroma.
4. Regression is seen clinically and dermoscopically.
5. The large area of regression is characterized by milky-red color, homogeneous gray color, and
polymorphous vessels.
236 DERMOSCOPY: AN ILLUSTRATED SELF-ASSESSMENT GUIDE

RISK
❑ Low
❑ Intermediate
✔ High
❑ 1

DIAGNOSIS 2
❑ Nevus
❑ Seborrheic keratosis
❑ Basal cell carcinoma
❑ Vascular
❑ Dermatofibroma
❑ Squamous cell carcinoma 3

❑ Melanoma

3-69c
❑ Other

ANSWERS
DISPOSITION
Answers: 1,2,4,5
❑ No intervention
❑ Follow-up Discussion:
✔ Histopathologic diagnosis
❑ ■ The brown globules are hard to see, nevertheless identify a melanocytic lesion.
■ This could also be considered a melanocytic lesion by default if one does

not agree that there are brown globules.


■ Reticular depigmentation (negative pigment network/white pigment
network/white network) seems to blend in with stellate bony-white scar
DERMOSCOPIC CRITERIA tissue in some areas.
■ Asymmetry of color and ■ Milky-red/pink color fills most of the lesion.
structure ■ Pink color in any form is always a red flag for concern.

■ Multicomponent global ■ However, pink color is not always associated with high risk pathology.

pattern (1,2,3) ■ Asymmetry of color and structure and the multicomponent global pattern
■ Irregular brown dots and are melanoma-specific criteria that can be seen in basal cell carcinoma and
globules (black circles) melanoma.
■ Irregular streaks (black arrows) ■ One focus of irregular streaks is a melanoma-specific criterion but of little
importance in the context of the other well-developed high risk criteria.
■ Reticular depigmentation
■ The area of regression is made up of:
(white boxes)
■ Bony-white color
■ Regression (stars) ■ Milky-red/pink color
■ Polymorphous vessels ■ Polymorphous vessels
(yellow boxes) ■ Pinpoint
■ Homogenous gray color and ■ Linear
peppering (yellow circles) ■ Homogenous gray color and peppering

■ Arborizing vessels (red boxes) ■ An atypical dermatofibroma is in the dermoscopic but not the clinical
differential diagnosis.
■ The arborizing vessels surrounding the lesion are a sign of chronic sun dam-
age and have nothing to do with the lesion itself.

PEARLS
■ It cannot be overemphasized: If there’s pink, stop and think!
■ While not specific, pink color is an independent potentially high risk criterion.
Chapter 3 Trunk and Extremities 237

RISK
❑ Low
❑ Intermediate
❑ High

DIAGNOSIS
❑ Nevus
❑ Seborrheic keratosis
❑ Basal cell carcinoma
❑ Vascular

3-70a
❑ Dermatofibroma
❑ Squamous cell carcinoma
❑ Melanoma
❑ Other

DISPOSITION
❑ No intervention
❑ Follow-up
❑ Histopathologic diagnosis
3-70b

CASE 70
HISTORY
A 64-year-old man was seen for his melanoma follow-up when this lesion was found on his back.
1. A central white patch and pigment network put a dermatofibroma in the differential diagnosis.
2. Pigment network identifies a melanocytic lesion.
3. An amelanotic nodule with a few pinpoint vessels puts melanoma in the differential diagnosis.
4. Slightly irregular pigment network, a hypomelanotic central area, and diffuse erythema put an
irritated nevus in the differential diagnosis.
5. Irregular pigment network and multifocal hypopigmentation put a dysplastic nevus in the
differential diagnosis.
238 DERMOSCOPY: AN ILLUSTRATED SELF-ASSESSMENT GUIDE

RISK
❑ Low
✔ Intermediate

❑ High

DIAGNOSIS

❑ Nevus
❑ Seborrheic keratosis
❑ Basal cell carcinoma
❑ Vascular

3-70c
❑ Dermatofibroma
❑ Squamous cell carcinoma
❑ Melanoma ANSWERS
❑ Other Answers: 1,2,3,4,5

Discussion:
■ Pigment network identifies a melanocytic lesion.
■ It could be considered regular or slightly irregular.
DISPOSITION ■ The brown dots are too small to be considered globules, which would be
❑ No intervention another criteria identifying a melanocytic lesion.
❑ Follow-up ■ Dermatofibromas are nonmelanocytic lesions that can have a pigment
✔ Histopathologic diagnosis
❑ network.
■ The global pattern is reticular-homogeneous.
■ Reticular created by pigment network.

■ Homogeneous created by an amelanotic centrally located dermal nevus.

■ Even though this lesion was found in an adult, this pattern is more
DERMOSCOPIC CRITERIA commonly found on the scalp in children and is also referred to as:
■ Reticular-homogeneous global ■ Targetoid nevus

pattern ■ Reverse fried egg nevus

■ Asymmetry of color and ■ Fried egg nevus has a central dark area.

■ “ Crown on thorns” nevus (our preferred term).


structure
■ Irregular pigment network ■ The differential diagnosis for the hypopigmented areas includes:
■ Regression
(boxes)
■ Hypopigmentation seen in a dermal nevus
■ Irregular dots (black circles)
■ Not uncommonly, dermal nevi have very little if any pigment.
■ Hypopigmentation ■ Multifocal hypopigmentation commonly found in dysplastic nevi
(black stars) ■ A nodule of amelanotic melanoma
■ Regression (white stars) ■ The pinpoint vessels:
■ Pinpoint vessels (red circles) ■ Are nonspecific and could be seen in regression or in a dermal nevus.

■ Might easily be missed since they are not well developed.

■ Diffuse erythema is a nonspecific sign of inflammation that easily blanches


away with pressure from instrumentation.

PEARLS
■ The value of being able to create a dermoscopic differential diagnosis for
global patterns and local criteria is highlighted in this case.
■ “Crown of thorns” nevi are commonly found in the scalp of children and
are usually benign.
■ A “Crown of thorns” nevus in an adult is a red flag for concern.
■ Whatever is seen with dermoscopy, the main goal is to determine if it is
regular or irregular, low or high risk, good or bad.
Chapter 3 Trunk and Extremities 239

RISK
❑ Low
❑ Intermediate
❑ High

DIAGNOSIS
❑ Nevus
❑ Seborrheic keratosis

3-71a
❑ Basal cell carcinoma
❑ Vascular
❑ Dermatofibroma
❑ Squamous cell carcinoma
❑ Melanoma
❑ Other

DISPOSITION
❑ No intervention
❑ Follow-up
❑ Histopathologic diagnosis
3-71b

CASE 71
HISTORY
A pink spot was found on the back of an 85-year-old man
1. There are no criteria to diagnose a melanocytic lesion.
2. There are no criteria to diagnose a seborrheic keratosis.
3. There are no criteria to diagnose a dermatofibroma.
4. There are no criteria to diagnose a vascular lesion.
5. An absence of pigment network within the lesion and the presence of arborizing vessels,
pigmentation, and ulceration diagnose a basal cell carcinoma.
240 DERMOSCOPY: AN ILLUSTRATED SELF-ASSESSMENT GUIDE

RISK
❑ Low
❑ Intermediate
✔ High

DIAGNOSIS
❑ Nevus
❑ Seborrheic keratosis

❑ Basal cell carcinoma
❑ Vascular
❑ Dermatofibroma
❑ Squamous cell carcinoma
❑ Melanoma

3-71c
❑ Other

DISPOSITION ANSWERS
❑ No intervention Answers: 1,2,3,4,5
❑ Follow-up
✔ Histopathologic diagnosis
❑ Discussion:
■ This is a classic basal cell carcinoma.
■ Absence of pigment network within the lesion:

■ There is pigment network at the periphery and outside of the lesion on

the surrounding skin.


DERMOSCOPIC CRITERIA ■ It is not really part of the lesion but could be considered a criterion to

■ Arborizing vessels diagnose a melanocytic lesion → dermoscopic differential diagnosis.


■ Arborizing vessels:
(yellow arrows)
■ They are all small bore.
■ Pigmentation (white arrows)
■ Some branch while most do not.
■ Ulceration (black arrows)
■ Basal cell-like vessels do not always branch, which is a variation of the
■ Pigment network (boxes) morphology that is commonly encountered.
■ Pigmentation:

■ Brown

■ Gray

■ Blue → no leaf-like structures are seen

■ Ulceration with a differential diagnosis:

■ Brown globules of a melanocytic lesion

■ Pigmented pseudofollicular openings of a seborrheic keratosis

PEARLS
■ There are innumerable variations of global patterns and local criteria that
are routinely encountered in daily practice.
■ Being aware of the classic presentations of global patterns and local criteria
plus practice, practice and more practice allows one to identify variations of
morphology that routinely occur.
Chapter 3 Trunk and Extremities 241

RISK
❑ Low
❑ Intermediate
❑ High

DIAGNOSIS
❑ Nevus
❑ Seborrheic keratosis
❑ Basal cell carcinoma
❑ Vascular

3-72a
❑ Dermatofibroma
❑ Squamous cell carcinoma
❑ Melanoma
❑ Other

DISPOSITION
❑ No intervention
❑ Follow-up
❑ Histopathologic diagnosis
3-72b

CASE 72
HISTORY
This lesion was found on the thigh of a 30-year-old woman.
1. This could not be a melanocytic lesion because there is no pigment network.
2. Milia-like cysts, pigmented pseudofollicular openings, and hairpin vessels diagnose a
seborrheic keratosis.
3. Brown globules identify a melanocytic lesion.
4. Pink, gray, white colors and pinpoint vessels characterize a large area of regression.
5. Asymmetry of color and structure, a multicomponent global pattern, regular globules, regular
blotches, and regression characterize this Spitzoid melanoma.
242 DERMOSCOPY: AN ILLUSTRATED SELF-ASSESSMENT GUIDE

RISK
❑ Low
❑ Intermediate 1
✔ High

2

DIAGNOSIS
❑ Nevus 3 4
❑ Seborrheic keratosis
❑ Basal cell carcinoma
❑ Vascular
❑ Dermatofibroma
❑ Squamous cell carcinoma

❑ Melanoma
❑ Other

3-72c
DISPOSITION
❑ No intervention
❑ Follow-up ANSWERS
✔ Histopathologic diagnosis
❑ Answers: 3,4

Discussion:
■ With a quick glance, this lesion looks clinically and dermoscopically similar
to Case 71.
DERMOSCOPIC CRITERIA ■ Brown globules identify a melanocytic lesion.
■ Asymmetry of color and ■ Pigment network is not needed to make that determination.

structure ■ There is one milia-like cyst.


■ Mucoid degeneration found in melanomas could look like a milia-like cyst.
■ Multicomponent global
pattern (1,2,3,4) ■ Pigmented pseudofollicular openings of a seborrheic keratosis are in the
differential diagnosis of the focus of large brown irregular globules and
■ Irregular dots and globules
irregular brown blotches at 3 o’clock.
(circles)
■ The irregular brown blotches are poorly defined with different shades of
■ Irregular dark blotches brown color.
(yellow arrows) ■ The large area of regression is made up of:
■ Regression (stars) ■ Milky-red/pink color ■ Gray homogeneous color without peppering
■ Gray color in the area of ■ Bony-white color ■ Pinpoint vessels
regression (black arrows) ■ It is hard to tell if the dots are brown or red.

■ Pinpoint vessels (box) ■ The absence of arborizing vessels and/or spoke-wheel structures goes against
■ Milia-like cyst (white arrow) the diagnoses of a basal cell carcinoma.
■ Six colors ■ Of the potential six Spitzoid patterns, only the atypical pattern would be in
the differential diagnosis of this case.
■ Histopathologically, there was a nodular component that was not seen
clinically or with dermoscopy.

PEARLS
■ Once this lesion was determined to be melanocytic, it looked more
malignant than benign.
■ The most atypical dermoscopic features could be associated with a benign lesion.
Chapter 3 Trunk and Extremities 243

RISK
❑ Low
❑ Intermediate
❑ High

DIAGNOSIS
❑ Nevus
❑ Seborrheic keratosis
❑ Basal cell carcinoma
❑ Vascular

3-73a
❑ Dermatofibroma
❑ Squamous cell carcinoma
❑ Melanoma
❑ Other

DISPOSITION
❑ No intervention
❑ Follow-up
❑ Histopathologic diagnosis
3-73b

CASE 73
HISTORY
An 84-year-old man with a history of multiple skin cancers had this lesion on his back.
1. A milky-red area, irregular brown globules, polymorphous vessels, and milia-like cysts
characterize this melanoma.
2. Multiple milia-like cysts, pigmented pseudofollicular openings, and hairpin vessels diagnose a
seborrheic keratosis.
3. This could be a collision tumor: a seborrheic keratosis and amelanotic melanoma.
4. The bluish-white color is not diagnostic of a melanoma and could be seen in a seborrheic
keratosis.
5. Multiple milia-like cysts and multiple pigmented pseudofollicular openings help differentiate a
seborrheic keratosis from a melanocytic lesion that can have a few of these criteria.
244 DERMOSCOPY: AN ILLUSTRATED SELF-ASSESSMENT GUIDE

RISK
❑ Low
✔ Intermediate

❑ High

DIAGNOSIS
❑ Nevus 1 2 3

❑ Seborrheic keratosis
❑ Basal cell carcinoma
❑ Vascular
❑ Dermatofibroma
❑ Squamous cell carcinoma
❑ Melanoma

3-73c
Other

DISPOSITION
ANSWERS
❑ No intervention
Answers: 2,3,4,5
❑ Follow-up
✔ Histopathologic diagnosis
❑ Discussion:
■ Asymmetry of color and structure and a multicomponent global pattern are
commonly seen in seborrheic keratosis and are a nonspecific finding of little
significance if there are criteria that definitively diagnose a seborrheic
keratosis.
DERMOSCOPIC CRITERIA ■ Multiple milia-like cysts and pigmented pseudofollicular openings make the
■ Asymmetry of color and diagnosis.
structure ■ The milia-like cysts look like “stars in the sky.”

■ Multicomponent global ■ This seborrheic keratosis is heavily pigmented, which is the norm.
pattern (1,2,3) ■ The milky-red/pink area has a differential diagnosis.
■ Milia-like cysts (circles) ■ Collision tumor:

■ Seborrheic keratosis and amelanotic melanoma


■ Pigmented pseudofollicular
■ Seborrheic keratosis and basal cell carcinoma
openings (arrows)
■ Seborrheic keratosis and eccrine porocarcinoma
■ Hairpin vessels (black box)
■ Focus of irritation
■ Milky-red area (yellow box)
■ One has to use their imagination to diagnose the hairpin vessels.
■ Bluish-white color (stars) ■ Polymorphous vessels (eg, linear, corkscrew) are in the differential diag-

nosis.

PEARLS
■ Classic seborrheic keratosis without areas of potentially high risk pathology
are routinely seen.
■ Collision tumors commonly have seborrheic keratosis as one component.
■ Beware! Melanomas are commonly mistaken for seborrheic keratosis.
Chapter 3 Trunk and Extremities 245

RISK
❑ Low
❑ Intermediate
❑ High

DIAGNOSIS
❑ Nevus
❑ Seborrheic keratosis
❑ Basal cell carcinoma
❑ Vascular
3-74a

❑ Dermatofibroma
❑ Squamous cell carcinoma
❑ Melanoma
❑ Other

DISPOSITION
❑ No intervention
❑ Follow-up
❑ Histopathologic diagnosis
3-74b

CASE 74
HISTORY
A 34-year-old woman noticed a change in a nevus on her arm, which she had for many years.
1. Pigment network and brown globules identify a melanocytic lesion.
2. There is symmetry of color and structure, a globular global pattern, regular pigment network,
and comma-shaped vessels.
3. Milky-red/pink color and pinpoint vessels are a red flag for concern.
4. Foci of the globular global pattern suggest that this melanoma arose in a pre-existing nevus.
5. Grayish homogeneous color without peppering suggests that there is regression in this melanoma.
246 DERMOSCOPY: AN ILLUSTRATED SELF-ASSESSMENT GUIDE

RISK
❑ Low
❑ Intermediate 1
✔ High

3 2

DIAGNOSIS
❑ Nevus 2
❑ Seborrheic keratosis
❑ Basal cell carcinoma
❑ Vascular 4
❑ Dermatofibroma
❑ Squamous cell carcinoma

❑ Melanoma 2
❑ Other 3

DISPOSITION

3-74c
❑ No intervention
❑ Follow-up
✔ Histopathologic diagnosis

ANSWERS
Answers: 1,3,4,5
DERMOSCOPIC CRITERIA Discussion:
■ Asymmetry of color and ■ Clinically and dermoscopically this is clearly a high risk lesion.
■ There would not be a good clinico–dermoscopic–pathologic correlation if
structure
the pathologic report was not melanoma.
■ Multicomponent global
■ Remnants of the cobblestone global pattern indicate that the melanoma
pattern (1,2,3,4)
arose in a pre-existing nevus.
■ Irregular pigment network ■ The differential diagnosis includes reticular depigmentation (white
(black boxes) network/negative pigment network).
■ Irregular purplish blotch ■ The milky-red/pink color with pinpoint vessels are dramatic and the first
(white arrows) clue one might notice that this is a high risk lesion.
■ Cobblestone-like globules ■ If there’s pink stop and think!

(red boxes) ■ There is a component of bluish-white color that can be a component of a

■ Milky-red/pink color (stars) milky-red area.


■ Pinpoint vessels (circles) ■ The milky-red/pink area has a differential diagnosis.
■ Neovascularization in a melanoma.
■ Bluish-white color
■ Amelanotic component of the pigmented melanoma.
(yellow arrows)
■ The irregular purplish blotch is another melanoma-specific criterion that
■ Gray homogeneous color and
has no diagnostic significance.
peppering (black arrows)
■ The homogeneous grayish color with peppering represents foci of regression.

PEARLS
■ No pearls here, this is a clear-cut melanoma both clinically and with
dermoscopy.
■ At this point the diagnosis should be easy.
Chapter 3 Trunk and Extremities 247

RISK
❑ Low
❑ Intermediate
❑ High

DIAGNOSIS
❑ Nevus
❑ Seborrheic keratosis
❑ Basal cell carcinoma
❑ Vascular

3-75a
❑ Dermatofibroma
❑ Squamous cell carcinoma
❑ Melanoma
❑ Other

DISPOSITION
❑ No intervention
❑ Follow-up
❑ Histopathologic diagnosis
3-75b

CASE 75
HISTORY
A 50-year-old woman found this lesion on her thigh one day, when she was in the shower.
1. Comma-shaped vessels, regular blotches, and regular globules diagnose a compound nevus.
2. This is a melanocytic lesion by default.
3. Diffuse bluish-white color, irregular dark blotches, milky-red color, and milky-red globules
diagnose an invasive melanoma.
4. Hairpin vessels, pigmented pseudofollicular openings with sharp border demarcation diagnose
a seborrheic keratosis.
5. Glomerular vessels and pigmentation diagnose pigmented Bowen disease.
248 DERMOSCOPY: AN ILLUSTRATED SELF-ASSESSMENT GUIDE

RISK
❑ Low
❑ Intermediate
✔ High

DIAGNOSIS 3 2
❑ Nevus
❑ Seborrheic keratosis
❑ Basal cell carcinoma
❑ Vascular
❑ Dermatofibroma
❑ Squamous cell carcinoma

❑ Melanoma

3-75c
❑ Other

DISPOSITION ANSWERS
❑ No intervention Answers: 2,3
❑ Follow-up
✔ Histopathologic diagnosis
❑ Discussion:
■ This is a melanocytic lesion by default.
■ There is an absence of criteria to diagnose a melanocytic lesion, seborrheic

keratosis, basal cell carcinoma, dermatofibroma, or hemangioma.


■ Two small dots are not enough to diagnose a melanocytic lesion.
DERMOSCOPIC CRITERIA ■ The irregular dark blotches are clues that this could be a melanocytic lesion.
■ Asymmetry of color and ■ Blotches are bigger than dot and globules.

structure ■ The asymmetry of color and structure and multicomponent global pattern
■ Multicomponent global are minor points in the context of the other high risk criteria.
pattern (1,2,3) ■ Diffuse bluish-white color, milky-red/pink color, and multiple milky-red
■ Irregular dark blotches globules are clues that this is an invasive melanoma.
■ Milky-red globules should not be confused with lacunae of a benign
(white arrows)
hemangioma:
■ Diffuse bluish-white color
■ Lacunae are sharply demarcated vascular spaces, milky-red globules are not.
■ Irregular dots (circle) ■ Glomerular vessels of Bowen disease should be grouped, sharply demarcated
■ Milky-red globules coiled red spots.
(black arrows) ■ There is one milia-like cyst that has no diagnostic significance.
■ Milia-like cyst (red arrow) ■ It could represent mucoid degeneration.

PEARLS
■ If there’s pink stop and think!
■ When there’s white control your fright!
■ If there’s blue they might sue!
■ Words to live by!

■ Nodular melanoma can come out of nowhere and grow rapidly


■ It is an easy diagnosis to make and miss!
■ It competes with amelanotic melanoma for the prize of best masquerader!
Chapter 3 Trunk and Extremities 249

RISK
❑ Low
❑ Intermediate
❑ High

DIAGNOSIS
❑ Nevus
❑ Seborrheic keratosis
❑ Basal cell carcinoma
❑ Vascular

3-76a
❑ Dermatofibroma
❑ Squamous cell carcinoma
❑ Melanoma
❑ Other

DISPOSITION
❑ No intervention
❑ Follow-up
❑ Histopathologic diagnosis
3-76b

CASE 76
HISTORY
This pink scaly slightly raised area was found on the forearm of a 50-year-old-golfer.
1. Pink scaly macules, papules, patches, plaques, or nodules could be melanocytic,
nonmelanocytic, benign, malignant, or inflammatory.
2. Dermoscopy is not helpful to diagnose pink scaly lesions.
3. Pinpoint and glomerular vessels plus pigmentation diagnose a pigmented Bowen disease.
4. Glomerular vessels are pathognomonic for Bowen disease.
5. Irregular linear, glomerular, and pinpoint vessels, brown homogeneous pigmentation and
white color put melanoma in the differential diagnosis.
250 DERMOSCOPY: AN ILLUSTRATED SELF-ASSESSMENT GUIDE

RISK
❑ Low
❑ Intermediate
✔ High

DIAGNOSIS
2
❑ Nevus
❑ Seborrheic keratosis
❑ Basal cell carcinoma 3
❑ Vascular
❑ Dermatofibroma

❑ Squamous cell carcinoma
❑ Melanoma

3-76c
❑ Other

DISPOSITION ANSWERS
❑ No intervention Answers: 1,3,5
❑ Follow-up
✔ Histopathologic diagnosis
❑ Discussion:
■ Pink scaly lesions are very common in the elderly sun-damaged population.
■ Lesions are not always melanocytic by default when there are criteria to
suggest another diagnosis not in that algorithm.
■ Pink scaly lesions could be melanocytic, nonmelanocytic benign, malignant,
DERMOSCOPIC CRITERIA or inflammatory, and dermoscopy is helpful to make a diagnosis.
■ Asymmetry of color and ■ Pink lesions that lack melanoma-specific criteria usually have telangiectatic
structure vessels that help make the diagnosis.
■ Multicomponent global ■ Rarely pink lesions are completely featureless.

■ One might have to look hard to identify and categorize the vessels.
pattern (1,2,3)
■ Homogeneous light brown ■ By definition, glomerular vessels are grouped and coiled.
■ Bowen disease can be:
color (black arrows)
■ Pigmented or nonpigmented
■ Pinpoint vessels (yellow circles)
■ Contain glomerular and/or pinpoint vessels
■ Glomerular vessels ■ Statistically, a pink scaly macule, papule, patch, plaque in an adult containing
(black circles) only pinpoint, and/or glomerular vessels usually is Bowen disease.
■ Irregular linear vessels ■ Amelanotic melanoma always is in the differential diagnosis.
(black box) ■ The focus of irregular linear vessels is not typically seen in Bowen disease
and suggests a melanoma.

PEARLS
■ Vessels seen with dermoscopy are nonspecific, yet have the possibility of
suggesting a specific diagnosis.
■ Arborizing—Basal cell carcinoma ■ Comma—Nevus

■ Glomerular—Bowen disease ■ Linear pinpoint-Clear cell


■ Polymorphous—Melanoma acanthoma
■ Hairpin-Seborrheic keratosis
Chapter 3 Trunk and Extremities 251

RISK
❑ Low
❑ Intermediate
❑ High

DIAGNOSIS
❑ Nevus
❑ Seborrheic keratosis
❑ Basal cell carcinoma
❑ Vascular
❑ Dermatofibroma
❑ Squamous cell carcinoma
❑ Melanoma
❑ Other

DISPOSITION
3-77a
❑ No intervention
❑ Follow-up
❑ Histopathologic diagnosis

CASE 77
HISTORY
This lesion was found in an adult on the thigh and clinically looks exactly like the lesion in Case 76.
1. This lesion is filled with pinpoint vessels.
2. A solitary pink lesion with diffuse pinpoint vessels could represent psoriasis.
3. A solitary pink lesion filled with pinpoint vessels could represent Bowen disease.
4. This lesion also demonstrates asymmetry of color and structure and bluish-white color that
puts amelanotic melanoma in the differential diagnoses.
5. The fat serpiginous purple lines represent underlying blood vessels in a thin-skinned
sun-damaged person.
252 DERMOSCOPY: AN ILLUSTRATED SELF-ASSESSMENT GUIDE

RISK
❑ Low
❑ Intermediate
✔ High

DIAGNOSIS
❑ Nevus
❑ Seborrheic keratosis
❑ Basal cell carcinoma
❑ Vascular
❑ Dermatofibroma

❑ Squamous cell carcinoma
❑ Melanoma

3-77b
❑ Other

DISPOSITION
ANSWERS
Answers: 1,2,3,4,5
❑ No intervention
❑ Follow-up Discussion:
✔ Histopathologic diagnosis
❑ ■ Once again, a solitary pink scaly lesion filled with pinpoint vessels has a
differential diagnosis.
■ In an adult, a lesion on sun-exposed skin with diffuse pinpoint vessels

indicates that the number one possibility is Bowen disease.


■ This is a classic example, the most common presentation of nonpig-
DERMOSCOPIC CRITERIA mented Bowen disease that one will encounter.
■ Asymmetry of color and ■ A plaque of psoriasis could be indistinguishable.
structure ■ One should perform a total body skin examination to discover other

■ Pinpoint vessels (circles) stigmata of psoriasis that might be present and easily clinch the correct
■ Hypopigmentation (black stars) diagnosis.
■ Bluish-white color (blue stars) ■ Amelanotic melanoma is lower on the differential diagnosis list.

■ One would like to see a less homogeneous distribution of polymor-


■ Underlying cutaneous blood
vessels (arrows) phous vessels.
■ A solitary lesion of nonspecific dermatitis can have this clinical and

dermoscopic picture.
■ Pigmented and nonpigmented Bowen disease are said to be diagnosed by
finding glomerular vessels.
■ Glomerular and/or pinpoint vessels can be seen.

■ It might be more common to see pinpoint than true glomerular vessels.

■ Nonpigmented Bowen disease is much more common than the

pigmented variety.
■ There can be a single or multiple lesions.

■ The bluish-white color has no diagnostic significance in this case.

PEARLS
■ Bowen disease is ubiquitous in the elderly and not so elderly population.
■ One of the major benefits of dermoscopy is patient reassurance.
■ When one encounters a solitary pink lesion that could be amelanotic
melanoma but classic vessels of Bowen disease are identified, it would be a
nice gesture to pass that positive information to your patient.
Chapter 3 Trunk and Extremities 253

RISK
❑ Low
❑ Intermediate
❑ High

DIAGNOSIS
❑ Nevus
❑ Seborrheic keratosis
❑ Basal cell carcinoma
❑ Vascular

3-78a
❑ Dermatofibroma
❑ Squamous cell carcinoma
❑ Melanoma
❑ Other

DISPOSITION
❑ No intervention
❑ Follow-up
❑ Histopathologic diagnosis
3-78b

CASE 78
HISTORY
This barely perceptible, asymptomatic pink macule was found on the upper back of a 50-year-old
man while performing a total body skin examination.
1. Clinically and dermoscopically this is a classic example of Bowen disease.
2. Comma and hairpin-shaped vessels help diagnose a seborrheic keratosis.
3. Linear glomerular vessels suggest that this could be a clear cell acanthoma.
4. The string of pearls circular arrangement of glomerular vessels diagnoses a clear cell
acanthoma.
5. Different shades of pink color and milky-red globules put amelanotic melanoma in the
differential diagnosis.
254 DERMOSCOPY: AN ILLUSTRATED SELF-ASSESSMENT GUIDE

RISK
✔ Low

❑ Intermediate
❑ High

DIAGNOSIS
❑ Nevus
❑ Seborrheic keratosis
❑ Basal cell carcinoma
❑ Vascular
❑ Dermatofibroma
❑ Squamous cell carcinoma
❑ Melanoma

3-78c
❑ Other

ANSWERS
DISPOSITION Answers: 3,4,5
❑ No intervention Discussion:
❑ Follow-up ■ Clinically, this is not a typical clinical clear cell acanthoma which usually are:
✔ Histopathologic diagnosis
❑ ■ Well circumscribed ■ Peripheral scale

■ 1-2 cm reddish moist nodules ■ Lower extremities

■ The linear and circular arrangement of glomerular-like vessels immediately


puts clear cell acanthoma at the top of the differential diagnosis list.
■ To date, this pattern of vessels is very sensitive and specific if not diagnostic
DERMOSCOPIC CRITERIA of a clear cell acanthoma.
■ Different shades of pink color ■ A large series of cases is needed to disprove this observation.

■ String of pearls glomerular ■ The vessels are said to be arranged like a string of pearls and in this case
vessels (black arrows) they appear to be forming a circular necklace.
■ Linear glomerular vessels ■ Pinpoint, linear, and comma-shaped vessels are also seen.
■ Some of the vessels form circles or look like hairpins.
(boxes)
■ Pinpoint vessels (circles) ■ The differential diagnosis includes:
■ Psoriasis → Usually has a homogeneous distribution of pinpoint vessels
■ Linear vessels (yellow arrows)
throughout the entire lesion.
■ Comma vessel (yellow box)
■ Amelanotic melanoma → different shades of pink color, pinpoint and linear

vessels, and a few milky-red globules should raise a red flag for concern.
■ Flat seborrheic keratosis → hairpin vessels.

■ This is very low on the differential diagnostic list.

■ One can see flat pink seborrheic keratosis without milia-like cysts,

pseudofollicular openings, or other important criteria.

PEARLS
■ Once again, this case points out how useful dermoscopy can be to help diag-
nose nonspecific pink lesions.
■ Even though there are several different-shaped telangiectatic vessels, the
predominant pattern with linear and a circular arrangement of glomerular
vessels makes the diagnoses.
■ Don’t expect tiny vessels to always fit into specific categories of shapes. There
can be shapes that have never been described and are hard to describe.
■ Don’t get too hung up on the shapes!
Chapter 3 Trunk and Extremities 255

RISK
❑ Low
❑ Intermediate
❑ High

DIAGNOSIS
❑ Nevus
❑ Seborrheic keratosis

3-79a
❑ Basal cell carcinoma
❑ Vascular
❑ Dermatofibroma
❑ Squamous cell carcinoma
❑ Melanoma
❑ Other

DISPOSITION
❑ No intervention
❑ Follow-up
❑ Histopathologic diagnosis
3-79b

CASE 79
HISTORY
The daughter of an 80-year-old man found this reddish lesion on the back of her father one day at
the beach.
1. This is a featureless pink lesion.
2. This is a feature poor pink lesion.
3. There are no melanoma-specific criteria.
4. There are pinpoint, linear, and hairpin vessels.
5. Different shades of pink color and polymorphous vessels could be seen in amelanotic
melanoma and acute lichen planus-like keratosis (LPLK).
256 DERMOSCOPY: AN ILLUSTRATED SELF-ASSESSMENT GUIDE

RISK 1

❑ Low
3
❑ Intermediate
✔ High
❑ 2

DIAGNOSIS
❑ Nevus
❑ Seborrheic keratosis
❑ Basal cell carcinoma
❑ Vascular 4
❑ Dermatofibroma
❑ Squamous cell carcinoma

❑ Melanoma

3-79c
❑ Other

DISPOSITION ANSWERS
❑ No intervention Answers: 2,4,5
❑ Follow-up
✔ Histopathologic diagnosis
❑ Discussion:
■ This could be considered a melanocytic lesion by default.
■ It is not featureless because there are polymorphous vessels.
■ It is feature-poor because there are no other well-developed melanoma-
specific criteria.
DERMOSCOPIC CRITERIA ■ The clinical differential diagnosis includes Bowen disease, basal cell carcino-
■ Asymmetry of color and ma, a pink lichen planus-like keratosis, nonspecific dermatitis and amelan-
structure otic melanoma.
■ Multicomponent global ■ Different shades of pink color are nonspecific and could be seen in
pattern (1,2,3,4) amelanotic melanoma and pink lichen planus-like keratosis.
■ Different shades of pink color ■ Pinpoint and linear vessels are also nonspecific.
■ It might be difficult to differentiate the pinpoint and linear vessels.
■ Pinpoint vessels (black boxes)
■ The presence of hairpin vessels favors the diagnosis of amelanotic
■ Linear vessels (yellow boxes)
melanoma.
■ Hairpin vessels (white boxes) ■ Hairpin vessels are not seen in pink lichen planus-like keratosis.
■ Clothing fibers (arrows) ■ Completely amelanotic 100% featureless melanomas are not commonly
found.
■ Statistically, a lesion that looks like this clinically and dermoscopically will

not be a melanoma.

PEARLS
■ Don’t be surprised that even the most innocuous looking pink lesion with
minimal dermoscopic features could be an amelanotic melanoma.
■ A patient can have multiple similar appearing pink lesions and only one
might turn out to be an amelanotic melanoma.
■ Don’t ignore pink lesions.
■ Don’t fear pink lesions, respect them!
Chapter 3 Trunk and Extremities 257

RISK
❑ Low
❑ Intermediate
❑ High

DIAGNOSIS
❑ Nevus
❑ Seborrheic keratosis
❑ Basal cell carcinoma
❑ Vascular

3-80a
❑ Dermatofibroma
❑ Squamous cell carcinoma
❑ Melanoma
❑ Other

DISPOSITION
❑ No intervention
❑ Follow-up
❑ Histopathologic diagnosis
3-80b

CASE 80
HISTORY
A 65-year-old woman presented with a slightly pruritic, pink scaly papule on her right thigh, which
started a month ago and was slowly getting bigger.
1. Diffuse pinpoint vessels and scale characterize this isolated plaque of psoriasis.
2. Different shades of pink and brown color, pinpoint vessels, and regression diagnose this
amelanotic melanoma.
3. This could be a melanocytic lesion by default.
4. This could be an acute pink lichen planus-like keratosis.
5. Clinically and dermoscopically it is not possible to make a specific diagnosis.
258 DERMOSCOPY: AN ILLUSTRATED SELF-ASSESSMENT GUIDE

RISK
❑ Low
❑ Intermediate
✔ High

2 1

DIAGNOSIS
❑ Nevus
5
❑ Seborrheic keratosis
3
❑ Basal cell carcinoma 4
❑ Vascular
❑ Dermatofibroma

3-80c
Squamous cell carcinoma
❑ Melanoma

❑ Other
ANSWERS
Answers: 3,4,5
DISPOSITION Discussion:
❑ No intervention ■ There is no way to differentiate this nonspecific clinical and dermoscopic
❑ Follow-up picture from amelanotic melanoma.
✔ Histopathologic diagnosis
❑ ■ The history of sudden appearance, slight enlargement, and pruritus favors

an acute lichen planus-like keratosis over a melanoma.


■ Pink lichen-planus like keratosis develop de novo and are not associated
with pre-existing lesions (eg, flat seborrheic keratosis, solar lentigo).
■ Lichen planus-like keratosis can be:
DERMOSCOPIC CRITERIA ■ Acute → less than 3 months

■ Sub-acute → 3 months to a year


■ Asymmetry of color and
■ Chronic → more than a year
structure
■ This is an acute short-lived lichen planus-like keratosis.
■ Multicomponent global
■ Acute lichen planus-like keratoses are usually pink.
pattern (1,2,3,4,5)
■ Solitary lesion.
■ Different shades of pink color ■ Patients can have two or three lesions at the same time.
■ Different shades of brown ■ Patients can develop new lesions at any time in the future.
color (black arrows) ■ Different shades of pink color with or without polymorphous vessels and
■ Pinpoint vessels (circles) scale characterize typical pink lichen planus-like keratosis.
■ Scale (yellow arrows) ■ It is not possible to differentiate pink lichen planus-like keratosis from

■ Normal skin (stars) other feature-poor scaly pink lesions.


■ The white color does not represent regression.
■ Typically some or all of the pink color blanches away with pressure from

instrumentation.

PEARLS
■ Eliminate scale or dryness on a lesion to get a better dermoscopic view.
■ Acute pink lichen planus-like keratosis can be seasonal and one can expect
to see several cases in short order in the same or different patients.
■ A patient’s luck is very important with melanoma survival, especially when a
pink lesion that could be a deep amelanotic melanoma turns out to be an
innocuous pink lichen planus-like keratosis.
■ What a nice and welcomed surprise!
■ Let your patient know how lucky they are.
Chapter 3 Trunk and Extremities 259

RISK
❑ Low
❑ Intermediate
❑ High

DIAGNOSIS
❑ Nevus
❑ Seborrheic keratosis
❑ Basal cell carcinoma
❑ Vascular

3-81a
❑ Dermatofibroma
❑ Squamous cell carcinoma
❑ Melanoma
❑ Other

DISPOSITION
❑ No intervention
❑ Follow-up
❑ Histopathologic diagnosis
3-81b

CASE 81
HISTORY
A 43-year-old woman had this nonchanging, asymptomatic lesion on her right lower leg for 9 months.
1. Brown globules identify a melanocytic lesion.
2. Regression characterized by grayish-white color and peppering fill the lesion.
3. Pinpoint vessels and brown color diagnose pigmented Bowen disease.
4. The red dots represent pinpoint vessels.
5. The clinical and dermoscopic differential diagnosis includes regressive melanoma and a sub-
acute lichen planus-like keratosis.
260 DERMOSCOPY: AN ILLUSTRATED SELF-ASSESSMENT GUIDE

RISK 1
❑ Low
❑ Intermediate
✔ High
❑ 2

DIAGNOSIS
❑ Nevus
❑ Seborrheic keratosis
❑ Basal cell carcinoma 3
❑ Vascular
❑ Dermatofibroma
❑ Squamous cell carcinoma
❑ Melanoma

3-81c
❑ Other

DISPOSITION ANSWERS
❑ No intervention
Answers: 2,4,5
❑ Follow-up
✔ Histopathologic diagnosis
❑ Discussion:
■ The clinical and dermoscopic pictures are worrisome:
■ This is melanoma until proven otherwise with a histopathologic

diagnosis.
■ The differential diagnosis includes sub-acute lichen planus-like keratosis.
DERMOSCOPIC CRITERIA ■ A relatively rare lesion.

■ Asymmetry of color and ■ There are well-developed melanoma-specific criteria:


structure ■ Asymmetry of color and structure

■ Multicomponent global pattern


■ Multicomponent global
■ Pinpoint vessels
pattern (1,2,3)
■ Widespread regression
■ Pinpoint vessels (circles)
■ Sub-acute lichen planus-like keratosis are characterized by:
■ Homogeneous gray color and
■ Dusky-red or violaceous color.
peppering (boxes) ■ Irregularly pigmented with different shades of brown, gray, and/or white
■ Bluish-white color of color.
regression (stars) ■ Have a variable amount of regression with peppering.

■ The older the lesion, one sees more regression with peppering.

■ A small sub-set of lichen planus-like keratosis is thought to represent


immunologically-mediated regression of an existing solar lentigo or flat
seborrheic keratosis.
■ One might see criteria associated with these two pathologies associated

with lichen planus-like keratosis.

PEARL
■ Put lichen planus-like keratosis in your differential diagnosis of regressive
lesions with peppering and you won’t be too surprised when a lesion you are
concerned is a bad melanoma turns out not to be.
Chapter 3 Trunk and Extremities 261

RISK
❑ Low
❑ Intermediate
❑ High

DIAGNOSIS
❑ Nevus
❑ Seborrheic keratosis
❑ Basal cell carcinoma
❑ Vascular

3-82a
❑ Dermatofibroma
❑ Squamous cell carcinoma
❑ Melanoma
❑ Other

DISPOSITION
❑ No intervention
❑ Follow-up
❑ Histopathologic diagnosis
3-82b

CASE 82
HISTORY
You discover this gray macule on the chest of a 75-year-old man with evidence of severe sun damage.
1. The patient should bathe and wash the dirt off.
2. The gray dots represent a classic example of peppering that represents melanophages and free
melanin in the dermis.
3. This is a classic radiation tattoo.
4. Remnants of the fingerprint pattern and peppering suggest that this could be a chronic lichen
planus-like keratosis.
5. The focus of brown globules could represent remnants of the fingerprint pattern.
262 DERMOSCOPY: AN ILLUSTRATED SELF-ASSESSMENT GUIDE

RISK
✔ Low

❑ Intermediate
❑ High

DIAGNOSIS
❑ Nevus
❑ Seborrheic keratosis
❑ Basal cell carcinoma
❑ Vascular
❑ Dermatofibroma

3-82c
Squamous cell carcinoma
❑ Melanoma

❑ Other

ANSWERS
Answers: 2,4,5
DISPOSITION
Discussion:
❑ No intervention ■ If one performs a focused total body skin examination, subtle gray macules
✔ Follow-up
❑ like this are not uncommonly found in people with extensive sun damage.
❑ Histopathologic diagnosis ■ The white color and peppering puts a lesion with regression at the top of the
differential diagnosis list.
■ With the identification of criteria to diagnose a solar lentigo, a chronic
lichen planus-like keratosis now tops the differential diagnostic list.
■ The solar lentigo consists of:
DERMOSCOPIC CRITERIA ■ Irregular tan color.
■ Peppering (black boxes) ■ Remnants of a fingerprint pattern with fine parallel line segments.

■ Solar lentigo (black arrows) ■ The brown globules look like the globules of a melanocytic lesion but

■ Irregular tan color represent broken up remnants of the fingerprint pattern.


■ Fingerprint pattern ■ The white color is seen clinically on the skin surrounding the lesion and
(yellow boxes) does not represent regression.
■ Diffuse white scar-like discoloration is one of the many findings that
■ Brown globules (yellow arrows)
result from chronic sun exposure.
■ Scarred sun-damaged skin
■ For the novice dermoscopist, a histopathologic diagnosis is indicated. With
(stars)
experience, after evaluating all of the criteria one will not feel compelled to
make a histopathologic diagnosis.
■ A radiation tattoo is characterized by:
■ A small perfectly round blue macule.

■ Homogeneous blue color similar to a blue nevus.

■ Several similar lesions can be seen in the same area.

■ The patient will tell you that they had radiation in the area.

PEARLS
■ This is an example of a chronic lichen planus-like keratosis in which there
appears to be regression of a pre-existing lesion.
■ The coexistence of a pre-existing lesion is an important clue that the

lesion might not be a regressive melanoma.


■ In our experience, one does not routinely find evidence of a pre-existing
solar lentigo or flat seborrheic keratosis associated with acute, sub-acute, or
chronic lichen planus-like keratosis.
Chapter 3 Trunk and Extremities 263

RISK
❑ Low
❑ Intermediate
❑ High

DIAGNOSIS
❑ Nevus
❑ Seborrheic keratosis
❑ Basal cell carcinoma
❑ Vascular

3-83a
❑ Dermatofibroma
❑ Squamous cell carcinoma
❑ Melanoma
❑ Other

DISPOSITION
❑ No intervention
❑ Follow-up
❑ Histopathologic diagnosis
3-83b

CASE 83
HISTORY
A 63-year-old female discovered this dark spot on her thigh while drying off in the shower.
1. Clinically and dermoscopically, this is similar to Case 82 and represents a chronic lichen
planus-like keratosis.
2. Globules identify a melanocytic lesion.
3. There are several melanoma-specific criteria without evidence of a solar lentigo or flat
seborrheic keratosis.
4. The focus of bluish-white color in the brown area is a red flag for concern that this could be a
melanoma.
5. Asymmetry of color and structure, a multicomponent global pattern, different shades of
brown color, irregular dots and globules, plus regression characterize this melanoma.
264 DERMOSCOPY: AN ILLUSTRATED SELF-ASSESSMENT GUIDE

RISK
❑ Low
❑ Intermediate
✔ High

1

3 2

DIAGNOSIS
❑ Nevus
❑ Seborrheic keratosis
❑ Basal cell carcinoma
❑ Vascular
1
❑ Dermatofibroma 1
❑ Squamous cell carcinoma

❑ Melanoma

3-83c
❑ Other

DISPOSITION ANSWERS
❑ No intervention Answers: 2,3,4,5
❑ Follow-up
✔ Histopathologic diagnosis
❑ Discussion:
■ Clinically and dermoscopically, this is similar to Case 82.
■ Both are subtle gray macules clinically with regression dermoscopically

but this is where the similarities end.


■ The suggestion of brown color seen clinically is a clue that this might be a
DERMOSCOPIC CRITERIA high risk lesion.
■ Asymmetry of color and ■ There are no criteria associated with a solar lentigo or flat seborrheic
structure keratosis.
■ Multicomponent global ■ The large irregular pigmented area is another clue that this could be a
pattern (1,2,3) melanoma and contains:
■ Different shades of brown color.
■ Irregular dots and globules
■ Poorly-developed irregular dots and globules.
(circles)
■ A focus of bluish-white color.
■ Different shades of brown ■ Regression with gray color and foci of peppering is a nonspecific finding.
color ■ The bluish-white color suggests this could be a melanoma.
■ Regression (black stars) ■ Histopathologically, this was diagnosed as an in situ melanoma arising in a
■ Gray color and peppering compound nevus without any mention of regression.
(boxes) ■ This is not a good dermoscopic-pathologic correlation.

■ Bluish-white color (arrows) ■ One should communicate with the pathologist that there was regression

seen with dermoscopy but it was not mentioned in the pathology report.
■ Further investigation is indicated to rule out a more invasive melanoma

with regression.

PEARLS
■ Take the time to check all gray macules with dermoscopy.
■ Perform focused skin examinations so that they are not overlooked!
■ Once discovered, passing them over could be a deadly mistake!
■ Don’t be lazy!
Chapter 3 Trunk and Extremities 265

RISK
❑ Low
❑ Intermediate
❑ High

DIAGNOSIS
❑ Nevus
❑ Seborrheic keratosis
❑ Basal cell carcinoma

3-84a
Vascular
❑ Dermatofibroma
❑ Squamous cell carcinoma
❑ Melanoma
❑ Other

DISPOSITION
❑ No intervention
❑ Follow-up
❑ Histopathologic diagnosis
3-84b

CASE 84
HISTORY
This lesion was found on the back of a 66-year-old man.
1. Remnants of the fingerprint pattern, milia-like cysts, and regression diagnose a chronic lichen
planus-like keratosis.
2. Pigment network identifies a melanocytic lesion.
3. There is asymmetry of color and structure plus a multicomponent global pattern.
4. Irregular pigment network and widespread regression are the other melanoma-specific criteria.
5. Even though there is widespread regression, the presence of regular pigment network and the
absence of pinpoint vessels rules out a melanoma.
266 DERMOSCOPY: AN ILLUSTRATED SELF-ASSESSMENT GUIDE

RISK 1

❑ Low
❑ Intermediate
✔ High

DIAGNOSIS
2
❑ Nevus 3
❑ Seborrheic keratosis
❑ Basal cell carcinoma
❑ Vascular
❑ Dermatofibroma
❑ Squamous cell carcinoma 2

❑ Melanoma

3-84c
❑ Other

DISPOSITION ANSWERS
❑ No intervention Answers: 2,3,4
❑ Follow-up
✔ Histopathologic diagnosis
❑ Discussion:
■ Clinically the lesion is difficult to see because it is almost completely
regressed.
■ Melanomas can completely regress leaving a residual depigmented area or

nothing at all to see.


DERMOSCOPIC CRITERIA ■ The widespread regression should not be confused with hypopigmentation.
■ Asymmetry of color and ■ Regression has a bony-white color.

■ Hypopigmentation can have different shades of light brown color.


structure
■ Gray color and peppering are not found in hypopigmentation.
■ Multicomponent global
■ The regression is comprised of:
pattern (1,2,3)
■ Bony-white color
■ Irregular pigment network
■ Homogeneous gray color
(boxes) ■ Peppering (very difficult to see)
■ Regression ■ There are only remnants of mildly atypical pigment network.
■ Bony-white color (stars) ■ Another dermoscopist might consider the pigment network to be regular.
■ Homogeneous gray ■ The criterion that looks like milia-like cysts could represent mucoid
color and peppering degeneration.
(black arrows) ■ Milia-like cysts can be found in melanomas.

■ Milia-like cysts (yellow arrows) ■ The presence or absence of pinpoint vessels does not make or break the
diagnosis of a melanoma. None are found in this regressive melanoma.

PEARLS
■ Wood’s light examination in a dark room will help accentuate the bony-
white color of clinical regression and superficially located pigmentation.
■ The Wood’s light can make invisible lesions become visible.
■ If a primary cutaneous melanoma cannot be found, a Wood’s light examina-
tion should be performed in patients that present with metastatic melanoma
to lymph nodes or other organs to try to identify the residual depigmented
area of a completely regressed melanoma.
Chapter 3 Trunk and Extremities 267

RISK
❑ Low
❑ Intermediate
❑ High

DIAGNOSIS
❑ Nevus
❑ Seborrheic keratosis
❑ Basal cell carcinoma
❑ Vascular

3-85a
❑ Dermatofibroma
❑ Squamous cell carcinoma
❑ Melanoma
❑ Other

DISPOSITION
❑ No intervention
❑ Follow-up
❑ Histopathologic diagnosis
3-85b

CASE 85
HISTORY
A 17-year-old male came in for his routine 6-month skin examination. He was a surfer with a history
of extensive sun exposure and a few mildly dysplastic nevi. Digital monitoring found a change in this
long-time-stable lesion on his left upper back. The patient was not aware of the change. He commented
before this lesion was discovered that he did not think he needed skin examinations anymore!
1. Blue ovoid nests, white and brown colors diagnose a pigmented basal cell carcinoma.
2. Brown globules identify a melanocytic lesion.
3. Regular blotches, hypopigmentation, and regular globules diagnose a dysplastic nevus.
4. The bony-white color with blue globules is never seen in true regression.
5. The well-developed melanoma-specific criteria diagnose melanoma with certainty.
268 DERMOSCOPY: AN ILLUSTRATED SELF-ASSESSMENT GUIDE

RISK
❑ Low
❑ Intermediate
✔ High
❑ 1

3
1
DIAGNOSIS 2

❑ Nevus
❑ Seborrheic keratosis
1
❑ Basal cell carcinoma 3
❑ Vascular
❑ Dermatofibroma
❑ Squamous cell carcinoma
❑ Melanoma 3

3-85c
❑ Other

DISPOSITION ANSWERS
❑ No intervention
Answers: 2
❑ Follow-up
✔ Histopathologic diagnosis
❑ Discussion:
■ Brown globules identify a melanocytic lesion.
■ There are well-developed melanoma-specific criteria:
■ Asymmetry of color and structure ■ Irregular brown blotches

■ Multicomponent global pattern ■ Regression


DERMOSCOPIC CRITERIA ■ The irregular brown dots and globules are few and far between, and a minor
■ Asymmetry of color and criterion when compared to the other well-developed high risk criteria.
structure ■ The regression is composed of:
■ Multicomponent global ■ Bony-white color

■ Bluish-gray blotches
pattern (1,2,3)
■ Peppering is absent.
■ Irregular dots and globules
■ The bluish tinge means that the melanophages and free melanin are
(circles)
deeper in the dermis → the Tyndall effect.
■ Irregular dark brown blotches
■ Gray and/or blue color with or without peppering can be seen in regression.
(white arrows) ■ This moderately dysplastic nevus looks more malignant than benign.
■ Regression ■ There is not a good dermoscopic-pathologic correlation since it is very
■ Bony-white color (stars) suspicious for a melanoma.
■ Bluish-gray color (red arrows) ■ A second dermatopathologists opinion was indicated.

■ Another experienced dermatopathologist could not find criteria to

diagnose a melanoma with regression.


■ An experienced dermatopatholgist can differentiate regression associated

with a benign or malignant lesion.

PEARLS
■ The patient commented at the beginning of the consultation that he felt
there was no longer a need for follow-up skin examinations.
■ Patients with a history of dysplastic nevi are at a higher risk to develop
melanoma at any time in the future and should continue with periodic self-
skin examinations and 6-month total body skin examinations, which of
course should include dermoscopy.
Chapter 3 Trunk and Extremities 269

RISK
❑ Low
❑ Intermediate
❑ High

DIAGNOSIS
❑ Nevus
❑ Seborrheic keratosis
❑ Basal cell carcinoma
❑ Vascular

3-86a
❑ Dermatofibroma
❑ Squamous cell carcinoma
❑ Melanoma
❑ Other

DISPOSITION
❑ No intervention
❑ Follow-up
❑ Histopathologic diagnosis
3-86b

CASE 86
HISTORY
A primary care physician found this solitary lesion on the back of a 52-year-old man and referred
him to you for your evaluation.
1. The darks dots and globules could be seen in a melanocytic lesion.
2. The dark dots and globules could be seen in a pigmented basal cell carcinoma.
3. The foci of pinpoint and linear vessels are hard to see and rule out a basal cell carcinoma.
4. This could be a basal cell carcinoma without arborizing vessels.
5. Pigmented pseudofollicular openings and milia-like cysts diagnose a seborrheic keratosis.
270 DERMOSCOPY: AN ILLUSTRATED SELF-ASSESSMENT GUIDE

RISK
❑ Low
❑ Intermediate
✔ High

DIAGNOSIS 2
❑ Nevus
❑ Seborrheic keratosis

❑ Basal cell carcinoma 3
❑ Vascular
❑ Dermatofibroma
❑ Squamous cell carcinoma
❑ Melanoma

3-86c
❑ Other

ANSWERS
DISPOSITION Answers: 1,2,4
❑ No intervention
❑ Follow-up Discussion:
✔ Histopathologic diagnosis
❑ ■ At first blush this is not a classic basal cell carcinoma.
■ There is an absence of well-developed criteria to make the diagnosis.

■ Arborizing vessels ■ Spoke-wheel structures

■ Ulceration

■ The conspicuous irregular grayish-black dots and globules have a differential


DERMOSCOPIC CRITERIA diagnosis.
■ A variation of typical peppering with larger dots and globules.
■ Asymmetry of color and
■ Dots and globules of a melanocytic lesion (eg, melanoma).
structure
■ Pigmentation found in basal cell carcinoma.
■ Multicomponent global
■ There are innumerable variations of shapes and colors of pigmentation that
pattern (1,2,3)
can be found in basal cell carcinomas.
■ Irregular grayish-black dots ■ There is no need to try to categorize pigmentation into special shapes
and globules (circles) and/or descriptions.
■ Pinpoint and linear vessels ■ Leaf-like structures ■ Ovoid nests
(boxes) ■ There are no arborizing vessels.
■ Ulcerations (arrows) ■ The diagnosis of basal cell carcinoma can be made without finding

■ Bluish-white color (stars) arborizing vessels.


■ There are pinpoint and linear vessels, which can be found in basal cell
carcinomas but are more suggestive of a melanocytic lesion.
■ There are poorly defined ulcerations that have a differential diagnosis.
■ Crust formation

■ There are no criteria to diagnose a seborrheic keratosis.

PEARLS
■ In vivo, the pigmentation typically found in basal cell carcinomas appears to
be in the context of a lesion with a semi-translucent clear look, which does
not always come through in digital clinical images.
■ Semi-translucency is an important clue that a lesion could be a basal cell
carcinoma.
■ If pigmented, it could appear like a dirty ice cube.
Chapter 3 Trunk and Extremities 271

RISK
❑ Low
❑ Intermediate
❑ High

DIAGNOSIS
❑ Nevus
❑ Seborrheic keratosis
❑ Basal cell carcinoma
❑ Vascular

3-87a
❑ Dermatofibroma
❑ Squamous cell carcinoma
❑ Melanoma
❑ Other

DISPOSITION
❑ No intervention
❑ Follow-up
❑ Histopathologic diagnosis
3-87b

CASE 87
HISTORY
A 31-year-old man complained that this lesion on his right lower leg that has been present for years
has lately become sensitive.
1. Clinically, the lesion looks like a basal cell carcinoma with a semi-translucent appearance.
However, the polymorphous vessels rule this out.
2. Gray blotches diagnose a melanocytic lesion.
3. Irregular gray blotches and well-developed polymorphous vessels put melanoma in the
differential diagnosis.
4. The differential diagnosis of the two large irregular dark blotches includes ulceration and
pigmented blotches of a melanocytic lesion.
5. Polymorphous and arborizing vessels can be found in basal cell carcinomas.
272 DERMOSCOPY: AN ILLUSTRATED SELF-ASSESSMENT GUIDE

RISK
❑ Low
❑ Intermediate
✔ High

2
DIAGNOSIS
1
❑ Nevus
❑ Seborrheic keratosis

❑ Basal cell carcinoma
3
❑ Vascular
❑ Dermatofibroma 4
❑ Squamous cell carcinoma
❑ Melanoma

3-87c
❑ Other

DISPOSITION ANSWERS
❑ No intervention Answers: 3,4,5
❑ Follow-up
✔ Histopathologic diagnosis
❑ Discussion:
■ The semi-translucent clinical appearance is a clue that the lesion could be a
basal cell carcinoma.
■ This case points out that basal cell carcinomas can have polymorphous
vessels in addition to typical arborizing vessels.
DERMOSCOPIC CRITERIA ■ The irregular gray blotches (homogeneous gray color) are the pigmented
■ Asymmetry of color and component of the basal cell and not a criterion that can diagnose a
structure melanocytic lesion.
■ Multicomponent global ■ There are two components to the ulceration:
■ Ulceration
pattern (1,2,3,4)
■ Dried dark blood
■ Irregular gray blotches
■ There appears to be vessels in the ulceration, which is unexplained and has a
(black boxes)
differential diagnosis:
■ Ulceration (black arrows)
■ An irregular dark blotch of a melanocytic lesion with polymorphous
■ Dried blood in ulcerations vessels.
(stars) ■ Ulcerations can be found in both basal cell carcinomas and melanomas, and
■ Polymorphous vessels are not a differentiating point.
■ Pinpoint (black circles) ■ Statistically, ulcerations are more commonly found in basal cell carcino-
■ Linear (yellow arrows) mas.
■ Hairpin (white box) ■ The final thought after evaluating all of the clinical and dermoscopic criteria
■ Corkscrew (yellow box) is that this could be a melanoma with ulceration and polymorphous vessels,
■ Arborizing (yellow circles) or an unusual basal cell carcinoma with polymorphous vessels and lacking
■ Comma (red circle)
classic arborizing vessels. Either way, prompt histopathologic diagnosis is
indicated.

PEARLS
■ This is a great case to study the different shapes of telangiectatic vessels.
■ Be aware that a sub-set of basal cell carcinomas lack well-developed arboriz-
ing vessels and can contain polymorphous vessels or no vessels at all.
Chapter 3 Trunk and Extremities 273

RISK
❑ Low
❑ Intermediate
❑ High

DIAGNOSIS
❑ Nevus
❑ Seborrheic keratosis
❑ Basal cell carcinoma
❑ Vascular

3-88a
❑ Dermatofibroma
❑ Squamous cell carcinoma
❑ Melanoma
❑ Other

DISPOSITION
❑ No intervention
❑ Follow-up
❑ Histopathologic diagnosis
3-88b

CASE 88
HISTORY
A 64-year-old man noticed this lesion on his back 4 weeks before going to his dermatologist.
1. Clinically and dermoscopically this looks like a compound nevus.
2. A focus of brown globules identifies a melanocytic lesion.
3. The bony- and bluish-white colors do not represent regression because there is no peppering.
4. A central white patch and a fine peripheral pigment network diagnose a dermatofibroma.
5. At least 50% of the lesion is replaced with regression, which is suspicious for a melanoma.
274 DERMOSCOPY: AN ILLUSTRATED SELF-ASSESSMENT GUIDE

RISK
❑ Low 2
1
❑ Intermediate
✔ High

1
2
DIAGNOSIS
❑ Nevus
❑ Seborrheic keratosis
❑ Basal cell carcinoma 1
❑ Vascular
❑ Dermatofibroma 2

❑ Squamous cell carcinoma



❑ Melanoma 3

3-88c
❑ Other

DISPOSITION ANSWERS
❑ No intervention
Answers: 2,5
❑ Follow-up
✔ Histopathologic diagnosis
❑ Discussion:
■ Clinically, the regression is hard to see and dermoscopy makes it glaringly
obvious.
■ The focus of globules is not well developed and one could consider this to
be a melanocytic lesion by default.
DERMOSCOPIC CRITERIA ■ The regression is extensive and consists of bony- and bluish-white colors
■ Asymmetry of color and without peppering.
structure ■ Peppering is not always found in regression.

■ Multicomponent global ■ Asymmetry of color and structure and the multicomponent global pattern
pattern (1,2,3) are the only other well-developed melanoma-specific criteria.
■ Irregular brown globules ■ The focus of brown globules are asymmetrically located, and irregular in
(circle) size and shape.
■ They are a minor component of the entire atypical picture.
■ Regression
■ Dermatofibromas do have significant variation of morphology that can be
■ Bony-white color (black
seen along with the central white patch.
stars) ■ The regression does not fall within the possibility of being considered a
■ Bluish-white color (blue stars)
central white patch.
■ The lesion does not have pigment network.

■ Dermatofibromas do not all have pigment network.

PEARLS
■ Once again, this case points out how much more can be seen with
dermoscopy over naked eye examination or with the other nondermoscopic
magnification clinicians use.
■ Regression by itself is an independent, potentially high risk criterion.
■ The more the regression filling a lesion, the greater the chance that it is a
melanoma.
■ Regression can be found in benign lesions.
Chapter 3 Trunk and Extremities 275

RISK
❑ Low
❑ Intermediate
❑ High

DIAGNOSIS
❑ Nevus
❑ Seborrheic keratosis
❑ Basal cell carcinoma
❑ Vascular

3-89a
❑ Dermatofibroma
❑ Squamous cell carcinoma
❑ Melanoma
❑ Other

DISPOSITION
❑ No intervention
❑ Follow-up
❑ Histopathologic diagnosis
3-89b

CASE 89
HISTORY
This firm papule was found on the right shin of a 48-year-old woman.
1. The pigment network could be seen in a melanocytic lesion or dermatofibroma.
2. The irregular bony-white color could represent regression in a melanoma or an atypical central
white patch of a dermatofibroma.
3. Giant milia-like cysts and pseudofollicular openings diagnose a seborrheic keratosis.
4. The differential diagnosis includes melanoma and an atypical dermatofibroma.
5. Atypical dermatofibromas warrant a histopathologic diagnosis.
276 DERMOSCOPY: AN ILLUSTRATED SELF-ASSESSMENT GUIDE

1
RISK
1
❑ Low
❑ Intermediate 2
✔ High

1

2 2

DIAGNOSIS 1
❑ Nevus 2
3
❑ Seborrheic keratosis
❑ Basal cell carcinoma
❑ Vascular

❑ Dermatofibroma

3-89c
❑ Squamous cell carcinoma
❑ Melanoma
❑ Other
ANSWERS
Answers: 1,2,4,5
DISPOSITION Discussion:
❑ No intervention ■ A strict interpretation of the global pattern and local criteria favors the
❑ Follow-up diagnosis of a melanoma over dermatofibroma.
✔ Histopathologic diagnosis
❑ ■ The history of a long-standing, nonchanging lesion that is firm to

palpation favors the diagnosis of a dermatofibroma.


■ A classic nonatypical dermatofibroma is made up of:
■ Centrally located bony-white homogeneous color.

■ Fine peripheral pigment network.


DERMOSCOPIC CRITERIA ■ This pattern is commonly encountered.

■ Asymmetry of color and ■ When atypical, the white color of a dermatofibroma is indistinguishable
structure from regression seen in melanomas and can take many forms (eg, reticular
■ Multicomponent global depigmentation, multifocal white areas).
pattern (1,2,3) ■ Giant milia-like cysts exist. However, they have sharp well-defined borders that
differ from the white color seen here where the borders are not sharp at all.
■ Regular pigment network
■ One must be aware that there are innumerable variations of morphology
(boxes)
and color that can be found in dermatofibromas.
■ Purplish-white color ■ The diffuse erythema seen clinically blanches away with pressure and is a
(black arrows) nonspecific finding.
■ Fingerprint pattern ■ The irregular purplish-white blotches could represent deep pigmentation
(yellow arrows) of a melanocytic lesion or vascularization that could be seen in a
■ Central white patch (stars) melanocytic lesion or in a dermatofibroma.
■ Ring-like structures (circles) ■ It is not the milky-red color typically seen in melanomas.

■ The pseudofollicular opening-like circles could represent ring-like fragments


of pigment network.
■ Dermatofibromas can have thick ring-like circles making up the pigment

network.
■ The parallel line segments suggesting a fingerprint pattern represent
fragments of the pigment network.

PEARLS
■ Be aware that atypical dermatofibromas exist.
■ In order not to miss dermatofibroma-like melanomas, all atypical der-
matofibromas warrant a histopathologic diagnosis.
Chapter 3 Trunk and Extremities 277

RISK
❑ Low
❑ Intermediate
❑ High

DIAGNOSIS
❑ Nevus
❑ Seborrheic keratosis
❑ Basal cell carcinoma
❑ Vascular

3-90a
❑ Dermatofibroma
❑ Squamous cell carcinoma
❑ Melanoma
❑ Other

DISPOSITION
❑ No intervention
❑ Follow-up
❑ Histopathologic diagnosis
3-90b

CASE 90
HISTORY
A 19-year-old developed white color around a pre-existing nevus.
1. Globules identify a melanocytic lesion.
2. Irregular globules, irregular dark blotches, and regression characterize this regressive
melanoma.
3. A peripheral white patch and irregular pigment network characterize this reverse
dermatofibroma.
4. A symmetrical white halo around the central pigmentation characterizes this halo nevus.
5. Asymmetrical pigmentation that extends outside of a white halo could represent a recurrent
melanoma.
278 DERMOSCOPY: AN ILLUSTRATED SELF-ASSESSMENT GUIDE

RISK
❑ Low
✔ Intermediate

❑ High

DIAGNOSIS

❑ Nevus
❑ Seborrheic keratosis
❑ Basal cell carcinoma
❑ Vascular
❑ Dermatofibroma
❑ Squamous cell carcinoma
❑ Melanoma

3-90c
❑ Other

ANSWERS
DISPOSITION Answers: 1,4,5
❑ No intervention Discussion:
❑ Follow-up ■ The clinical diagnosis of a halo nevus is usually straightforward.
✔ Histopathologic diagnosis
❑ ■ Identify if pigmented component is low, intermediate, or high risk.
■ In most but not all cases melanocytic lesion are low risk (eg, nevus).
■ Halo phenomenon can be found around melanomas.
■ In this case, centrally located criteria are atypical and raise a red flag.
■ Irregular dots and globules with an asymmetric location and irregular size
DERMOSCOPIC CRITERIA and shape
■ Irregular dark brown blotches with an irregular size and shape
■ Irregular dots and globules
(boxes) ■ Low risk criteria in a halo nevus might include a featureless pink color,
homogeneous brown color with/or without regular dots and/or globules.
■ Irregular dark brown blotches
■ The follicular openings and perifollicular hypopigmentation have no
(black arrows)
diagnostic significance.
■ Hypopigmentation (blue stars) ■ Perifollicular hypopigmentation can be seen in congenital melanocytic nevi.
■ Depigmented Halo ■ Recurrent nevus or recurrent melanoma is in the differential diagnosis and
(black stars) the history is important.
■ Follicular openings ■ A halo forming around a pre-existing pigmented skin lesion favors the
(yellow arrows) diagnosis of a halo nevus.
■ Perifollicular hypopigmentation ■ The recurrence of pigmentation in a previously excised melanocytic lesion

(circles) scar could be seen in a recurrent nevus or recurrent melanoma.


■ It is essential to review the original pathology of a recurrent pigmented

skin lesion.
■ Centrally-located pigmentation favors a benign process.
■ Halo phenomenon with asymmetrical pigmentation especially if it extends
outside of the halo, is a red flag that could represent recurrent melanoma.
■ In this case a histopathologic diagnosis is indicated because of atypical
centrally located criteria.
■ To date a reverse dermatofibroma has not been reported.

PEARLS
■ New halo nevi or the development of vitiligo in a patient with a history of
melanoma could be a sign of metastatic melanoma.
■ A metastatic work-up is indicated even if the patient is asymptomatic.
Chapter 3 Trunk and Extremities 279

RISK
❑ Low
❑ Intermediate
❑ High

DIAGNOSIS
❑ Nevus
❑ Seborrheic keratosis
❑ Basal cell carcinoma
❑ Vascular

3-91a
❑ Dermatofibroma
❑ Squamous cell carcinoma
❑ Melanoma
❑ Other

DISPOSITION
❑ No intervention
❑ Follow-up
❑ Histopathologic diagnosis
3-91b

CASE 91
HISTORY
A 24-year-old man had 10 melanocytic nevi on his back and this was one of them. It was clinically
and dermoscopically an “ugly duckling” lesion.
1. This case is similar to Case 89 with regression and an irregular dark blotch.
2. With a multicomponent global pattern, an irregular dark blotch, and irregular globules,
melanoma is in the differential diagnosis.
3. This lesion demonstrates symmetry of color and structure that favors low risk pathology.
4. This is a benign “fried egg” appearing nevus with a centrally located regular dark blotch and
hypopigmentation.
5. This appears to be an “ugly duckling” lesion that warrants an immediate excision to rule out
melanoma.
280 DERMOSCOPY: AN ILLUSTRATED SELF-ASSESSMENT GUIDE

RISK
✔ Low

❑ Intermediate
❑ High 1

DIAGNOSIS 3

❑ Nevus
❑ Seborrheic keratosis
❑ Basal cell carcinoma
❑ Vascular
❑ Dermatofibroma
❑ Squamous cell carcinoma
❑ Melanoma

3-91c
❑ Other

DISPOSITION ANSWERS
✔ No intervention
❑ Answers: 3,4
❑ Follow-up
❑ Histopathologic diagnosis Discussion:
■ Clinically and dermoscopically the lesion looks low risk:
■ Symmetry of color and structure

■ Absence of melanoma-specific criteria

■ One might consider the central dark blotch to be irregular.


DERMOSCOPIC CRITERIA ■ Most “fried egg” appearing nevi are not high risk:
■ Symmetry of color and ■ Beware! Melanoma can have a “fried egg” clinical and/or dermoscopic

structure appearance.
■ There should be melanoma-specific criteria.
■ Multicomponent global
pattern (1,2,3) ■ Hypopigmentation should not be confused with regression.
■ It is not bony-white.
■ Hypopigmentation (stars)
■ It is light brown color.
■ Regular dark brown blotch
■ The focus of barely visible brown dots has no diagnostic significance and
(arrows) could be easily overlooked.
■ Regular brown dots (circle) ■ If one thinks they are globules that would be a criterion to diagnose a
■ White dots (boxes) melanocytic lesion.
■ They are too small to be considered globules.

■ This is melanocytic by default.

■ There are no melanoma-specific criteria.


■ The white specks represent reflection artifact from the mineral oil used to
take the image, and are not milia-like cysts.

PEARLS
■ “Ugly duckling” and “fried egg” appearing melanocytic nevi are not always
high risk.
■ Every case should be evaluated based on its own merits by a full evaluation
of all of the clinical and dermoscopic criteria.
Chapter 3 Trunk and Extremities 281

RISK
❑ Low
❑ Intermediate
❑ High

DIAGNOSIS
❑ Nevus
❑ Seborrheic keratosis
❑ Basal cell carcinoma
❑ Vascular

3-92a
❑ Dermatofibroma
❑ Squamous cell carcinoma
❑ Melanoma
❑ Other

DISPOSITION
❑ No intervention
❑ Follow-up
❑ Histopathologic diagnosis
3-92b

CASE 92
HISTORY
A 34-year-old woman noticed an enlarging skin lesion on her left shoulder. She waited 5 months
before going to her dermatologist.
1. Globules identify a melanocytic lesion.
2. There is asymmetry of color and structure, and a homogeneous global pattern.
3. One cannot be sure whether there is true regression or hypopigmentation.
4. The differential diagnosis includes nodular melanoma, atypical dermatofibroma, and
hypopigmented seborrheic keratosis.
5. This could not be a nodular melanoma because the lesion is dark clinically and light
dermoscopically.
282 DERMOSCOPY: AN ILLUSTRATED SELF-ASSESSMENT GUIDE

RISK
❑ Low
❑ Intermediate
✔ High

DIAGNOSIS
❑ Nevus
❑ Seborrheic keratosis
❑ Basal cell carcinoma
❑ Vascular
❑ Dermatofibroma

3-92c
❑ Squamous cell carcinoma

❑ Melanoma
❑ Other
ANSWERS
Answers: 1,2,3

Discussion:
DISPOSITION ■ One does not need dermoscopy to realize that this is a high risk lesion that
❑ No intervention needs to be removed as soon as possible.
❑ Follow-up ■ There is not a good clinico–dermoscopic correlation, which should be a red
✔ Histopathologic diagnosis
❑ flag for concern.
■ The lesion is dark clinically and hypopigmented dermoscopically.

■ Irregular globules identify a melanocytic lesion.


■ The exact description of the global pattern is debatable and is considered to
be homogeneous because it is practically devoid of local criteria (eg, dots,
DERMOSCOPIC CRITERIA globules, pigment network) and composed mainly of different homoge-
■ Asymmetry of color and neous colors.
■ It could also be considered a multicomponent global pattern.
structure
■ Homogeneous global pattern ■ The diffuse light color is a clue that the lesion could be high risk and has a
differential diagnosis.
■ Irregular dots and globules
■ Hypopigmentation vs regression:
(boxes)
■ The gray color favors regression over hypopigmentation. However, it is
■ Irregular streaks not bony-white that favors hypopigmentation over regression.
(yellow arrows) ■ Thus, one sees that local criteria cannot always be categorized with certainty.
■ Regression: ■ A focus of streaks is debatably present but is of no diagnostic significance.
■ White color (yellow stars) ■ Even with a stretch of one’s imagination, there are no criteria to diagnose a
■ Grayish color (black stars) dermatofibroma or seborrheic keratosis.
■ Hypopigmented melanomas are more difficult to diagnose because there is
usually a paucity of well-developed high risk criteria.
■ Putting the history, and clinical and dermoscopic features together, one can
only come up with one conclusion: This is a melanoma until proven
otherwise and should be removed posthaste.

PEARLS
■ Sequential digital monitoring of a potential nodular melanoma is
contraindicated.
■ Valuable time will be lost that could adversely affect the patients prognosis.
■ This entire clinical scenario would be a perfect example of when one should
consider removing a lesion at first consultation, so that valuable time is not
lost and an unreliable patient is not lost to follow-up.
Chapter 3 Trunk and Extremities 283

RISK
❑ Low
❑ Intermediate
❑ High

DIAGNOSIS
❑ Nevus
❑ Seborrheic keratosis
❑ Basal cell carcinoma
❑ Vascular

3-93a
❑ Dermatofibroma
❑ Squamous cell carcinoma
❑ Melanoma
❑ Other

DISPOSITION
❑ No intervention
❑ Follow-up
❑ Histopathologic diagnosis
3-93b

CASE 93
HISTORY
A 71-year-old man was concerned about this skin lesion on his left upper thigh.
1. Milia-like cysts can be seen clinically suggesting that this irregularly pigmented nodule is a
seborrheic keratosis.
2. Asymmetry of color and structure, irregular dark globules, regression, and bluish-white color
diagnose a nodular melanoma.
3. Multiple milia-like cysts and pseudofollicular openings are the main criteria to diagnose this
seborrheic keratosis.
4. One large pigmented crypt also favors the diagnosis of a seborrheic keratosis.
5. The diffuse bluish-white color is pathognomonic for a melanoma.
284 DERMOSCOPY: AN ILLUSTRATED SELF-ASSESSMENT GUIDE

RISK
✔ Low

❑ Intermediate
❑ High

DIAGNOSIS
❑ Nevus

❑ Seborrheic keratosis
❑ Basal cell carcinoma
❑ Vascular

3-93c
Dermatofibroma
❑ Squamous cell carcinoma
❑ Melanoma
❑ Other ANSWERS
Answers: 1,3,4

Discussion:
DISPOSITION ■ Clinically, but not dermoscopically, this dark nodule is somewhat suspicious
for a nodular melanoma.
✔ No intervention
❑ ■ At times, dermoscopic criteria can be seen clinically and that clue helps
❑ Follow-up point one in the direction of the correct diagnosis.
❑ Histopathologic diagnosis ■ Some dermoscopic criteria that can be seen clinically include:

■ The white milia-like cysts of a seborrheic keratosis.

■ The dark streaks of a Spitz nevus.

■ Peripheral globules in a melanocytic nevus.

■ This is a classic seborrheic keratosis with some, but not all, of the criteria
DERMOSCOPIC CRITERIA used to make the diagnosis:
■ Asymmetry of color and ■ Sharp border demarcation ■ Multiple pigmented and nonpigmented

structure ■ Multiple milia-like cysts pseudofollicular openings


■ Sharp border demarcation ■ Crypts are large pseudofollicular openings which:
(black arrows) ■ Vary in size and shape

■ Are not exclusively seen in seborrheic keratoses


■ Milia-like cysts (white arrows)
■ Are also found in papillomatious melanocytic nevi
■ Pigmented pseudofollicular
■ Absent criteria used to make the diagnosis include:
openings (yellow arrows)
■ Fissures and ridges, which can, but may not always, form a brain-like pattern
■ Nonpigmented
■ Fat-fingers ■ Hairpin vessels
pseudofollicular openings ■ Milia-like cysts can be of an opaque white color or can be brilliant like
(blue arrows) shinning “stars in the sky.”
■ Pigmented crypt (red arrows) ■ Different sizes of these roundish structures are the norm.

■ Pseudofollicular openings are more irregular in size and shape and can be
pigmented or nonpigmented.
■ Oxidation of sebaceous material in the epidermal invaginations results in

a dark color, creating the pigmentation of the openings.


■ The diffuse bluish-white color is a nonspecific finding that is not diagnostic
of a high risk lesion (eg, melanoma).

PEARL
■ It is fun to be able to recognize dermoscopic criteria clinically.
If you can do this, you can feel proud that you have taken your dermoscopic
skills to a higher level. Congratulations!
Chapter 3 Trunk and Extremities 285

RISK
❑ Low
❑ Intermediate
❑ High

DIAGNOSIS
❑ Nevus
❑ Seborrheic keratosis

3-94a
❑ Basal cell carcinoma
❑ Vascular
❑ Dermatofibroma
❑ Squamous cell carcinoma
❑ Melanoma
❑ Other

DISPOSITION
❑ No intervention
❑ Follow-up
❑ Histopathologic diagnosis
3-94b

CASE 94
HISTORY
A 23-year-old woman developed this new nodule that seemed to be growing rapidly.
1. Globules identify a melanocytic lesion.
2. There is significant asymmetry of color and structure, a red flag for concern.
3. Diffuse bluish- and bony-white colors along with peppering diagnoses regression.
4. Irregular dots and globules, an irregular dark blotch, and polymorphous vessels are more
criteria to help diagnose a melanoma.
5. If this was felt to be a benign lesion histopathologically, there would not be a good
dermoscopic-pathologic correlation.
286 DERMOSCOPY: AN ILLUSTRATED SELF-ASSESSMENT GUIDE

RISK
1
❑ Low
❑ Intermediate
✔ High

DIAGNOSIS 2
❑ Nevus
3
❑ Seborrheic keratosis
❑ Basal cell carcinoma
❑ Vascular 3
❑ Dermatofibroma
❑ Squamous cell carcinoma

❑ Melanoma

3-94c
❑ Other

DISPOSITION ANSWERS
❑ No intervention Answers: 1,2,3,4,5
❑ Follow-up
✔ Histopathologic diagnosis
❑ Discussion:
■ Clinically and dermoscopically, this is highly suspicious for a nodular
melanoma: a rapidly growing nodule with dramatic melanoma-specific criteria.
■ It would be very difficult to misdiagnose this as a benign lesion.
■ Melanoma-specific criteria are dramatically present:
DERMOSCOPIC CRITERIA ■ Asymmetry of color and structure

■ Asymmetry of color and ■ Multicomponent global pattern

structure ■ There is so much going on that it is hard to divide the lesion into three

■ Multicomponent global or more separate components, which is a minor point in the context of
pattern (1,2,3) the entire atypical picture:
■ Irregular dots and globules
■ Irregular dots and globules
■ Irregular dark brown blotch
(black boxes)
■ Regression
■ Irregular dark brown blotch ■ The polymorphous vessels are hard to see, but at any rate represent only a
(yellow arrows) minor criterion in the context of the overall atypical picture.
■ Polymorphous vessels ■ There are no differential diagnostic possibilities here other than a bad
(red boxes) melanoma!
■ Regression
■ Bluish-white color (stars) PEARLS
■ Peppering (yellow boxes)
■ Even experienced clinicians could miss this case if their eyes and minds are
closed to the benefits of dermoscopy.
■ It is remarkable how many of the patients we have seen with a history of

melanoma have been told by one or several dermatologists that their


melanomas were nothing to worry about! Invariably, they were never
examined with dermoscopy.
■ Dermoscopy should be like the seat belts in a car. One should not leave
home without using them!
■ Once you have begun to incorporate dermoscopy into your daily practice,
you will fully understand this point: It is impossible to feel comfortable
practicing without it.
Chapter 3 Trunk and Extremities 287

RISK
❑ Low
❑ Intermediate
❑ High

DIAGNOSIS
❑ Nevus
❑ Seborrheic keratosis
❑ Basal cell carcinoma
❑ Vascular

3-95a
❑ Dermatofibroma
❑ Squamous cell carcinoma
❑ Melanoma
❑ Other

DISPOSITION
❑ No intervention
❑ Follow-up
❑ Histopathologic diagnosis
3-95b

CASE 95
HISTORY
This lesion was found on the back of a 19-year-old woman who frequented tanning parlors.
1. Pigment network identifies a melanocytic lesion.
2. The global pattern is reticular.
3. There are foci of irregular pigment network, and irregular dots and globules.
4. Hypopigmentation not regression fills the lesion.
5. There are enough melanoma-specific criteria to diagnose a melanoma.
288 DERMOSCOPY: AN ILLUSTRATED SELF-ASSESSMENT GUIDE

RISK
❑ Low
✔ Intermediate

❑ High

DIAGNOSIS

❑ Nevus
❑ Seborrheic keratosis
❑ Basal cell carcinoma
❑ Vascular
❑ Dermatofibroma

3-95c
❑ Squamous cell carcinoma
❑ Melanoma
❑ Other
ANSWERS
Answers: 1,2,3,4

DISPOSITION Discussion:
❑ No intervention ■ This is a mildly dysplastic nevus without enough criteria to diagnose a
melanoma.
❑ Follow-up
■ This is a classic example of a dysplastic nevus:
✔ Histopathologic diagnosis

■ Asymmetry of color and structure

■ Foci of irregular pigment network and irregular dots and globules

■ No greater than two colors ■ Multifocal hypopigmentation

■ The global pattern is reticular because most of the lesion is filled with
DERMOSCOPIC CRITERIA pigment network.
■ Whatever the global pattern is, criteria must be determined as regular or
■ Reticular global pattern
irregular.
■ Asymmetry of color and ■ In this case the pigment network is irregular without a uniform
structure honeycomb pattern. It is thickened, branched, and broken up.
■ Irregular pigment network ■ The dots and globules are of different sizes and shapes and asymmetri-
(boxes) cally located, making them irregular.
■ Irregular dots and globules ■ Hypopigmentation that fills the lesion should not be confused with regression.
(circles) ■ It is light tan color not bony-white.

■ Multifocal hypopigmentation ■ An experienced dermoscopist might consider this a low risk lesion that does
(stars) not need a histopathologic diagnosis.
■ Sequential digital monitoring would be an alternative to excision.

■ With sequential monitoring, most nevi that look like this do not change.

■ Of those that do change, most, but not all, remain low risk.

■ All nevi that change do not need to be removed if there are no high

risk criteria.
■ If this were felt to be a melanoma histopathologically, there would
not be a good clinico–dermoscopic–pathologic correlation and another
dermatopathologist’s opinion should be obtained.

PEARLS
■ One of the major benefits of dermoscopy is avoiding unnecessary surgery as
would be the case here with an experienced dermoscopist.
■ A novice might decide a lesion like this needs a histopathologic diagnosis.
■ With experience, this clinical scenario will look less and less worrisome to you.
Chapter 3 Trunk and Extremities 289

RISK
❑ Low
❑ Intermediate
❑ High

DIAGNOSIS
❑ Nevus
❑ Seborrheic keratosis
❑ Basal cell carcinoma

3-96a
Vascular
❑ Dermatofibroma
❑ Squamous cell carcinoma
❑ Melanoma
❑ Other

DISPOSITION
❑ No intervention
❑ Follow-up
❑ Histopathologic diagnosis
3-96b

CASE 96
HISTORY
A 61-year-old man noticed this lesion on his left shoulder.
1. Clinically and dermoscopically, this is more worrisome than the last case.
2. This looks more malignant than benign but it is not a clear cut melanoma.
3. There is a large regular dark blotch with regular dots and globules.
4. There is an irregular dark blotch and irregular dots and globules.
5. The differential diagnosis includes a dysplastic nevus or in situ melanoma.
290 DERMOSCOPY: AN ILLUSTRATED SELF-ASSESSMENT GUIDE

RISK 1

❑ Low
❑ Intermediate 2
✔ High

2 3

DIAGNOSIS
3

❑ Nevus 2
❑ Seborrheic keratosis
❑ Basal cell carcinoma
❑ Vascular

3-96c
3
❑ Dermatofibroma
❑ Squamous cell carcinoma
❑ Melanoma
❑ Other ANSWERS
Answers: 1,2,4,5

Discussion:
■ Pigment network and globules identify a melanocytic lesion.
DISPOSITION ■ Even though the pigment network in Case 95 is more widespread and
❑ No intervention irregular, the global picture here is more atypical.
❑ Follow-up ■ The irregular dark brown blotch is much larger, darker with a suggestion
✔ Histopathologic diagnosis
❑ of bluish color in the center.
■ The dots and globules are larger, darker, and numerous.

■ Both lesions have hypopigmentation with regression in the differential

diagnosis. In this case, there is clear cut hypopigmentation with lighter


shades of brown but also a much whiter area.
DERMOSCOPIC CRITERIA ■ It is questionable if the white area is true regression because it is not whiter
■ Asymmetry of color and than the surrounding skin.
structure ■ Not uncommon, one can see normal skin within a melanocytic lesion,

■ Multicomponent global which appears to be the case here.


pattern (1, 2, 3) ■ A dark blotch by definition should be devoid of local criteria. That does not
■ Regular pigment network always hold true, as seen in this case because there are foci or irregular globules.
(boxes) ■ This dysplastic nevus appears more malignant than benign, with a dysplastic
nevus and in situ melanoma in the differential diagnosis.
■ Irregular brown dots and
■ There are criteria in favor of a superficial vs invasive melanoma:
globules (circles)
■ Flat lesion clinically
■ Irregular dark brown blotch ■ Color associated with a superficial location (eg, shades of brown)
(white arrows) ■ Absence of criteria associated with invasive melanoma (eg, raised or nodular
■ Bluish-white color (yellow star) lesion, 5 or 6 colors including pink and blue, polymorphous vessels).
■ Hypopigmentation (black stars)
■ Normal skin (blue stars) PEARLS
■ One can see a progression of atypical features in Cases 95 and 96.
■ It is essential to be able to create a differential diagnosis of global patterns
and local criteria.
■ With experience, one will have mental images of past cases to draw on when
dealing with real life clinical situations.
■ You will remember specific global dermoscopic patterns or specific pat-

terns of local criteria with their histopathologic diagnoses, and realize that
the lesion you are looking at looks similar to the past lesions.
■ The more one uses dermoscopy, the larger a mental data base will be

created → Experience!
Chapter 3 Trunk and Extremities 291

RISK
❑ Low
❑ Intermediate
❑ High

DIAGNOSIS
❑ Nevus
❑ Seborrheic keratosis
❑ Basal cell carcinoma
❑ Vascular

3-97a
❑ Dermatofibroma
❑ Squamous cell carcinoma
❑ Melanoma
❑ Other

DISPOSITION
❑ No intervention
❑ Follow-up
❑ Histopathologic diagnosis
3-97b

CASE 97
HISTORY
This solitary lesion was found on the back of a 40-year-old woman.
1. Regular pigment network and regular globules identify a melanocytic lesion.
2. There is symmetry of color and structure and a reticular global pattern.
3. Multifocal hypopigmentation and regular pigment network diagnose a dysplastic nevus.
4. The pigment network, dots and globules, and dark blotches are all irregular.
5. Regression with milky-red globules and the other melanoma-specific criteria diagnose an
invasive melanoma.
292 DERMOSCOPY: AN ILLUSTRATED SELF-ASSESSMENT GUIDE

RISK
1
❑ Low
2
❑ Intermediate
✔ High
❑ 3

DIAGNOSIS
4
❑ Nevus
❑ Seborrheic keratosis
❑ Basal cell carcinoma
❑ Vascular 5
❑ Dermatofibroma 5
❑ Squamous cell carcinoma

❑ Melanoma

3-97c
❑ Other

DISPOSITION ANSWERS
❑ No intervention Answers: 4,5
❑ Follow-up
✔ Histopathologic diagnosis
❑ Discussion:
■ Irregular, not regular, pigment network and irregular globules diagnose a
melanocytic lesion.
■ The pigment network is thickened, branched, and in some areas broken up.

■ The dots and globules are of different sizes and shapes, dark and asym-
DERMOSCOPIC CRITERIA metrically located in the lesion.
■ Asymmetry of color and ■ They are larger than in Cases 95 and 96.

structure ■ Compared to Cases 95 and 96, with similar criteria, the global picture is
■ Multicomponent global more irregular.
■ There are, however, features that are less irregular:
pattern (1,2,3,4,5)
■ The irregular brown blotches are much smaller compared to those in
■ Irregular pigment network
Case 96.
(boxes) ■ Bluish-white color with foci of milky-red globules fills the lesion and repre-
■ Irregular dots and globules sents the invasive component of this melanoma.
(black circles) ■ The milky-red globules could be considered pinpoint vessels.
■ Irregular dark brown blotches ■ A pinkish hue can also be appreciated.
(arrows) ■ The two milia-like cysts have no diagnostic significance.
■ Regression (stars) ■ Milia-like cysts are not exclusively found in seborrheic keratoses and can

■ Milky-red globules be found in benign and malignant melanocytic lesions.


(yellow circles) ■ One could say with near certainty that this is a melanoma.
■ Never tell a patient that they have melanoma 100% because a very atypi-
■ Milia-like cysts (yellow box)
cal historical, clinical, and dermoscopic picture could turn out
to be benign.
■ A dysplastic nevus is not in the differential diagnosis.

PEARLS
■ This is an easy melanoma to diagnose.
■ If you had trouble, are unsure of yourself, or missed it completely, go
directly to jail. Do not pass. Go and do not collect $100!
Chapter 3 Trunk and Extremities 293

RISK
❑ Low
❑ Intermediate
❑ High

DIAGNOSIS
❑ Nevus
❑ Seborrheic keratosis
❑ Basal cell carcinoma

3-98a
Vascular
❑ Dermatofibroma
❑ Squamous cell carcinoma
❑ Melanoma
❑ Other

DISPOSITION
❑ No intervention
❑ Follow-up
❑ Histopathologic diagnosis
3-98b

CASE 98
HISTORY
A 3-year-old was referred for evaluation of a darkening lesion on his right shin.
1. Irregular pigment network and irregular globules identify a melanocytic lesion.
2. The global pattern is “Spitzoid”.
3. There are foci of irregular streaks at the periphery.
4. Regular streaks surround the lesion at the periphery.
5. Asymmetry of color and structure, irregular pigment network, dots, and streaks diagnose a
nodular melanoma.
294 DERMOSCOPY: AN ILLUSTRATED SELF-ASSESSMENT GUIDE

RISK
❑ Low
✔ Intermediate

❑ High

DIAGNOSIS

❑ Nevus
❑ Seborrheic keratosis
❑ Basal cell carcinoma
❑ Vascular
❑ Dermatofibroma
❑ Squamous cell carcinoma
❑ Melanoma

3-98c
❑ Other

ANSWERS
DISPOSITION Answers: 1,2,4
❑ No intervention Discussion:
❑ Follow-up ■ A dark lesion developing in a child is more likely to be a Spitz nevus than
✔ Histopathologic diagnosis
❑ melanoma.
■ Clinically, one can appreciate streaks at the periphery before using
dermoscopy.
■ Irregular pigment network and irregular globules identify a melanocytic lesion.
■ They are, however, symmetrically located within the lesion.
DERMOSCOPIC CRITERIA ■ They intermingle with each other, which has no diagnostic significance.
■ Symmetry of color and ■ The global pattern is “Spitzoid”.
structure ■ This is the black pigment network/reticular variant.

■ “Spitzoid “ global pattern ■ Any of the six global patterns suggestive of a Spitz nevus are referred to as

■ Irregular pigment network having a “Spitzoid” global pattern:


(white boxes) ■ Starburst ■ Black pigment network/reticular

■ Globular ■ Homogeneous
■ Irregular dots and globules
■ The pink and nonspecific patterns often cannot be diagnosed as
(red circles)
being “Spitzoid”.
■ Regular streaks (arrows)
■ The streaks are considered to be regular because they symmetrically sur-
round the lesion at the periphery.
■ Shape does not determine if streaks are regular or irregular.

■ Foci of streaks at the periphery define them as being irregular.

■ Dermoscopically, the diagnosis is clear with the caveat that a small


percentage of symmetrical “Spitzoid” patterns turns out to be melanoma.
■ A new and changing black lesion in a patient of any age, even with a sym-

metrical “Spitzoid” pattern, is a red flag for concern and should be removed.

PEARLS
■ Be confident and reassure your patient and/or their parents that this will
turn out in most cases to be a benign Spitz nevus.
■ Spend time with the patient for a good explanation of what a Spitz nevus is,
to allay anxiety.
■ Sequential digital follow-up of a “Spitzoid” lesion has been reported, is fool
hearted, puts the patient at risk, and is contraindicated.
Chapter 3 Trunk and Extremities 295

RISK
❑ Low
❑ Intermediate
❑ High

DIAGNOSIS
❑ Nevus
❑ Seborrheic keratosis
❑ Basal cell carcinoma
❑ Vascular

3-99a
❑ Dermatofibroma
❑ Squamous cell carcinoma
❑ Melanoma
❑ Other

DISPOSITION
❑ No intervention
❑ Follow-up
❑ Histopathologic diagnosis
3-99b

CASE 99
HISTORY
A 71-year-old woman was not sure how long this lesion has been present on her back.
1. Regular pigment network identifies a melanocytic lesion.
2. Moth-eaten borders and the fingerprint pattern diagnose a solar lentigo.
3. An irregular black blotch plus irregular pigment network diagnose a small nodular melanoma.
4. The black blotch is regular and created by a “black lamella.”
5. The “black lamella” is created by pigmented parakeratosis and is not associated with
melanoma.
296 DERMOSCOPY: AN ILLUSTRATED SELF-ASSESSMENT GUIDE

RISK
✔ Low

❑ Intermediate
❑ High

DIAGNOSIS

❑ Nevus
❑ Seborrheic keratosis
❑ Basal cell carcinoma
❑ Vascular
❑ Dermatofibroma
❑ Squamous cell carcinoma
❑ Melanoma

3-99c
❑ Other

DISPOSITION ANSWERS
✔ No intervention
❑ Answers: 1,4,5
❑ Follow-up
❑ Histopathologic diagnosis Discussion:
■ The global pattern is reticular-homogeneous:
■ Reticular created by the pigment network.

■ Homogeneous created by the regular black blotch.

■ This is a nice example of global symmetry of color and structure.


DERMOSCOPIC CRITERIA ■ The pigment network symmetrically surrounds the lesion and is not
■ Symmetry of color and considered to be irregular.
structure ■ It looks the same throughout the lesion.

■ The fingerprint pattern has parallel, not honeycomb-like line segments,


■ Homogeneous-reticular global
pattern which make up the pigment network.
■ The black blotch is considered to be regular:
■ Regular pigment network
■ It has a uniform oval shape and symmetrically fills most of the lesion.
(boxes)
■ The black blotch is also shinny, and is referred to as a “black lamella” created
■ Regular black blotch (star) by pigmented parakeratosis.
■ Tape stripping usually, but not always, results in specks of the “black lamel-
la” sticking to the tape.
■ Melanoma is not in the differential diagnosis of this symmetrical black lesion.
■ A black lesion or black color, seen with dermoscopy, is not always high risk.

■ Trans-epidermal elimination of melanin dermoscopically looks black;

however, it is not high risk histopathologically.

PEARLS
■ Patients are impressed with tape stripping especially if specks of the
“black lamella” stick to the tape.
■ Play it up, your patients will think you are a genius and the best dermatolo-
gist in town! You might get a referral or two.
Chapter 3 Trunk and Extremities 297

RISK
❑ Low
❑ Intermediate
❑ High

DIAGNOSIS
❑ Nevus
❑ Seborrheic keratosis
❑ Basal cell carcinoma

3-100a
❑ Vascular
❑ Dermatofibroma
❑ Squamous cell carcinoma
❑ Melanoma
❑ Other

DISPOSITION
❑ No intervention
❑ Follow-up
❑ Histopathologic diagnosis
3-100b

CASE 100
HISTORY
This “ugly duckling” black spot was found on the back of a 79-year-old woman.
1. Globules identify a melanocytic lesion.
2. Multiple milia-like cysts and pigmented pseudofollicular openings diagnose a seborrheic
keratosis.
3. Irregular dark blotches and regression diagnose a melanoma.
4. The irregular dark blotches represent the hyperpigmentation typically found in seborrheic
keratosis.
5. Hyperpigmentation is rarely found in seborrheic keratosis.
298 DERMOSCOPY: AN ILLUSTRATED SELF-ASSESSMENT GUIDE

RISK
✔ Low

❑ Intermediate
❑ High

DIAGNOSIS
❑ Nevus

❑ Seborrheic keratosis
❑ Basal cell carcinoma
❑ Vascular
❑ Dermatofibroma
❑ Squamous cell carcinoma
❑ Melanoma
❑ Other

3-100c
DISPOSITION
✔ No intervention

❑ Follow-up ANSWERS
❑ Histopathologic diagnosis
Answers: 2,4

Discussion:
■ There is an absence of criteria to diagnose a melanocytic lesion.
■ Multiple well-developed milia-like cysts and poorly developed pigmented
DERMOSCOPIC CRITERIA pseudofollicular openings diagnose a seborrheic keratosis.
■ Sharp border demarcation ■ The dots and globular-like structures seen at the periphery of the lesion and
■ Milia-like cysts (white arrows) in the surrounding skin are not part of the lesion.
■ Pigmented pseudofollicular ■ Some of the pigmented pseudofollicular openings look like the brown
openings (yellow arrows) globules of a melanocytic lesion.
■ Dots and globular-like ■ It is not always possible to perfectly categorize local criteria, hence the

structures (boxes) need for dermoscopic differential diagnosis.


■ Hyperpigmentation (stars) ■ The irregular dark blotches represents hyperpigmentation commonly found
in seborrheic keratosis.

PEARLS
■ The multiple well-developed milia-like cysts clinch the diagnosis.
■ For the less-experienced dermoscopist, always remember, “If in doubt,
cut it out!”
Chapter 3 Trunk and Extremities 299

RISK
❑ Low
❑ Intermediate
❑ High

DIAGNOSIS
❑ Nevus
❑ Seborrheic keratosis
❑ Basal cell carcinoma

3-101a
❑ Vascular
❑ Dermatofibroma
❑ Squamous cell carcinoma
❑ Melanoma
❑ Other

DISPOSITION
❑ No intervention
❑ Follow-up
❑ Histopathologic diagnosis
3-101b

CASE 101
HISTORY
A 66-year-old man had this long-standing asymptomatic lesion on his chest.
1. Clinically and dermoscopically, this is similar to the last case especially with the regular blotch
that fills most of the lesion.
2. There is symmetry of color and structure, a homogeneous global pattern with regular dots and
globules, and a regular blotch.
3. Multifocal hypopigmentation plus an irregular pigment network diagnose a dysplastic nevus.
4. There is a good clinico–dermoscopic correlation because this dark nodule looks like a
melanoma and there are well-developed melanoma-specific criteria.
5. Irregular dots and globules, an irregular dark blotch, and bluish-white color suggest that this
could be a melanoma.
300 DERMOSCOPY: AN ILLUSTRATED SELF-ASSESSMENT GUIDE

RISK
❑ Low
❑ Intermediate
✔ High

2
DIAGNOSIS 1

❑ Nevus
2
❑ Seborrheic keratosis 3
❑ Basal cell carcinoma
❑ Vascular 1
❑ Dermatofibroma
❑ Squamous cell carcinoma

❑ Melanoma 4

3-101c
❑ Other

DISPOSITION ANSWERS
❑ No intervention
Answers: 4,5
❑ Follow-up
✔ Histopathologic diagnosis
❑ Discussion:
■ Irregular globules diagnose a melanocytic lesion.
■ There is no pigment network.

■ Global melanoma-specific criteria include:


■ Asymmetry of color and structure
DERMOSCOPIC CRITERIA ■ A multicomponent global pattern

■ Asymmetry of color and ■ Local melanoma-specific criteria include:


structure ■ Irregular brown dots and globules

■ Multicomponent global ■ Irregular dark brown blotch

■ Bluish-white color
pattern (1,2,3,4)
■ Irregular brown dots and ■ There is multifocal hypopigmentation, which is more commonly associated
with dysplastic nevi.
globules (circles)
■ The dermoscopic picture in this case is diametrically opposed to Case 99.
■ Irregular dark brown blotch
■ Symmetry of color and structure vs asymmetry of color and structure.
(white arrows) ■ A flat vs nodular lesion.
■ Multifocal hypopigmentation ■ An absence of melanoma-specific criteria vs well-developed melanoma-
(stars) specific criteria.
■ Bluish-white color (white box) ■ There is a good clinico–dermoscopic–pathologic correlation because
clinically, dermoscopically, and histopathologically this is a melanoma.

PEARLS
■ There is no pearl in this straightforward case.
■ It is a nice study of contrasts between good and evil!
Chapter 3 Trunk and Extremities 301

RISK
❑ Low
❑ Intermediate
❑ High

DIAGNOSIS
❑ Nevus
❑ Seborrheic keratosis
❑ Basal cell carcinoma

3-102a
❑ Vascular
❑ Dermatofibroma
❑ Squamous cell carcinoma
❑ Melanoma
❑ Other

DISPOSITION
❑ No intervention
❑ Follow-up
❑ Histopathologic diagnosis
3-102b

CASE 102
HISTORY
The patient’s dermatologist followed this lesion for 3 years and said it was a benign seborrheic
keratosis. It was never examined with dermoscopy. The patient came for a second opinion because
his wife was worried, thinking it could be a skin cancer.
1. Multiple milia-like cysts and pigmented pseudofollicular openings diagnose a seborrheic
keratosis.
2. Hairpin vessels and fat-fingers clinch the diagnosis of a seborrheic keratosis.
3. Irregular globules and a large irregular dark blotch suggest this is a melanoma.
4. This could be a collision tumor consisting of a melanoma and a seborrheic keratosis.
5. Melanoma as part of a collision tumor has never been reported.
302 DERMOSCOPY: AN ILLUSTRATED SELF-ASSESSMENT GUIDE

RISK
❑ Low
1
❑ Intermediate
✔ High

DIAGNOSIS
2
❑ Nevus

❑ Seborrheic keratosis
❑ Basal cell carcinoma
❑ Vascular 3
❑ Dermatofibroma
❑ Squamous cell carcinoma

❑ Melanoma

3-102c
❑ Other

DISPOSITION ANSWERS
❑ No intervention Answers: 3,4
❑ Follow-up
✔ Histopathologic diagnosis
❑ Discussion:
■ Clinically, this dark nodule with an eccentric bony-white halo is very suspi-
cious for a melanoma.
■ The verrucous component suggests this could be a collision tumor.
■ The lesion looks like a seborrheic keratosis with milia-like cysts and
DERMOSCOPIC CRITERIA pigmented pseudofollicular openings or hyperpigmentation.
■ Asymmetry of color and ■ This is by far not a classic seborrheic keratosis.

structure ■ Poorly defined irregular brown globules identify a melanocytic lesion.


■ Multicomponent global ■ The global melanoma-specific criteria include:
■ Asymmetry of color and structure.
pattern (1,2,3)
■ A multicomponent global pattern.
■ Irregular brown globules
■ Compared to Case 94 in which the zones of the multicomponent global
(yellow boxes) pattern overlap, here they are very sharply demarcated.
■ Irregular dark black blotch ■ The local melanoma-specific criteria include:
(white stars) ■ Poorly developed irregular brown globules.
■ Regression ■ A well-developed irregular black blotch.
■ White color (black stars) ■ Regression composed of bony-white color and peppering.

■ Peppering (black box) ■ The regression is more obvious clinically than dermoscopically.

■ Seborrheic keratosis ■ Melanoma as part of a collision tumor is a known phenomenon.


(black arrows) ■ Melanoma and seborrheic keratosis is one of the most common

■ Milia-like cysts
associations.
(yellow arrows)
■ Pigmented pseudofollicular
PEARLS
openings (white arrows) ■ When there is a possibility that you are dealing with a collision tumor, alert
your pathologist so that they will know to search for both pathologies.
■ In a collision tumor that does not contain a potential melanoma, if it is pos-
sible, submit two specimens in separate formalin containers to help make
the diagnosis.
Chapter 3 Trunk and Extremities 303

RISK
❑ Low
❑ Intermediate
❑ High

DIAGNOSIS
❑ Nevus
❑ Seborrheic keratosis
❑ Basal cell carcinoma
3-103a

❑ Vascular
❑ Dermatofibroma
❑ Squamous cell carcinoma
❑ Melanoma
❑ Other

DISPOSITION
❑ No intervention
❑ Follow-up
❑ Histopathologic diagnosis
3-103b

CASE 103
HISTORY
This lesion was found on the chest of a 65-year-old man.
1. This is a melanocytic lesion by default.
2. The bluish-white color is a clue that this dark nodule is a melanoma.
3. There is a paucity of superficially located local criteria that puts a nodular melanoma at the top
of the differential diagnosis list.
4. There is an irregular dark blotch which is not sharply demarcated.
5. Ovoid nests of bluish-white color and a focus of milky-red color diagnose a pigmented basal
cell carcinoma.
304 DERMOSCOPY: AN ILLUSTRATED SELF-ASSESSMENT GUIDE

RISK
❑ Low
❑ Intermediate
✔ High
❑ 1

DIAGNOSIS
1
❑ Nevus
2
❑ Seborrheic keratosis
❑ Basal cell carcinoma
❑ Vascular
❑ Dermatofibroma
❑ Squamous cell carcinoma

❑ Melanoma
❑ Other
3

4
1
DISPOSITION
❑ No intervention

3-103c
❑ Follow-up
✔ Histopathologic diagnosis

ANSWERS
Answers: 1,2,3,4

DERMOSCOPIC CRITERIA Discussion:


■ This is melanocytic by default.
■ Asymmetry of color and
■ Clinically and dermoscopically this is a good example of a nodular
structure melanoma.
■ Multicomponent global ■ The melanoma-specific criteria are well developed:
pattern (1,2,3,4) ■ Asymmetry of color and structure ■ Regression
■ Irregular dark brown blotch ■ Multicomponent global pattern ■ Milky-red area
(white arrows) ■ Irregular dark brown blotch ■ Five colors

■ Bluish-white color (white stars) ■ Bluish-white color

■ Regression (black stars) ■ The irregular dark blotch correlates clinically with what appears to be
■ Milky-red area (yellow box) ulceration.
■ Ulceration was reported histopathologically.
■ Five colors
■ Typically, nodular melanomas have a paucity of superficially located local
criteria (eg, pigment network, brown dots, and globules).
■ Compared with the last two cases, this is the end of the line of how bad a
primary melanoma can be!

PEARLS
■ The goal of performing periodic total body skin examinations utilizing
dermoscopy is to prevent melanoma from getting this bad.
■ It is not the luck of the Irish if one presents with an ulcerated deep nodular
melanoma. It is one of life’s disasters that have terrible consequences for
the patient and their loved ones.
■ Every clinician’s motivation should be to be as good as they can be every
minute of the working day, to prevent this from ever happening.
Chapter 3 Trunk and Extremities 305

RISK
❑ Low
❑ Intermediate
❑ High

DIAGNOSIS
❑ Nevus
❑ Seborrheic keratosis
❑ Basal cell carcinoma

3-104a
❑ Vascular
❑ Dermatofibroma
❑ Squamous cell carcinoma
❑ Melanoma
❑ Other

DISPOSITION
❑ No intervention
❑ Follow-up
❑ Histopathologic diagnosis
3-104b

CASE 104
HISTORY
This was found on the upper back of a 67-year-old woman.
1. Clinically, there is a semi-translucent quality suggesting this could be a basal cell carcinoma.
2. This cannot be a basal cell carcinoma because there are no arborizing vessels or ulceration.
3. The presence of spoke-wheel structures suggests this is a basal cell carcinoma.
4. Irregular streaks and regression diagnose a hypomelanotic melanoma.
5. Spoke-wheel structures always have a perfect set of lines radiating from the central hub.
306 DERMOSCOPY: AN ILLUSTRATED SELF-ASSESSMENT GUIDE

RISK
❑ Low
❑ Intermediate
✔ High

1

2
DIAGNOSIS
❑ Nevus
❑ Seborrheic keratosis

❑ Basal cell carcinoma
❑ Vascular
3
❑ Dermatofibroma
❑ Squamous cell carcinoma
❑ Melanoma

3-104c
❑ Other

DISPOSITION ANSWERS
❑ No intervention Answers: 1,3
❑ Follow-up
✔ Histopathologic diagnosis
❑ Discussion:
■ A focused skin examination with a high index of suspicion and dermoscopy
are needed to find and diagnose a small basal cell carcinoma like this one.
■ The suggestion of the presence of atypical spoke-wheel structures was the
first clue that this could be a basal cell carcinoma.
DERMOSCOPIC CRITERIA ■ Classic spoke-wheel structures are made up of:
■ Asymmetry of color and ■ A central pigmented hub that looks like a globule.

■ Fine pigmented lines that radiate from all points of the hub (like the
structure
■ Multicomponent global spokes of a bicycle wheel).
■ In this case, the central hubs are irregular, the spokes look like thickened
pattern (1,2,3)
streaks, and they do not radiate from all points of the central hub.
■ Brown pigmentation (circles)
■ Often one has to use their imagination to identify atypical spoke-wheel
■ Blue ovoid pigmentation structures.
(red arrow) ■ It is not uncommon to find multiple well-developed spoke-wheel
■ Spoke-wheel structures structures.
■ Central hub (yellow arrows) ■ One blue ovoid area of pigmentation associated with pigmented basal cell
■ Spokes (black arrows) carcinomas appears to have three spoke-like structures radiating from it.
■ Arborizing vessels and ulceration are not needed to make the diagnosis.

PEARLS
■ Inter-observer agreement is not always good even among experienced
dermoscopists. Some might not think there are spoke-wheel structures in
this lesion.
■ We identified the spoke-wheel structures immediately, and based on that
made the correct dermoscopic diagnosis before the lesion was removed.
■ With knowledge and imagination, there is no telling where dermoscopy
will take you!
Chapter 3 Trunk and Extremities 307

RISK
❑ Low
❑ Intermediate
❑ High

DIAGNOSIS
❑ Nevus
❑ Seborrheic keratosis
❑ Basal cell carcinoma
❑ Vascular
❑ Dermatofibroma

3-105a
❑ Squamous cell carcinoma
❑ Melanoma
❑ Other

DISPOSITION
❑ No intervention
❑ Follow-up
❑ Histopathologic diagnosis
3-105b

CASE 105
HISTORY
White color developed around several nevi on a 19-year-old woman.
1. Globules identify a melanocytic lesion.
2. This could represent the fissure and ridge pattern of a halo seborrheic keratosis.
3. Asymmetry of color and structure, irregular dots and globules, and regression diagnose a
melanoma.
4. This could be a halo nevus with a benign cobblestone global pattern.
5. A wide diameter halo around a melanocytic lesion is diagnostic of melanoma.
308 DERMOSCOPY: AN ILLUSTRATED SELF-ASSESSMENT GUIDE

RISK
✔ Low

❑ Intermediate
❑ High

DIAGNOSIS

❑ Nevus
❑ Seborrheic keratosis
❑ Basal cell carcinoma
❑ Vascular
❑ Dermatofibroma
❑ Squamous cell carcinoma
❑ Melanoma

3-105c
❑ Other

DISPOSITION ANSWERS
✔ No intervention
❑ Answers: 1,2,4
❑ Follow-up
❑ Histopathologic diagnosis Discussion:
■ Brown globules identify a melanocytic lesion.
■ Large angulated globules with a few dots make up the cobblestone global
pattern.
■ The fissure and ridge pattern of a seborrheic keratosis is in the differential
DERMOSCOPIC CRITERIA diagnosis of a cobblestone global pattern.
■ Cobblestone global pattern ■ This is a halo nevus with an unusually wide halo.
■ Regular dots and globules ■ The size of a white halo has no diagnostic significance and does not

(boxes) diagnose melanoma.


■ Rarely, you may see halo seborrheic keratosis.
■ Regression (stars)

PEARLS
■ Sequential digital clinical and dermoscopic monitoring is a cutting-edge way
to follow this patient.
■ Halo nevi usually disappear over a variable period of time.

■ The “wobble sign” created by pressure from instrumentation directly on a


lesion and side-to-side movement can be used to differentiate a seborrheic
keratosis from a melanocytic nevus.
■ A seborrheic keratosis is usually fixed to the skin and is immobile →

a negative “wobble sign.”


■ An elevated nevus usually moves easily from side-to-side → a positive

“wobble sign.”
Chapter 3 Trunk and Extremities 309

RISK
❑ Low
❑ Intermediate
❑ High

DIAGNOSIS
❑ Nevus
❑ Seborrheic keratosis
❑ Basal cell carcinoma

3-106a
❑ Vascular
❑ Dermatofibroma
❑ Squamous cell carcinoma
❑ Melanoma
❑ Other

DISPOSITION
❑ No intervention
❑ Follow-up
❑ Histopathologic diagnosis
3-106

CASE 106
HISTORY
This lesion was found on the knee of an 82-year-old man. There could be a history of trauma while
working in his yard. However, the patient was not sure.
1. Clinically and dermoscopically this is consistent with an abrasion and topical antibiotics
should be recommended.
2. If you miss the focus of brown globules, the homogeneous brown color suggests this red
nodule could be a melanocytic lesion.
3. Different shades of pink color are a red flag for concern.
4. Pinpoint vessels and milky-red globules suggest this could be an amelanotic melanoma.
5. This feature-poor lesion does not have well-developed melanoma-specific criteria.
310 DERMOSCOPY: AN ILLUSTRATED SELF-ASSESSMENT GUIDE

RISK
1
❑ Low
❑ Intermediate
✔ High

2

3
DIAGNOSIS
❑ Nevus
❑ Seborrheic keratosis
❑ Basal cell carcinoma
❑ Vascular
❑ Dermatofibroma
❑ Squamous cell carcinoma
4

❑ Melanoma
❑ Other

3-106c
DISPOSITION ANSWERS
❑ No intervention Answers: 2,3,4,5
❑ Follow-up
✔ Histopathologic diagnosis
❑ Discussion:
■ Clinically, but not dermoscopically, an infectious process is in the
differential diagnosis.
■ Dermoscopic criteria for infectious lesions have not been well studied.

■ A focus of irregular brown globules identifies a melanocytic lesion.


DERMOSCOPIC CRITERIA ■ If the globules are not identified, this could be considered to be

■ Asymmetry of color and melanocytic by default.


■ The focus of homogeneous brown color suggests the lesion could be melanocytic.
structure
■ Amelanotic melanoma, devoid of the typical well-developed, melanoma-
■ Multicomponent global
specific criteria seen in pigmented melanomas, often have foci of homo-
pattern (1,2,3,4) geneous brown pigmentation at the periphery.
■ Irregular brown dots and ■ Different shades of pink color are an important yet nonspecific clue that this
globules (black box) lesion is an amelanotic melanoma.
■ Homogeneous brown color ■ These can be seen in melanocytic, nonmelanocytic, benign, malignant, or
(yellow arrows) inflammatory pink lesions.
■ Different shades of pink color ■ Foci of milky-red globules in a background of pink color are another
■ Milky-red globules important clue that this could be an amelanotic melanoma.
(black arrows) ■ The pinpoint vessels could be considered milky-red globules. The

■ Pinpoint vessels (yellow boxes) differentiation is a minor point because both types of vessels indicate
■ Ulceration (white box) the lesion might be high risk.
■ Hypopigmentation is created by the pressure of the camera.
■ Hypopigmentation (stars)
■ In most pink lesions pink blanches away with adequate pressure.

■ The small ulceration has no diagnostic significance.

PEARLS
■ Finding as many clues as possible might be the only way to diagnose
amelanotic melanoma.
■ It would be a rare lesion that is 100% featureless. Most have vessels that can be
identified with minimal pressure or the use of ultrasound or hand sanitizer gel.
■ Polarizing light is better than nonpolarizing for visualizing blood vessels.
Chapter 3 Trunk and Extremities 311

RISK
❑ Low
❑ Intermediate
❑ High

DIAGNOSIS
❑ Nevus
❑ Seborrheic keratosis
❑ Basal cell carcinoma
❑ Vascular
❑ Dermatofibroma
❑ Squamous cell carcinoma
❑ Melanoma
❑ Other

DISPOSITION
3-107a ❑ No intervention
❑ Follow-up
❑ Histopathologic diagnosis

CASE 107
HISTORY
You were e-mailed this case for your dermoscopic evaluation. No clinical image was sent. It was a
firm papule on the right shin of a 22-year-old woman. She was not aware of its presence.
1. Brown globules identify a melanocytic lesion.
2. Fine peripheral pigment network and a central white patch diagnose a dermatofibroma.
3. The central white patch is composed of homogeneous bony-white color and reticular
depigmentation.
4. The globular-like structures are created by the reticular depigmentation over a background of
dark brown color.
5. Reticular depigmentation is diagnostic of a dermatofibroma.
312 DERMOSCOPY: AN ILLUSTRATED SELF-ASSESSMENT GUIDE

RISK
✔ Low

❑ Intermediate
❑ High

DIAGNOSIS
❑ Nevus
❑ Seborrheic keratosis
❑ Basal cell carcinoma
❑ Vascular

❑ Dermatofibroma
❑ Squamous cell carcinoma
❑ Melanoma

3-107b
❑ Other

DISPOSITION ANSWERS
✔ No intervention
❑ Answers: 2,3,4
❑ Follow-up
❑ Histopathologic diagnosis Discussion:
■ A clinical image is not needed to help make the diagnosis. The history and
dermoscopic criteria are sufficient.
■ This is not the classic central white patch that is made up of centrally-located,
homogeneous, bony-white color, characteristic of a dermatofibroma.
DERMOSCOPIC CRITERIA ■ Reticular depigmentation (negative or white network) is made up of sharply
■ Regular pigment network in-focus honeycomb-like or reticular bony-white line segments.
(black boxes) ■ The thickness, extent of the line segments, and intensity of the color

■ Central white patch varies from lesion to lesion.


■ Homogeneous bony-white
■ Not uncommonly, reticular depigmentation makes up the central white
color (stars) patch of a dermatofibroma.
■ The fine, thin peripheral pigment network is one of the major criteria to
■ Reticular depigmentation
diagnose dermatofibromas.
(white boxes)
■ This is the exception to the rule that pigment network diagnoses a
■ Globular-like structures melanoctyic lesion.
(yellow arrows) ■ Dermatofibromas are not melanocytic, and pigment network helps make

their diagnosis.
■ The supposedly well-developed brown globules are not what they appear to
be. They are created by reticular depigmentation over a background of
homogeneous dark brown color.
■ The novice dermoscopist could be easily led astray and consider them to

be the brown globules of a melanocytic lesion.

PEARL
■ Beware! While not the case here, reticular depigmentation/white network
might be the only clue that a pigmented lesion is a melanoma!
Chapter 3 Trunk and Extremities 313

RISK
❑ Low
❑ Intermediate
❑ High

DIAGNOSIS
❑ Nevus
❑ Seborrheic keratosis
❑ Basal cell carcinoma

3-108a
❑ Vascular
❑ Dermatofibroma
❑ Squamous cell carcinoma
❑ Melanoma
❑ Other

DISPOSITION
❑ No intervention
❑ Follow-up
❑ Histopathologic diagnosis
3-108b

CASE 108
HISTORY
At a routine follow-up skin examination, this small pigmented lesion was found on the shin of a
30-year-old woman.
1. There is not a good clinico–dermoscopic correlation because the lesion looks low risk based on
ABCD clinical criteria, yet with dermoscopy it looks suspicious.
2. Globules identify a melanocytic lesion.
3. A large irregular dark blotch and reticular depigmentation diagnose a dermatofibroma.
4. A large irregular dark blotch, irregular dots and globules, and reticular depigmentation suggest
this is a melanoma.
5. Sequential digital monitoring is indicated for this gray zone slightly suspicious pigmented skin
lesion.
314 DERMOSCOPY: AN ILLUSTRATED SELF-ASSESSMENT GUIDE

RISK
❑ Low
❑ Intermediate
✔ High

DIAGNOSIS
❑ Nevus 1
❑ Seborrheic keratosis 2
❑ Basal cell carcinoma
❑ Vascular 3

❑ Dermatofibroma
❑ Squamous cell carcinoma

❑ Melanoma 4

3-108c
❑ Other

ANSWERS
DISPOSITION Answers: 1,2,4
❑ No intervention Discussion:
❑ Follow-up ■ This is a melanoma- incognito aka false negative clinical melanoma that
✔ Histopathologic diagnosis
❑ would be missed if it were not examined with dermoscopy.
■ It lacks the ABCD clinical criteria and looks low risk.

■ Irregular brown globules identify a melanocytic lesion.


■ Some of the melanoma-specific criteria are well-developed and include:
■ Asymmetry of color and structure
DERMOSCOPIC CRITERIA ■ Multicomponent global pattern with four different areas
■ Asymmetry of color and ■ A large irregular dark blotch ■ Reticular depigmentation
structure ■ There are poorly defined irregular dots and globules and hypopigmentation
■ Multicomponent global both of which are of minor diagnostic significance.
pattern (1,2,3,4) ■ Section 4 could be an area of regression.

■ Irregular brown dots and ■ Reticular depigmentation (negative/white network) is an important clue that
globules (circles) a lesion might be melanoma.
■ Irregular dark brown blotch ■ Reticular depigmentation can be found in:
■ Banal nevi ■ Dermatofibromas
(yellow arrows)
■ Dysplastic nevi ■ Melanoma
■ Reticular depigmentation
■ Spitz nevi
(yellow boxes)
■ Hypopigmentation (stars) PEARLS
■ To improve your dermoscopic skills, it is best to have your instrumentation
handy at all times.
■ What a shame for you and more importantly your patient, to miss being

able to examine an interesting lesion because for one reason or another


you do not have dermoscopic instrumentation!
■ Always keep in mind that the most banal appearing lesion could be
melanoma-incognito.
■ Dermoscopy should not only be used to examine clinically atypical lesions
but also to gather a reference base from typical lesions, which is important.
■ Refrain from outbursts of surprise when you find a strikingly bad dermo-
scopic picture. It will scare your patient; it is unprofessional yet instinctive,
often uncontrollable!
Chapter 3 Trunk and Extremities 315

RISK
❑ Low
❑ Intermediate
❑ High

DIAGNOSIS
❑ Nevus
❑ Seborrheic keratosis

3-109a
❑ Basal cell carcinoma
❑ Vascular
❑ Dermatofibroma
❑ Squamous cell carcinoma
❑ Melanoma
❑ Other

DISPOSITION
❑ No intervention
❑ Follow-up
❑ Histopathologic diagnosis
3-109b

CASE 109
HISTORY
This new lesion was found on the anterior thigh of a 19-year-old tanning parlor enthusiast.
1. Pigment network and globules identify a melanocytic lesion.
2. There is a significant asymmetry of color and structure and the multicomponent global
pattern.
3. Some of the high risk criteria are easy to see while others are not.
4. The irregular dots and globules, irregular black blotches, and bluish-white color are easy to see.
5. Irregular pigment network, irregular streaks, and regression are not that easy to see.
316 DERMOSCOPY: AN ILLUSTRATED SELF-ASSESSMENT GUIDE

RISK
❑ Low
❑ Intermediate
✔ High

1

DIAGNOSIS 1
1
❑ Nevus
2
❑ Seborrheic keratosis 3
❑ Basal cell carcinoma
❑ Vascular
❑ Dermatofibroma
❑ Squamous cell carcinoma

❑ Melanoma

3-109c
❑ Other

DISPOSITION ANSWERS
❑ No intervention Answers: 1,2,3,4,5
❑ Follow-up
✔ Histopathologic diagnosis
❑ Discussion:
■ Clinically and dermoscopically, this lesion is high risk and easy to diagnose.
■ Most of the high risk criteria are easy to identify:
■ Asymmetry of color and structure

■ Multicomponent global pattern


DERMOSCOPIC CRITERIA ■ Each zone has several local criteria within it.

■ Asymmetry of color and ■ Irregular brown dots and globules

structure ■ Irregular black blotches

■ Multicomponent global ■ Bluish-white color

pattern (1,2,3) ■ There is one focus of irregular pigment network with some streaks coming
■ Irregular pigment network off it at the periphery.
■ There are irregular streaks scattered around at the periphery.
(yellow box)
■ Regression is seen clinically and with dermoscopy but it is not clear cut and
■ Irregular streaks (arrows)
could be missed.
■ Irregular brown dots and
globules (white boxes)
PEARLS
■ Irregular black blotches
(yellow stars) ■ If you were insecure about the analysis of this lesion at this point in the
■ Bluish-white color (white stars) book:
■ Review Chapter 1.
■ Regression (gray stars)
■ Find a colleague who has experience to help you.
■ Five colors
■ Find a dermoscopy meeting to attend ASAP.

■ Don’t give up, the learning curve is steep yet not insurmountable.
Chapter 3 Trunk and Extremities 317

RISK
❑ Low
❑ Intermediate
❑ High

DIAGNOSIS
Baseline 6 month follow-up
❑ Nevus
❑ Seborrheic keratosis
❑ Basal cell carcinoma
❑ Vascular
❑ Dermatofibroma
❑ Squamous cell carcinoma
❑ Melanoma
3-110a ❑ Other

12 month follow-up 18 month follow-up


DISPOSITION
❑ No intervention
❑ Follow-up
❑ Histopathologic diagnosis

CASE 110
HISTORY
This is one of several nevi that were being followed with digital dermoscopy to look for important
changes over time.
1. Asymmetry of color and structure, the multicomponent global pattern, a very irregular
pigment network, and regression in the baseline image diagnose an in situ melanoma. The
lesion should have been removed at that time.
2. Asymmetry of color and structure, a reticular-homogeneous global pattern, a mildly irregular
pigment network, different shades of brown color, and multifocal hypopigmentation diagnose
a mildly dysplastic nevus.
3. A side-by-side comparison of the baseline image with the same lesion at 6, 12, and 18 months
shows that the lesion is basically without change.
4. The lesion has been changing slowly over time with high risk criteria in the 18-month follow-
up image.
5. Significant dermoscopic changes over time include: asymmetrical enlargement, the presence of
new melanoma-specific criteria, or new colors.
318 DERMOSCOPY: AN ILLUSTRATED SELF-ASSESSMENT GUIDE

RISK
✔ Low

❑ Intermediate
❑ High

DIAGNOSIS Baseline 6 month follow-up


❑ Nevus
❑ Seborrheic keratosis
❑ Basal cell carcinoma
❑ Vascular
❑ Dermatofibroma
❑ Squamous cell carcinoma
❑ Melanoma

3-110b
Other
12 month follow-up 18 month follow-up

ANSWERS
DISPOSITION
Answers: 2,3,5
❑ No intervention
✔ Follow-up
❑ Discussion:
❑ Histopathologic diagnosis ■ The criteria in the baseline image diagnose a mildly dysplastic nevus and
include:
■ Asymmetry of color and structure

■ Reticular-homogeneous global pattern:

■ Mildly irregular pigment network creating the reticular component


DERMOSCOPIC CRITERIA ■ Multifocal hypopigmentation not regression creating the homogeneous

■ Asymmetry of color and component


structure ■ Light and dark brown color

■ Reticular-homogeneous global ■ There are no significant melanoma-specific criteria.


■ In this case, the asymmetry of color and structure and slightly irregular
pattern
pigment network are not high risk.
■ Irregular pigment network
■ A side-by-side comparison of the baseline image and the lesion at 6, 12, and
(all colored boxes)
18 months show that they are essentially without significant changes.
■ Multifocal hypopigmentation ■ If you compare the different-colored boxes and stars, the changes are minimal.
(all colored stars) ■ For long-term digital dermoscopic monitoring every 6 to 12 months, poten-
tially significant changes would include:
■ Asymmetrical enlargement

■ Asymmetrical changes within the lesion without enlargement

■ The presence of new melanoma-specific criteria (eg, irregular pigment

network, dots, globules, streaks, blotches, regression)


■ Disappearance of well-developed local criteria

■ New colors

PEARLS
■ Most nevi that change over time are not high risk.
■ All nevi that change do not need to be removed.
■ Beware! There are melanomas that do not appear to be high risk clinically or with
dermoscopy. They are only diagnosed after monitoring and finding dermoscopic
changes over time when comparing baseline with subsequent digital images.
■ Consider adding digital dermoscopy to your practice.
Chapter 3 Trunk and Extremities 319

Baseline 6 month fol


RISK
❑ Low
❑ Intermediate
❑ High

3-111a
DIAGNOSIS
❑ Nevus
❑ Seborrheic keratosis
❑ Basal cell carcinoma
❑ Vascular
❑ Dermatofibroma
❑ Squamous cell carcinoma
❑ Melanoma
❑ Other

DISPOSITION
❑ No intervention
❑ Follow-up
❑ Histopathologic diagnosis
3-111b

CASE 111
HISTORY
A 43-year-old man returned for a routine 6-month follow-up digital dermoscopic examination.
He had a baseline database of 15 nevi that were being followed. He performed monthly self-skin
examinations and did not report any new, changing, or symptomatic lesions.
1. The baseline image does not have enough criteria to diagnose a melanoma. There is mild
asymmetry of color and structure, foci of minimally irregular pigment network, different
shades of brown color, and a focus of reticular depigmentation.
2. A side-by-side comparison of the baseline and follow-up image demonstrates that the lesion
has not changed at all.
3. A side-by-side comparison of the baseline and follow-up image demonstrates symmetrical
enlargement without any new high risk criteria.
4. There has been a dramatic asymmetrical enlargement that is a red flag for concern. Without
well-developed melanoma-specific criteria continued monitoring is indicated.
5. Even though the lesion does not have well-developed, melanoma-specific criteria, the change
itself is significant and an indication for an excision.
320 DERMOSCOPY: AN ILLUSTRATED SELF-ASSESSMENT GUIDE

Baseline 6 month follow-up

RISK
❑ Low
❑ Intermediate
✔ High

DIAGNOSIS
❑ Nevus

3-111c
❑ Seborrheic keratosis
❑ Basal cell carcinoma
❑ Vascular
❑ Dermatofibroma
❑ Squamous cell carcinoma

❑ Melanoma
ANSWERS
❑ Other
Answers: 1,5

Discussion:
■ The baseline image does not have enough criteria to diagnose a high risk
DISPOSITION dysplastic nevus or melanoma. There is only mild asymmetry of color and
❑ No intervention structure, a minimally irregular pigment network, and a small focus of
❑ Follow-up reticular depigmentation.
✔ Histopathologic diagnosis
❑ ■ A side-by-side comparison of the baseline image and the same nevus, 6 months
later, demonstrates significant and dramatic changes.
■ The shape has asymmetrically expanded and it looks like a completely

different lesion.
■ When there is such a dramatic change, make sure that you are compar-

DERMOSCOPIC CRITERIA: ing the same lesions.


■ The focus of irregular pigment network has expanded.
BASELINE IMAGE
■ The hyperpigmentation within the lesion has expanded and is darker.
■ Asymmetry of color and ■ The hyperpigmentation is not well-demarcated and cannot be consid-
structure ered an irregular blotch.
■ Homogeneous-reticular global ■ Even though the lesion has changed significantly, if it were examined by
pattern itself, it would not have enough criteria to diagnose a high risk dysplastic
■ Irregular pigment network nevus or melanoma.
(red box) ■ The focus of reticular depigmentation in the baseline image is no longer
■ Reticular depigmentation seen in the follow-up image and is of no diagnostic significance.
(white arrows) ■ Reticular depigmentation should always be a red flag for concern but that

does not mean that it is always found in high risk lesions.


■ The overall changes are significant and warrant a histopathologic diagnosis
to rule out melanoma-incognito.

PEARLS
■ This case points out the benefits of digital dermoscopic follow-up
examination.
■ The patient was not aware of the change.

■ The change was only found by digital-dermoscopic follow-up.


■ The dermoscopic changes were significant even though the lesion by itself

could have been overlooked as not being high risk with an absence of
well-developed melanoma-specific criteria.
■ There are cases when melanoma can only be diagnosed by finding

dermoscpic changes over time.


Chapter 3 Trunk and Extremities 321

RISK
❑ Low
❑ Intermediate
❑ High

DIAGNOSIS

3-112a
Baseline 6 month follow-up ❑ Nevus
❑ Seborrheic keratosis
❑ Basal cell carcinoma
❑ Vascular
❑ Dermatofibroma
❑ Squamous cell carcinoma
❑ Melanoma
❑ Other

DISPOSITION
❑ No intervention
❑ Follow-up
❑ Histopathologic diagnosis
3-112b

CASE 112
HISTORY
This is a follow-up of another nevus on the back of the patient from the last case.
1. The baseline image looks low risk with mildly irregular features.
2. A side-by-side comparison of the baseline and follow-up image demonstrates symmetrical
darkening and that the lesion has expanded in size.
3. Even though the lesion is the same size at follow-up, there is asymmetrical darkening within
the lesion.
4. At a higher magnification, one can see in the follow-up lesion asymmetry of color and
structure, foci of irregular pigment network, a few irregular globules, bluish-white color, and
multifocal hypopigmentation.
5. The changes are enough to warrant an excision yet the lesion might still not be a melanoma.
322 DERMOSCOPY: AN ILLUSTRATED SELF-ASSESSMENT GUIDE

RISK
❑ Low
✔ Intermediate

❑ High

DIAGNOSIS

❑ Nevus
❑ Seborrheic keratosis
❑ Basal cell carcinoma
❑ Vascular
❑ Dermatofibroma
❑ Squamous cell carcinoma
❑ Melanoma
❑ Other

DISPOSITION
❑ No intervention
❑ Follow-up

3-112c
✔ Histopathologic diagnosis

ANSWERS
Answers: 1,3,4,5
DERMOSCOPIC CRITERIA:
FOLLOW-UP IMAGE Discussion:
■ Asymmetry of color and ■ You can clearly see that even though the size is the same at follow-up, the
nevus has changed:
structure
■ Asymmetrical darkening
■ Reticular global pattern
■ At a higher magnification you can see the criteria to diagnose a mildly
■ Irregular pigment network dysplastic nevus:
(black boxes) ■ Asymmetry of color and structure ■ A few irregular globules
■ Irregular globules ■ Foci of irregular pigment network ■ Multifocal hypopigmentation
(yellow boxes) ■ The bluish-white color has no diagnostic significance.
■ Multifocal hypopigmentation ● The changes are significant and warrant a histopathologic diagnosis.
(stars) ● There is a good dermoscopic–pathologic correlation with the diagnosis of a
■ Bluish-white color mildly dysplastic nevus.
■ The follow-up lesion looks more benign than malignant.

■ There are not enough criteria to diagnose a high grade dysplastic nevus or

melanoma.

PEARLS
■ Experienced dermoscopists might still feel comfortable monitoring this
mildly dysplastic nevus.
■ Beware! There is not always a good correlation with the dermoscopic criteria
to differentiate mild, moderate, or severely dysplastic nevi.
■ Nevi with very irregular criteria might be diagnosed as being mildly dys-

plastic nevi and feature-poor lesions might actually be severely dysplastic


nevi.
Chapter 3 Trunk and Extremities 323

RISK
❑ Low
❑ Intermediate
❑ High

DIAGNOSIS
❑ Nevus
❑ Seborrheic keratosis
❑ Basal cell carcinoma
❑ Vascular
❑ Dermatofibroma
❑ Squamous cell carcinoma
❑ Melanoma
❑ Other

3-113a
Baseline 6 month follow-up

DISPOSITION
❑ No intervention
❑ Follow-up
❑ Histopathologic diagnosis

CASE 113
HISTORY
This is the 6-month follow-up of a nevus on the abdomen of a 30-year-old man.
1. The criteria in the baseline image diagnose a mildly dysplastic nevus.
2. A side-by-side comparison of the baseline and follow-up image demonstrates symmetrical
enlargement.
3. A side-by-side comparison of the baseline and follow-up image demonstrates an asymmetrical
enlargement, asymmetrical darkening, and a diffuse pinkish hue.
4. The differential diagnosis of the lesion at the 6-month follow-up includes: a moderate to
severely dysplastic nevus or melanoma arising in a dysplastic nevus.
5. The changes are significant and warrant a histopathologic diagnosis.
324 DERMOSCOPY: AN ILLUSTRATED SELF-ASSESSMENT GUIDE

RISK
❑ Low
❑ Intermediate
✔ High

DIAGNOSIS
❑ Nevus

3-113b
Seborrheic keratosis
❑ Basal cell carcinoma Baseline 6 month follow-up
❑ Vascular
❑ Dermatofibroma
❑ Squamous cell carcinoma

❑ Melanoma ANSWERS
❑ Other Answers: 1,3,4,5

Discussion:
■ Once again, the baseline image is not atypical enough to warrant an exci-
sion, yet not perfectly banal appearing. This is an indication for sequential
DISPOSITION monitoring.
❑ No intervention ■ It looks more benign than malignant, a “gray zone” lesion.

❑ Follow-up ■ The lesion has changed significantly over time.


✔ Histopathologic diagnosis
❑ ■ Expansion of the right side of the lesion (black arrows).

■ Asymmetrical expansion of the centrally located hyperpigmentation

(white arrows).
■ Histopathologically, this was felt to be an in situ melanoma arising in a dys-
plastic nevus, which is a good dermoscopic–pathologic correlation.
DERMOSCOPIC CRITERIA:
BASELINE IMAGE PEARLS
■ Asymmetry of color and ■ Critics of sequential digital dermoscopic monitoring might argue that with
structure this technique, you could miss a melanoma at the initial examination.
■ Homogeneous-reticular global ■ Studies have shown that most melanomas that are diagnosed with sequential
pattern digital dermoscopic monitoring are either in situ or early invasive lesions.
■ Irregular pigment network ■ In one’s career, everyone misses or misdiagnoses melanoma with or

■ Multifocal hypopigmentation without special diagnostic techniques.


■ Sequential digital dermoscopic monitoring allows you to follow patients
with multiple nevi over time and helps to avoid unnecessary surgery. Yet, it
facilitates the early diagnosis of obvious melanomas or melanoma-incognito.
■ Digital dermoscopy takes your skills to the highest level and might be the
only way to diagnose melanoma that is 100% curable!
Chapter 4

Palms, Soles, Nails


General Instructions: You will find a list of True/False statements following each case
history. Select any statements, which you believe to be true. There may be one, more
than one or no true statements for any given case. Choose the correct risk, diagnosis
and disposition for each case. Then, turn the page to find a detailed discussion and
pearls for each case.
Chapter 4 Palms, Soles, Nails 327

RISK
❑ Low
❑ Intermediate
❑ High

DIAGNOSIS
❑ Nevus
❑ Seborrheic keratosis
❑ Basal cell carcinoma
❑ Vascular

4-1a
❑ Dermatofibroma
❑ Squamous cell carcinoma
❑ Melanoma
❑ Other

DISPOSITION
❑ No intervention
❑ Follow-up
❑ Histopathologic diagnosis
4-1b

CASE 1
HISTORY
The parents of this 5-year-old girl noticed a dark spot on her palm.
1. There is asymmetry of color and structure and the malignant parallel ridge pattern (PRP).
2. The parallel brown lines are in the furrows of the skin diagnosing the malignant parallel
furrow pattern.
3. The parallel brown lines are in the furrows of the skin diagnosing the benign parallel furrow
pattern.
4. The parallel furrow pattern is the most common benign pattern found on glabrous skin.
5. The parallel furrow pattern is the most common benign pattern found on nonglabrous skin.
328 DERMOSCOPY: AN ILLUSTRATED SELF-ASSESSMENT GUIDE

RISK
✔ Low

❑ Intermediate
❑ High

DIAGNOSIS

❑ Nevus
❑ Seborrheic keratosis
❑ Basal cell carcinoma
❑ Vascular
❑ Dermatofibroma

4-1c
❑ Squamous cell carcinoma
❑ Melanoma
❑ Other ANSWERS
Answers: 3, 4

Discussion:
■ There is perfect symmetry of color and structure.
DISPOSITION
■ This nevus is on the volar surface of the hand (ie, glabrous skin).
❑ No intervention ■ Glabrous skin is nonhair-bearing skin.
✔ Follow-up
❑ ■ The unique anatomy of the skin on the palms and soles with its distinctive
❑ Histopathologic diagnosis dermatoglyphic pattern is made up of ridges (gyri) and furrows (sulci or
fissures) that create parallel site-specific patterns.
■ Parallel patterns can also be found on mucosal surfaces.

■ The parallel brown lines are in the furrows of the skin creating the benign
parallel furrow pattern.
DERMOSCOPIC CRITERIA ■ This is the most common benign pattern seen on glabrous skin.
■ Symmetry of color and ■ There is no such entity as the malignant parallel furrow pattern.

structure ■ The ridges and furrows can be seen clinically, which is a clue to the

■ Parallel furrow pattern dermoscopic pattern.


■ Furrows (white arrows) ■ The lighter, hypopigmented parallel lines represent the ridges of the skin:
■ When pigmentation is found in the ridges that creates the parallel ridge
■ Ridges (yellow arrows)

■ Acrosyringia (black arrows)


pattern (PRP), which is very sensitive and specific yet not completely
diagnostic for acral melanoma.
■ Acrosyringia are:
■ The intra-epidermal section of eccrine sweat gland ducts (“eccrine pores”).

■ Always in the ridges and never in the furrows of the skin.

■ Appear as centrally located uniform white dots with a linear arrangement.

■ They are said to look like “a string of pearls.”

PEARLS
■ Digital sequential clinical and dermoscopic monitoring is a cutting-edge way
to follow such a lesion for changes over time that might warrant a
histopathologic diagnosis in the future.
■ Patients and/or their parents are always very impressed when digital dermo-
scopic images are taken for follow-up purposes. It is cutting-edge: Why
shouldn’t they be?
■ When trying to remember whether pigment in ridges or furrows represents
a malignant or benign pattern, think only that “furrows are fine!”
■ Dermoscopy helps to prevent surgical intervention in most, but not all, acral
pigmented skin lesions especially in children!
Chapter 4 Palms, Soles, Nails 329

RISK
❑ Low
❑ Intermediate
❑ High

DIAGNOSIS
❑ Nevus
❑ Seborrheic keratosis
❑ Basal cell carcinoma
❑ Vascular

4-2a
❑ Dermatofibroma
❑ Squamous cell carcinoma
❑ Melanoma
❑ Other

DISPOSITION
❑ No intervention
❑ Follow-up
❑ Histopathologic diagnosis
4-2b

CASE 2
HISTORY
This lesion was found on the foot of a 43-year-old woman.
1. This is the benign parallel furrow pattern, double-line variant.
2. Irregular dots and globules, irregular thickened lines, and wide ridges are a red flag for concern.
3. There are variations of the parallel furrow pattern, which can contain a single line of pigment in
the furrows or two parallel lines on each side of the furrows with or without dots and globules.
4. The parallel furrow pattern could be made up completely of a linear arrangement of dots and
globules.
5. The parallel furrow pattern always contains a single row of pigmentation in the furrows.
330 DERMOSCOPY: AN ILLUSTRATED SELF-ASSESSMENT GUIDE

RISK
✔ Low

❑ Intermediate
❑ High

DIAGNOSIS

❑ Nevus
❑ Seborrheic keratosis
❑ Basal cell carcinoma
❑ Vascular
❑ Dermatofibroma
❑ Squamous cell carcinoma
❑ Melanoma

4-2c
Other

DISPOSITION ANSWERS
❑ No intervention Answers: 1,3,4
✔ Follow-up

❑ Histopathologic diagnosis Discussion:
■ Clinically, but not dermoscopically, this lesion looks similar to that in Case 1
except that it has the double line parallel furrow pattern.
■ The classic parallel furrow pattern is made up of a single brown line in the
skin furrows (“singles line variant”). However, there are several commonly
DERMOSCOPIC CRITERIA encountered variations of this benign pattern.
■ Symmetry of color and ■ This is the double-line variant with parallel brown lines on both sides of the
hypopigmented furrows.
structure
■ There are also a few globules.
■ Parallel furrow pattern
■ One can see dots and globules in combination with linear pigmentation or
■ Double lines in the furrows
dots and globules by themselves.
(black lines) ■ A single row of dots and globules in the furrows without linear pigmenta-
■ Singe lines in furrows tion (“the single-dotted line variant”).
(yellow arrows) ■ Two rows of dots and globules on both sides of hypopigmented furrows
■ Ridges (black arrows) without linear pigmentation (“the double-dotted line variant”).
■ Globules (circles) ■ The width of the ridges has no diagnostic significance in this case.
■ Irregularly pigmented and thickened ridges can be seen in acral

melanoma.

PEARLS
■ It is essential to be aware of all of the variations that can be seen in the
benign parallel furrow pattern, so that they are not mistaken for high risk
lesions resulting in unnecessary surgical procedures.
■ It is not necessary to remember the specific names for the variants, just
remember that the pigmentation has to be in the furrows (“furrows are
fine”) and there can be one or two lines with or without dots and globules,
or just dots and globules alone.
Chapter 4 Palms, Soles, Nails 331

RISK
❑ Low
❑ Intermediate
❑ High

DIAGNOSIS
❑ Nevus
❑ Seborrheic keratosis
❑ Basal cell carcinoma
❑ Vascular
❑ Dermatofibroma
❑ Squamous cell carcinoma
❑ Melanoma
❑ Other

4-3a
DISPOSITION
❑ No intervention
❑ Follow-up
❑ Histopathologic diagnosis

CASE 3
HISTORY
This lesion was found on the foot of a 6-year-old boy and looked clinically similar to the last two cases.
1. Asymmetry of color and structure, irregular globules, and irregular hyperpigmentation
diagnose a melanoma.
2. This is a variant of the benign parallel furrow pattern with brown globules and linear
hyperpigmentation in the skin furrows.
3. Irregularities of the parallel furrow pattern are routinely encountered and are not high risk.
4. Irregularities of the parallel furrow pattern warrant immediate excision.
5. If in doubt, cut it out!
332 DERMOSCOPY: AN ILLUSTRATED SELF-ASSESSMENT GUIDE

RISK
✔ Low

❑ Intermediate
❑ High

DIAGNOSIS

❑ Nevus
❑ Seborrheic keratosis
❑ Basal cell carcinoma
❑ Vascular
❑ Dermatofibroma
❑ Squamous cell carcinoma
❑ Melanoma

4-3b
Other

ANSWERS
DISPOSITION
Answers: 2,3,5
❑ No intervention
✔ Follow-up
❑ Discussion:
❑ Histopathologic diagnosis ■ The global picture in this case is more irregular than in the last two cases.
However, it is a variant of the benign parallel furrow pattern with foci of the
lattice-like pattern.
■ The young age of the patient is also important and favors a benign acral

nevus.
DERMOSCOPIC CRITERIA ■ Acral lentiginous melanoma in a child is extremely improbable.

■ Parallel furrow pattern ■ The parallel furrow pattern in this case is made up of:
■ Globules (white boxes) ■ Linear pigmentation in the furrows.

■ Brown globules in the furrows impinging into the hypopigmented ridges.


■ Skin furrows (black arrows)
■ Irregular hyperpigmentation in the furrows impinging into the ridges.
■ Hyperpigmentation (stars)
■ Criteria impinging into the ridges are commonly seen. They do not diag-
■ Skin ridges (yellow arrows)
nose the parallel ridge pattern and are not associated with melanoma.
■ Lattice-like pattern (white lines)
■ Lattice-like pattern:
■ Is made up of brown lines in the furrows and brown lines running

perpendicular to the furrows creating a ladder-like appearance.


■ Refer to Figure 1-5 in Chapter 1 to see a classic lattice like pattern.

■ There are three complete foci of this pattern with some of the globules

trying to form the rungs of a ladder in other areas.


■ It is not uncommon to have a combination of benign parallel patterns in
one lesion—as long as there are no criteria associated with melanoma, the
combinations in most cases still represent benign nevi.
■ The wide hypopigmented ridges are easily identified. In this case there are
no acrosyringia.
■ Ridges can be with or without acrosyringia.

PEARLS
■ There are innumerable variations of the benign parallel furrow pattern.
■ At times, the global picture in acral nevi is not clear cut. If this happens, and
if you have to think too long about what is going on, fall back on the very
important principle: If in doubt, cut it out.
Chapter 4 Palms, Soles, Nails 333

RISK
❑ Low
❑ Intermediate
❑ High

DIAGNOSIS
❑ Nevus
❑ Seborrheic keratosis
❑ Basal cell carcinoma
❑ Vascular

4-4a
❑ Dermatofibroma
❑ Squamous cell carcinoma
❑ Melanoma
❑ Other

DISPOSITION
❑ No intervention
❑ Follow-up
❑ Histopathologic diagnosis
4-4b

CASE 4
HISTORY
A 24-year-old man was concerned about a pigmented lesion on his left foot.
1. This is the classic parallel furrow pattern with pigmentation in the skin furrows.
2. This is a variation of the morphology that can be seen with the lattice-like pattern with
pigmentation in the furrows and running perpendicular to the furrows.
3. Acrosyringia are clearly seen in the furrows.
4. Acrosyringia are clearly seen in the ridges and on normal skin.
5. One does not see acrosyringia on normal skin.
334 DERMOSCOPY: AN ILLUSTRATED SELF-ASSESSMENT GUIDE

RISK
✔ Low

❑ Intermediate
❑ High

DIAGNOSIS

❑ Nevus
❑ Seborrheic keratosis
❑ Basal cell carcinoma
❑ Vascular
❑ Dermatofibroma
❑ Squamous cell carcinoma
❑ Melanoma

4-4c
Other

DISPOSITION ANSWERS
❑ No intervention Answers: 2,4
✔ Follow-up

❑ Histopathologic diagnosis Discussion:
■ The benign lattice-like pattern is more developed in this nevus compared to
the last case, with multiple pigmented linear lines in the furrows and pig-
mented lines running between furrows.
■ Refer to Figure 1-5 in Chapter 1 to see a classic lattice-like pattern.
DERMOSCOPIC CRITERIA ■ This is a variation of the morphology than can be seen in the lattice-like
■ Asymmetry of color and pattern and falls within the range of a benign acral nevus.
■ There are only foci of pigmentation within the furrows and pigmented
structure
■ Lattice-like pattern lines coming off them.
■ The angles of the rungs of the ladder in this case are not all 90°. They are
■ Furrows (black arrows)
not all perpendicular and do not have to be perpendicular to be within
■ Perpendicular lines (yellow
the range of the criteria to diagnose the lattice-like pattern.
arrows) ■ Multiple white dots representing eccrine pores (acrosyringia) are seen in a
■ Ridges (red arrows) linear and irregular distribution in the lesion and in the surrounding skin.
■ Acrosyringia ■ Acrosyringia are commonly seen on the palms and soles of normal skin,
■ Inside lesion (black boxes) the normal anatomy of the palms and soles.
■ Outside lesion (red boxes)

PEARLS
■ Variations of benign patterns can be found in acral lentiginous melanoma.
■ Make sure that you do not miss criteria associated with melanoma before
diagnosing a benign acral nevus.
Chapter 4 Palms, Soles, Nails 335

RISK
❑ Low
❑ Intermediate
❑ High

DIAGNOSIS
❑ Nevus
❑ Seborrheic keratosis
❑ Basal cell carcinoma
❑ Vascular

4-5a
❑ Dermatofibroma
❑ Squamous cell carcinoma
❑ Melanoma
❑ Other

DISPOSITION
❑ No intervention
❑ Follow-up
❑ Histopathologic diagnosis
4-5b

CASE 5
HISTORY
A nevus on the left fifth finger of a 15-year-old girl increased in size over the past year and itches.
1. Asymmetry of color and structure plus bluish-white color diagnose a nodular melanoma.
2. Foci of the parallel furrow pattern suggest that this could be a benign lesion.
3. Both a benign parallel furrow pattern and criteria to diagnose acral lentiginous melanoma can
be seen in the same lesion.
4. This is the dotted parallel ridge pattern with bluish-white color and irregular acrosyringia,
diagnosing a nodular melanoma.
5. Criteria associated with melanomas seen on nonglabrous skin can also be found on glabrous skin.
336 DERMOSCOPY: AN ILLUSTRATED SELF-ASSESSMENT GUIDE

RISK
❑ Low
❑ Intermediate
✔ High

DIAGNOSIS

❑ Nevus
❑ Seborrheic keratosis
❑ Basal cell carcinoma
❑ Vascular
❑ Dermatofibroma
❑ Squamous cell carcinoma
❑ Melanoma

4-5c
Other

DISPOSITION ANSWERS
❑ No intervention Answers: 2,3,5
❑ Follow-up
✔ Histopathologic diagnosis
❑ Discussion:
■ The history of a changing, itchy lesion is a red flag for concern.
■ Focus your attention and be on the look-out for high risk criteria.

■ The bluish-white color is another red flag for concern.


■ Criteria associated with melanoma (eg, irregular streaks, irregular blotch-
DERMOSCOPIC CRITERIA es, bluish-white color) on nonglabrous skin can also be found on the
■ Asymmetry of color and palms and soles (glabrous or nonhair-bearing skin).
structure ■ Foci of the benign parallel furrow pattern can be seen at the periphery:
■ Furrows (black arrows) ■ Pigmentation in the furrows on each side of hypopigmented ridges.

■ Acrosyringia are seen in some of the ridges.


■ Ridges (yellow arrows)
■ Beware! Benign acral patterns can be found in melanoma.
■ Acrosyringia (black circles)
■ The blend of a linear and nonlinear arrangement of acrosyringia within the
■ Brown globules (yellow circles)
bluish-white area is an irregular feature, indeed the most irregular feature in
■ Bluish-white color (stars)
this lesion.
■ The malignant parallel ridge pattern, which would be seen as linear pigmen-
tation filling the ridges and is associated with melanoma, is not identified
anywhere in this lesion. Rather there are a few brown globules in the ridges,
which have no diagnostic significance.
■ The differential diagnosis of what turned out to be histopathologically an
irritated melanoctyic nevus, includes a dysplastic nevus or melanoma.

PEARLS
■ If there are any clues that an acral lesion might be high risk, be cautious and
make a histopathologic diagnosis as soon as possible. Acral lentiginous
melanomas are among the most difficult melanomas to diagnose. A working
knowledge of the important dermoscopic patterns can help enormously.
■ If you do not put in time with book learning, classic image examination,
and using the technique as often as possible, your dermoscopic skills will not
get to the level where they should be. Laziness could be a fatal error that
should be avoided at all costs!
Chapter 4 Palms, Soles, Nails 337

RISK
❑ Low
❑ Intermediate
❑ High

DIAGNOSIS

4-6a
❑ Nevus
❑ Seborrheic keratosis
❑ Basal cell carcinoma
❑ Vascular
❑ Dermatofibroma
❑ Squamous cell carcinoma
❑ Melanoma
❑ Other
4-6b

DISPOSITION
❑ No intervention
❑ Follow-up
❑ Histopathologic diagnosis
4-6c

CASE 6
HISTORY
This lesion developed de novo on the foot of a 57-year-old man.
1. Even though this looks like a melanoma clinically and dermoscopically, without the malignant
parallel ridge pattern the diagnosis cannot be made.
2. One expects to find parallel patterns on the dorsal and ventral (volar) surfaces of the feet.
3. One expects to find parallel patterns only on the dorsal surfaces of the feet.
4. The criteria to diagnose melanoma are similar on the dorsal surface of the feet, legs, and trunk.
5. There are well-developed melanoma-specific criteria typically found on the trunk.
338 DERMOSCOPY: AN ILLUSTRATED SELF-ASSESSMENT GUIDE

RISK
❑ Low
❑ Intermediate
✔ High
❑ 1

DIAGNOSIS
❑ Nevus 3
❑ Seborrheic keratosis
❑ Basal cell carcinoma
❑ Vascular
4
❑ Dermatofibroma
❑ Squamous cell carcinoma

❑ Melanoma

4-6d
Other

DISPOSITION ANSWERS
❑ No intervention Answers: 4,5
❑ Follow-up
✔ Histopathologic diagnosis
❑ Discussion:
■ This case points out that parallel patterns are only found on the palms,
soles, and mucosa (glabrous skin) and not on the dorsal surfaces of the
hands and feet (hair-bearing nonglabrous skin).
■ Glabrous vs nonglabrous skin
DERMOSCOPIC CRITERIA ■ The dorsal surfaces of the hands and feet are evaluated the same way as are
■ Asymmetry of color and other areas of hair-bearing skin, such as the trunk, arms, legs, and scalp.
structure ■ Well-developed, melanoma-specific criteria are present:
■ Asymmetry of color and structure
■ Multicomponent global
■ Multicomponent global pattern (four different areas of criteria within the
pattern (1,2,3,4)
lesion)
■ Irregular pigment network ■ Irregular black dots/globules
(black arrows) ■ Irregular black blotches
■ Irregular black dots/globules ■ Irregular streaks
(circles) ■ Bluish-white color
■ Irregular streaks (red arrows) ■ Five colors

■ Irregular black blotches ■ Reticular depigmentation

(white stars) ■ The irregular pigment network is a minor features in this lesion.
■ Reticular depigmentation
(white boxes) PEARLS
■ Bluish-white color (black stars) ■ Parallel patterns are only found on the glabrous skin of the palms, soles, and
mucosae.
■ De novo dark lesions developing in any location in any patient young, old,
or in between, are not always high risk but rather are a red flag for concern
that should not be ignored.
■ Dermoscopy is a great tool to get an on the spot second opinion if an
equivocal new dark lesion is low, intermediate, or high risk!
Chapter 4 Palms, Soles, Nails 339

RISK
❑ Low
❑ Intermediate
❑ High

DIAGNOSIS
❑ Nevus
❑ Seborrheic keratosis
❑ Basal cell carcinoma
❑ Vascular

4-7a
❑ Dermatofibroma
❑ Squamous cell carcinoma
❑ Melanoma
❑ Other

DISPOSITION
❑ No intervention
❑ Follow-up
❑ Histopathologic diagnosis
4-7b

CASE 7
HISTORY
This lesion was found on the sole of a 46-year-old man while performing a routine total body skin
examination. The patient said the lesion was there since he was a child but might be changing.
1. Globules identify a melanocytic lesion.
2. Lacunae-like structures put a hemangioma in the differential diagnosis.
3. A bluish-white haze and globules put a Spitz nevus in the differential diagnosis.
4. Globular patterns are seen on the trunk but never on the palms or soles.
5. Irregular globules, irregular blotches, and bluish-white color diagnose a nodular melanoma.
340 DERMOSCOPY: AN ILLUSTRATED SELF-ASSESSMENT GUIDE

RISK
❑ Low
✔ Intermediate

❑ High

DIAGNOSIS

❑ Nevus
❑ Seborrheic keratosis
❑ Basal cell carcinoma
❑ Vascular

4-7c
Dermatofibroma
❑ Squamous cell carcinoma
❑ Melanoma
❑ Other
ANSWERS
Answers: 1,2,3

Discussion:
■ Clinically, this compound nevus is a low risk lesion. Dermoscopically, it is a
DISPOSITION gray-zone lesion that looks more benign than malignant. It is not a perfectly
❑ No intervention banal lesion.
❑ Follow-up ■ Dermoscopic differential diagnosis comes into play with this lesion.
✔ Histopathologic diagnosis
❑ ■ There are minimally irregular dots and globules with a reddish-brown color.

■ Melanocytic lesion (brown dots and globules) vs vascular lesion

(violaceous lacunae)
■ Irregular blotches ■ Confluent globules vs irregular blotches

■ The bluish-white haze puts the globular variant of a Spitz nevus into the
DERMOSCOPIC CRITERIA differential diagnosis.
■ Globular global pattern ■ When one sees bluish-white color, a Spitz nevus usually is in the differ-

■ Irregular dots and globules ential diagnosis.


■ Centrally located bluish/bluish-white color is a clue that the lesion
(black arrows)
might be a Sptiz nevus.
■ Irregular blotches (yellow
■ This is a gray-zone lesion that might be handled differently depending on
arrows)
the experience of the clinician.
■ Bluish-white haze ■ No intervention because this is a benign nevus with a globular pattern.

■ Sequential digital clinical and dermoscopic monitoring to look for impor-

tant changes over time.


■ Histopathologic diagnosis for the novice dermoscopist because of some

gray zone, nondiagnostic irregular features.


■ If in doubt, cut it out! ■ It is better to be safe than sorry!

PEARLS
■ One does not always find parallel patterns on glabrous skin.
■ Benign and malignant nonparallel patterns can be found.
■ One can find melanocytic lesions on the palms/soles that contain a pigment
network, dots/globules, or homogeneous color without parallel patterns.
■ Any shade, distribution or intensity of blue/bluish-white color could be
associated with a high risk lesion.
■ Remember: If there’s blue, they might sue!
■ As a general rule, whatever global pattern or local criteria are seen, you still
have to determine if they are regular or irregular, low or high risk, good or bad.
■ Don’t take things for granted! ■ Try to always keep on your thinking cap!
Chapter 4 Palms, Soles, Nails 341

RISK
❑ Low
❑ Intermediate
❑ High

DIAGNOSIS
❑ Nevus
❑ Seborrheic keratosis
❑ Basal cell carcinoma
❑ Vascular
4-8a

❑ Dermatofibroma
❑ Squamous cell carcinoma
❑ Melanoma
❑ Other

DISPOSITION
❑ No intervention
❑ Follow-up
❑ Histopathologic diagnosis
4-8b

CASE 8
HISTORY
This asymptomatic lesion was found on the foot of a 67-year-old man at a routine follow-up skin
examination.
1. Clinically and dermoscopically, this is a high risk lesion.
2. Clinically and dermoscopically, this is a gray-zone lesion.
3. Clinically and dermoscopically, this is a banal acral nevus.
4. Asymmetry of color and structure, irregular pigment network, and regression diagnose a
regressive melanoma.
5. Regular pigment network, indistinct borders, and island of normal skin diagnose a benign
acral nevus.
342 DERMOSCOPY: AN ILLUSTRATED SELF-ASSESSMENT GUIDE

RISK
✔ Low

❑ Intermediate
❑ High

DIAGNOSIS

❑ Nevus
❑ Seborrheic keratosis
❑ Basal cell carcinoma
❑ Vascular
❑ Dermatofibroma
❑ Squamous cell carcinoma
❑ Melanoma

4-8c
Other

DISPOSITION ANSWERS
❑ No intervention Answers: 3,5
✔ Follow-up

❑ Histopathologic diagnosis Discussion:
■ Clinically and dermoscopically, there are no red flags for concern. This is a
banal clinical and dermoscopic picture.
■ The pigment network is regular, not irregular, and diagnoses a melanocytic
lesion:
DERMOSCOPIC CRITERIA ■ The line segments are thin, light, and look the same throughout the

■ Asymmetry of color and lesion.


structure ■ This case points out that you can see pigment network without parallel pat-
■ Reticular global pattern terns on acral surfaces.
■ Regular pigment network ■ The global pattern is reticular because most of the lesion is filled with pig-
ment network.
(boxes)
■ There is mild asymmetry of color and structure, which in this case is not
■ Islands of normal skin (stars)
high risk.
■ Indistinct margins (arrows) ■ Indistinct margins without an abrupt cut-off is considered a sign of low risk
pathology.
■ Two shades of brown color and an absence of melanoma-specific criteria
point in the direction of banal melanocytic nevus.
■ Normal skin seen within a lesion, on any part of the body, is a common
occurrence and should not be confused with regression.
■ Regression can be seen on glabrous skin.

PEARLS
■ By definition, regression should be lighter than the surrounding skin or a
milky/bony-white color. The distinction is not always possible to make
especially for the novice dermoscopist.
■ Regression is an independently high risk criterion and might be the one clue
to help diagnose melanoma.
■ The more the regression that fills a lesion, the greater the chance it is a
melanoma.
Chapter 4 Palms, Soles, Nails 343

RISK
❑ Low
❑ Intermediate
❑ High

DIAGNOSIS
❑ Nevus
❑ Seborrheic keratosis
❑ Basal cell carcinoma
❑ Vascular

4-9a
❑ Dermatofibroma
❑ Squamous cell carcinoma
❑ Melanoma
❑ Other

DISPOSITION
❑ No intervention
❑ Follow-up
❑ Histopathologic diagnosis
4-9b

CASE 9
HISTORY
A 35-year-old woman discovered this asymptomatic and nonchanging lesion on the lateral surface
of her right foot.
1. Clinically and dermoscopically, this is a high risk lesion.
2. Dermoscopically, but not clinically, this is a high risk lesion.
3. Regular pigment network, islands of normal skin, and sharp border demarcation diagnose a
banal acral nevus.
4. Asymmetry of color and structure and a very irregular pigment network are red flags for
concern.
5. A dysplastic nevus and in situ melanoma are in the dermoscopic but not clinical differential
diagnosis.
344 DERMOSCOPY: AN ILLUSTRATED SELF-ASSESSMENT GUIDE

RISK
❑ Low
❑ Intermediate
✔ High

DIAGNOSIS

❑ Nevus
❑ Seborrheic keratosis
❑ Basal cell carcinoma
❑ Vascular
❑ Dermatofibroma
❑ Squamous cell carcinoma
❑ Melanoma

4-9c
Other

ANSWERS
DISPOSITION
Answers: 2,4,5
❑ No intervention
❑ Follow-up Discussion:
✔ Histopathologic diagnosis
❑ ■ The dermoscopic picture is a surprise because it is very irregular while
clinically the lesion does not look high risk:
■ This case points out how much more important information you can

obtain when using dermoscopy vs clinical examination alone.


■ The global pattern is reticular because the majority of the lesion is made up
DERMOSCOPIC CRITERIA of pigment network.
■ Compared to the last case, the pigment network is very irregular because
■ Asymmetry of color and
structure it is thickened, branched, and broken-up.
■ There is multifocal hypopigmentation typically found in dysplastic nevi, which
■ Reticular global pattern
should not be confused with regression, a common feature of melanoma.
■ Irregular pigment network
■ The foci of irregular streaks are a minor point compared to the irregular
(boxes) pigment network that fills the lesion.
■ Irregular streaks (red arrows) ■ The shades of brown color are much more intense compared to the last case:
■ Multifocal hypopigmentation ■ Intensities of color have diagnostic significance. In general, the darker
(stars) they are, the more high risk the lesion might be.
■ Sharp border demarcation ■ A sharp lesional border is considered a high risk criterion. An exception to
(black arrows) this rule is found in seborrheic keratosis that typically have sharp borders.
■ There can always be an exception to every rule!

■ Compare this dysplastic nevus with the last case to see the difference

between nonsharp vs sharp borders.


■ The dermoscopic differential diagnosis includes dysplastic nevus vs in situ
melanoma.
■ In situ melanoma can be diagnosed simply by having a very irregular

pigment network as the major melanoma-specific criterion.

PEARLS
■ There is not a good clinico-dermoscopic correlation, which should be a red
flag for concern.
■ Outbursts of shock and surprise with unexpected dramatic dermoscopic
pictures do not sit well with patients. Control yourself!
Chapter 4 Palms, Soles, Nails 345

RISK
❑ Low
❑ Intermediate
❑ High

DIAGNOSIS
❑ Nevus
❑ Seborrheic keratosis
❑ Basal cell carcinoma

4-10a
Vascular
❑ Dermatofibroma
❑ Squamous cell carcinoma
❑ Melanoma
❑ Other

DISPOSITION
❑ No intervention
❑ Follow-up
❑ Histopathologic diagnosis
4-10b

CASE 10
HISTORY
The podiatrist of this 57-year-old woman found this lesion and referred the patient to you because
he did not want to do a skin biopsy.
1. Clinically and dermoscopically, this is a low risk lesion.
2. Clinically and dermoscopically, this is a gray-zone lesion.
3. The fingerprint pattern and moth-eaten borders diagnose a solar lentigo.
4. A slightly irregular pigment network, asymmetry of color and structure, and multifocal
hypopigmentation are red flags for concern.
5. The high risk clinical picture outweighs the banal appearing dermoscopic picture and a
histopathologic diagnosis is indicated.
346 DERMOSCOPY: AN ILLUSTRATED SELF-ASSESSMENT GUIDE

RISK
❑ Low
❑ Intermediate
✔ High

DIAGNOSIS
❑ Nevus
❑ Seborrheic keratosis
❑ Basal cell carcinoma
❑ Vascular
❑ Dermatofibroma

4-10c
Squamous cell carcinoma

❑ Melanoma
❑ Other

ANSWERS
Answers: 2,4,5
DISPOSITION
Discussion:
❑ No intervention ■ Pigment network diagnoses a melanocytic lesion. There is no acral parallel
❑ Follow-up pattern because the lesion is located on the dorsal surface of the toe
✔ Histopathologic diagnosis
❑ (ie, nonglabrous skin).
■ By definition, this is the reticular global pattern because the lesion is filled

with a pigment network.


■ Criteria to diagnose a solar lentigo other than a focus of the fingerprint pat-
tern are not present (eg, moth-eaten borders) and the lesion does not look
DERMOSCOPIC CRITERIA like a solar lentigo clinically.
■ Asymmetry of color and ■ Beware! A solar lentigo can be associated with high risk melanocytic
structure pathology such as atypical melanocytic hyperplasia, dysplastic nevi, and
■ Reticular global pattern melanoma.
■ Irregular pigment network ■ The clinical picture looks more high risk than the banal appearing
(black boxes) dermoscopic picture of:
■ Mild asymmetry of color and structure
■ Multifocal hypopigmentation
■ Minimally irregular pigment network
(white stars)
■ Multifocal hypopigmentation
■ Fingerprint pattern (yellow
■ The two foci of bony-white color represent reflection artifact or scale
box)
formation. They are not areas of regression.
■ Subungual hematoma ■ This is a gray-zone lesion with red flags for concern but no definitive
(yellow arrow) diagnosis can be made with dermoscopy.
■ Nail plate (black stars)
■ Reflection artifact PEARLS
(black arrows)
■ In most cases, an atypical clinical picture is overridden by a banal
dermoscopic picture.
■ At times, an atypical clinical picture trumps a banal dermoscopic picture
and it is better to err on the side of conservatism and make a histopathologic
diagnosis.
■ Think in terms of a good clinico–dermoscopic–pathologic correlation.
■ Multiple skin biopsies are often needed to diagnose melanoma when the
clinical lesion is large.
■ Use the most atypical dermoscopic area for an incisional biopsy.
Chapter 4 Palms, Soles, Nails 347

RISK
❑ Low
❑ Intermediate
❑ High

DIAGNOSIS
❑ Nevus
❑ Seborrheic keratosis
❑ Basal cell carcinoma
❑ Vascular
4-11a

❑ Dermatofibroma
❑ Squamous cell carcinoma
❑ Melanoma
❑ Other

DISPOSITION
❑ No intervention
❑ Follow-up
❑ Histopathologic diagnosis
4-11b

(Reprinted, with permission, from Stolz W, Braun-Falco O, Bilek P et al. (2002) Color Atlas of
Dermatoscopy. Second Edition. Blackwell Publishing, Oxford.)

CASE 11
HISTORY
The patient says that this spot has been on his foot since he was a child and has not changed
in any way.
1. Clinically and dermoscopically, this is a high risk lesion.
2. Clinically but not dermoscopically, this is a high risk lesion.
3. This is a banal acral nevus with the lattice-like pattern.
4. This is a banal nevus with the parallel furrow pattern.
5. Fine oblique lines diagnose the benign fibrillar pattern.
348 DERMOSCOPY: AN ILLUSTRATED SELF-ASSESSMENT GUIDE

RISK
✔ Low

❑ Intermediate
❑ High

DIAGNOSIS

❑ Nevus
❑ Seborrheic keratosis
❑ Basal cell carcinoma
❑ Vascular
❑ Dermatofibroma
❑ Squamous cell carcinoma
❑ Melanoma

4-11c
❑ Other

DISPOSITION ANSWERS
❑ No intervention Answers: 5
✔ Follow-up

❑ Histopathologic diagnosis Discussion:
■ This is a classic fine filamentous fibrillar acral pattern, characterized by thin
oblique parallel line segments.
■ Oblique parallel line segments that are thickened and irregular can be seen
in melanoma.
DERMOSCOPIC CRITERIA ■ The history and clinical and dermoscopic pictures are all low risk.
■ Fibrillar global pattern ■ There are no high risk criteria.
■ Oblique parallel line
■ With pressure, the lattice-like pattern can transform into the fibrillar
segments (\\\) pattern.
■ Ridges (stars)
■ Furrows (arrows) PEARLS
■ Acral nevi should be followed like any other nevus on other areas of the
body.
■ It is a misconception held by some that acral nevi are more likely to trans-
form into melanoma and they should be removed as a preventative measure.
■ Contrary to popular belief, they do not need to be removed simply because
of their location unless there are high risk criteria.
■ Sequential digital clinical and dermoscopic monitoring is the best way to
follow low risk acral nevi.
Chapter 4 Palms, Soles, Nails 349

RISK
❑ Low
❑ Intermediate
❑ High

DIAGNOSIS
❑ Nevus
❑ Seborrheic keratosis
❑ Basal cell carcinoma
❑ Vascular

4-12a
❑ Dermatofibroma
❑ Squamous cell carcinoma
❑ Melanoma
❑ Other

DISPOSITION
❑ No intervention
❑ Follow-up
❑ Histopathologic diagnosis
4-12b

(Reprinted, with permission, from Stolz W, Braun-Falco O, Bilek P et al.


(2002) Color Atlas of Dermatoscopy. Second Edition. Blackwell
Publishing, Oxford.)

CASE 12
HISTORY
A 64-year-old woman had a biopsy on this acral lesion 2 years ago and a lentiginous nevus was
diagnosed. The lesion has been slowly getting bigger and darker.
1. This is a classic parallel furrow pattern with hypopigmented ridges and pigmented furrows.
2. This is a classic malignant parallel ridge pattern with pigmentation in the ridges.
3. Acrosyringia are clearly seen in the skin furrows.
4. Acrocringia are in the pigmented ridges.
5. The biopsy was benign and this is a gray-zone lesion that warrants sequential digital dermoscopic
follow-up.
350 DERMOSCOPY: AN ILLUSTRATED SELF-ASSESSMENT GUIDE

RISK
❑ Low
❑ Intermediate
✔ High

DIAGNOSIS
❑ Nevus
❑ Seborrheic keratosis
❑ Basal cell carcinoma

4-12c
❑ Vascular
❑ Dermatofibroma
❑ Squamous cell carcinoma

❑ Melanoma ANSWERS
❑ Other Answers: 2,4

Discussion:
■ One does not need dermoscopy to realize that this is a high risk lesion that
needs a histopathologic diagnosis.
DISPOSITION
■ This is a classic malignant parallel ridge pattern that confirms the clinical
❑ No intervention diagnosis of a melanoma.
❑ Follow-up ■ A review of the pathology of an older biopsy in a malignant lesion is indi-
✔ Histopathologic diagnosis
❑ cated in some situations. Possibly, the original biopsy was not read by an
experienced dermatopathologist and the correct diagnosis was missed.
■ Pigmentation is in the skin ridges.
■ There are ridges with and without acrosyringia.
■ One does not always see acrosyringia in the ridges.
DERMOSCOPIC CRITERIA ■ Acrosyringia are important landmarks to help determine where
■ Parallel ridge pattern pigmentation is. In a straightforward case, with furrows and ridges,
■ Pigmented ridges with linear they are always in the ridges.
acrosyringia (boxes) ■ In this case, the furrows are hypopigmented. The opposite of the benign
■ Pigmented ridges without parallel furrow pattern in which they are pigmented.
acrosyringia (black arrows) ■ It is not known why pigmentation favors skin ridges in malignant lesions vs
■ Furrows (black lines)
furrows in benign acral lesions. The sensitivity and specificity for this is,
■ Acrosyringia (yellow arrows)
however, is very high.
■ When a lesion is made up exclusively of the parallel ridge pattern, it is either
an in situ or early invasive acral lentiginous melanoma.
■ Foci of the parallel ridge pattern can be found in more invasive acral lentigi-
nous melanoma usually with other melanoma-specific criteria (eg, irregular
streaks, dark irregular blotches, bluish-white color, regression, pink color,
atypical vessels).

PEARLS
■ If you see the parallel ridge pattern with a nonmalignant pathology report, it
would indicate a poor dermoscopic–pathologic correlation and should raise
a red flag for concern.
■ Consider performing several biopsies with large lesions at acral sites.
■ Beware! Even experienced dermatopathologists can have trouble diagnosing
atypical acral melanocytic lesions. If there is any question, do not hesitate to
get another dermatopathologist’s opinion, preferably a national pigmented
lesion expert. Even the super experts are not always correct in diagnosing acral
melanomas. You can’t always win. Unfortunately, your patient might die!!
Chapter 4 Palms, Soles, Nails 351

RISK
❑ Low
❑ Intermediate
❑ High

DIAGNOSIS
❑ Nevus
❑ Seborrheic keratosis
❑ Basal cell carcinoma
❑ Vascular

4-13a
❑ Dermatofibroma
❑ Squamous cell carcinoma
❑ Melanoma
❑ Other

DISPOSITION
❑ No intervention
❑ Follow-up
❑ Histopathologic diagnosis
4-13b

CASE 13
HISTORY
A 72-year-old tennis player with a history of extensive sun exposure and nonmelanoma skin cancer
found this lesion on his right thumb. He came in right away because a friend had just died from
melanoma.
1. Clinically and dermoscopically, this is a worrisome lesion.
2. Irregular dots and globules and the parallel furrow pattern diagnose a melanoma.
3. Irregular dots and globules and the parallel ridge pattern diagnose melanoma.
4. Blood pebbles and blood in the skin ridges diagnose skin hemorrhage.
5. The parallel ridge pattern is diagnostic of melanoma. There is no differential diagnosis.
352 DERMOSCOPY: AN ILLUSTRATED SELF-ASSESSMENT GUIDE

RISK
✔ Low

❑ Intermediate
❑ High

DIAGNOSIS
❑ Nevus
❑ Seborrheic keratosis
❑ Basal cell carcinoma
❑ Vascular
❑ Dermatofibroma
❑ Squamous cell carcinoma
❑ Melanoma

4-13c

❑ Other

DISPOSITION ANSWERS
❑ No intervention Answers: 4
✔ Follow-up

❑ Histopathologic diagnosis Discussion:
■ This is a purplish, not brown or black, parallel ridge pattern associated with
skin hemorrhage.
■ The parallel ridge pattern is very sensitive and specific to diagnose
melanoma; however, it is not pathognomonic of melanoma.
DERMOSCOPIC CRITERIA ■ The parallel ridge pattern can also be seen in:
■ Parallel ridge pattern ■ Skin hemorrhage

■ Ridges (yellow arrows) ■ Banal acral melanocytic nevi in darker skinned races

■ Diffusely as a normal finding on the palms and soles of darker skin races
■ Furrows (black arrows)
■ Postinflammatory changes in darker skinned races
■ Blood pebbles (circles)
■ Acral hemangiomas

■ The peripheral dots/globules represent “blood pebbles.”


■ Dots/globules of a melanocytic lesion are in the differential diagnosis.

PEARLS
■ Sequential digital clinical and dermoscopic monitoring can be used to
follow the expected natural color changes one sees with the breakdown and
disappearance of blood.
■ For a faster way to diagnose hemorrhage one can:
■ Pick out the blood with a 20-gauge needle.

■ Perform a heme guaiac test—blood in the skin will be guaiac positive.

■ Beware! The presence of blood does not rule out melanoma. Acral
melanomas can be associated with blood. Always search for melanoma-
specific criteria before diagnosing pure skin hemorrhage.
Chapter 4 Palms, Soles, Nails 353

RISK
❑ Low
❑ Intermediate
❑ High

DIAGNOSIS
❑ Nevus
❑ Seborrheic keratosis
❑ Basal cell carcinoma
❑ Vascular

4-14a
❑ Dermatofibroma
❑ Squamous cell carcinoma
❑ Melanoma
❑ Other

DISPOSITION
❑ No intervention
❑ Follow-up
❑ Histopathologic diagnosis
4-14b

CASE 14
HISTORY
A 76-year-old man has this lesion on his foot for several years. There was a history of trauma in the
area that never healed.
1. With a history of trauma, gray color and peppering, one can diagnose post-inflammatory
hyperpigmentation.
2. Asymmetry of color and structure, and gray color are red flags for concern.
3. There are remnants of the parallel ridge pattern, which puts melanoma in the differential
diagnosis.
4. There are remnants of the benign parallel furrow pattern, which can be seen in invasive acral
lentiginous melanoma.
5. Clinically and dermoscopically, this is a high risk lesion that warrants a histopathologic
diagnosis posthaste.
354 DERMOSCOPY: AN ILLUSTRATED SELF-ASSESSMENT GUIDE

RISK
❑ Low
❑ Intermediate
✔ High

DIAGNOSIS
❑ Nevus
❑ Seborrheic keratosis
❑ Basal cell carcinoma
❑ Vascular
❑ Dermatofibroma
❑ Squamous cell carcinoma

❑ Melanoma

4-14c
❑ Other

DISPOSITION ANSWERS
❑ No intervention
Answers: 2,3,4,5
❑ Follow-up
✔ Histopathologic diagnosis
❑ Discussion:
■ The history of trauma is misleading and with gray color and peppering one
might mistakenly diagnose post-traumatic dyschromia if one does not
examine the entire lesion.
■ Regression is seen clinically and dermoscopically in the form of white color
DERMOSCOPIC CRITERIA and gray color with peppering.
■ Asymmetry of color and ■ Compared to the last case, the parallel patterns are poorly developed:
structure ■ The black arrows identify faint areas of the parallel ridge pattern without

■ Parallel ridge pattern acrosyringia in the ridges.


(black arrows) ■ One cannot be sure if the yellow arrows identify the parallel ridge or

■ Parallel furrow pattern parallel furrow pattern.


■ Both patterns can be found in acral lentiginous melanoma.
(yellow arrows)
■ Combinations of parallel patterns can be found in invasive acral melanomas as
■ Regression (stars)
opposed to in situ lesions that usually only contain the parallel ridge pattern.
■ Gray color and peppering

(boxes)
PEARLS
■ At times, it is difficult to differentiate the gray dots and globules of peppering
from dots and globules representing malignant melanocytes in melanoma.
■ Err on the side of caution and make the diagnosis as soon as possible.
■ Sequential digital clinical and dermoscopic monitoring is contraindicated in
this case. Valuable time would be lost in making the correct diagnosis.
Chapter 4 Palms, Soles, Nails 355

RISK
❑ Low
❑ Intermediate
❑ High

DIAGNOSIS
❑ Nevus
❑ Seborrheic keratosis
❑ Basal cell carcinoma

4-15a
Vascular
❑ Dermatofibroma
❑ Squamous cell carcinoma
❑ Melanoma
❑ Other

DISPOSITION
❑ No intervention
❑ Follow-up
❑ Histopathologic diagnosis
4-15b

(Reprinted, with permission, from Stolz W, Braun-Falco O, Bilek P et al.


(2002) Color Atlas of Dermatoscopy. Second Edition. Blackwell
Publishing, Oxford.)

CASE 15
HISTORY
A 68-year-old woman has a pigmented lesion on the plantar surface of her foot that has been
getting thicker over the past few years.
1. Without the parallel ridge pattern, melanoma cannot be diagnosed.
2. The bluish-white color is a red flag for concern that this might be a melanoma.
3. The irregular brown globular-like structures could represent remnants of the parallel ridge
pattern.
4. The nonlinear arrangement of acrosyringia favors a benign lesion.
5. The nonlinear arrangement of the acrosyringia is irregular and represents destruction of the
normal architecture by invasive melanoma.
356 DERMOSCOPY: AN ILLUSTRATED SELF-ASSESSMENT GUIDE

RISK
❑ Low
1
❑ Intermediate
✔ High

2

DIAGNOSIS 2
3
❑ Nevus
❑ Seborrheic keratosis
❑ Basal cell carcinoma
❑ Vascular 2
❑ Dermatofibroma
❑ Squamous cell carcinoma

❑ Melanoma

4-15c
❑ Other

DISPOSITION ANSWERS
❑ No intervention Answers: 2, 3, 5
❑ Follow-up
✔ Histopathologic diagnosis
❑ Discussion:
■ The history and clinical appearance are worrisome. The irregular dermo-
scopic picture confirms that this could be a high risk lesion→ melanoma.
■ The bluish-white color is a red flag for concern and represents deep involve-
ment of the pathologic process.
DERMOSCOPIC CRITERIA ■ Different shades of blue color represent pigmentation in the deeper der-

■ Asymmetry of color and mis.


structure ■ Without the expected parallel patterns, one should evaluate the lesions as if
■ Multicomponent global it were on nonglabrous skin.
■ There are several melanoma-specific criteria:
pattern (1,2,3)
■ Asymmetry of color and structure
■ Irregular brown globules
■ Multicomponent global pattern
(boxes) ■ Bluish-white color
■ Bluish-white color (stars) ■ The irregular brown globular-like structures have a differential diagnosis.
■ Acrosyringia (arrows) Irregular globules of a melanocytic lesion, or remnants of the parallel ridge
pattern that have been destroyed by the regressive process.
■ In some but not all areas they have a linear appearance, which favors

remnants of a parallel pattern.


■ The acrosyringia have foci of linear and nonlinear distribution, which is
irregular and a red flag for concern.

PEARLS
■ Everything in this case, from the history to the clinical and dermoscopic pic-
tures, points to the diagnosis of invasive acral lentiginous melanoma.
■ In reality, this case can only be a melanoma. Inform your patient about the
possibility of malignancy. Hold out hope that there is a chance that it might
be benign.
■ The most atypical clinical scenario can still turn out not to be malignant. It
happens!
Chapter 4 Palms, Soles, Nails 357

RISK
❑ Low
❑ Intermediate
❑ High

DIAGNOSIS
❑ Nevus
❑ Seborrheic keratosis
❑ Basal cell carcinoma
❑ Vascular
4-16a

❑ Dermatofibroma
❑ Squamous cell carcinoma
❑ Melanoma
❑ Other

DISPOSITION
❑ No intervention
❑ Follow-up
❑ Histopathologic diagnosis
4-16b

CASE 16
HISTORY
A 75-year-old woman developed persistent pain while walking after treatment for a callus on the
sole of her right foot.
1. The parallel furrow pattern and peppering diagnose a benign nevus with post-inflammatory
changes secondary to the treatment of the “callus”.
2. The malignant parallel ridge pattern is clearly seen indicating that this was not a callus but a
melanoma mistakenly treated as a callus.
3. Bluish-white, gray, and pink color plus pinpoint vessels indicate this is not an in situ
melanoma.
4. This represents the benign parallel ridge pattern created by hemorrhage secondary to the
treatment of the callus.
5. Sequential digital dermoscopic monitoring is indicated to follow the natural disappearance of
the hemorrhage.
358 DERMOSCOPY: AN ILLUSTRATED SELF-ASSESSMENT GUIDE

RISK
❑ Low
❑ Intermediate
✔ High

DIAGNOSIS
❑ Nevus
❑ Seborrheic keratosis
❑ Basal cell carcinoma
❑ Vascular
❑ Dermatofibroma
❑ Squamous cell carcinoma

❑ Melanoma

4-16c
❑ Other

DISPOSITION ANSWERS
❑ No intervention Answers: 2,3
❑ Follow-up
✔ Histopathologic diagnosis
❑ Discussion:
■ The malignant parallel ridge pattern is clearly seen with pigmentation in the
thicker ridges flanked on both sides by the lighter colored furrows.
■ The bluish-white and gray color has a differential diagnoses, invasive
melanoma vs post-inflammatory changes secondary to the treatment of a
DERMOSCOPIC CRITERIA “callus”.
■ Asymmetry of color and ■ Asymmetry of color and structure, pink color, and pinpoint vessels are high
structure risk features.
■ The color is not good for hemorrhage because it is brown not purplish,
■ Parallel ridge pattern
ruling out the parallel ridge pattern created by blood.
■ Ridges (black lines)
■ Sequential digital clinical and dermoscopic monitoring is contraindicated.
■ Furrows (arrows)
There is too much evidence that this is not a simple callus but rather an
■ Bluish-white color (stars) invasive acral lentiginous melanoma.
■ Gray color (boxes)
■ Diffuse pink color PEARLS
■ Pinpoint vessels (circles)
■ This will not be the first nor the last time acral lentiginous melanoma is
misdiagnosed.
■ Always keep melanoma in mind when dealing with acral lesions.
■ Equivocal lesions need close monitoring.
■ Don’t lose your patient to follow-up!
Chapter 4 Palms, Soles, Nails 359

RISK
❑ Low
❑ Intermediate
❑ High

DIAGNOSIS
❑ Nevus
❑ Seborrheic keratosis
❑ Basal cell carcinoma
❑ Vascular

4-17a
❑ Dermatofibroma
❑ Squamous cell carcinoma
❑ Melanoma
❑ Other

DISPOSITION
❑ No intervention
❑ Follow-up
❑ Histopathologic diagnosis
4-17b

CASE 17
HISTORY
A 44-year-old man was concerned about this spot on his foot that would not go away.
1. Clinically and dermoscopically this is a worrisome lesion.
2. There is asymmetry of color and structure but the malignant parallel ridge pattern is not
identified, therefore melanoma cannot be diagnosed.
3. There is subtle pigmentation in the ridges which is enough to diagnose the malignant parallel
ridge pattern.
4. Irregular dots and globules scattered throughout the lesion and irregular dark blotches are
high risk criteria.
5. The banal dermoscopic features trump the high risk clinical appearance, and a histopathologic
diagnosis is not indicated.
360 DERMOSCOPY: AN ILLUSTRATED SELF-ASSESSMENT GUIDE

RISK
❑ Low
❑ Intermediate
✔ High

DIAGNOSIS
❑ Nevus
❑ Seborrheic keratosis
❑ Basal cell carcinoma
❑ Vascular
❑ Dermatofibroma
❑ Squamous cell carcinoma

❑ Melanoma

4-17c
❑ Other

DISPOSITION ANSWERS
❑ No intervention Answers: 1,3,4
❑ Follow-up
✔ Histopathologic diagnosis
❑ Discussion:
■ Clinically and dermoscopically, this is a high risk lesion.
■ The parallel ridge pattern is present with tan pigmentation in some but not
all of the ridges.
■ Compared to some of the other cases, one can see that there can be subtle
DERMOSCOPIC CRITERIA presentations of this high risk criterion.
■ Asymmetry of color and ■ One has to focus their attention and specifically look for the pigmenta-

structure tion in the ridges so they are not missed.


■ Parallel ridge pattern (stars) ■ Foci of the parallel furrow pattern with linear pigmentation in some furrows
are also seen.
■ Parallel furrow pattern
■ A haphazard distribution of irregular brown dots/globules and irregular
(yellow arrows)
pigmentation are red flags for concern.
■ Ridges (yellow lines) ■ At times, a suspicious clinical appearance trumps a not so high risk looking
■ Furrows (black arrows) dermoscopic picture and a histopathologic diagnosis should be considered.
■ Irregular brown dots and
globules (circles) PEARLS
■ Irregular dark brown blotches
■ Before you diagnose a low risk lesion, make sure that you do not miss high
(red arrows)
risk criteria that might be present yet are not well-developed.
■ This is a busy image with lots of marks to identify. One has to focus this
attention to study the image just as one should focus their attention and
study the dermoscopic criteria in a difficult case.
■ If one has to spend too much time thinking about the dermoscopic picture,
it is better to err on the conservative side and make a histopathologic
diagnosis.
Chapter 4 Palms, Soles, Nails 361

RISK
❑ Low
❑ Intermediate
❑ High

DIAGNOSIS
❑ Nevus
❑ Seborrheic keratosis
❑ Basal cell carcinoma
❑ Vascular

4-18a
❑ Dermatofibroma
❑ Squamous cell carcinoma
❑ Melanoma
❑ Other

DISPOSITION
❑ No intervention
❑ Follow-up
❑ Histopathologic diagnosis
4-18b

CASE 18
HISTORY
A 32-year-old woman has had this lesion since childhood and thinks it has been getting slowly
bigger over many years.
1. This is a perfectly banal appearing homogeneous global pattern that is of no concern.
2. The black color is not homogeneously seen throughout the lesion, which is a red flag for
concern.
3. There are foci of pigmentation in some ridges, which diagnose a low risk parallel ridge pattern.
4. There are foci of pigmentation in some ridges, which diagnose the malignant parallel ridge
pattern.
5. There is a linear and haphazard distribution of acrosyringia, which is a red flag for concern.
362 DERMOSCOPY: AN ILLUSTRATED SELF-ASSESSMENT GUIDE

RISK
❑ Low
❑ Intermediate
✔ High

DIAGNOSIS
❑ Nevus
❑ Seborrheic keratosis
❑ Basal cell carcinoma
❑ Vascular
❑ Dermatofibroma
❑ Squamous cell carcinoma

❑ Melanoma

4-18c
❑ Other

DISPOSITION ANSWERS
❑ No intervention Answers: 2,4,5
❑ Follow-up
✔ Histopathologic diagnosis
❑ Discussion:
■ The long-term banal history conflicts with the high risk clinical and
dermoscopic picture.
■ There can be a nonparallel homogeneous pattern seen on glabrous skin.
■ A low risk homogeneous pattern is made up of uniform light color without
DERMOSCOPIC CRITERIA any high risk criteria.
■ Homogeneous global pattern ■ This lesion has several high risk features suggesting that it could be a
■ Asymmetry of color and melanoma:
■ Asymmetry of color and structure
structure
■ Bluish-white color
■ Bluish-white and black colors
■ Irregular black color filling the lesion
(stars)
■ A linear and haphazard distribution of acrosyringia
■ Ridges in normal skin
■ There are foci of the malignant parallel ridge pattern without acrosyringia.
(black lines) ■ Compared with the last case, this is a more advanced acral lentiginous
■ Furrows in normal skin melanoma that is easier to diagnose.
(black arrows) ■ There is no doubt about the significance of the black pigmentation.
■ Acrosyringia (yellow boxes) ■ The black color is intense and fills the lesion.
■ Parallel ridge pattern without ■ The bluish-white and black colors are signs of invasion.

acrosyringia (yellow arrows)


PEARLS
■ There are no clues clinically or dermoscopically that this is not a high risk
lesion.
■ The high risk clinical and dermoscopic pictures trumps the relatively low
risk history, and points out how much useful information can be obtained
with dermoscopy.
■ Keep in mind that patient’s histories are not always accurate.
Chapter 4 Palms, Soles, Nails 363

RISK
❑ Low
❑ Intermediate
❑ High

DIAGNOSIS
❑ Nevus
❑ Seborrheic keratosis
❑ Basal cell carcinoma
❑ Vascular

4-19a
❑ Dermatofibroma
❑ Squamous cell carcinoma
❑ Melanoma
❑ Other

DISPOSITION
❑ No intervention
❑ Follow-up
❑ Histopathologic diagnosis
4-19b

CASE 19
HISTORY
This lesion was present for 3 years on the heel of a 78-year-old woman. It started to bleed 2 weeks
before she came in for a consultation.
1. Clinically and dermoscopically, this is a typical “black heel” with ulceration secondary to
trauma.
2. Different shades of blue color are a red flag for concern.
3. Well-developed acrosyringia suggests that this is a melanocytic lesion.
4. The central area has a differential diagnosis that includes ulceration of a nodular melanoma.
5. Hemorrhagic crusts rule out melanoma and favor senile purpura, which is commonly seen in
this age group.
364 DERMOSCOPY: AN ILLUSTRATED SELF-ASSESSMENT GUIDE

RISK
❑ Low
❑ Intermediate
✔ High

DIAGNOSIS
❑ Nevus
❑ Seborrheic keratosis
❑ Basal cell carcinoma
❑ Vascular
❑ Dermatofibroma
❑ Squamous cell carcinoma

❑ Melanoma

4-19c
❑ Other

DISPOSITION ANSWERS
❑ No intervention Answers: 2,3,4
❑ Follow-up
✔ Histopathologic diagnosis
❑ Discussion:
■ There is nothing in the history, clinical, or dermoscopic picture to suggest
that this is not a high risk lesion.
■ The absence of the purplish color typical of blood and well-developed
acrosyringia rules out “black heel” (talon noir) and senile purpura and
DERMOSCOPIC CRITERIA favors a melanocytic lesion.
■ Homogeneous global pattern ■ The central darker area represents a deeper nodular component and the
■ Bluish-white color (white stars) bluish-white color with linear acrosyringia represents the more superficial
■ Dark blue homogeneous color malignant parallel ridge pattern.
■ Clinically and dermoscopically, there is ulceration with foci of hemorrhagic
(yellow stars)
crusts.
■ Acrosyringia (boxes)
■ This is the ultimate expression of how bad an acral melanoma can be.
■ Hemorrhagic crusts
(white arrows)
PEARLS
■ It is controversial if total body skin examinations should be the standard of
care for dermatologists and other categories of physicians. Our colleagues
that do not perform total body skin examinations would miss this lesion
if it were asymptomatic and not the chief complain of the patient.
■ One happier scenario would be that it was discovered at an in situ 100%
curable stage at an earlier visit to a physician for an unrelated complaint
and a complete skin examination was performed.
Chapter 4 Palms, Soles, Nails 365

RISK
❑ Low
❑ Intermediate
❑ High

DIAGNOSIS
❑ Nevus
❑ Seborrheic keratosis
❑ Basal cell carcinoma
❑ Vascular

4-20a
❑ Dermatofibroma
❑ Squamous cell carcinoma
❑ Melanoma
❑ Other

DISPOSITION
❑ No intervention
❑ Follow-up
❑ Histopathologic diagnosis
4-20b

CASE 20
HISTORY
A 75-year-old woman had a painful hyperkeratosis on the sole of her foot for one year.
1. The presence of dried blood rules out melanoma.
2. The absence of a malignant parallel ridge pattern rules out melanoma.
3. Bluish-white and milky-red colors are clues that this could be a melanoma.
4. One focus of irregular globules is enough to identify a melanocytic lesion.
5. Asymmetry of color and structure and a multicomponent global pattern are more clues that
this could be a melanoma.
366 DERMOSCOPY: AN ILLUSTRATED SELF-ASSESSMENT GUIDE

RISK
❑ Low
❑ Intermediate 1
✔ High

DIAGNOSIS
❑ Nevus 2
❑ Seborrheic keratosis
❑ Basal cell carcinoma 3
❑ Vascular
❑ Dermatofibroma
❑ Squamous cell carcinoma 4

❑ Melanoma

4-20c
❑ Other 5

DISPOSITION ANSWERS
❑ No intervention Answers: 3,4,5
❑ Follow-up
✔ Histopathologic diagnosis
❑ Discussion:
■ The last case represented how bad a pigmented acral melanoma can be. This
invasive melanoma is just as invasive yet it is not heavily pigmented.
■ The milky-red and bluish-white colors are clues that this could be a
melanoma.
DERMOSCOPIC CRITERIA ■ If there’s pink, stop and think!

■ Asymmetry of color and ■ If there’s blue, they might sue!

■ If there’s white, control your fright!


structure
■ Multicomponent global ■ Invasive acral melanomas that lack parallel patterns should be evaluated like
melanomas found on the trunk or extremities.
pattern (1,2,3,4,5)
■ There are well-developed melanoma-specific criteria:
■ Irregular brown globules (box)
■ Asymmetry of color and structure
■ Irregular brown blotches ■ A multicomponent global pattern with five different areas
(yellow arrows) ■ Bony-white color suggesting there is regression
■ Bluish-white color (blue star) ■ The irregular dark blotches and focus of irregular globules could represent
■ Milky-red color (black arrows) dried blood.
■ Regression (black stars) ■ Blood can be associated with acral melanomas.

■ A callus or plantar wart are in the historical, clinical but not dermoscopic
differential diagnosis.

PEARLS
■ Always maintain a high “melanoma index of suspicion” for equivocal cases.
■ Have your dermoscopic instrumentation handy and ready to be used at all
times, so important clues that can only be gained by dermoscopy are not
missed that might help diagnose a difficult case.
■ This is a difficult case, with subtle dermoscopic features that could easily be
missed even by an experienced dermoscopist.
Chapter 4 Palms, Soles, Nails 367

RISK
❑ Low
❑ Intermediate
❑ High

DIAGNOSIS
❑ Nevus
❑ Seborrheic keratosis
❑ Basal cell carcinoma
❑ Vascular

4-21a
❑ Dermatofibroma
❑ Squamous cell carcinoma
❑ Melanoma
❑ Other

DISPOSITION
❑ No intervention
❑ Follow-up
❑ Histopathologic diagnosis
4-21b

CASE 21
HISTORY
This pink macule was found on the left foot of a 65-year-old woman while performing a total body
skin examination. The patient was not aware of its presence.
1. Diffuse pinpoint vessels and pink color could represent Bowen disease.
2. Nonspecific dermatitis and psoriasis are in the clinical and dermoscopic differential diagnosis.
3. Amelanotic melanoma and acute lichen planus-like keratosis are in the differential diagnosis.
4. A solitary pink macule could be melanocytic, nonmelanocytic, benign, malignant, or
inflammatory.
5. Diagnostic sensitivity and specificity are higher when dermoscopy is used to diagnose
pigmented vs pink, feature poor or featureless skin lesions.
368 DERMOSCOPY: AN ILLUSTRATED SELF-ASSESSMENT GUIDE

RISK
❑ Low
✔ Intermediate

❑ High

DIAGNOSIS
❑ Nevus
❑ Seborrheic keratosis
❑ Basal cell carcinoma
❑ Vascular
❑ Dermatofibroma

4-21d
❑ Squamous cell carcinoma
❑ Melanoma

❑ Other
ANSWERS
Answers: 1,2,3,4,5

DISPOSITION Discussion:
■ A solitary pink lesion is nonspecific and could be melanocytic,
❑ No intervention nonmelanocytic, benign, malignant, or inflammatory.
❑ Follow-up ■ To our surprise, the histopathologic report was that of a nonspecific
✔ Histopathologic diagnosis
❑ dermatitis without evidence of a fungus.
■ Dermoscopy is usually not as good a tool to diagnose pink, feature poor, or
featureless vs pigmented lesions that have well-developed criteria (eg,
pigment network, streaks, dots, globules, blotches).
DERMOSCOPIC CRITERIA ■ Telangiectatic vessels can give us clues to help diagnose pink lesions.
However, they are also relatively nonspecific.
■ Pinpoint vessels (boxes) ■ This lesion is filled with pinpoint vessels.
■ Different shades of pink color ■ Pinpoint and glomerular vessels (diffuse or clustered coiled vessels) are a
■ White color (stars) variation of the same pathology. Small vessels running perpendicular to the
skin surface.
■ The differential diagnosis of this lesion includes:
■ Bowen disease ■ Nonspecific dermatitis

■ Amelanotic melanoma ■ Psoriasis

■ Acute lichen planus-like keratosis

■ Clues to narrow down the differential diagnosis can be obtained by knowing


the history of the lesion, age of the patient, distribution of the lesion, and if
it is symptomatic or not.
■ The absence of scale and the plantar location are points against an acute

lichen planus-like keratosis.


■ Even though one can see a solitary lesion of psoriasis, it is not very

common and is a point against this diagnosis.


■ The white color has a differential diagnosis that includes regression or
blanching of the blood supply with pressure when the image was taken.

PEARLS
■ Polarizing instrumentation, minimal pressure, and fluid is the best way to
visualize telangietatic vessels (eg, ultrasound gel).
■ It is not always necessary to make a histopathologic diagnosis of a solitary
pink lesion at the first visit. Quite often they disappear after a few weeks and
surgical interventions can be avoided.
Chapter 4 Palms, Soles, Nails 369

RISK
❑ Low
❑ Intermediate
❑ High

DIAGNOSIS
❑ Nevus
❑ Seborrheic keratosis
❑ Basal cell carcinoma
❑ Vascular

4-22a
❑ Dermatofibroma
❑ Squamous cell carcinoma
❑ Melanoma
❑ Other

DISPOSITION
❑ No intervention
❑ Follow-up
❑ Histopathologic diagnosis
4-22b

CASE 22
HISTORY
An adult male presented with this nonhealing area on the plantar surface of his left foot. There was
no history of trauma.
1. Milky-red color and regression diagnose an amelanotic melanoma.
2. Homogeneous red color and a peripheral white collarette diagnose a pyogenic granuloma.
3. There is an absence of color associated with melanin, which favors a pyogenic granuloma.
4. Homogeneous red color and a peripheral white collarette are seen in 100% of pyogenic
granulomas.
5. Homogeneous red color, peripheral white collarette, white lines within the lesion, and
ulceration can all be found in pyogenic granulomas.
370 DERMOSCOPY: AN ILLUSTRATED SELF-ASSESSMENT GUIDE

RISK
✔ Low

❑ Intermediate
❑ High

DIAGNOSIS
❑ Nevus
❑ Seborrheic keratosis
❑ Basal cell carcinoma
❑ Vascular
❑ Dermatofibroma
❑ Squamous cell carcinoma
❑ Melanoma

4-22c

❑ Other

DISPOSITION ANSWERS
❑ No intervention Answers: 2,3,5
❑ Follow-up
✔ Histopathologic diagnosis
❑ Discussion:
■ Pyogenic granuloma is a misnomer. It is neither pyogenic nor is it a granuloma.
■ Histopathologically, it is a vascular tumor.

■ The etiology is unknown, and it is thought to be associated with trauma.

■ A reactive process “proud flesh.”

DERMOSCOPIC CRITERIA ■ In most cases, it is a clinical diagnosis based on:


■ Recent onset ■ Reddish color
■ Homogeneous red color
■ Areas associated with trauma (eg, fingers, hands, mucosal surfaces)
■ White peripheral collarette
■ Friability and easily bleeding
(stars)
■ The dermoscopic criteria used to diagnose pyogenic granulomas include:
■ Acrosyringia (boxes) ■ Homogeneous reddish color
■ Furrows (arrows) ■ White collarette at the periphery
■ Ridges (black lines) ■ The thickness of the collarette varies from very thin to thick as in this case.
■ Pinpoint vessels (circles) ■ White lines within the lesion ■ Ulceration
■ Thread of material from a sock ■ None of the criteria are seen all the time:
(yellow arrow) ■ Homogeneous red color and peripheral white collarette are the most fre-

quently found criteria.


■ Amelanotic melanoma is the most important tumor that should always be
in the differential diagnosis of a pyogenic granuloma.
■ Criteria in favor of melanoma vs pyogenic granuloma include:
■ Criteria seen in melanocytic lesions (eg, pigment network, dots, globules,

streaks, blotches)
■ Color associated with melanin (eg, black, brown, gray, blue)

■ The differential diagnosis of the pinpoint vessel-like dots includes pinpoint


vessels and dots created by blood.

PEARLS
■ Amelanotic melanoma is the great masquerader, and should always be
included in the differential diagnosis of a “pyogenic granuloma.”
■ No age group is excluded from this general principle.
Chapter 4 Palms, Soles, Nails 371

RISK
❑ Low
❑ Intermediate
❑ High

DIAGNOSIS
❑ Nevus
❑ Seborrheic keratosis
❑ Basal cell carcinoma
❑ Vascular

4-23a
❑ Dermatofibroma
❑ Squamous cell carcinoma
❑ Melanoma
❑ Other

DISPOSITION
❑ No intervention
❑ Follow-up
❑ Histopathologic diagnosis
4-23b

CASE 23
HISTORY
This 7-year-old boy was referred for a biopsy of pigmentation of the nail apparatus to rule out
melanoma. The lesion had been present for 2 years without change.
1. Pigmented nail bands are rare in childhood. The main differential diagnosis includes nail
apparatus melanoma.
2. The longitudinal bands are irregular in thickness with a loss of parallelism, which confirms the
clinical diagnosis of nail apparatus melanoma.
3. The periungual pigmentation seen here is known as Hutchinson sign and is diagnostic of nail
apparatus melanoma.
4. The pigmented bands are homogeneous in color without loss of parallelism, which favors the
diagnosis of a benign melanocytic nevus.
5. Nail apparatus melanoma is rare in children but should always be included in the differential
diagnosis of melanonychia striata.
372 DERMOSCOPY: AN ILLUSTRATED SELF-ASSESSMENT GUIDE

RISK
❑ Low
❑ Intermediate
✔ High

DIAGNOSIS

❑ Nevus
❑ Seborrheic keratosis

4-23c
❑ Basal cell carcinoma
❑ Vascular
❑ Dermatofibroma ANSWERS
❑ Squamous cell carcinoma Answers: 4,5
❑ Melanoma
Discussion:
❑ Other
■ Melanonychia striata (longitudinal pigmented banding) is usually a worri-
some clinical finding in any age group. This history and the clinical and
dermoscopic findings help to determine if the pigmentation is low risk (eg,
nevus, lentigo, ethnic variation, drug induced, post-inflammatory,
DISPOSITION post-traumatic) or high risk (eg, nail apparatus melanoma).
❑ No intervention ■ A history that points in a benign direction includes:
■ Onset during childhood
❑ Follow-up
■ Stable without change over time
✔ Histopathologic diagnosis

■ A family history of similar nail pigmentation
■ Exposure to pigmentogenic medication (eg, acyclovir, minocyline, hydroxyurea)
■ History of trauma or inflammation
■ Low risk dermoscopic features include:
■ A light or dark brown homogeneous background may or may not be present.
DERMOSCOPIC CRITERIA
■ Regular longitudinal line segments with regard to the color, spacing,
■ Homogeneous light and dark thickness, parallelism.
brown background ■ Parallelism refers to uniform unbroken line segments:
■ Uniform thick dark bands ■ Loss of parallelism (broken line segments) created by irregular melanin

(double red arrows) production can be associated with nail apparatus melanoma.
■ Uniform thin dark bands ■ Pigmentation that extends into the periungual tissue is called Hutchinson
sign and can be associated with nail apparatus melanoma, but not always.
(white arrows)
■ Pseudo-Hutchinson sign refers to pigment in the cuticle, and is not
■ Hutchinson sign associated with melanoma.
(yellow arrows) ■ The criteria in this case that favors a benign nevus include:
■ Onset during childhood ■ Stable over time
■ Even though the color of the bands is uniform, the widths are not, which is
a red flag for concern.
■ The periungual pigmentation (Hutchinson sign) is another red flag for concern.
■ There were enough red flags for concern which prompted a biopsy. The
histopathologic diagnosis was that of a severely atypical melanocytic process
and the lesion was excised.

PEARLS
■ Nail apparatus biopsy can be avoided in most children because the incidence
of melanoma is extremely low.
■ A red flag for concern in a child is a rapid change in any type of nail
apparatus pigmentation.
■ Sequential digital clinical and dermoscopic monitoring is the cutting-edge
way to follow patients with nail apparatus pigmentation.
■ Oil or gel must be used to see through the thick, dry nail plate.
Chapter 4 Palms, Soles, Nails 373

RISK
❑ Low
❑ Intermediate
❑ High

DIAGNOSIS
❑ Nevus
❑ Seborrheic keratosis
❑ Basal cell carcinoma
❑ Vascular

4-24a
❑ Dermatofibroma
❑ Squamous cell carcinoma
❑ Melanoma
❑ Other

DISPOSITION
❑ No intervention
❑ Follow-up
❑ Histopathologic diagnosis
4-24b

CASE 24
HISTORY
A 17-year-old girl has had this pigmentation on her fifth finger for 2 years. There is no history of
trauma and the color seems to be changing.
1. Uniform bands diagnose a benign nevus.
2. There is no loss of parallelism.
3. There is no Hutchinson sign.
4. The lines are irregular in thickness and spacing with loss of parallelism which suggests this
could be a melanoma.
5. Compared to the last case, the color of the bands is much lighter with minimal periungual
pigmentation which rules out nail apparatus melanoma.
374 DERMOSCOPY: AN ILLUSTRATED SELF-ASSESSMENT GUIDE

RISK
❑ Low
❑ Intermediate
✔ High

DIAGNOSIS
❑ Nevus
❑ Seborrheic keratosis
❑ Basal cell carcinoma
❑ Vascular
❑ Dermatofibroma
❑ Squamous cell carcinoma

❑ Melanoma
❑ Other

DISPOSITION

4-24c
❑ No intervention
❑ Follow-up
✔ Histopathologic diagnosis

ANSWERS
Answers: 4
DERMOSCOPIC CRITERIA Discussion:
■ Irregular pigmented bands ■ Even though the patient is young, the history and clinical and dermoscopic
(black arrows) features are worrisome.
■ There is a history of change.
■ Loss of parallelism (red arrow)
■ There is no history of a potentially benign cause for the pigmentation
■ Hutchinson sign
(trauma, inflammation).
(yellow arrows)
■ The bands are irregular:
■ Variable thickness

■ Irregular spacing

■ Variegate color (light/dark brown, gray)

■ The intensity of the band color is not a defining point to help differen-

tiate benign from malignant pathology.


■ Adding up the high risk criteria help diagnose Hutchinson sign vs pseudo-
Hutchinson sign with periungual pigmentation.
■ There is no feature of this case other than the patient’s age that favors a
benign lesion.
■ This is a melanoma until proven otherwise!

PEARLS
■ Do not put a histopathologic diagnosis off if melanoma is in the differential
diagnosis of melanonychia striata.
■ Digital monitoring is contraindicated in this case.
■ Be persuasive in your convictions that a patient might have a nail apparatus
melanoma if you have to convince a colleague to perform the biopsy for you.
Chapter 4 Palms, Soles, Nails 375

RISK
❑ Low
❑ Intermediate
❑ High

DIAGNOSIS
❑ Nevus
❑ Seborrheic keratosis
❑ Basal cell carcinoma
❑ Vascular

4-25a
❑ Dermatofibroma
❑ Squamous cell carcinoma
❑ Melanoma
❑ Other

DISPOSITION
❑ No intervention
❑ Follow-up
❑ Histopathologic diagnosis
4-25b

CASE 25
HISTORY
A 36-year-old woman noticed this discoloration on one big toe nail a few months ago. There was
no history of trauma even though she jogs three times a week.
1. Gray color diagnoses post-inflammatory hyperpigmentation secondary to trauma.
2. The presence of blood spots rules out melanoma and confirms that this is secondary to the
patients jogging.
3. There is no Hutchinson sign, which must be present to diagnose melanoma.
4. The bands have different widths with a loss of parallelism, which suggests this could be a nail
apparatus melanoma.
5. The homogeneous light and dark brown bands diagnose a benign nail apparatus nevus.
376 DERMOSCOPY: AN ILLUSTRATED SELF-ASSESSMENT GUIDE

RISK
❑ Low
❑ Intermediate
✔ High

DIAGNOSIS
❑ Nevus
❑ Seborrheic keratosis
❑ Basal cell carcinoma
❑ Vascular
❑ Dermatofibroma
❑ Squamous cell carcinoma

❑ Melanoma

4-25c
❑ Other

DISPOSITION ANSWERS
❑ No intervention Answers: 4
❑ Follow-up
✔ Histopathologic diagnosis
❑ Discussion:
■ All aspects of this case, the history, clinical, and dermoscopic features are
high risk and suggest this could be a nail apparatus melanoma.
■ The clinical appearance is more atypical than what is seen with dermoscopy.
■ There are subtle high risk criteria that could be missed by the novice dermo-
DERMOSCOPIC CRITERIA scopist or experienced dermoscopist who is in a hurry and/or does not make
a focused examination:
■ Irregular thick pigmented
■ Thick pigmented bands with different widths
bands (yellow and white
■ Multiple fine thin bands
double arrows) ■ The loss of parallelism is questionably present
■ Thin fine pigmented bands ■ The presence of irregular dark dots and globules could be confused with
(red arrow) blood spots which can be associated with nail apparatus melanoma.
■ Loss of parallelism ■ Post-inflammatory melanonychia striata created by trauma is usually
(black arrows) symmetrically seen on both feet and has homogeneous gray bands.
■ Irregular dark dots and
globules (circles) PEARLS
■ Do not diagnose low risk melanonychia striata before a focused search is
made for high risk criteria that might not be well-developed.
■ If only one nail is involved, raise a red flag for concern.
Chapter 4 Palms, Soles, Nails 377

RISK
❑ Low
❑ Intermediate
❑ High

DIAGNOSIS
❑ Nevus
❑ Seborrheic keratosis
❑ Basal cell carcinoma
❑ Vascular
4-26a

❑ Dermatofibroma
❑ Squamous cell carcinoma
❑ Melanoma
❑ Other

DISPOSITION
❑ No intervention
❑ Follow-up
❑ Histopathologic diagnosis
4-26b

CASE 26
HISTORY
This solitary pigmented band has been present for 15 years on the index finger of a 47-year-old man.
1. The pressure of the micro-Hutchinson sign diagnoses a slow growing nail apparatus
melanoma.
2. Irregular bands and loss of parallelism diagnose a melanoma.
3. Homogeneous light bands with definite areas of gray color put lentigo simplex in the
differential diagnosis.
4. Gray bands in one or multiple nails can be found in lentigo simplex, ethnic pigmentation, after
trauma, inflammation, or after the intake of pigmentogenic drugs.
5. Gray bands are an indication for a nail apparatus biopsy no matter what the history or clinical
appearance of the nail.
378 DERMOSCOPY: AN ILLUSTRATED SELF-ASSESSMENT GUIDE

RISK
✔ Low

❑ Intermediate
❑ High

DIAGNOSIS
❑ Nevus
❑ Seborrheic keratosis
❑ Basal cell carcinoma
❑ Vascular
❑ Dermatofibroma
❑ Squamous cell carcinoma
❑ Melanoma

4-26c
❑ Other

DISPOSITION ANSWERS
❑ No intervention Answers: 3,4
✔ Follow-up

❑ Histopathologic diagnosis Discussion:
■ The long-term stable history, absence of trauma or inflammation, and gray
color put a lentigo simplex at the top of the differential diagnostic list.
■ The thin lines are not well-developed and the loss of parallelism might be
considered to be present.
DERMOSCOPIC CRITERIA ■ Loss of parallelism is not diagnostic of melanoma.

■ Uniform thick gray band ■ There is no history or other physical findings used to diagnose other
(black double arrows) etiologies of gray melanonychia striata.
■ Ethnic bands are usually polydactylic (multiple nails) with a positive fam-
■ Uniform thin gray bands
(yellow arrows) ily history of similar bands.
■ There is no history of ingestion of pigmentogenic drugs (eg, acyclovir,
■ Darker homogenous gray color
minocyline, hydroxyurea) and only one nail is involved. Drug-induced
(box)
bands are polydactylic.
■ Dry cuticle (red arrows) ■ A negative history of trauma, or inflammation, rules out post-

inflammatory melanonychia striata.


■ Micro-Hutchinson sign is defined as pigmentation in the cuticle that can
only be seen with dermoscopy. It is a nonspecific finding that has to be eval-
uated in the context of the other dermoscopic findings. It is not diagnostic
of nail apparatus melanoma.

PEARLS
■ There are always exceptions to every rule. Gray bands are usually associated
with low risk pathology but theoretically they could be seen in nail apparatus
melanoma.
■ What looks like gray color to one person might be blackish to another. If in
doubt, cut it out!
Chapter 4 Palms, Soles, Nails 379

RISK
❑ Low
❑ Intermediate
❑ High

DIAGNOSIS
❑ Nevus
❑ Seborrheic keratosis
❑ Basal cell carcinoma
❑ Vascular

4-27a
❑ Dermatofibroma
❑ Squamous cell carcinoma
❑ Melanoma
❑ Other

DISPOSITION
❑ No intervention
❑ Follow-up
❑ Histopathologic diagnosis
4-27b

CASE 27
HISTORY
The patient hit his index finger with a hammer but the spot does not seem to be going away.
1. Irregular pigmented bands seen clinically proximal to the main area of discoloration, plus
several melanoma-specific criteria, diagnose nail bed melanoma.
2. Nail bed melanoma exists but is rare compared to nail matrix melanoma.
3. Different shades of purple color plus blood spots diagnose subungual hemorrhage.
4. A round proximal edge and a filamentous distal edge can be seen in some, but not all, cases of
subungual hemorrhage.
5. The reddish longitudinal bands seen clinically represent the normal, nonpathologic vasculature
that can be seen in some but not all nails.
380 DERMOSCOPY: AN ILLUSTRATED SELF-ASSESSMENT GUIDE

RISK
✔ Low

❑ Intermediate
❑ High

DIAGNOSIS
❑ Nevus
❑ Seborrheic keratosis
❑ Basal cell carcinoma
❑ Vascular
❑ Dermatofibroma
❑ Squamous cell carcinoma
❑ Melanoma

4-27c
❑ Other

ANSWERS
DISPOSITION
Answers: 2,3,4,5
❑ No intervention
✔ Follow-up
❑ Discussion:
❑ Histopathologic diagnosis ■ No feature of this case points to the diagnosis of a nail apparatus melanoma.
■ Criteria that diagnose subungual hemorrhage include:
■ History of trauma

■ Distal location

■ Purple color
DERMOSCOPIC CRITERIA ■ Purple round blood spots (blood pebbles)
■ Dried blood (white arrows) ■ An absence of pigmented longitudinal lines

■ Blood spots (blood pebbles) ■ The other colors, excluding white, represent heme breakdown.
(boxes) ■ Trauma created the white color and it should not be confused with the
■ Heme breakdown (stars) bony-white color seen with regression.
■ Longitudinal erythronychia ■ Lateral nail fold and nail bed melanoma exist but is rare compared to nail
(black arrows clinical image) matrix melanoma.
■ The faint reddish longitudinal bands seen clinically represent longitudinal
erythronychia.
■ In a single nail, it could be associated with amelanotic nail apparatus

melanoma.
■ In multiple nails it is a normal variant of the nail apparatus vasculature.

■ The reddish bands should partially or totally blanch easily with pressure

and are usually seen on several but not all fingernails.


■ The incidence of longitudinal erythronychia in the general population is

not known.
■ Melanoma is the most common pathology included in the differential diag-
nosis of a subungual hemorrhage.

PEARLS
■ Longitudinal erythronychia is a common finding that can easily be over
looked.
■ Longitudinal erythronychia should not be confused with pigmented nail
apparatus bands.
Chapter 4 Palms, Soles, Nails 381

RISK
❑ Low
❑ Intermediate
❑ High

DIAGNOSIS
❑ Nevus
❑ Seborrheic keratosis

4-28a1
❑ Basal cell carcinoma
❑ Vascular
❑ Dermatofibroma
❑ Squamous cell carcinoma
❑ Melanoma
❑ Other

DISPOSITION
❑ No intervention
❑ Follow-up
❑ Histopathologic diagnosis
4-28a2
4-28b

CASE 28
HISTORY
There is a questionable history of trauma in this long-standing blue discoloration in a 35-year-old man.
1. Purplish color and a distal filamentous border diagnose subungual hemorrhage.
2. Nail dystrophy and blood spots are seen clinically, which confirm that this is subungual
hemorrhage.
3. Homogeneous blue color suggests that this could be a blue nevus.
4. Bluish-white color, polymorphous vessels, and blood spots put a malignant blue nevus in the
differential diagnosis.
5. A long-standing history eliminates the need to make a histopathologic diagnosis.
382 DERMOSCOPY: AN ILLUSTRATED SELF-ASSESSMENT GUIDE

RISK
❑ Low
✔ Intermediate

❑ High

DIAGNOSIS

❑ Nevus
❑ Seborrheic keratosis
❑ Basal cell carcinoma
❑ Vascular
❑ Dermatofibroma
❑ Squamous cell carcinoma
❑ Melanoma

4-28c
❑ Other

DISPOSITION
❑ No intervention ANSWERS
❑ Follow-up
Answers: 3,4
✔ Histopathologic diagnosis

Discussion:
■ This is a difficult case with a rare, unstudied tumor that brings the concept
of differential diagnosis into play.
■ The bluish color seen clinically with a long-standing stable history rules out
DERMOSCOPIC CRITERIA subungual hemorrhage.
■ Bluish-white color (stars) ■ A banal dermoscopic presentation of a nail apparatus blue nevus would
■ Irregular hairpin vessels include homogeneous blue color similar to a blue nevus seen on another
(dark arrows) part of the body that fills the lunula.
■ Polymorphous vessels (box) ■ Atypical features include:
■ Nonhomogeneous bluish-white color filling the lunula
■ Subungual hemorrhage
■ Irregular hairpin vessels
(red arrows)
■ Polymorphous vessels (eg, pinpoint and irregular linear vessels)

■ Nail plate dystrophy (black arrows in the clinical images)

■ Foci of subungual hemorrhages (boxes in the clinical image)

■ The irregular hairpin vessels could be confused with the distal filamentous
component that can be seen in subungual hemorrhage, which would be a
solid purple color.
■ The differential diagnosis includes a blue nevus vs malignant blue nevus.
■ The stable history favors a benign blue nevus.

■ The atypical dermoscopic features favor a malignant blue nevus.

■ Histopathology confirmed the benign nature of this lesion. Unusual vascula-


ture, mild inflammation, and extravasated red blood cells help
to explain what is seen clinically and dermoscopically.

PEARLS
■ Sequential digital monitoring was used to confirm the stable nature of this lesion.
■ One cannot see what one does not know. Now you know about this rare
tumor that you might encounter someday.
Chapter 4 Palms, Soles, Nails 383

RISK
❑ Low
❑ Intermediate
❑ High

4-29a
DIAGNOSIS
❑ Nevus
❑ Seborrheic keratosis
❑ Basal cell carcinoma
❑ Vascular
❑ Dermatofibroma
❑ Squamous cell carcinoma
❑ Melanoma
❑ Other

DISPOSITION
❑ No intervention
❑ Follow-up
4-29b

❑ Histopathologic diagnosis

CASE 29
HISTORY
The physician of a 62-year-old woman noticed this nail discoloration and referred her for dermoscopic
evaluation to rule out melanoma.
1. Bluish-white color and irregular streaks diagnose nail apparatus nodular melanoma.
2. Cuticular mega-capillary loops confirm the diagnosis of melanoma.
3. Bluish-white color and irregular streaks diagnose a malignant blue nevus.
4. Bluish-white homogeneous color, well-demarcated lateral and proximal borders, and a
purplish filamentous distal border diagnose a subungual hemorrhage.
5. Bluish-white color rules out a subungual hemorrhage.
384 DERMOSCOPY: AN ILLUSTRATED SELF-ASSESSMENT GUIDE

RISK
✔ Low

❑ Intermediate
❑ High

DIAGNOSIS
❑ Nevus
❑ Seborrheic keratosis
❑ Basal cell carcinoma
❑ Vascular
❑ Dermatofibroma
❑ Squamous cell carcinoma
❑ Melanoma

4-29c

❑ Other

DISPOSITION ANSWERS
❑ No intervention Answers: 4
✔ Follow-up

❑ Histopathologic diagnosis Discussion:
■ This is a classic example of a subungual hemorrhage:
■ Homogeneous bluish-white and purple color

■ Well-demarcated proximal and lateral borders

■ Purple solid distal filamentous border


DERMOSCOPIC CRITERIA ■ The filamentous lines are solid and have no resemblance to large irregular
■ Homogeneous bluish-white hairpin vessels seen in the last case.
color (stars) ■ There are normal small hairpin-shaped cuticular capillary loops, which
■ Well-demarcated proximal and should be compared to the irregular mega-capillary loops (Figure 1-34 in
lateral borders (yellow arrows) Chapter 1) that can be seen in collagen vascular diseases.
■ Cuticular capillary loops are not a criterion used to diagnose nail
■ Purple filamentous distal
apparatus melanoma.
border (black arrows)
■ Purple is not the only color that can be seen in subungual hemorrhage.
■ Cuticular hairpin-shaped Shades of black, blue, brown, and yellow can also be seen and depend on the
capillary loops (boxes) state of heme breakdown

PEARLS
■ Even though the solid purplish filamentous distal border is not seen all the
time, it is a major clue to diagnose subungual hemorrhage and should
specifically be looked for with any nail pigmentation.
■ Be cutting-edge and take digital clinical and dermoscopic images of
subungual hemorrhage to ensure that you can document the expected distal
migration of the discoloration.
Chapter 4 Palms, Soles, Nails 385

RISK
❑ Low
❑ Intermediate
❑ High

DIAGNOSIS
❑ Nevus
❑ Seborrheic keratosis
❑ Basal cell carcinoma

4-30a

Vascular
❑ Dermatofibroma
❑ Squamous cell carcinoma
❑ Melanoma
❑ Other

DISPOSITION
❑ No intervention
❑ Follow-up
❑ Histopathologic diagnosis
4-30b

CASE 30
HISTORY
A podiatrist noticed a progressive darkening of the first toe nail on the right foot of a 58-year-old
woman and referred her to rule out melanoma.
1. A normal nail plate and homogeneous brown color diagnose a nail matrix nevus.
2. Purplish color and partial nail plate destruction diagnose a subungual hemorrhage secondary
to trauma.
3. Homogeneous brown color and irregular longitudinal bands suggest this could be a nail
apparatus melanoma.
4. Loss of parallelism is another point that favors a diagnosis of nail apparatus melanoma.
5. The absence of the Hutchinson sign does not rule out melanoma.
386 DERMOSCOPY: AN ILLUSTRATED SELF-ASSESSMENT GUIDE

RISK
❑ Low
❑ Intermediate
✔ High

DIAGNOSIS
❑ Nevus
❑ Seborrheic keratosis
❑ Basal cell carcinoma
❑ Vascular
❑ Dermatofibroma
❑ Squamous cell carcinoma

❑ Melanoma

4-30c
❑ Other

DISPOSITION ANSWERS
❑ No intervention Answers: 3,4,5
❑ Follow-up
✔ Histopathologic diagnosis Discussion:

■ There are no criteria to suggest that this could be a subungual hemorrhage.
■ All of the features suggest that this is a melanoma:
■ No history of trauma or inflammation

■ Progressive darkening

DERMOSCOPIC CRITERIA ■ Partial nail plate destruction

■ Homogeneous dark color


■ Diffuse dark homogeneous
■ Bluish-white color
color (stars)
■ Irregular bands
■ Irregular longitudinal bands ■ Loss of parallelism
(black arrows) ■ The absence of the Hutchinson sign does not rule out melanoma.
■ Loss of parallelism ■ Amelanotic nail apparatus melanoma, the antithesis of this case, is not rare
(yellow arrows) and could be devoid of any pigmentation to suggest the correct diagnosis.
■ Nail plate destruction ■ Nail plate destruction is an important clue to suggest this diagnosis in a

(white arrows) nonpigmented or pink nail apparatus tumor.

PEARLS
■ You will not have the opportunity to diagnose a case like this if you do not
perform a total body skin examination which includes the feet.
■ Have nail polish remover handy so that you can check your female patient’s
nails that use nail polish.
Chapter 4 Palms, Soles, Nails 387

RISK
❑ Low
❑ Intermediate
❑ High

DIAGNOSIS
❑ Nevus
❑ Seborrheic keratosis
❑ Basal cell carcinoma
❑ Vascular

4-31a
❑ Dermatofibroma
❑ Squamous cell carcinoma
❑ Melanoma
❑ Other

DISPOSITION
❑ No intervention
❑ Follow-up
❑ Histopathologic diagnosis
4-31b

CASE 31
HISTORY
A dermatologist was treating this finger nail as a fungus for 2 years without improvement.
1. Bluish-green color seen clinically and nail plate destruction diagnose a chronic pseudomonas
infection.
2. Nail plate destruction and different colors seen with dermoscopy suggest that this could be a
melanocytic lesion.
3. Gray globules identify a melanocytic lesion.
4. There is bony-white color that could represent scarring or regression.
5. The dermoscopic features are nonspecific and melanoma should be included in the differential
diagnosis.
388 DERMOSCOPY: AN ILLUSTRATED SELF-ASSESSMENT GUIDE

RISK
❑ Low
❑ Intermediate
✔ High

DIAGNOSIS
❑ Nevus
❑ Seborrheic keratosis
❑ Basal cell carcinoma
❑ Vascular
❑ Dermatofibroma
❑ Squamous cell carcinoma

❑ Melanoma

4-31c
❑ Other

DISPOSITION ANSWERS
❑ No intervention Answers: 2,4,5
❑ Follow-up
✔ Histopathologic diagnosis
❑ Discussion:
■ For the astute clinician, the brown color seen clinically and dermoscopically
is a clue that this might be a melanoma.
■ Brown not gray dots and/or globules (peppering) are one criterion used

to diagnose a melanocytic lesion.


DERMOSCOPIC CRITERIA ■ Tinea is clearly in the differential diagnosis and the unresponsiveness to
■ Light and dark brown color treatment does not rule tinea out.
(arrows) ■ The dermoscopic criteria are nonspecific yet there are clues that point in the
■ Bluish-gray color (black boxes) direction of the correct diagnosis:
■ Foci of brown color
■ Peppering (yellow boxes)
■ Peppering (gray dots)
■ Scarring (stars)
■ Foci of bluish color

■ The bony-white color represents the scarring typically found in desmoplas-


tic melanoma, which could be missed by the most experienced dermo-
scopist.
■ Blanching of the nail apparatus vasculature is in the differential diagnosis

of the white color.


■ This is a desmoplastic melanoma which is usually a histopathologic surprise
diagnosis for most clinicians. This should not prevent it from being in the
differential diagnosis of a scar-like lesion anywhere on the body.

PEARLS
■ No one can win them all, especially when it comes to diagnosing melanoma.
■ Dermoscopy, knowledge, experience, and a high index of suspicion increase
the odds that you will not get burned!
Chapter 5

Genitalia
General Instructions: You will find a list of True/False statements following each case
history. Select any statements, which you believe to be true. There may be one, more
than one or no true statements for any given case. Choose the correct risk, diagnosis
and disposition for each case. Then, turn the page to find a detailed discussion and
pearls for each case.
Chapter 5 Genitalia 391

RISK
❑ Low
❑ Intermediate
❑ High

DIAGNOSIS
❑ Nevus
❑ Seborrheic keratosis
❑ Basal cell carcinoma
❑ Vascular

5-1a
❑ Dermatofibroma
❑ Squamous cell carcinoma
❑ Melanoma
❑ Other

DISPOSITION
❑ No intervention
❑ Follow-up
❑ Histopathologic diagnosis
5-1b

CASE 1
HISTORY
A 53-year-old woman was referred by her gynecologist who thought she had a melanoma.
1. Pigment network puts a melanocytic lesion in the differential diagnosis.
2. Pigment network puts a lentigo in the differential diagnosis.
3. The pigment network is prominent yet falls within the low risk category.
4. The pigment network is highly irregular, which favors the diagnosis of a vulvar melanoma.
5. Melanoma specific-criteria are absent, which favors the diagnosis of a benign vulvar lentigo.
392 DERMOSCOPY: AN ILLUSTRATED SELF-ASSESSMENT GUIDE

RISK
❑ Low
✔ Intermediate

❑ High

DIAGNOSIS
❑ Nevus
❑ Seborrheic keratosis
❑ Basal cell carcinoma
❑ Vascular
❑ Dermatofibroma
❑ Squamous cell carcinoma
❑ Melanoma

5-1c
Other

DISPOSITION ANSWERS
❑ No intervention Answers: 1,2,3,5
❑ Follow-up
✔ Histopathologic diagnosis
❑ Discussion:
■ Clinically, this genital lentigo is more worrisome than the dermoscopic picture.
■ A melanocytic lesion and lentigo both can have a pigment network.
■ The pigment network is thick with sharp border demarcation, reminiscent
of an ink-spot lentigo.
DERMOSCOPIC CRITERIA ■ An ink-spot lentigo is usually found on sun-exposed areas and does not

■ Regular pigment network have this clinical appearance. Typically, they are flat black macules as
(white boxes) opposed to this lesion that appears raised.
■ Homogeneous black color ■ The jet black color is unusual and a red flag for concern. Genital lentigines
(stars) are usually a light or dark brown color, but can be difficult to differentiate
clinically and/or dermoscopically from melanoma.
■ Irregular black globules
■ Genital melanoma is rare (4% of all melanomas) and presents with
(yellow box)
melanoma-specific criteria similar to those seen on the trunk and extremi-
■ Irregular black dots (circles) ties (eg, asymmetry of color and structure, multicomponent global pattern,
irregular pigment network, irregular dots, globules, irregular blotches, irreg-
ular streaks, multiple colors, regression, polymorphous vessels).
■ The focus of irregular globules represents fragments of the pigment
network.
■ The irregular black dots have no diagnostic significance.

PEARLS
■ Dermoscopic evaluation can help to avoid potentially aggressive, often muti-
lating, surgery to rule out melanoma in this very sensitive area.
■ In this case, sequential digital clinical and dermoscopic monitoring rather
than histopathologic evaluation can be used by the more experienced and
confident dermoscopist.
■ The presence of a chaperone should be the standard of care when examining
genital lesions.
Chapter 5 Genitalia 393

RISK
❑ Low
❑ Intermediate
❑ High

DIAGNOSIS
❑ Nevus
❑ Seborrheic keratosis
❑ Basal cell carcinoma
❑ Vascular

5-2a
❑ Dermatofibroma
❑ Squamous cell carcinoma
❑ Melanoma
❑ Other

DISPOSITION
❑ No intervention
❑ Follow-up
❑ Histopathologic diagnosis
5-2b

(Reprinted, with permission, from Stolz W, Braun-Falco O, Bilek P et al.


(2002) Color Atlas of Dermatoscopy. Second Edition. Blackwell
Publishing, Oxford.)

CASE 2
HISTORY
A 34-year-old woman has this dark brown pigmentation on her vulva for many years.
1. A multicomponent global pattern, irregular dots, globules, streaks, and irregular black blotch
diagnose a melanoma.
2. This is a gray-zone lesion with a benign globular and benign parallel pattern and an irregular
black blotch.
3. If it were not for the irregular black blotch, this would be one of the benign patterns that can
be seen in a genital lentigo.
4. The parallel line segments are similar to the parallel patterns that can be seen on the palms
and soles.
5. Combinations of benign patterns can be seen in the genital area.
394 DERMOSCOPY: AN ILLUSTRATED SELF-ASSESSMENT GUIDE

RISK
❑ Low
✔ Intermediate

❑ High
1
2

DIAGNOSIS
❑ Nevus
❑ Seborrheic keratosis
❑ Basal cell carcinoma 4
❑ Vascular 3

❑ Dermatofibroma
❑ Squamous cell carcinoma
❑ Melanoma

❑ Other

5-2c
DISPOSITION ANSWERS
❑ No intervention Answers: 2,3,4,5
❑ Follow-up
✔ Histopathologic diagnosis

Discussion:
■ The history is low risk, whereas clinically and dermoscopically there are
worrisome features.
■ There is a multicomponent global pattern (melanoma-specific criterion)
with four different areas that put melanoma in the differential diagnosis:
DERMOSCOPIC CRITERIA ■ Globular pattern made up of regular dots and globules (zone 1).

■ Parallel pattern made up of regular thick parallel line segments (zone 2).
■ Multicomponent global
■ Irregular black blotch (zone 3).
pattern (1,2,3,4)
■ Homogeneous gray color (zone 4).
■ Parallel pattern (black arrows)
■ Other than the prominent irregular black blotch, this lesion has a combina-
■ Globular pattern (black circles) tion of benign patterns that can be seen in a lentigo.
■ Irregular black blotch ■ A multicomponent global pattern is not diagnostic for a melanoma.
(white arrows) ■ More than one benign pattern can be seen in a genital lentigo.
■ Homogeneous gray color (box) ■ The focus of homogeneous gray color represents melanophages that can
■ White dots (white circles) result from inflation or trauma.
■ A specific name for the white dots is not known. They have no diagnostic
significance.
■ The differential diagnosis includes a lentigo, melanocytic nevus, combined
nevus (blue nevus underlying junctional/compound nevus), and collision
tumor (eg, lentigo and nevus, lentigo and melanoma).

PEARLS
■ If it is necessary to make a histopathologic diagnosis, one can use the most
atypical dermoscopic area for an incisional biopsy and/or biopsies to avoid
more aggressive surgical techniques.
■ To rule out a collision tumor, more than one incisional biopsy should
be performed.
■ Alert your pathologist to be on the look-out for a possible collision tumor
when one is in the differential diagnosis.
Chapter 5 Genitalia 395

RISK
❑ Low
❑ Intermediate
❑ High

DIAGNOSIS
5-3a

❑ Nevus
❑ Seborrheic keratosis
❑ Basal cell carcinoma
❑ Vascular
❑ Dermatofibroma
❑ Squamous cell carcinoma

5-3a
❑ Melanoma
❑ Other

DISPOSITION
❑ No intervention
❑ Follow-up
❑ Histopathologic diagnosis
5-3b

CASE 3
HISTORY
The gynecologist of a 62-year-old woman discovered this lesion and referred the patient for a
biopsy.
1. Irregular dark blotches, irregular dots and globules, plus bluish-white color diagnose a mucosal
melanoma.
2. This is a melanocytic nevus with a globular pattern.
3. Ring-like structures put a lentigo in the differential diagnosis.
4. Irregular dark blotches and the clinical appearance of the lesion trump the low risk, ring-like
pattern, and a histopathologic diagnosis is indicated.
5. The completely banal dermoscopic picture trumps the worrisome clinical appearance and a
histopathologic diagnosis is not indicated.
396 DERMOSCOPY: AN ILLUSTRATED SELF-ASSESSMENT GUIDE

RISK
❑ Low
✔ Intermediate

❑ High

DIAGNOSIS
❑ Nevus
❑ Seborrheic keratosis
❑ Basal cell carcinoma
❑ Vascular
❑ Dermatofibroma
❑ Squamous cell carcinoma
❑ Melanoma

5-3c
Other

DISPOSITION ANSWERS
❑ No intervention Answers: 3, 4
❑ Follow-up
✔ Histopathologic diagnosis
❑ Discussion:
■ This genital lentigo another gray-zone lesion that is not perfectly banal nor
with sufficient high risk criteria to diagnose a melanoma.
■ Ring-like structures and not dots and globules are identifiable:
■ Multiple round to oval white or tan structures with central clearing.
DERMOSCOPIC CRITERIA ■ Well-demarcated hyperpigmented borders.

■ Ring-like pattern (boxes) ■ They can be diffuse, clustered, and appear grape-like.

■ Irregular dark blotches ■ A ring-like pattern can be seen in genital lentigo, bowenoid papulosis, der-
(arrows) matofibromas, seborrheic keratosis, and lentigo maligna on the face (circle
within a circle).
■ The irregular dark blotches are a red flag for concern and together with the
irregular clinical appearance is an indication to make a histopathologic
diagnosis.

PEARLS
■ Glabrous skin (eg, lips, palms, soles, mucocutaneous junction, and mucosa)
can have the same dermoscopic patterns that are considered site-specific.
■ The dermoscopic features of the genitalia have not been well studied. If
there are any clues (eg, history, clinical appearance, melanoma-specific crite-
ria) that you might be dealing with a high risk lesion, it is better to err on
the side of caution and make a histopathologic diagnosis.
Chapter 5 Genitalia 397

RISK
❑ Low
❑ Intermediate
❑ High

DIAGNOSIS
❑ Nevus
❑ Seborrheic keratosis
❑ Basal cell carcinoma
❑ Vascular
5-4a

❑ Dermatofibroma
❑ Squamous cell carcinoma
❑ Melanoma
❑ Other

DISPOSITION
❑ No intervention
❑ Follow-up
❑ Histopathologic diagnosis
5-4b

CASE 4
HISTORY
A 21-year-old woman discovered this lesion and asked to have it checked during a visit for an
unrelated skin complaint.
1. Ring-like, fish scale-like, and parallel patterns diagnose a lentigo.
2. There is no pattern characterized as fish scale-like.
3. There are no high risk dermoscopic criteria to suggest this is a melanoma.
4. The fish scale-like pattern is a variant of the ring pattern and is thought to be created by
pressure from instrumentation.
5. In this case, sequential digital clinical and dermoscopic monitoring can be used instead of
making a histopathologic diagnosis.
398 DERMOSCOPY: AN ILLUSTRATED SELF-ASSESSMENT GUIDE

RISK
✔ Low

❑ Intermediate
❑ High

DIAGNOSIS
❑ Nevus
❑ Seborrheic keratosis
❑ Basal cell carcinoma
❑ Vascular
❑ Dermatofibroma
❑ Squamous cell carcinoma
❑ Melanoma

❑ Other

5-4c
DISPOSITION ANSWERS
❑ No intervention Answers: 1,3,4,5
✔ Follow-up

❑ Histopathologic diagnosis Discussion:
■ Clinically, but not dermoscopically, this genital lentigo is worrisome charac-
terized by asymmetry, irregular borders, and different shades of brown color.
■ Dermoscopically, the picture is low risk with variants of the ring, fish
scale-like, and parallel patterns.
DERMOSCOPIC CRITERIA ■ Only a few of the ring structures have hyperpigmented borders.

■ Ring-like pattern ■ When there are only two parallel line segments they are referred to as

(white boxes) “train tracks.”


■ Fish scale-like pattern ■ The fish-scale pattern is thought to be a variant of the ring-like pattern
(black boxes) created by pressure from instrumentation and is characterized by multiple
■ Parallel pattern (arrows) curves or semicircular U- or V-shapes mimicking fish scales.
■ The hyphal pattern not seen here, is made up of short-branched line

segments that look like fungal hyphae and is considered a variant of


the ring-like and fish-scale pattern.
■ There are no dermoscopic criteria to suggest that this is a melanoma.

PEARLS
■ Dermoscopy is an invaluable tool to help diagnose genital pigmentation that
may or may not look high risk clinically.
■ It is essential to learn all of the site-specific criteria and patterns to avoid
misdiagnosis.
Chapter 5 Genitalia 399

RISK
❑ Low
❑ Intermediate
❑ High

DIAGNOSIS
❑ Nevus
❑ Seborrheic keratosis
❑ Basal cell carcinoma
❑ Vascular

5-5a
❑ Dermatofibroma
❑ Squamous cell carcinoma
❑ Melanoma
❑ Other

DISPOSITION
❑ No intervention
❑ Follow-up
❑ Histopathologic diagnosis
5-5b

CASE 5
HISTORY
A 50-year-old woman was at the gynecologist who found this pigmented lesion.
1. A multicomponent global pattern and bluish-white and gray colors put melanoma in the
differential diagnosis.
2. The bluish-white and gray colors could represent discoloration secondary to trauma or
inflammation.
3. Foci of ring and fish scale-like structures and parallel line segments put a lentigo in the
differential diagnosis.
4. This is a worrisome lesion clinically and dermoscopically. A wide excision without a
preliminary incisional biopsy is indicated because it must be a melanoma.
5. To avoid aggressive and unnecessary surgery, because this could be a low risk lesion, multiple
incisional biopsies should be considered in order to make an accurate diagnosis.
400 DERMOSCOPY: AN ILLUSTRATED SELF-ASSESSMENT GUIDE

RISK 1

❑ Low
❑ Intermediate
✔ High

3

DIAGNOSIS 4 2
❑ Nevus
❑ Seborrheic keratosis
❑ Basal cell carcinoma
❑ Vascular
❑ Dermatofibroma 5
❑ Squamous cell carcinoma
❑ Melanoma

❑ Other

5-5c
DISPOSITION
❑ No intervention ANSWERS
❑ Follow-up Answers: 1,2,3,5
✔ Histopathologic diagnosis

Discussion:
■ This is a worrisome dermoscopic picture because there are several
melanoma-specific criteria:
■ Asymmetry of color and structure
DERMOSCOPIC CRITERIA ■ Multicomponent global pattern

■ Bluish-white color
■ Asymmetry of color and
■ Gray color
structure
■ There are foci of poorly-developed low risk patterns:
■ Multicomponent global
■ Ring-like (individual rings)
pattern (1,2,3,4,5)
■ Parallel
■ Bluish-white and gray colors ■ The fish scale-like pattern is well developed with inverted v-shaped line
(arrows) segments.
■ Ring-like structures ■ Bluish-white color should always be a red flag for concern that a lesion
(black boxes) might be a melanoma.
■ Parallel structures (yellow box) ■ Gray color indicates the presence of melanophages created by inflammation
■ Fish scale-like pattern or trauma both commonly seen in the genital area.
(white box) ■ Even with the identification of low risk criteria, the overall picture is high
■ Homogeneous brownish-gray risk and a histopathologic diagnosis is indicated posthaste.
color (stars)
PEARLS
■ The diagnosis of a lentigo was not a good dermoscopic–pathologic correlation.
Make sure that your pathologist has experience diagnosing pigmented geni-
tal lesions.
■ One should not be politically correct when a patient’s well-being is on the
line. Asking for second histopathologic opinion from a more experienced
dermatopathologist should be your standard of care in certain situations.
■ Multiple incisional biopsies are indicated at times with multicomponent
lesions so that the correct diagnosis is not missed.
Chapter 5 Genitalia 401

RISK
❑ Low
❑ Intermediate
❑ High

DIAGNOSIS
❑ Nevus
❑ Seborrheic keratosis
❑ Basal cell carcinoma
❑ Vascular
5-6a

❑ Dermatofibroma
❑ Squamous cell carcinoma
❑ Melanoma
❑ Other

DISPOSITION
❑ No intervention
❑ Follow-up
❑ Histopathologic diagnosis
5-6b

CASE 6
HISTORY
A 45-year-old woman with biopsy-proven genital lichen sclerosus et atrophicus was seen at her
routine 6-month follow-up examination. Historically, the area was bony-white. She was not aware
of the new dark discoloration.
1. Milky-red areas and bluish-white color diagnose a melanoma arising in chronic lichen
sclerosus et atrophicus.
2. Polymorphous vessels confirm this to be a melanoma.
3. This represents a partially pigmented and amelanotic genital melanoma.
4. Purple color suggests that the new discoloration could simply represent hemorrhage.
5. A wide excision is indicated. There is no need for a preliminary incisional biopsy.
402 DERMOSCOPY: AN ILLUSTRATED SELF-ASSESSMENT GUIDE

RISK
✔ Low

❑ Intermediate
❑ High

DIAGNOSIS
❑ Nevus
❑ Seborrheic keratosis
❑ Basal cell carcinoma
❑ Vascular
❑ Dermatofibroma
❑ Squamous cell carcinoma
❑ Melanoma

5-6c
Other

ANSWERS
DISPOSITION
Answers: 4
❑ No intervention
❑ Follow-up Discussion:
✔ Histopathologic diagnosis
❑ ■ The purple color is a clue that this new discoloration might be secondary to
hemorrhage, and it is.
■ One should still think in terms of dermoscopic differential diagnosis.
■ The bluish-white color could represent invasive melanoma or different

shades of hemorrhage.
DERMOSCOPIC CRITERIA ■ The homogeneous purplish-red color could represent blood, the milky-

■ Hemorrhage (stars) red areas of a melanoma, or the amelanotic component of a partially


■ Blood pebbles (box) pigmented melanoma.
■ Lichen sclerosus et atrophicus ■ The bony-white color could represent regression in a melanoma or lichen

(arrows) sclerosus et atrophicus.


■ There are no polymorphous vessels (eg, pinpoint, irregular linear,
corkscrew).
■ Squamous cell carcinoma and not melanoma has been reported to develop
in chronic lichen sclerosus et atrophicus. This is not the dermoscopic picture
of squamous cell carcinoma:
■ Glomerular or irregular hairpin-shaped vessels

■ Since the differential diagnosis includes melanoma, a histopathologic


diagnosis is indicated.

PEARLS
■ At the time the discoloration was first discovered, since hemorrhage was at
the top of the differential diagnosis, short-term (q 3 months) digital clinical
and dermoscopic monitoring would be an alternative to a surgical interven-
tion. One should see the expected changes in heme over time, which would
confirm the diagnosis.
■ Close observation is not always a bad idea in certain situations.
Chapter 5 Genitalia 403

RISK
❑ Low
❑ Intermediate
❑ High

DIAGNOSIS
❑ Nevus
❑ Seborrheic keratosis
❑ Basal cell carcinoma
❑ Vascular
5-7a

❑ Dermatofibroma
❑ Squamous cell carcinoma
❑ Melanoma
❑ Other

DISPOSITION
❑ No intervention
❑ Follow-up
❑ Histopathologic diagnosis
5-7b

CASE 7
HISTORY
According to the patient, this lesion literally came out of nowhere within a 4-month period. There
was no history of trauma or any pre-existing genital problem.
1. Clear cut polymorphous vessels suggest this could be a melanoma.
2. Different shades of purple and bluish-white color diagnose hemorrhage.
3. Bluish-white color is a red flag for concern that this could be a melanoma.
4. There is sufficient clinical and dermoscopic evidence to justify a wide excision to rule out
invasive melanoma.
5. One does not need dermoscopy to realize that this could be a bad melanoma, but it helps to
confirm the clinical impression and guide one’s surgical approach.
404 DERMOSCOPY: AN ILLUSTRATED SELF-ASSESSMENT GUIDE

RISK
❑ Low
❑ Intermediate
✔ High

DIAGNOSIS
❑ Nevus
❑ Seborrheic keratosis
❑ Basal cell carcinoma
❑ Vascular
❑ Dermatofibroma
❑ Squamous cell carcinoma

❑ Melanoma
❑ Other

5-7c
DISPOSITION ANSWERS
❑ No intervention
Answers: 1,3,4,5
❑ Follow-up
✔ Histopathologic diagnosis
❑ Discussion:
■ The presence of polymorphous vessels (eg, pinpoint, irregular linear, cork
screw) rules out hemorrhage.
■ Polymorphous vessels may or may not be sharply in focus and easy to

identify.
DERMOSCOPIC CRITERIA ■ This lesion has a reddish amelanotic and bluish-white pigmented compo-
■ Polymorphous vessels nent. Both colors are associated with invasive melanoma.
■ Pinpoint (white boxes) ■ There are no differential diagnostic possibilities. A wide excision of the
■ Irregular linear and entire lesion is indicated without the need for preliminary incisional biopsy.
corkscrew (red box) ■ This is the end of the line as far as how bad a genital/mucosal melanoma can
■ Combinations of irregular be. The goal is to diagnose a melanoma before it gets this bad or better yet, a
shapes (yellow boxes) benign premalignant lesion.
■ Bluish-white color (white stars)
■ Amelanotic component PEARLS
(red stars) ■ One can see a progression of severity in the last three cases characterized by
bluish-white color.
■ Bluish-white color is always a red flag for concern, and a careful focused
dermoscopic evaluation is indicated.
Chapter 5 Genitalia 405

RISK
❑ Low
❑ Intermediate
❑ High

DIAGNOSIS
❑ Nevus
❑ Seborrheic keratosis
❑ Basal cell carcinoma
❑ Vascular

5-8a
❑ Dermatofibroma
❑ Squamous cell carcinoma
❑ Melanoma
❑ Other

DISPOSITION
❑ No intervention
❑ Follow-up
❑ Histopathologic diagnosis
5-8b

CASE 8
HISTORY
A 35-year-old man has this pigmentation for several years without change.
1. Brown globules identify a melanocytic lesion.
2. Asymmetry of color and structure, irregular brown globules, and irregular streaks diagnose a
dysplastic nevus.
3. Multifocal pigmentation seen clinically plus globular and parallel patterns diagnose benign
penile melanosis.
4. The globular pattern is only found in melanocytic nevi and never in the genital area.
5. The extensive nature of the pigmentation and irregular fingerprint pattern diagnose a
melanoma.
406 DERMOSCOPY: AN ILLUSTRATED SELF-ASSESSMENT GUIDE

RISK
✔ Low

❑ Intermediate
❑ High

DIAGNOSIS
❑ Nevus
❑ Seborrheic keratosis
❑ Basal cell carcinoma
❑ Vascular
❑ Dermatofibroma
❑ Squamous cell carcinoma
❑ Melanoma

❑ Other

5-8c
DISPOSITION ANSWERS
❑ No intervention Answers: 3
✔ Follow-up

❑ Histopathologic diagnosis Discussion:
■ There are no clinical or dermoscopic features to suggest this could be a
melanoma.
■ This is a classic clinical and dermoscopic example of penile melanosis
(lentigines).
DERMOSCOPIC CRITERIA ■ Extensive genital melanosis can have a single or multiple dermoscopic
■ Linear globular pattern patterns. In this case there is the globular and parallel pattern.
(yellow boxes) ■ The globular pattern is made up of aggregated round structures that have
■ Nonlinear globular pattern a linear and nonlinear distribution.
■ There are only a few foci of parallel lines, which can be similar to the
(black boxes)
parallel patterns seen on the palms and soles.
■ Parallel pattern (arrows)
■ More extensive parallel lines can resemble the fingerprint pattern seen in
lentigines on sun-exposed areas.

PEARLS
■ In order not to miss high risk dermoscopic criteria (eg, irregular dark
blotches, bluish-white color, polymorphous vessels) one should examine
the entire lesion.
■ There is no association between melanosis and melanoma. The only prob-
lem for the novice dermoscopist is to not know the important patterns and
criteria, and make a misdiagnosis that could result in inappropriate aggres-
sive surgery or no surgery at all.
Chapter 5 Genitalia 407

RISK
❑ Low
❑ Intermediate
❑ High

DIAGNOSIS
❑ Nevus
❑ Seborrheic keratosis
❑ Basal cell carcinoma
❑ Vascular

5-9a
❑ Dermatofibroma
❑ Squamous cell carcinoma
❑ Melanoma
❑ Other

DISPOSITION
❑ No intervention
❑ Follow-up
❑ Histopathologic diagnosis
5-9b

CASE 9
HISTORY
A 46-year-old man, with a history of lichen sclerosus et atrophicus and condyloma, has
pigmentation on his penis for several years.
1. The pigmentation is post-inflammatory, secondary to the destructive treatment of his
condyloma.
2. The scarring seen clinically, regression seen dermoscopically, and irregular globules diagnose a
penile melanoma.
3. This is a classic example of melanoma arising in a condyloma.
4. The globular and fingerprint patterns diagnose a lentigo.
5. Fingerprint parallel patterns can have thick and/or thin line segments.
408 DERMOSCOPY: AN ILLUSTRATED SELF-ASSESSMENT GUIDE

RISK
✔ Low

❑ Intermediate
❑ High

DIAGNOSIS
❑ Nevus
❑ Seborrheic keratosis
❑ Basal cell carcinoma
❑ Vascular
❑ Dermatofibroma
❑ Squamous cell carcinoma
❑ Melanoma

5-9c
❑ Other

DISPOSITION
❑ No intervention ANSWERS
✔ Follow-up
❑ Answers: 4,5
❑ Histopathologic diagnosis
Discussion:
■ The patient has an irregularly pigmented lesion with scarring secondary to
lichen sclerosus et atrophius.
■ Even though the clinical appearance is worrisome the dermoscopic criteria
DERMOSCOPIC CRITERIA are low risk:
■ Foci of a globular pattern
■ Globular pattern (black boxes)
■ Foci of the parallel fingerprint pattern with thick line segments
■ Fingerprint pattern
■ There are no high risk dermoscopic features.
(yellow boxes) ■ The grayish color has no diagnostic significance.
■ Scarring (stars) ■ The wart seen clinically has no relationship to the lentigo.

PEARL
■ Once again, dermoscopy gives us a cleaner picture of a clinically-worrisome
pigmentation, eliminating the need for a surgical intervention.
Chapter 5 Genitalia 409

RISK
❑ Low
❑ Intermediate
❑ High

DIAGNOSIS
❑ Nevus
❑ Seborrheic keratosis
❑ Basal cell carcinoma
❑ Vascular

5-10a
❑ Dermatofibroma
❑ Squamous cell carcinoma
❑ Melanoma
❑ Other

DISPOSITION
❑ No intervention
❑ Follow-up
❑ Histopathologic diagnosis
5-10b

CASE 10
HISTORY
A 26-year-old man noticed a fast-growing lesion on his penis, which has been stable for 3 months.
1. Brown globules identify a melanocytic lesion.
2. Grayish -black color is a red flag for concern.
3. There is asymmetry of color and structure and a multicomponent global pattern that can be
seen in banal and high risk pathology.
4. This is a high risk lesion without a differential diagnosis that warrants a complete excision even
though it will be very destructive in this sensitive area.
5. Picking the most atypical dermoscopic area for an incisional biopsy is the better way to go
rather than a complete excision.
410 DERMOSCOPY: AN ILLUSTRATED SELF-ASSESSMENT GUIDE

RISK
❑ Low
✔ Intermediate

❑ High
1
2

DIAGNOSIS 3

❑ Nevus
❑ Seborrheic keratosis
4
❑ Basal cell carcinoma
❑ Vascular
❑ Dermatofibroma
❑ Squamous cell carcinoma
❑ Melanoma

5-10c
❑ Other

ANSWERS
DISPOSITION Answers: 1,2,3,5
❑ No intervention
❑ Follow-up Discussion:
✔ Histopathologic diagnosis ■ At first blush, the global dermoscopic picture of what turned out to be a

dermal nevus with blue nevus component (combined nevus) is worrisome
but one must keep in mind the concept of a dermoscopic differential
diagnosis.
■ Even a very irregular clinical and/or dermoscopic picture might not

DERMOSCOPIC CRITERIA represent high risk pathology.


■ The lesion has well-developed melanoma-specific criteria:
■ Asymmetry of color and
■ Asymmetry of color and structure
structure
■ Multicomponent global pattern
■ Multicomponent global ■ Irregular black dots/globules
pattern (1,2,3,4) ■ Irregular grayish-black blotches/color
■ Irregular brown dots/globules ■ Melanoma-specific criteria are very sensitive and specific for high risk
(white box) pathology (eg, dysplastic nevi, melanoma) but not diagnostic and can
■ Irregular black dots/globules be seen in melanocytic, nonmelanocytic benign, and malignant
(black boxes) pathology.
■ Irregular grayish-black ■ Diffuse erythema is always a red flag for concern that can be seen in banal
blotches (yellow boxes) and in high risk pathology.
■ Multifocal hypopigmentation ■ The differential diagnosis includes: nevus, dysplastic nevus, combined nevus
(stars) (blue nevus with overlying junctional/compound/dermal nevus), melanoma.
■ Diffuse erythema (pink color)
PEARLS
■ Aggressive surgical intervention should be avoided if the differential
diagnosis includes low risk pathology.
■ If diffuse erythema blanches away leaving low risk criteria, in most cases the
lesion will be low risk. If erythema blanches away leaving high risk or
nonspecific criteria then you could be dealing with a high risk lesion.
■ If you refer a patient with a lesion in this location to a colleague in or out of
our field to make the histopathologic diagnosis for you, make sure that you
convey the concept of a conservative surgical approach to them.
■ If in doubt, cut it out “does not always mean with a heavy hand!”
Chapter 5 Genitalia 411

RISK
❑ Low
❑ Intermediate
❑ High

DIAGNOSIS
❑ Nevus
❑ Seborrheic keratosis
❑ Basal cell carcinoma
❑ Vascular

5-11a
❑ Dermatofibroma
❑ Squamous cell carcinoma
❑ Melanoma
❑ Other

DISPOSITION
❑ No intervention
❑ Follow-up
❑ Histopathologic diagnosis
5-11b

CASE 11
HISTORY
A 20-year-old man had this lesion for 6 months.
1. This could be diagnosed a melanocytic lesion because there are globules or by default criteria.
2. There is a pigmented and amelanotic component, which are potentially high risk criteria.
3. Well-demarcated lacunae and hemorrhage diagnose a thrombosed hemangioma.
4. This is the banal homogeneous pattern that can be seen in a lentigo.
5. A dysplastic nevus and melanoma are in the differential diagnosis; therefore, a histopathologic
diagnosis should be made posthaste.
412 DERMOSCOPY: AN ILLUSTRATED SELF-ASSESSMENT GUIDE

RISK
❑ Low
❑ Intermediate
✔ High

DIAGNOSIS 1


❑ Nevus 2
❑ Seborrheic keratosis 3
❑ Basal cell carcinoma
❑ Vascular
❑ Dermatofibroma
❑ Squamous cell carcinoma
❑ Melanoma
❑ Other

5-11c
DISPOSITION
❑ No intervention
ANSWERS
❑ Follow-up Answers: 1,2,5
✔ Histopathologic diagnosis
❑ Discussion:
■ Clinically and dermoscopically, this is a high risk lesion (eg, dysplastic nevus,
melanoma) until proven otherwise by making a histopathologic diagnosis.
■ The histopathologic diagnosis was a dysplastic nevus with “Spitzoid”

DERMOSCOPIC CRITERIA features.


■ There are well-developed melanoma-specific criteria with a differential
■ Asymmetry of color and diagnosis:
structure ■ Multicomponent global pattern
■ Multicomponent global ■ Irregular grayish-black blotches that could represent atypical melanocytes
pattern (1,2,3) or melanophages.
■ Irregular brown globules (box) ■ Amelanotic component that represents one component of a benign nevus

■ Irregular grayish-black blotch or the amelanotic component of a partially pigmented melanoma.


(black arrows) ■ The focus of brown globules and polymorphous vessels are not easy to identify.
■ Polymorphous vessels The vessels should be compared to the well-developed vessels in Case 7.
(yellow arrows) ■ One can see a homogeneous global pattern in a lentigo or melanoma char-
■ Amelanotic component (stars) acterized by homogeneous color without local criteria.
■ A banal homogeneous global pattern has uniform light and/or dark color.

■ A high risk homogeneous global pattern has different shades of black,

brown, gray, and/or blue color.


■ There are no criteria to diagnose a hemangioma (eg, lacunae) or a collision
tumor.

PEARLS
■ Based on the history, clinical, and dermoscopic findings, this could be a high
risk lesion. It is small and an incisional biopsy is not practical and could
miss the important pathology. Therefore, in this case a complete, yet conser-
vative excision is indicated.
■ You can always go back and make a wider excision with high risk pathology.
However, an aggressive wide excision cannot be undone if the pathology
turns out to be banal!
Chapter 5 Genitalia 413

RISK
❑ Low
❑ Intermediate
❑ High

DIAGNOSIS

5-12a
Nevus
❑ Seborrheic keratosis
❑ Basal cell carcinoma
❑ Vascular
❑ Dermatofibroma
❑ Squamous cell carcinoma
❑ Melanoma
❑ Other

DISPOSITION
❑ No intervention
❑ Follow-up
❑ Histopathologic diagnosis
5-12b

CASE 12
HISTORY
A 54-year-old man had this lesion on his penis for several years without symptoms or any changes.
1. Fat-fingers diagnose a pigmented seborrheic keratosis.
2. The homogeneous pattern diagnoses a lentigo.
3. Irregular brown globules put a melanocytic lesion in the differential diagnosis.
4. Nonhomogeneous hyperpigmentation with a bluish-white tinge, irregular brown globules, and
irregular streaks put melanoma in the differential diagnosis.
5. Without banal criteria a histopathologic diagnosis is indicated.
414 DERMOSCOPY: AN ILLUSTRATED SELF-ASSESSMENT GUIDE

RISK
❑ Low
❑ Intermediate
✔ High

DIAGNOSIS
❑ Nevus
❑ Seborrheic keratosis
❑ Basal cell carcinoma
❑ Vascular
❑ Dermatofibroma
❑ Squamous cell carcinoma
❑ Melanoma

5-12c
❑ Other

DISPOSITION ANSWERS
❑ No intervention Answers: 3,4,5
❑ Follow-up
✔ Histopathologic diagnosis
❑ Discussion:
■ Clinically and dermoscopically, this is a worrisome lesion that warrants a
histopathologic diagnosis.
■ There are no criteria associated with a lentigo (eg, homogeneous, globular,
ring-like patterns) or seborrheic keratosis.
DERMOSCOPIC CRITERIA ■ Fat-fingers with digit-like line segments are not identifiable.

■ Asymmetry of color and ■ There are melanoma-specific criteria that put a dysplastic nevus or
structure melanoma in the differential diagnoses:
■ Irregular brown globules ■ Asymmetry of color and structure

■ Irregular brown globules


(yellow boxes)
■ Irregular streaks
■ Nonhomogeneous
■ Nonhomogeneous/irregular hyperpigmentation with a bluish-white tinge
hyperpigmentation with
■ This turned out to be bowenoid papulosis which is a clinical and
bluish-white tinge (stars)
pathologic diagnosis. The dermoscopic features have not been studied.
■ Irregular streaks (arrows ) ■ Bowenoid papulosis may be considered to be a transition state between a
■ Parallel line segments genital wart and Bowen disase.
(red boxes)
PEARLS
■ Dermoscopy has done its job by ruling out low risk pathology that does not
need a surgical intervention.
■ Getting up close is not fun but a must in the genital area with a suspicious
lesion that warrants dermoscopic examination.
■ The presence of a chaperone should be the standard of care in these
situations.
Chapter 5 Genitalia 415

RISK
❑ Low
❑ Intermediate
❑ High

DIAGNOSIS
❑ Nevus
❑ Seborrheic keratosis
❑ Basal cell carcinoma

5-13a
Vascular
❑ Dermatofibroma
❑ Squamous cell carcinoma
❑ Melanoma
❑ Other

DISPOSITION
❑ No intervention
❑ Follow-up
❑ Histopathologic diagnosis
5-13b

CASE 13
HISTORY
A colleague referred this 70-year-old man for a routine skin examination. There was a history of
squamous cell carcinoma on his penis. The patient brought this spot to our attention.
1. Clinically, but not dermoscopically, this is a worrisome lesion.
2. Well-developed, ring-like structures put a lentigo at the top of the differential diagnosis.
3. Regression, polymorphous vessels, and irregular globules diagnose an in situ acral lentiginous
melanoma.
4. Bowenoid papulosis is in the clinical but not dermoscopic differential diagnosis.
5. Bowenoid papulosis should be in the clinical and dermoscopic differential diagnosis.
416 DERMOSCOPY: AN ILLUSTRATED SELF-ASSESSMENT GUIDE

RISK
❑ Low
✔ Intermediate

❑ High

DIAGNOSIS
❑ Nevus
❑ Seborrheic keratosis
❑ Basal cell carcinoma
❑ Vascular
❑ Dermatofibroma
❑ Squamous cell carcinoma
❑ Melanoma

5-13c

❑ Other

DISPOSITION
❑ No intervention ANSWERS
❑ Follow-up Answers: 1,2,4,5
✔ Histopathologic diagnosis

Discussion:
■ We now have a series of bowenoid papulosis cases that look completely dif-
ferent clinically and dermoscopically. One with melanoma-specific criteria
and one with the ring-like pattern associated with a genital lentigo.
DERMOSCOPIC CRITERIA ■ Ring-like structures are well-developed and should not be confused with the
■ Ring-like structures (boxes) globules associated with a melanocytic lesion.
■ Normal skin color (stars) ■ The normal skin color should not be confused with regression.
■ There are no high risk dermoscopic criteria.
■ The irregular clinical appearance trumped the low risk dermoscopic pattern
and led to an incisional biopsy.

PEARLS
■ Do not ignore gut feelings that a lesion might be high risk, especially in the
genital area where much work needs to be done to clarify the global patterns
and local criteria associated with low, intermediate, and high risk pathology.
■ There was not a good clinico–dermoscopic–pathologic correlation since a
ring-like pattern has only been associated with lentigines in the genital area.
A review of the pathology by another experienced dermatopathologist con-
firmed the diagnosis of bowenoid papulosis.
Chapter 5 Genitalia 417

RISK
❑ Low
❑ Intermediate
❑ High

DIAGNOSIS

5-14a
❑ Nevus
❑ Seborrheic keratosis
❑ Basal cell carcinoma
❑ Vascular
❑ Dermatofibroma
❑ Squamous cell carcinoma
❑ Melanoma
❑ Other
5-14b1

DISPOSITION
❑ No intervention
❑ Follow-up
❑ Histopathologic diagnosis
5-14b2

CASE 14
HISTORY
The brown discoloration and pink nodule were found in a 72-year-old woman when her primary
care physician was about to perform a routine rectal examination.
1. The bluish-white and brown discoloration seen in both dermoscopic images are a red flag for
concern that this could be a rectal melanoma.
2. Rectal melanoma has never been reported and this represents a thrombosed hemorrhoid.
3. Multiple melanoma-specific criteria are seen in dermoscopy Image 2 (eg, irregular brown dots
and globules, irregular hyperpigmentation, polymorphous vessels, and bluish-white color), and
put melanoma at the top of the differential diagnostic list.
4. Purplish-brown color puts senile purpura in the differential diagnosis.
5. Since this is a difficult area to biopsy and the clinical and dermoscopic features are equivocal,
sequential digital clinical and dermoscopic monitoring is indicated instead of making a
histopathologic diagnosis. The patient should return in 6 months for a follow-up examination.
418 DERMOSCOPY: AN ILLUSTRATED SELF-ASSESSMENT GUIDE

RISK
❑ Low
❑ Intermediate
✔ High

DIAGNOSIS
❑ Nevus

5-14c1
❑ Seborrheic keratosis
❑ Basal cell carcinoma
❑ Vascular
❑ Dermatofibroma
❑ Squamous cell carcinoma

❑ Melanoma
❑ Other

DISPOSITION
❑ No intervention
❑ Follow-up

5-14c2
✔ Histopathologic diagnosis

ANSWERS
DERMOSCOPIC CRITERIA Answers: 1,3
■ Image 1 Discussion:
■ Bluish-white color (stars) ■ Any mucosal surface can develop melanoma. Theoretically, the genital area
■ Homogeneous brown color should routinely be included in a total body skin examination, but for many
(arrows) reasons that is not possible.
■ Irregular brown dots and ■ There are several high risk criteria that put melanoma at the top of the dif-
globules (boxes) ferential diagnostic list:
■ Bluish-white color ■ Irregular brown dots and globules
■ Image 2
■ Diffuse irregular hyperpigmentation ■ Polymorphous vessels
■ Irregular brown dots and
■ There are no criteria to suggest that this is hemorrhage.
globules (black boxes)
■ The criteria seen in the second image come from the pink nodule.
■ Irregular hyperpigmentation
■ Several years after the initial diagnosis, the patient was found to have
(yellow stars) metastatic melanoma to the brain and recently died!
■ Bluish-white color

(black stars) PEARLS


■ Polymorphous vessels ■ It is the luck of the draw who will develop a melanoma in an unusual place
(red boxes) and who will find a dermatologist/physician that will try to be as good as
they can be and routinely perform total body skin examinations and become
proficient in dermoscopy – a tissue sparing, potentially lifesaving technique.
■ Dermoscopy can improve the diagnosis of melanocytic, nonmelanocytic,
benign, malignant, and inflammatory lesions. It can help avoid unnecessary
surgical procedures and help one decide which would be the best way to
make a histopathologic diagnosis. The learning curve is relatively steep but
no comparison to subjects every physician has had to learn (eg, human
anatomy, biochemistry, genetics) to get his/her medical degree.
Index

Page numbers followed by “f ” denote figures; those followed by “t” denote tables.

A Asymmetrical follicular pigmentation, 15, 40, 62, 66, 78, 82, 84, 86,
ABCD rule, 3, 4t 88, 90, 92, 94
Abdomen Atypical dermatofibroma, 10f, 170
melanoma of, 165–166, 323–324 Atypical melanonychia striata, 16
nevus on, 161–162, 205–206, 217–218 Atypical Spitzoid pattern, 18–19, 25, 45–46, 56, 148, 191–192,
Acquired nevus 194–196, 198, 217–218
clinical features of, 17 Axillary seborrheic keratosis, 127–128
illustration of, 5f, 7f, 13f
Acral hemorrhage, 7f B
Acral lentiginous melanoma, 7f, 356, Back
362, 366 basal cell carcinoma of, 225–226, 239–240, 269–270, 305–306
Acral nevus, 6f, 348 melanoma of, 101–102, 105–108, 151–152, 167–168, 207–208,
Acral patterns, 6 231–232, 255–256, 265–266, 273–274, 291–292, 301–302,
Acrosyringia, 7, 328, 334, 336, 350, 356, 362 319–320
Actinic keratosis nevus of, 129–130, 143–144, 175–176, 193–194, 209–210,
characteristics of, 21 237–238, 267–268, 287–288, 295–296, 307–308, 321–322
pigmented, 80 seborrheic keratosis of, 131–132, 213–214, 223–224, 243–244,
Actinic lentigo, 16, 21 297–298, 301–302
African-Americans, 198 vascular lesions of, 215–216
Agminated nevi, 109 Basal cell carcinoma
Alopecia areata, 23 arborizing vessels of, 9, 9f, 60, 226, 228, 240, 250
Amelanotic melanoma, 5f, 11, 11f, 15, 17, 19–21, 20f, 25, of back, 225–226, 239–240, 269–270, 305–306
55–56, 113–115, 122, 178, 202, 212, 215, 227, 238, bony-white color of, 36, 228
243–244, 246, 248, 250–258, 309–310, 367–370, of chest, 227–228
380, 386, 401–402, 412 criteria for, 5t, 9f, 9–10, 23
Amelanotic nail-apparatus melanoma, 386 of face, 57–58
Amelanotic nodule, 114 irregular dots and globules of, 48, 182
Androgenic alopecia, 23 of leg, 271–272
Angiokeratoma, 120, 122 nonarborizing vessels of, 74, 228
Annular-granular structures, 12t, 16f, 19, 21, 42, 64 of nose, 35–35, 73–74
description of, 15 pigmented, 9, 9f, 142, 181–182, 225–226
of face, 79–80, 85–86, 88, 90 polymorphous vessels of, 228, 272
peppering vs., 90 of scalp, 47–48, 59–60
Appendageal openings, 70, 74 spoke-wheel structures in, 9–10, 181–182, 305–306
Arborizing telangiectases, 73 ulceration in, 9, 36, 48, 58, 60, 181–182, 228, 240, 270, 272
Arborizing vessels, 5t, 18, 23–25, 35–37, 47–48, 53–54, 57–60, Basal cell-like vessels, 38
73–74, 166, 175–176, 182, 200, 225–233, 235–236, 239–240, Benign pigmented nail bands, 16
242, 269–272, 305–306 “Black” blue nevi, 32
of basal cell carcinoma, 9, 9f, 60, 226, 228, 240, 250 Black dots, 4t, 5, 22–23, 270, 338, 392, 410
description of, 9, 9f “Black heel,” 363–364
of melanoma, 176, 230, 236 “Black lamella,” 32, 77–78, 189, 193–194, 204, 296
of nevus, 175–176 “Blood pebbles,” 352, 380, 402
Areola melanoma, 203–204 Blotches
Arm black, 44, 46, 62, 64, 76, 78, 160, 164, 172
melanoma of, 155–156, 171–172, 185–186, 211–212, 235–236, blue, 118, 162, 168, 172
245–246 brown, 13, 17f, 22, 34, 175–176, 176, 230, 242, 268, 278, 280,
nevus of, 179–180, 187–188, 191–192 286, 290, 292, 300, 304, 314, 360, 366
420 Index

Blotches (Cont.): Children


description of, 13 nail-apparatus melanoma in, 371
grayish-black, 40, 48 nevus in
irregular, 44 of back, 107–108, 193–194
of nevus, 50 blue, 31–32
Blue blotches, 118, 162, 168, 172 congenital melanocytic nevi, 149–150
Blue globules, 74, 166 of ear, 55–56
Blue nevus of face, 69–70
of arm, 180 of fingers, 335–336
case studies of, 31–32, 180 of foot, 331–332
in child, 31–32 of scalp, 49–52
clinical features of, 6, 6f, 17 Spitzoid melanoma in, 194
polychromatic, 32 Chronic lichen planus-like keratosis, 261–262
Blue ovoid nest of pigmentation Cicatricial alopecia, 23
of basal cell carcinoma, 182, 184, 303, 306 Clear cell acanthoma, 250, 253–254
differential diagnosis of, 182 Cobblestone globules, 138, 164
Blue-white veil, 13, 104 Cobblestone pattern, 12, 92, 108, 138–140,
Bluish-white color 246, 308
asymmetry of, 212 Collision tumors, 21, 21f, 62, 116, 244, 302
of basal cell carcinoma, 60 Combined nevus, 17, 177–178, 410
description of, 14 Comma-shaped vessels, 250, 254
of genitalia, 400, 402 Compound nevus, 264, 340
of melanoma, 44, 58, 92, 104, 126, 166, 172, 186, 212, 230, 300, Congenital melanocytic nevi, 136, 149–150
316, 356, 366, 404, 418 Congenital nevi, 17, 17f, 163–164
of nevus, 124, 164, 174, 290, 336 “Crown on thorns” nevus, 238
of seborrheic keratosis, 128, 214, 224 “Crown vessels,” 38, 142
Bony-white color Crypts, 142, 284
of basal cell carcinoma, 36, 228 Cutaneous lupus, 22f
of desmoplastic melanoma, 388 Cutaneous metastatic melanoma
of melanoma, 274 description of, 20, 118
of regression, 156 features of, 11f, 178, 187–188
Bowen disease
characteristics of, 15, 15f, 21, 226, 250 D
glomerular vessels of, 250, 252 Dark homogeneous areas, 12t, 15, 188, 364, 372, 386
Bowenoid papulosis, 413–414, 415–416 Depigmentation
Breast melanoma, 203–204 of basal cell carcinoma, 226
Brown blotches, 13, 17f, 22, 34, 175–176, 176, 230, 242, 268, 278, reticular
280, 286, 290, 292, 300, 304, 314, 360, 366 in dermatofibroma, 312
Brown dots, 4t, 5, 15, 17, 24, 74, 80, 105, 106, 114, 166, 220, 224, 226, in melanoma, 154, 156, 216, 236, 314,
232, 236, 238, 268, 280, 290, 300, 304, 310, 314, 316, 340, 360, 320, 338
410, 417–418 in nevus, 218
Brown globules in tumoral melanosis, 200
of basal cell carcinoma, 74, 240 Dermatofibroma
of melanoma, 118, 232, 236, 242, 274, 308 atypical, 10f, 276
of nevus, 336 characteristics of, 208, 274
in parallel furrows, 332 criteria for, 5t, 10, 10f, 24–25, 152
Brown pigmentation, irregular, 110 of extremities, 153–154
of leg, 153–154, 275–276, 311–312
C nonatypical, 276
Cherry hemangiomas, 120 pigment network of, 10
Chest polymorphous vascular structures, 10, 10f
basal cell carcinoma of, 227–228 reticular depigmentation in, 312
melanoma of, 108–109, 125–126, 133–136, 159–160, 195–196, ring-like structures of, 10
221–222, 299–300, 303–304 senile, 159
nevus of, 173–174 Dermatomyositis, 22
Index 421

Dermoscopy Fish scale-like pattern, 397–398, 400


benefits of, 2 Fissures, 8, 140, 142
definition of, 2 Flat melanoma, 20
digital monitoring uses of, 2 Follicular openings, 80
two-step algorithm for, 3–7 in basal cell carcinoma, 36, 62
Desmoplastic melanoma, 388 in melanoma, 40, 44, 64, 66, 78, 82, 84, 86, 88
Diffuse erythema, 108, 154, 238, 410 in nevus, 30, 32
Dots and globules. See also Globules Follicular pigmentation, 90
black, 4t, 5, 13, 22–23, 270, 338, 392, 410 Foot
brown, 4t, 5, 13, 15, 17, 24, 74, 80, 105, 106, 114, 166, 220, 224, melanoma of, 337–338, 345–346, 349–350, 353–354, 355–356,
226, 232, 236, 238, 268, 280, 290, 300, 304, 310, 314, 316, 357–358, 359–360, 361–362, 363–364
340, 360, 410, 417–418 nevus of, 329–330, 331–332, 333–334, 341–342, 343–344,
cobblestone, 138 347–348
color of, 5 Forearm, squamous cell carcinoma of, 249–250
description of, 5 Forehead melanoma, 89–92
gray, 5, 13 Free melanin, 268
irregular. See Irregular dots and globules “Fried egg” appearance, 33–34, 238, 279–280
melanocytic, 13 Furrows, 6, 6f. See also Parallel furrow patterns
nevus and, 34
red, 5, 13 G
regular, 13, 70, 138 Genital lentigines, 392
Dysplastic nevus Genital lentigo, 391–392, 395–398
clinical features of, 108, 168, 268, 288, 290, 317–318 Genitalia
criteria for, 18, 18f, 24–25, 322 Bowenoid papulosis of, 413–414, 415–416
melanoma of, 403–404, 417–418
E nevus of, 409–410, 411–412
Ear Glabrous skin
melanoma of, 12t, 45–46, 65–66, 77–78 description of, 327–328, 396
nevus of, 55–56 volar surfaces of, 6
Eccrine pores. See Acrosyringia Global patterns
Erythronychia, 17, 20, 380 description of, 12
Eumelanin, 14 homogeneous, 12, 364, 412
Extremities. See also Arm; Leg multicomponent. See Multicomponent global pattern
dermatofibroma of, 153–154 Globular pattern
melanoma of, 12t, 111–114, 117–118, 145–146, 155–158, description of, 150
169–170 of nevus, 206
nevi of, 97–98, 147–148 nonlinear, 406
in situ melanoma of, 19 of penile melanosis, 406
Globular-like structures, 154, 183–184
F Globules. See also Dots and globules
Face blue, 74, 166
annular-granular structures of, 79–80 brown
basal cell carcinoma of, 57–58 of basal cell carcinoma, 74, 240
lentigo maligna of, 57–58 of melanoma, 118, 232, 236, 242, 274, 308
melanoma of, 12t, 43–44, 61–64, 83–90, 93–94 of nevus, 336
nevus of, 31–34, 69–70 in parallel furrows, 332
seborrheic keratosis of, 29–30 cobblestone, 138, 164
solar lentigo of, 67–68, 88 description of, 5, 14f
False negative melanoma, 20 Glomerular vessels
“Fat fingers,” 8, 8f, 138, 140, 142 Bowen disease and, 250, 254
Fibrillar acral pattern, 6, 348 description of, 15, 15f
Fibrous border, 120 linear, 254
Fibrous septa, 120 Gray dots, 5
Fingerprint pattern, 8, 15f, 68, 90, 132, 136, 408 Gray macules, 262
Fingers, nevus of, 335–336 Grayish-black blotches, 40, 48
422 Index

H K
Hair shafts Keratoacanthoma, 21
abnormalities of, 22–23 Knee melanoma, 309
illustration of, 90
Hairpin vessels, 8, 8f, 54, 56, 244, 256, 382 L
Halo nevus, 118, 277–278, 308 Lacunae, 10–11, 116, 120, 123–124, 216, 248
Halo phenomenon, 278 Lagoons, 10–11
Heel, 363–364 Lanugo hairs, 150
Hemangioma, 10f, 11, 120, 194, 216 Latinos, 198
Hemorrhage Lattice-like pattern, 6, 332, 334
acral, 7f “Leaf-like” structures, 142, 182
skin, 351–352 Leg
Hemorrhagic crusts, 122, 364 basal cell carcinoma of, 271–272
Homogeneous blue pigmentation, 6 dermatofibroma of, 153–154, 275–276, 311–312
Homogeneous pattern, 6, 12, 130, 364, 412 melanoma of, 157–158, 169–170, 201–202, 313–314
Hutchinson sign, 16, 371–372, 374 nevus of, 293–294
Hyperkeratosis, 106, 210, 222, 234 Lentigo
Hyperpigmentation actinic, 21
description of, 298 genital, 391–392, 395–396, 397–398
irregular, 130 solar
Hyperpigmented globules, 150 case studies of, 67–68, 71–72
Hypopigmentation characteristics of, 21, 262, 346
description of, 14, 98, 133, 144, 196 of chest, 133
differential diagnosis, 238, 280 of face, 88
of melanoma, 160, 282, 310, 314 of lip, 71–72
multifocal, 84, 98, 100, 108, 110, 112, 148, 300, 318, 322, Lentigo maligna
344, 410 characteristics of, 5, 11, 16–17, 19, 21, 29–30, 57, 62, 77–80,
of nevus, 162, 164, 206, 238, 288, 290 82–86, 88–90, 132, 145, 396
perifollicular, 150, 278 of ear lobe, 16f
regression vs., 280, 282 of face, 57–58, 88
of seborrheic keratosis, 30 Lentigo maligna melanoma, 11, 15–16, 19, 30, 77–78
of squamous cell carcinoma, 252 Lichen planus, 22
Hypopigmented fissures, 138 Lichen planus-like keratosis
acute, 258
I characteristics of, 15f, 80
In situ melanoma, 19, 19f, 264, 344 chronic, 261–262
Ink-spot lentigo pink, 18
case studies of, 41–42, 392 subacute, 259–260
criteria for, 21 Lichen sclerosus et atrophicus
Invasive melanoma, 14–15, 47, 62, 72, 94, 102, 104, 110, 114, 118, of penis, 407–408
126, 133, 135, 146, 157, 203, 222–223, 247–248, 264, of vulva, 401–402
290–291, 355, 358, 366, 402–404 Linear globules, irregular, 72
Irregular black blotches, 44, 46, 62, 64, 76, 78, 160, 164, 172 Linear glomerular vessels, 54, 56, 254, 256, 270
Irregular dots and globules Linear hypopigmentation, 100
of basal cell carcinoma, 48, 182 Linear pinpoint vessels, 250
black, 234 Lip, solar lentigo of, 71–72
of blue nevus, 180 Lupus erythematosus, 22
description of, 13 Lymphangiomas, 120
of melanoma, 44, 46, 64, 66, 76, 82, 84, 92, 94, 102, 104, 110, Lymph-hemangiomas, 120
126, 136, 146, 152, 156, 158, 168, 174, 186, 192, 202, 204,
220, 242, 310, 314, 360 M
of nevus, 50, 52, 108, 162, 278, 288 Melanocytic lesions
of Spitzoid pattern, 196 analysis of, 11–21
of tumoral melanosis, 200 criteria for, 4–7
Irregular linear globules, 72 diagnosis of, 23
Index 423

Melanocytic lesions (Cont.): Melanoma incognito, 2, 20, 314


global patterns of, 12 Melanoma-specific criteria, 12, 12t, 14f, 16–17, 19–21, 24–25, 64,
local criteria for, 13–17, 23 66, 70, 84, 91–92, 98, 112, 114, 132, 134, 136, 154, 159–160,
pattern analysis of, 11–12 168, 171, 174, 176, 191–192, 198, 201–202, 206–207, 210,
pigment network, 14f 214, 231–232, 234, 236, 250, 255, 260, 263, 265, 267–268,
Melanoma 274, 280, 286–287, 299–300, 302, 304, 309, 314, 317–320,
of abdomen, 165–166, 323–324 337–338, 342, 350, 356, 366, 379, 392, 396, 400, 410, 412,
acral lentiginous, 7f, 356, 362, 366 414, 416–417
amelanotic, 5f, 11, 11f, 15, 17, 19–21, 20f, 25, 55–56, 113–115, Melanonychia striata, 16, 372, 376
122, 178, 202, 212, 215, 227, 238, 243–244, 246, 248, Melanophages, 14, 268, 394
250–258, 309–310, 362, 367–370, 380, 386, 401–402, 412 Melanosis
arborizing vessels of, 176, 230, 236. See also Arborizing vessels description of, 210
of areola, 203–204 penile, 405–406
of arm, 155–156, 171–172, 185–186, 211–212, 235–236, 245–246 tumoral, 197–198, 199–200
of back, 101–102, 105–108, 151–152, 167–168, 207–208, Menzies method, 3, 4t
231–232, 255–256, 265–266, 273–274, 291–292, 301–302, Micro-Hutchinson sign, 16, 377–378
319–320 Milia-like cysts, 7–8, 30, 56, 128, 166, 180, 186, 208, 214, 244, 248,
characteristics of, 123 266, 276, 284, 292
of chest, 108–109, 125–126, 133–136, 159–160, 195–196, Milky-red areas
221–222, 299–300, 303–304 description of, 15, 116
desmoplastic, 388 of melanoma, 114, 186, 216, 232, 236, 246, 292, 304, 310, 366
of ears, 45–46, 65–66, 77–78 of nevus, 52, 174
of extremities, 12t, 111–114, 117–118, 145–146, 155–156, of seborrheic keratosis, 244
157–158, 169–170 of vascular lesions, 124
of face, 43–44, 61–64, 83–90, 93–94 Molluscum contagiosum, 38
featureless, 20 Monilethrix, 22
feature-poor, 20 “Moth-eaten borders,” 9, 30, 68, 82, 84, 86, 88, 132
of foot, 337–338, 345–346, 349–350, 353–354, 355–356, “Mountain and valley” pattern, 140
357–358, 359–360, 361–362, 363–364 Multicomponent global pattern, 12, 64, 72, 76, 92, 104, 106, 108,
of forehead, 89–92 114, 118, 126, 136, 146, 152, 156, 162, 164, 168, 170, 172,
of genitalia, 403–404, 417 174, 176, 256, 300, 302, 306, 316
in situ, 19, 19f, 264, 344 Multifocal hypopigmentation, 84, 98, 100, 108, 110, 112, 148, 300,
invasive, 14–15, 47, 62, 72, 94, 102, 104, 110, 114, 118, 126, 133, 318, 322, 344, 410
135, 146, 157, 203, 222–223, 247–248, 264, 290–291, 355,
358, 366, 402–404 N
of knee, 309 Nail(s)
of leg, 157–158, 169–170, 201–202, 313–314 gray bands in, 377–378
lentigo maligna, 11, 15–16, 19, 30, 77–78 nevus of, 371–372, 381–382
nail-apparatus. See Nail-apparatus melanoma subungual hematoma, 16, 17f, 346, 379–380, 383–384
of nose, 39–40, 81–82 Nail bands, 16, 371, 377–378
polymorphous vessels of, 222, 232, 234, 250 Nail folds, 22, 22f
pyogenic granuloma vs., 370 Nail matrix melanoma, 380
recurrent, 278 Nail-apparatus melanoma
regression of, 46, 64, 66, 76, 94, 102, 104, 106, 110, 118, 134, 136, amelanotic, 386
146, 152, 158, 172, 242, 266, 274, 304, 316, 366 in children, 371
of scalp, 75–76 criteria for, 12t
of shoulder, 229–230, 281–282 features of, 16f, 20–21, 371, 373–374, 375–376, 385–386
of sole, 12t, 365–366 Netherton syndrome, 23
Spitzoid, 19, 191–192 Nevus
superficial spreading, 19–20, 24–25 of abdomen, 161–162, 205–206, 217–218
of thigh, 199–200, 219–220, 241–242, 247–248, 263–264, acquired, 5f, 7f, 13f, 17
315–316 acral, 6f, 348
of trunk, 103–104, 197–198, 233–234 of arm, 179–180, 187–188
types of, 11 of back, 129–130, 143–144, 175–176, 209–210, 237–238,
ulceration in, 40, 94, 310 267–268, 287–288, 295–296, 307–308, 321–322
424 Index

Nevus (Cont.): Parallel furrow patterns (Cont.):


blue. See Blue nevus in melanoma, 354
of chest, 173–174 of palm, 327–328
in children. See Children, nevus in single-line variant, 330
combined, 17, 410 Parallel line segments, 90, 276, 414
compound, 264, 340 Parallel patterns, 6
congenital, 17, 17f, 136, 163–164 Parallel ridge pattern
“crown on thorns,” 238 acrosyringia in, 7
dysplastic. See Dysplastic nevus description of, 7, 328
of ears, 55–56 differential diagnosis, 352
of extremities, 97–98, 147–148 malignant, 336, 358, 362
of face, 31–34, 69–70 in melanoma, 354, 360, 362
of fingers, 335–336 pigmented, 350
of foot, 329–330, 331–332, 333–334, 341–342, 343–344, Parallelism
347–348 description of, 372
“fried egg” appearance of, 33–34 loss of, 386
of genitalia, 409–410, 411–412 Pattern analysis, 3, 4t, 11–12. See also specific pattern
halo, 118, 277–278, 308 Pediculosis capitis, 22
of leg, 293–294 Pediculosis pubis, 22
of nails, 371–372, 381–382 Penis
of palm, 327–328 Bowenoid papulosis of, 413–414, 415–416
of penis, 409–410, 411–412 lichen sclerosus et atrophicus of, 407–408
recurrent, 17–18, 18f, 278 melanosis of, 405–406
of scalp, 49–52 nevus of, 409–410, 411–412
of shoulder, 289–290 Peppering
of sole, 339–340 description of, 5
Spitz, 18–19, 19f, 24–25, 143–144, 187–188, 193–194, differential diagnosis of, 202
218, 294 general images of, 50, 71–72, 80, 106, 128, 262
of trunk, 97–100, 115–116, 137–140 of melanoma, 64, 90, 106, 152, 156, 172, 202, 232, 246
types of, 11 of seborrheic keratosis, 128
Nevus spilus, 109 of tumoral melanosis, 200
Nodular melanoma, 20, 121–122, 212, 284, 304 Perifollicular hypopigmentation, 150, 278
Nonarborizing vessels, of basal cell carcinoma, 74, 228 Peripheral erythema, 122, 192
Nonlinear globular pattern, 406 Periungual pigmentation, 371
Nonpigmented pseudofollicular opening, 132, 142, 224 Pigment networks
Nose of dermatofibroma, 10
basal cell carcinoma of, 35–36, 73–74 description of, 13
melanoma of, 12t, 39–40, 81–82 irregular, 98, 100, 104, 108, 110, 112, 114, 126, 130, 134, 136,
146, 152, 160, 176, 238, 266, 292, 294, 320, 338
O of melanoma, 230, 292, 338
Ovoid nests of nevus, 98, 290, 294, 342
of basal cell carcinoma, 182, 184, 303, 306 regular, 98, 102, 114, 116, 134, 154
differential diagnosis of, 182 Pigmentation
asymmetrical, 15
P basal cell carcinoma, 9, 9f, 142, 181–182, 240
Palm blue ovoid nest of
melanoma of, 12t of basal cell carcinoma, 182, 184, 303, 306
nevus of, 327–328 differential diagnosis of, 182
Parallel furrow patterns homogenous blue, 6
brown globules in, 332 hyperpigmentation, 130, 298
description of, 6 hypopigmentation. See Hypopigmentation
double-line variant, 329–330 Pigmented actinic keratosis, 79–80, 85–86, 89
of foot, 329–330, 332 Pigmented pseudofollicular openings, 30, 128, 224, 242, 284, 298
irregularities of, 331–332 Pili annulati, 23
lattice-like pattern, 334 Pink lichen planus-like keratosis, 17, 18, 18f
Index 425

Pinpoint vessels, 54, 56, 170, 238, 242, 252, 254, 256, 258, Scalp (Cont.):
270, 310, 368 nevus of, 49–52
Polychromatic blue nevus, 32 seborrheic keratosis of, 53–54
Polymorphous vascular pattern, 14–15 Scleroderma
Polymorphous vessels nail-fold capillaries in, 22f
of basal cell carcinoma, 228, 272 pattern associated with, 22
of melanoma, 40, 62, 106, 222, 232, 234, 404 Sebaceous gland hyperplasia
of nevus, 412 case study of, 37–38
Pseudofollicular openings, in melanoma, 36, 44 characteristics of, 21
Pseudo-Hutchinson sign, 16, 372 Seborrheic keratosis
Pseudomelanoma, 17–18 atypical, 224
Pseudonetwork, 67–68, 80, 82, 86 of axilla, 127–128
Pseudopigment network, 70 of back, 131–132, 213–214, 223–224, 243–244, 297–298,
Pseudopods, 13, 194 301–302
Psoriasis, 22, 252, 254 criteria for, 5t, 7–9, 8f, 23
Purplish color of face, 29–30
of basal cell carcinoma, 58 flat, 254
of melanoma, 146, 246 incipient, 142
of nevus, 162 milia-like cysts associated with, 7f, 7–8
of vascular lesions, 183–184 papillomatous, 8
Pyogenic granuloma, 20, 119–120, 369–370 of scalp, 53–54
of thigh, 283–284
R of trunk, 141–142
Radial streaming, 194 Senile dermatofibroma, 159
Radiation tattoo, 262 Senile hemangiomas, 120
Recurrent melanoma, 278 Seven-point checklist, 3, 4t
Recurrent nevus, 17–18, 18f, 278 Sharp border demarcation, 142
Red dots, 5, 13 Sharp demarcation, 9
Reflection artifact, 346 Shoulder
Regression melanoma of, 229–230, 281–282
bony-white color of, 156 nevus of, 289–290
characteristics of, 13, 186, 238, Skin hemorrhage, 351–352
264, 302 Skin lesions
hypopigmentation vs., 280, 282 criteria for, 5t
of melanoma, 46, 64, 66, 76, 94, 102, 104, 106, 110, 118, 134, melanocytic. See Melanocytic lesions
136, 146, 152, 158, 172, 242, 266, 274, 304, 316, 366 Solar lentigines
of tumoral melanosis, 200 characteristics of, 10, 68
white color with, 13, 170 on lips, 72
Reticular depigmentation Solar lentigo
in dermatofibroma, 312 case studies of, 67–68, 71–72
in melanoma, 154, 156, 158, 216, 236, 314, 320, 338 characteristics of, 21, 262, 346
in nevus, 218 of chest, 133
in tumoral melanosis, 200 of face, 88
Reticular global pattern, 12, 238, 288, 296, 320, 346 of lip, 71–72
Rhomboid structures, 15, 62, 78 Sole
Ridges, 8 melanoma of, 12t, 365–366
Ring-like pattern, 10, 395–396, 397–398, 400, 416 nevus of, 339–340
Spitz nevus, 18–19, 19f, 24–25, 143–144, 187–188, 193–194, 218, 294
S Spitzoid melanoma
Saccules, 10–11 in children, 194
Scabies, 21 description of, 19, 191–192
Scaliness, 222 Spitzoid pattern
Scalp atypical, 18–19, 25, 45–46, 56, 148, 191–192, 194–196, 195, 198,
basal cell carcinoma of, 47–48, 59–60 217–218
melanoma of, 75–76 description of, 45, 143–144, 148
426 Index

Spitzoid pattern (Cont.): Trunk


irregular, 191–192, 208 basal cell carcinoma of, 181–182
of nevus, 218, 294 melanoma of, 12t, 103–104, 197–198, 233–234
regular, 188 nevi of, 97–100, 115–116, 137–140
Spoke-wheel structures, of basal cell carcinoma, 9–10, seborrheic keratosis of, 141–142
181–182, 305–306 in situ melanoma of, 19
Squamous cell carcinoma vascular lesions of, 119–124, 183–184
of forearm, 249–250 Tumoral melanosis, 197–198, 199–200
of thigh, 251–252 Tyndall effect, 14, 128, 180, 268
“Starburst” pattern, 12, 14f, 18, 19f, 46, 187–188, 191–194
Sternal melanoma, 221–222 U
Streaks “Ugly duckling” appearance, 98, 163–164, 213–214, 279–280, 297
description of, 13 Ulceration
irregular, 13, 46, 126, 236, 316, 414 in basal cell carcinoma, 9, 36, 48, 58, 60, 181–182, 228, 240,
of melanoma, 46, 126, 236, 316 270, 272
of nevus, 187–188, 193–194 in melanoma, 40, 94, 310
regular, 13, 194
Subacute lichen planus-like keratosis, 259–260 V
Subungual hematoma, 16, 17f, 346, 379–380, 383–384 Vascular lesions
Sun-damaged skin, 262–263 assessment of, 10–11
Superficial spreading melanoma of back, 215–216
characteristics of, 19–20 criteria for, 10–11, 24–25
criteria for, 24–25 milky-red areas of, 124
Synonyms, 2 of thigh, 183–184
of trunk, 119–124, 183–184
T Vulva
Tape stripping, 296 lichen sclerosus et atrophicus of, 401–402
Thick melanoma, 20 melanoma of, 417
Thigh
melanoma of, 199–200, 219–220, 241–242, 247–248, 263–264, W
315–316 Warts, 22
seborrheic keratosis of, 283–284 White color
squamous cell carcinoma of, 251–252 differential diagnosis, 234, 262
vascular lesions of, 183–184 with regression, 13
Three-point checklist, 3, 4t of vascular lesions, 183–184
Thrombosed hemangioma, 123 white network/white pigment network, 10, 19, 22, 153–154, 192,
Thrombosed lacunae, 124 215, 216, 218, 236, 246, 312, 314
“Train tracks,” 72, 398 Whitish dots of melanoma, 222
Trichoscopy, 22 “Wobble” sign, 138, 308
Wood’s light examination, 266

Anda mungkin juga menyukai